You are on page 1of 274

1. – 18. sorularda, cümlede boş bırakılan yerlere 9.

Many engineers ---- the thrill of designing a


uygun düşen sözcük ya da ifadeyi bulunuz. novel product that then ---- mass production.

1. In the life sciences, biologists need to be more A) are having / is entering


aware of the ---- between science and technology. B) had had / had entered
C) would have / has entered
A) approach B) departure C) complexity D) have had / enters
D) extremity E) gap E) were having / have entered

2. The theory is interesting enough, but is it ----? 10. Over the past eight years, the TES instrument
---- that Martian rocks and sands ---- almost
A) solvable B) expressive C) intensive entirely of volcanic minerals.
D) coherent E) resistant
A) would discover / had been composed
B) has discovered / are composed
3. However incredible we may now find it, C) would discover / were composed
engineers did not ---- welcome the idea of a D) had discovered / had been composed
general purpose microchip. E) was discovering / would be composed

A) instantly B) urgently C) crudely


D) scarcely E) reliably 11. In order ---- a good sheep-shearing robot I had
to understand sheep shearers and the skill of
shearing, as well as the technology ---- in building
4. His efforts to ---- the threat of global warming a robot.
with new forms of energy have been much
appreciated. A) having built / to have been involved
B) building / to be involved
A) excuse B) counter C) pursue C) to build / involved
D) uphold E) deliver D) to be building / involving
E) to have built / having been involved

5. The amount of fish caught did not ---- the


massive increase in vessel tonnage and fishing 12. It is estimated that sulphur pollution costs
effort. China nearly 45 billion dollars each year ---- lost
productivity, health care and damage ---- forests
A) pull through B) make do with and crops.
C) keep pace with D) come across
E) break out of A) from / at B) in / to C) under / of
D) over / through E) by / for

6. Home heating, which ---- less than 7 per cent of


all energy consumed in the US, has had a 13. One of the great advances ---- astronomy ----
commendable efficiency record. the past decade has been the discovery of planets
outside our solar system.
A) accounts for B) comes with C) joins in
D) picks up E) brings out A) of / over B) through /at C) in / to
D) for / by E) with / for

7. For a long time now, biologists ---- that bits of


tissue placed next to each other ----. 14. He realized that the world could run out of key
resources, ---- he was a harsh critic of the
A) had known / may fuse wastefulness of modern industrial society.
B) have known / can fuse
C) knew / had fused A) so that B) whether C) so long as
D) would have known / have fused D) and so E) but
E) know / would have fused
15. ---- the car is equipped with a sophisticated
8. It ---- that Brazil’s new surveillance system ---- a protection system, you know you are fully
useful tool in the protection of the rain forests. protected.

A) may be hoped / would have proved A) Until B) Even if C) Although


B) was hoped / had proved D) So E) Since
C) has been hoped / would prove
D) could be hoped / might have proved
E) is hoped / will prove 16. Certain reactions, ---- catalytic methanation,
appear to stop before they are complete.

A) instead of B) such as C) in place of


D) in case of E) with reference to
1
17. Engineering structures must conform ---- to 25. In biology, isolating particular enzymes is a
their type ---- to the laws of physics. tedious process of trial and error ----.

A) not only / but also B) as / as A) though silicon is not the best choice of material
C) more / but D) both / as B) if several hundred steps were involved
E) either / and C) which involves many different experiments
D) until other problems could be eliminated
E) as lab experiments may prove unnecessary
18. The harder a material is, ---- ductile or
workable it tends to be.
26. Just try shooting at bullet-proof glass ----.
A) the most B) as much C) the less
D) more E) so much A) but you can stand a few meters away
B) if you want to be quite sure that it really is bullet
proof
19. – 23. sorularda, aşağıdaki parçada C) while the glass remained unharmed
numaralanmış yerlere uygun düşen sözcük ya da D) why some gangs have started to fire anti-tank
ifadeyi bulunuz. missiles at armoured cars
E) that it withstood every attack
Natural disturbances, including hurricanes and
earthquakes, have affected coral reefs for millions of
years. They are typically acute but have short-lived 27. The vast oil output of the Caspian must be
(19) ----. Reef areas (20) ---- human influences often piped overland to, say, the Mediterranean, ----.
recover within a few years (21) ---- water and
substratum quality remain high. Indeed, acute natural A) that it is not a landlocked sea
disturbances can actually help (22) ---- diversity on B) if a pipeline is laid across Iran
coral reefs by knocking back dominant species and C) before it can be pumped into tankers
allowing (23) ---- competitive species to re-establish D) which would also pass through Georgia
themselves. E) as a great deal of diplomacy would be required

19.
A) products B) conditions C) concerns 28. Their latest digital radio is supplied with a pair
D) effects E) explanations of active speakers ----.

A) if the number keys have been moved to the sides


20. B) that an FM radio is fitted into its compact
A) down to B) up to C) up against dimensions
D) out of E) away from C) though the screen itself seemed to be touch-
sensitive
D) so you don’t have to plug it into an amplifier
21. E) how the graphic display shows the programme
A) so that B) unless C) though you are listening to
D) if E) whether
29. The engineers ---- took their know-how to
22. Japan and China in the late 19th and early 20th
A) to have maintained B) maintaining centuries.
C) having maintained D) to be maintained
E) to maintain A) that it would be the world’s largest dam
B) who continue to travel around the world
transferring technology
23. C) who had built railroads and dams across America
A) much B) little C) as D) as engineering problems can attract worldwide
D) less E) least interest
E) though the spread of technology is not likely to be
halted
24. – 35. sorularda, verilen cümleyi uygun şekilde
tamamlayan ifadeyi bulunuz.

24. Battlefield radios may become obsolete ----.

A) if software programmable units take over


B) until they are all built to a common standard
C) when local police and fire departments would
also benefit
D) since other versions had become tailored to their
needs
E) while security features are also innovative

2
30. Although global warming was outside the 35. Because each leaf is characteristic of the plant
parameters of their study, ----. on which it grows, ----.

A) countries with high gasoline prices are more A) most leaves are composed of three parts: a
innovative in the field of personal transportation blade, a petiole and a pair of stipules
vehicles B) water loss by evaporation from the leaf’s surface
B) battery-powered electric vehicles would not have is unavoidable
been disregarded C) many plants can be identified by their leaves
C) fossil fuel consumption habits will have to be alone
curtailed D) all parts of a plant can be damaged by air
D) for the present the focus is on the efficient use of pollution, but leaves are particularly susceptible
fossil fuels E) many leaves have special structures through
E) it is nevertheless a fact that should have been which water is literally forced out
faced

36. – 38. sorularda, verilen Đngilizce cümleye


31. ---- as they are today. anlamca en yakın Türkçe cümleyi bulunuz.

A) Satellites are providing clear photographs 36. The findings of a ten-year study reveal that
B) The fluctuating magnetic field lies deep in the two species of seahorses have been living in the
centre of Earth waters around Britain without anyone realizing
C) Several other bodies in the solar system they were there.
generate their own magnetic fields
D) Earth’s magnetic poles have not always been A) Đngiltere’nin çevresindeki sularda daha önce
oriented onların orada yaşadığını kimsenin bilmediği iki tür
E) Many intriguing explanations are being put denizatının olduğu on yıllık bir çalışmayla ortaya
forward çıktı.
B) On yıllık bir çalışmanın bulguları Đngiltere’nin
çevresindeki sularda onların orada olduğunu hiç
32. Since mines may have been laid there, ----. kimsenin fark etmediği iki tür denizatının
yaşamakta olduğunu gösteriyor.
A) these fields had not been cultivated C) Đngiltere’nin çevresindeki sularda iki tür
B) large areas of valuable farmland are being denizatının yaşadığını ilk kez ortaya çıkaran on
overgrown by bamboo yıllık çalışma bu türlerin hiç fark edilmediğini
C) immense bamboo thickets would have come into belirtiyor.
being D) On yıllık çalışmanın sonuçlarına göre Đngiltere’nin
D) the costs of restoring farmland to full production çevresindeki sularda hiç kimsenin bilmediği iki tür
have always been high denizatının yaşamakta olduğu bildiriliyor.
E) faster techniques were called for E) Đki tür denizatı hiç kimseye belli etmeden
Đngiltere’nin çevresindeki sularda yaşarken on
yıllık bir çalışma onları ortaya çıkardı.
33. ---- that solar flares triggered geomagnetic
storms.
37. Though stars appear to the eye as single
A) Scientists used to think points of light, very many of them turn out to be
B) The findings were ambiguous double when seen through a telescope.
C) The astronauts were taken by surprise
D) The facts have to be suppressed A) Teleskopla bakıldığında çift ışık olan yıldızların
E) The results of the experiment surprised everyone çoğu göze tek ışık noktası gibi görünür.
B) Çıplak gözle bakıldığında yıldızlar tek ışık noktası
gibi görünür, çoğuna sadece teleskopla
34. ----, though none are yet being grown on a bakıldığında çift olduğu anlaşılabilir.
commercial scale. C) Göze tek ışık noktası gibi görünen yıldızlara
teleskopla bakıldığında çoğunun çift olduğu
A) Conifers are dominant trees in northern latitudes gözlenir.
B) Trees produce copious pollen, which travels up to D) Yıldızlar çıplak gözle bakıldığında tek ışık noktası
16 kilometres gibi algılansa da teleskopla bakıldığında aslında
C) Genetically modified trees have not been planted çift olduğu görülür.
in natural woods and forests E) Her ne kadar yıldızlar göze tek ışık noktası gibi
D) Plants demonstrate a remarkable diversity in görünse de pek çoğunun teleskopla bakıldığında
size, habit and form çift olduğu ortaya çıkar.
E) Genetically modified trees are being developed
for a variety of uses

3
38. Entropy is a physical property like temperature 40. Aşağı yukarı aynı büyüklükte ve Güneş’e aynı
and pressure, and measures how close a system uzaklıkta olan Dünya ve Venüs sıklıkla ikiz
has come to reaching stagnant “equilibrium”. gezegenler olarak nitelendirilir.

A) Sıcaklık ve basınç gibi fiziksel bir özellik olan A) It is usual to refer to Earth and Venus as twin
entropi bir sistemin durgun “denge”ye ulaşıp planets as they are almost the same size and
ulaşmadığını ölçer. distance from the Sun.
B) Entropi sıcaklık ve basınç gibi fiziksel bir özelliktir B) Since Earth and Venus are roughly the same size
ve bir sistemin durgun “denge”ye ne kadar and distance from the Sun it is only natural that
ulaştığını ölçer. they should be regarded as twin planets.
C) Bir sistemin durgun “denge”ye ulaşıp ulaşmadığı C) Earth and Venus, being roughly the same size
sıcaklık ve basınç gibi fiziksel bir özellik olan and distance from the Sun, are often regarded as
entropi ile ölçülür. twin planets.
D) Entropi hem sıcaklık ve basınç gibi fiziksel D) It is because they are exactly the same size and
özellikler arasındadır hem de bir sistemin durgun distance from the Sun that Earth and Venus are
“denge”ye ne kadar ulaştığını belirlemede known as the twin planets.
yardımcıdır. E) By the twin planets we mean Earth and Venus
E) Sistemlerin durgun” denge”ye ulaşıp which are nearly the same size and are
ulaşmadığını ölçen entropinin sıcaklık ve equidistant from the Sun.
basınçla ortak olan yanı, fiziksel bir özellik
olmasıdır.
41. Dünyanın iç kısmını araştırma konusunda
uzmanlaşan bilim adamları, uzun süre, Dünya’nın
39. – 41. sorularda, verilen Türkçe cümleye derin iç kısmındaki hareketin yüzeydeki dikey
anlamca en yakın Đngilizce cümleyi bulunuz. değişimlerin arkasında olduğundan şüphelendiler.

39. Bilim adamları bir büyük depremin, bir A) Scientists who specialize in studying Earth’s
sonrakinin zamanı ve yeri üzerinde kayda değer interior have long suspected that activity deep
bir etkisinin olmadığını düşünürlerdi; fakat son inside Earth is behind vertical changes at the
araştırmalar durumun böyle olmayabileceğine surface.
işaret ediyor. B) Scientists studying Earth’s interior have come to
the conclusion that what goes on deep inside
A) Scientists used to think that there was no Earth affects vertical changes at the surface.
connection between one major earthquake and C) It is generally agreed by scientists studying
subsequent smaller ones, but new studies Earth’s interior that events deep inside Earth’s
suggest there may be. core influence vertical changes at the surface.
B) Formerly scientists thought that one large D) Scientists specializing in the study of Earth’s
earthquake could not possibly affect the timing or interior have long been aware of the fact that
location of the next, but recent research suggests activity deep inside Earth may be responsible for
it may. vertical change at its surface.
C) Contrary to what scientists used to think, recent E) Scientists specializing in happenings deep within
research suggests that a major earthquake may Earth’s core have, for a long time, suspected a
considerably affect the timing and location of relationship between them and vertical changes
subsequent earthquakes. at the surface.
D) Recent research suggests that a major
earthquake may influence the time and place of
subsequent earthquakes though this has not
been the traditional view of scientists.
E) Scientists used to think that one large earthquake
had no notable influence on the timing or location
of the next one but recent research suggests this
may not be the case.

4
42. – 46. sorularda, boş bırakılan yere, parçanın 45. Isaac Newton presented the earliest scientific
anlam bütünlüğünü sağlamak için getirilebilecek definition of mass in 1687 in his landmark work
cümleyi bulunuz. Principium: “The quantity of matter is the
measure of the same arising from its density and
42. Fragile ecosystems like the Arctic could face bulk conjointly.” That very basic definition was
many more years of contamination from PCBs good enough for Newton and other scientists for
(polychlorinated biphenyls), despite international more than 200 years. ----. In recent years,
treaties banning their use. ----. An estimated 1.3 however, the why of mass has become a research
million tonnes of PCBs were made between the topic in physics.
1930s and 1990s around the world for use in the
manufacture of pesticides, lubricants, and A) The laws of gravity predict that gravity acts on
plastics. But an investigation to determine the fate mass and energy
of these PCBs has failed to locate most of them. B) Most people think they know what mass is, but
actually they understand only a very small part of
A) On the other hand, PCBs may be carried by wind what it entails
to cold countries where they condense out in the C) Fundamental particles have an intrinsic mass
cold air known as their rest mass
B) Indeed, soils in temperate lands have captured D) Energy and mass are related, as described by
most of the PCBs so far released into the Einstein’s famous equation, E=mc2
environment E) They understood that science should proceed
C) On the contrary, PCBs could pose a threat to first by describing how things work and later by
polar bears for years to come understanding why
D) That is the conclusion of a study into the fate of
PCBs manufactured worldwide during much of
the 20th century 46. Can coal ever become a friend of the
E) As a result, urban air contains more PCBs than environment? Coal-fired power stations supply
rural air half the electricity used in many industrial
countries. ----. This, of course, is the most
worrisome of the so-called “greenhouse gases.”
43. The Kavli Foundation’s approach differs from
the increasingly utilitarian focus of most funded A) New ones will have to comply with the Clean Air
research. ----. Kavli opposes this practice for he Act
believes you have to be willing to fund science B) They are, however, responsible for 80% of the
without knowledge of the benefits. power industry’s emissions of carbon dioxide
C) Energy engineers are already talking about
A) To obtain funding from any source, scientists “clean coal” technology
must usually frame their ideas in the context of D) Clean coal means different things to different
studies already completed and short-term impact people
B) Knowledge about materials and processes in the E) Coal treatment and refining processes are rightly
universe could open up benefits that we can’t getting a lot of attention as well
even imagine
C) In fact, the foundation pays for nondirected
research in its three main areas of interest: 47. – 51. sorularda, karşılıklı konuşmanın boş
astrophysics, nanoscience and neuroscience bırakılan kısmını tamamlayabilecek ifadeyi
D) It is unrealistic of such agencies to expect these bulunuz.
programmes to deliver useful tools and
applications rapidly 47. Roy: - If you haven’t already read this account
E) The foundation has chosen disciplines that are of Philip Morrison, make sure you do.
already acknowledged as “growth” areas in Michael: - ----
science Roy: - That’s what impressed me most. He made
important contributions in quantum
electrodynamics among other things, and then
44. Traditionally, the study of planet formation has gave courses on physics for poets!
proved frustrating, as astronomers have never
been sure whether their theories apply to other A) Yes, I will. I’ve seen him on TV on several
planetary systems. ----. Now, however, the occasions; he’s both charming and amusing.
observations of debris discs around stars of B) Yes, I intend to. What was it that impressed you?
different masses and ages are helping to place C) I’ve already done so. The range of his interests
our solar system in context. and activities is amazing.
D) Did you realize he was an assembler of the first
A) Some discs look like gigantic versions of the rings atomic bomb?
of Saturn E) Of course. I was his student, you know, at Cornell
B) Most of the discs, however, could not be seen University.
directly
C) What the recent images show is wonderfully
unexpected
D) This is because the solar system is the only
known example of a planetary system
E) The dust particles probably result from collisions
among asteroids

5
48. Gary: - Is this the website you like best when it 51. Larry: - They’re holding a young designers’
comes to scientific news? competition for designing a robot to put out a
Philip: - Yes, I suppose it is. It’s updated weekly house fire.
and well-linked to related websites. Tony: - I think you mean to blow out a candle!
Gary: ---- Larry: - ----
Philip: - That’s hard to say. So many scientific Tony: - But you are right. The final aim is, of
“facts” are being questioned these days. course, to put out house fires.

A) Is it university-owned? A) Well, at this stage, that’s all they’re asking for.


B) Does it keep up with recent developments? B) Do you think they ever will?
C) What’s the level? College stuff? C) That shouldn’t be too difficult. The real problem is
D) From a scientific point of view, how reliable is it? to locate the fire.
E) Does it cover all the sciences? D) If it could set off an alarm even, that would be
useful, wouldn’t it?
E) Once a fire takes hold it becomes a major
49. Brian: - Have you read this book, Water problem.
Follies?
Peter: - No I haven’t; but I’ve heard a lot about it. It
focuses on how much water is being wasted, 52. – 56. sorularda, cümleler sırasıyla
doesn’t it? okunduğunda parçanın anlam bütünlüğünü bozan
Brian: - ---- cümleyi bulunuz.
Peter: - Good! It’s time someone took a firm stand
against the waste. 52. (I) Computer researchers predict that quantum
computers will become a reality within 10 to 15 years.
A) That’s right. And it’s pretty critical of man for (II) However, these machines pose a security threat,
being so unconcerned about this waste. because their ability to perform many calculations at
B) No. It actually concentrates on ground water. once means they will be able to uncover the
C) Yes. Most people seem to think ground water is encryption keys that are, for practical purposes,
boundless. untraceable by today’s “classical” conventional
D) And the gold-mining industry is attacked for its computers. (III) If that happens, people will be able to
vast “dewatering” operations. tap into cell phone calls. (IV) A quantum computer
E) And the consequences include dry rivers and can represent a 0 and a 1 at the same time in a
land subsidence. quantum bit (called a qubit). (V) Furthermore, secure
e-commerce will be a thing of the past.
50. Sam: - Do you think NASA’s emergency A) I B) II C) III D) IV E) V
escape plan for space-bound astronauts will
work?
Robert: - That’s hard to say. I suppose really it will 53. (I) Everyone knew that freezing rain could be bad,
depend on the kind of emergency that presents but what the researchers learned was frightening. (II)
itself. All aircraft designers are familiar with the challenge of
Sam: - ---- icing. (III) Manufacturers must demonstrate that their
Robert: - Actually, the colour is a survival feature aircraft are capable of flying safely in cold, wet
too. It makes a search for the crew easier. conditions where they might ice up. (IV) In the US,
those conditions are specified by the Federal Aviation
A) It reminds one of science-fiction films, with Administration’s (FAA) “Appendix C”. (V) This
everything neatly planned. appendix clearly states the kinds of cold weather
B) It seems a bit like a game to me. The suits are a which an aircraft must be able to deal with.
brilliant orange colour.
C) The antigravity suit squeezes the legs to prevent A) I B) II C) III D) IV E) V
blood from pooling in them.
D) Apparently shuttle bailout is a last resort, to be
used only if landing becomes impossible. 54. (I) Mother Columbian rainbow boas, Epicrates
E) Let’s hope the Challenger catastrophe is not cenchria maurus, have the strange habit of eating
repeated. some of their own young. (II) Now new research
reveals why. (III) Within two weeks, these mothers
regained their lost muscle. (IV) Stillborns and
undeveloped eggs in a clutch, which the mother
consumes soon after laying or giving birth, are rich in
energy and contain a diversity of proteins and
essential nutrients. (V) Therefore, maternal
cannibalism leads to a quick recovery after giving
birth.

A) I B) II C) III D) IV E) V

6
55. (I) Nyos is a crater lake formed by a volcanic 58. It is pointed out in the passage that the
eruption roughly five centuries ago. (II) In these lakes diversity of life on Earth ----.
the gas saturates the bottom water. (III) It is one of
many such lakes, found the world over in volcanic A) is far more extensive in temperate climates than
chains (IV) It is, however, one of only two lakes in colder ones
known to have exploded. (V) And when it exploded a B) results from the variety of ways whereby
jet of gas-laden water rose 80 meters high and carbon organisms meet environmental challenges
dioxide filled the air. C) is related to plants rather than other organisms
D) becomes far more apparent in spring than in
A) I B) II C) III D) IV E) V winter
E) must be maintained through the conservation of
the environment
56. (I) Most of the military robotic systems currently
operating are airborne. (II) This is because robots for
use in a ground war present serious problems as 59. One can understand from the passage that, for
conditions are more complex and less predictable. deciduous trees, the shedding of leaves ----.
(III) The Army’s first ground robots were devoted to
mine-sweeping. (IV) For instance, what works in a A) increases the amount of water loss, which is a
desert will be unsuited to jungles. (V) Similarly, a serious environmental challenge
machine designed for use in cities is unlikely to adapt B) increases their metabolic activity throughout
to mountains. winter
C) is an effective mechanism of resistance to heat
A) I B) II C) III D) IV E) V D) is a regular metabolic activity which is not related
to environmental conditions
E) is a kind of hibernation that enables them to
57. – 60. soruları aşağıdaki parçaya göre survive the cold winter months
cevaplayınız.

How have terrestrial organisms met the environmental 60. It is clear from the passage that, for some
challenges of living on land? Life began in the animals, migration ----.
oceans, but many life forms have since adapted to
terrestrial life in a sea of air. Every single organism A) and hibernation are equally viable options
living on land has to meet the same environmental B) is comparatively easy
challenges: obtaining enough water; preventing C) is indispensable for survival
excessive water loss; getting enough energy; and in D) causes a great deal of energy loss
Polar Regions, tolerating widely varying temperature E) involves various environmental challenges
extremes. How those challenges are met varies from
one organism to another, and in large part explains
the diversity of life encountered on land today. Some
animals avoid colder temperatures by migrating to
warmer climates for the winter, whereas others avoid
the cold by passing the winter in a dormant state
called hibernation. Many plants also spend winter in a
dormant state. The aerial parts of some plants die
during the winter, but the underground parts remain
alive; the following spring they resume metabolic
activity and develop new aerial shoots. Many trees
are deciduous; that is, they shed their leaves for the
duration of their dormancy. Shedding leaves is
actually an adaptation to the “dryness” of winter.
Roots cannot absorb water from ground that is cold or
frozen; by shedding its leaves the plant reduces water
loss during the cold winter months when obtaining
water from the soil is impossible.

57. It is pointed out in the passage that all


terrestrial organisms ----.

A) in warm regions find it very hard to tolerate


extreme temperatures
B) in polar regions live out the winter through
hibernation
C) face the danger of extinction due to
environmental challenges
D) have, one way or another, adapted themselves to
environmental conditions
E) are most adversely affected by excessive water
loss and cold temperatures

7
61. – 64. soruları aşağıdaki parçaya göre 63. We understand from the passage that the
cevaplayınız. Dutch research centre, the TNO, ----.

Henrik-Jan van Veen has carried out a great deal of A) works in very close association with the Dutch
research into spinning. This is especially true for armed forces
“graveyard spins”, the term for what happens when B) oversees all major research projects being
fighter pilots get so disoriented they miscalculate how carried out in the country
to get their plane back on course. They can end up in C) has put a lot of pressure on van Veen to extend
a dangerous and often fatal spin. Van Veen works at the uses of vibrotactile devices
a research lab run by the Netherlands Organization D) encourages the application of research for the
for Applied Scientific Research, the TNO. The range solving of problems
of research covered by the TNO is vast, and it sees E) puts safety devices high on its list of priorities
itself as a practical problem solver. And for the Dutch
air force, the graveyard spin is certainly a problem
that needs solving. Van Veen’s specialty is 64. It is clear from the passage that van Veen’s
“vibrotactile devices”, which use vibrations to convey immediate purpose in developing the vibrotactile
information. His latest project is a vest studded all vest is to ----.
over with small discs that can each vibrate
independently. In a test room, a pilot is strapped into A) help fighter pilots to calculate their course more
a seat in a “cockpit”. At the push of a button, the lights accurately
go out and the chair starts spinning. After a while the B) warn pilots in advance that a spin is building up
chair is stopped. “He’ll think he’s spinning the other C) help the blind find their way about
way now”, says van Veen. The pilot is told to correct D) make it possible to reduce the amount of fuel
the spin, but instead, he overcorrects massively, and used by aircraft
the chair begins spinning again. In the next test, the E) enable fighter pilots to get over the effects of a
pilot dons van Veen’s vest and is told that the patch of spin and thus, avoid a crash
the vest that is vibrating will indicate the direction he
should force the joystick to correct a spin. This time,
when the chair stops spinning the pilot manages to
keep the seat still. Van Veen thinks the vibrotactile
vest could do more than save the lives of fighter
pilots. He’s now working on linking the system to a
GPS receiver so that tourists in a foreign city or blind
people in an unfamiliar environment can use the vest
to find their way around.

61. It is clear from the passage that the


vibrotactile vest ----.

A) has contributed significantly to environmental


research
B) could be developed to serve a variety of
purposes
C) has been in use in military aviation for many
decades
D) is the product of an expensive research project
undertaken by the Dutch military
E) could be used to transmit secret military
information

62. It is understood from the passage that,


because spins in fighter flights can be fatal, ----.

A) they are referred to as “graveyard spins”


B) van Veen has been testing a number of devices
that could prevent spins
C) the TNO is making spin-prevention devices a
major research project
D) many aircraft have been indefinitely grounded
E) all pilots are required to wear a vibrotactile vest

8
65. – 68. soruları aşağıdaki parçaya göre 68. It is clear from the passage that, in the past,
cevaplayınız. the church ----.

For almost 200 years, the idea of cosmic events A) regarded global catastrophes as acts of God
affecting life on Earth was viewed as heretical by the B) was particularly interested in the movements of
church, which regarded catastrophe as proof of divine comets
intervention, and as nonsense by the scientific C) consistently banned any research into cosmic
establishment, which dismissed it as superstition. Yet events
in the end, the sheer weight of evidence has swept D) encouraged scientists to find ways of preventing
away all doubt about the reality of global global catastrophes
catastrophes. Attempts to make scientific sense of the E) wished to suppress all thoughts of cosmic events
many legends of global catastrophes date back to the
dawn of modern science itself, in the 17th century.
Following the publication of Newton’s laws of motion
and universal gravitation in 1687, Edmond Halley
decided to apply them to the mystery of comets. By
studying records of their appearance, Halley argued
that the bright comets of 1456, 1531, 1607 and 1682
were in fact one comet, later known as the “Halley”
comet, that followed a vast elliptical orbit around the
Sun in agreement with Newton’s laws. But Halley
noted something else as well: a comet crossing the
orbit of the Earth might one day collide with us with
devastating consequences.

65. According to the passage, Newton’s laws of


motion and gravitation ----.

A) had no impact whatsoever on the rise of modern


science
B) were approached sceptically by Halley and other
contemporary scientists
C) helped Halley to identify the comet that bears his
name
D) convinced Halley that catastrophes were in fact
acts of divine intervention
E) were dismissed right away by the scientific
establishment of his time

66. It is pointed out in the passage that, prior to


the rise of modern science in the 17th century,----.

A) various studies had been made of comets, but


Halley disregarded them all
B) the way people viewed cosmic events varied
greatly
C) all kinds of learning had been subject to the
approval of the church
D) the Earth had experienced several collisions with
cosmic objects
E) attempts had been made to explain certain
catastrophes with reference to gravitational laws

67. We learn from the passage that in the opinion


of Halley, ----.

A) the mystery surrounding comets could never be


cleared up
B) Newton’s laws of motion and gravitation needed
to be further clarified and elaborated
C) the earlier appearances of the comet “Halley” had
not been properly recorded
D) global catastrophes could be prevented through
new scientific developments
E) a comet may, at some point in the future, strike
Earth

9
69. – 72. soruları aşağıdaki parçaya göre 72. The passage suggests that the inhabitants of
cevaplayınız. small isolated islands ----.

Except perhaps for some remote island dwellers, A) will be adversely affected if the oceans continue
most people have a natural tendency to view to be polluted
continents as fundamental, permanent and even B) have frequently chosen to live in comparative
characteristic features of Earth. One easily forgets isolation in preference to living in a crowded city
that the world’s continental platforms amount only to C) always demonstrate a keen interest in the solar
scattered and isolated masses on a planet that is system
largely covered by water. But when viewed from D) depend for their living more on the sea than on
space, the correct picture of Earth becomes the land
immediately clear. It is a blue planet. From this E) probably have a better perception of the reality of
perspective it seems quite extraordinary that over its Earth than the majority of us
long history, Earth could manage to hold a small
fraction of its surface always above the sea —
enabling, among other things, human evolution to
proceed on dry land. Is the persistence of high-
standing continents just an accident? How did Earth’s
complicated crust come into existence? Has it been
there all the time, like some primeval icing on a
planetary cake, or has it evolved through the ages?
Such questions engendered debates that divided
scientists for many decades, but the fascinating story
of how the terrestrial surface came to take its present
form is now partly resolved. That understanding
shows, remarkably enough, that the conditions
required to form the continents of Earth may be
unmatched in the rest of the solar system.

69. One important point made in the passage is


that ----.

A) new questions concerning the solar system are


constantly coming to the fore
B) scientists have finally been able to understand
fully the mystery of Earth’s crust
C) people living on remote islands are so cut off
from the rest of the world that they have no idea
about what is happening elsewhere
D) the formation of the continents of Earth may have
no parallel elsewhere in the solar system
E) the growing pollution of the oceans is causing a
great deal of concern among scientists

70. According to the passage, the question of how


Earth’s continents came into being ----.

A) has never attracted much attention


B) has been one of the concerns of space research
and exploration
C) can best be answered through a comprehensive
study of the other planets in the solar system
D) is not likely to be resolved in the near future
E) gave rise to considerable disagreement among
scientists

71. The passage calls Earth the “blue planet” to


underline the fact that ----.

A) the waters of the oceans are crystal clear


B) the geographical features of Earth are not very
distinct when viewed from space
C) many things on Earth are blue
D) there is actually very little land on Earth
E) it is man’s duty to keep the seas clean

10
73. – 76. soruları aşağıdaki parçaya göre 76. It is clear from the passage that there ----.
cevaplayınız.
A) is an ongoing debate on the uses of ballistic
In his preface to Spaceflight Revolution, David missiles
Ashford recalls how he started his research into B) is some discrepancy between Ashford’s words
rocket motors. As he later explains, these were and his actions
motors that would power a space plane — one that C) is much public support for Ashford’s project
would launch space travelers and satellites cheaply D) are many technological differences between
and reliably into orbit. That was 1961. Ashford admits Ashford’s rocket motors and the conventional
he would probably have taken another job if he’d rockets currently in use
known that, 42 years later, satellites would still be E) is much concern among space scientists,
launched by rockets descended from ballistic missiles. including Ashford, about the ever-growing costs
The technology is there, but political and budgetary of the space programme in general and of shuttle
decisions have so far stopped space planes getting flights in particular
off the ground. But Ashford presents a compelling
argument that a small orbital space plane would cost
relatively little to design and develop — the equivalent
of just two shuttle flights.

73. As we understand from the passage,


Ashford’s space plane project ----.

A) has made space travel extremely cheap and


reliable
B) has been welcomed by political authorities and
received much attention
C) has received no political or financial support
since the early 1960s
D) has been proved faulty in the course of several
trials
E) was originally inspired by ballistic missile
technology

74. According to the passage, Ashford ----.

A) has not yet completed his research into rocket


motors
B) feels that his decades-long work on rocket motors
has been unjustly ignored
C) has written his book Spaceflight Revolution
mainly to criticize politicians
D) has been recognized as a leading scientist in
space research and rocket technology for quite
some time now
E) has proposed a project which can only be
realized if a sizeable budget is available

75. As can be understood from the passage,


Ashford is firmly convinced that ----.

A) the development of a space plane would cost no


more than two shuttle flights
B) politicians and financial authorities need to be
careful about investing money in space projects
C) rockets bear no relation to ballistic missiles
D) there are many people eager to be space
travellers and willing and able to pay a
reasonable fare
E) his work on rocket motors has greatly contributed
to space research

11
77. – 80. soruları aşağıdaki parçaya göre TEST BĐTTĐ.
cevaplayınız. CEVAPLARINIZI KONTROL EDĐNĐZ.

The Wireless Museum has several of the earliest


crystal wireless sets from the 1920s which ran on
electromagnetic waves with no external power
source, and were easily made at home. Valve radios,
which came along in the 1930s, needed electricity to
heat up the valves and the museum has both mains
and battery-powered valve radios on display. The
collection also has some rare wartime civilian
receivers — the only type of valve radio manufactured
during the Second World War. This was by order of
the government, because at this time most
manufacturing was focused on the war effort. There
are also plenty of modern day transistor radios
including a collection of novelty radios dating from the
sixties and seventies.

77. It is pointed out in the passage that, during


World War II, ----.

A) transistor radios began to replace traditional


valve radios
B) the production of wireless sets was almost
entirely for military purposes
C) various types of radios requiring no external
power source were developed
D) the government banned all kinds of civilian
receivers
E) the efficiency of valve radios was upgraded
through the introduction of new designs

78. We learn from the passage that the very early


crystal wireless sets ----.

A) were the models out of which transistor radios


were later developed
B) were manufactured in large quantities before the
arrival of valve radios
C) are among the museum’s most precious exhibits
D) were still in use during World War II, though in
limited numbers
E) were unconnected to an outside power system

79. It is clear from the passage that valve radios ---


-.

A) were still in widespread use in the sixties and


seventies
B) originally operated on electromagnetic waves
C) consumed more electricity than one might expect
D) are of two types: mains and battery-powered
E) were costly products and the government
disapproved of them

80. This passage is concerned with ----.

A) the exhibits of a wireless museum which cover a


considerable variety
B) the government’s war efforts and production
policies regarding radios
C) the technical features of transistor radios
D) the way a valve radio works
E) why the wireless museum was originally set up

12
1. – 18. sorularda, cümlede boş bırakılan yerlere 8. If things ---- according to plan, the book ---- by
uygun düşen sözcük ya da ifadeyi bulunuz. this time next year.

1. The Maastricht Treaty, which laid down the A) go / will have been published
rules for euro membership, says that B) have gone / would have been published
governments may not have budget ---- of more C) went / was to be published
than 3% of their GDP. D) were going / has been published
E) will go / will be published
A) deficits B) restrictions C) rates
D) allocations E) assets
9. Included with the account of his journey down
the Amazon there ---- engaging stories ---- by the
2. The European Commission is opposed to any unusual people he meets.
--- change in current banking practices.
A) have been / recounting
A) reluctant B) emotional C) relentless B) were / being recounted
D) crucial E) resentful C) are / recounted
D) will be / to have been recounted
E) would be / to be recounted
3. For the information systems to work properly,
you need to ---- the technology to suit the
situation. 10. The US presidential election of 1800 ----
notorious on account of the unforeseen
A) convince B) adjust C) reduce constitutional problems it ----.
D) sustain E) explain
A) is / has presented
B) has been / presented
4. Though it is fashionable to denounce negative C) would be / presents
campaigning, every political expert knows it can D) had been / would present
be ---- effective. E) was / presented

A) defiantly B) suitably C) extremely


D) sensitively E) lately 11. Having found the appropriate archives, it is
now possible ---- with some degree of certainty
what really ----.
5. There were plenty of people willing to ---- the
experiment as the subject, “controlling emotions”, A) reconstructing / happens
attracted them. B) to reconstruct / happened
C) to have reconstructed / has happened
A) fill out B) take part in C) watch out for D) having reconstructed / had happened
D) make do with E) open up E) to be reconstructed / was happening

6. Finally, the commissioners settled on a short, 12. Moreover, ---- its own statutes, the EU itself is
simple, constitutional amendment granting obliged to consult the trade unions ---- a number
Congress the authority to ---- guidelines for of topics.
selecting temporary members in an emergency.
A) under / on B) with / at C) from / for
A) hold up B) bring down C) call out D) on / against E) to / over
D) serve up E) set up
13. After the collapse of the Soviet Union and the
7. Some Italian film-makers believe that once opening of formerly closed borders, traders ----
Americans ---- making films in Rome again, local Georgia and Armenia, crossed over the borders ---
films ---- as well. Turkey to make a living.

A) would start / had flourished A) at / with B) of / about


B) have started / will have flourished C) from / to D) between / back
C) will start / are flourishing E) on / towards
D) start / will flourish
E) started / would flourish
14. They are installing a great deal of information
technology, ---- the staff are quite incapable of
using it.

A) owing to B) whenever C) as if
D) even so E) even though

13
15. She can have the job ---- she is willing to work 24. – 35. sorularda, verilen cümleyi uygun şekilde
on Saturdays. tamamlayan ifadeyi bulunuz.

A) apart from B) owing to C) except 24. Until fairly recently, management could have
D) provided E) whereas bought peace with generous increases in pay, ----.

A) but that is no longer an easy option


16. Middle children are obviously affected by the B) since that was not to be recommended
fact that they never have their parents’ attention C) however ambiguous this had seemed
all to ----. D) as long as the profits would have remained
steady
A) themselves B) each C) itself E) unless fringe benefits were reduced
D) one another E) each other

25. The most stunning recent museums, ----, are


17. Today virtually all country and suburban art objects themselves.
weeklies and small dailies are produced by offset
lithography, a procedure ---- photographs can be A) why they were designed by a Japanese architect
reproduced inexpensively. B) of which the one in Bilbao must be the finest
C) as the Pritzker is architecture’s biggest prize
A) why B) that C) which D) since there were smooth glass and aluminium
D) whether E) by which panels
E) since space and proportion get flawless
Treatment
18. The belief that anyone can aspire to anything
is ---- America’s greatest gifts to the world.
26. Why should astronauts be sent into space ----?
A) as much as B) some of C) one of
D) another E) the other A) so that life on earth will have been improved
B) but the costs involved were especially horrific
C) even if it were no longer dangerous
19. – 23. sorularda, aşağıdaki parçada D) as if no harm would have come of it
numaralanmış yerlere uygun düşen sözcük ya da E) if there is nothing meaningful for them to do there
ifadeyi bulunuz.

The poet Robert Browning was lucky in his parents. 27. No one event, ----, can reorder politics in a
His father, who (19) ---- in a bank, was a man (20) ---- country as populous as the US.
genuine intelligence, who owned a library of six
thousand volumes and seems (21) ---- them all. His A) as if there had never been a cold war
mother, (22) ---- was a devoutly religious woman, B) whichever appears the more dangerous
gave Browning a tenderness and optimism he would C) however shocking it may be
need (23) ---- in life. D) even if the assassination attempt had succeeded
E) since the impact was felt in far-off places
19.
A) having been employed B) has been employed
C) has employed D) was employed 28. There is a museum in the centre of Bristol ----.
E) will be employed
A) as one wants to spend a pleasant morning there
B) that there are over 150 animal species
20. C) as some of the strangest animals and plants on
A) from B) about C) for earth are to be found there
D) in E) of D) so one could learn more about the natural world
E) which is home to a living rainforest

21.
A) reading B) to read C) to have read 29. ---- who built the world’s first business
D) to be reading E) have read computer.

A) Everyone knows
22. B) It wasn’t to be expected
A) which B) who C) whose C) It came as a surprise
D) that E) whom D) The question was unreasonable
E) We took it for granted

23.
A) never B) hardly C) scarcely
D) frequently E) fairly

14
30. Since he ignores certain fundamental facts 35. ---- before he became a film-maker.
about the past century, ----.
A) The director of the Titanic now plans to make a
A) there will be many different kinds of authoritarian film of the sinking of the Bismarck
leaders B) He is best known as the director of the film
B) the conclusions had been unreliable Titanic
C) good and evil alike have to be judged C) The director of the film Titanic was a scuba diver
D) he presents a distorted picture of reality and wreck diver
E) someone should, nevertheless, still be blamed for D) The film’s director has always been fascinated by
the unjustifiable deaths wrecks
E) He is interested in why the Titanic sank

31. Although conventional IQ tests are good


predictors of college grades, ----. 36. – 38. sorularda, verilen Đngilizce cümleye
anlamca en yakın Türkçe cümleyi bulunuz.
A) there are actually very few really exceptional
individuals 36. Charles Dickens is one of the few novelists
B) they are still the best single predictor of overall whose works did not become unpopular after his
success death.
C) they are less valid for predicting later job success
D) some weaker students have become charismatic A) Charles Dickens, ölümünden sonra eserleri
leaders popülerliğini kaybetmeyen birkaç romancıdan
E) brain damage does not necessarily impair certain biridir.
types of intelligence B) Charles Dickens, ölümünden sonra da
popülerliğini koruyan eserler vermiş bir
romancıdır.
32. Because unemployment is rising and wages C) Ünlü bir romancı olan Charles Dickens’in eserleri,
are falling, ----. ölümünden sonra bile popülerliğini
kaybetmemiştir.
A) the government would soon be out-of-favour D) Ölümünden sonra eserleri popülerliğini hep
B) Americans are naturally worried about the state koruyan birkaç romancıdan biri de Charles
of the economy Dickens’tır.
C) the state of panic was regarded as unfounded E) Eserleri, ölümünden sonra bile popülerliğini hiç
D) no one could lose on the stock market kaybetmeyen az sayıdaki romancıdan biri de
E) it would have been a good time to change your Charles Dickens’tır.
car

37. Gorbachev came to power in 1985 in the


33. ---- that it produces national cohesion in the former Soviet Union partly because he was
target countries. promising to make sweeping changes.
A) The single most important effect of terrorism is A) 1985’te eski Sovyetler Birliği’nde köklü
B) The bombing of resorts is a recent development değişiklikler yapmaya söz vermiş olan Gorbachev
C) In a globalized world small groups of people can iktidara getirilmiştir.
cause big trouble B) Eski Sovyetler Birliği’nde 1985’te Gorbachev’in
D) Terrorists are at an inherent disadvantage iktidara gelmesinin nedeni, beklenen köklü
E) Suicide-bomb attacks against ordinary citizens değişikliklerin bir kısmını yapmaya söz vermiş
followed olmasıdır.
C) Gorbachev çok kapsamlı değişiklikler yapmaya
söz verdiği için, Sovyetler Birliği’nde 1985’te
34. Whatever Luhrmann’s New York critics may iktidara getirilmiştir.
think of his new production of La Bohème, ----. D) Gorbachev, kısmen, geniş kapsamlı değişiklikler
yapmaya söz verdiği için, eski Sovyetler
A) people had waited in long queues for last-minute Birliği’nde 1985’te iktidara gelmiştir.
cancellations E) Gorbachev, eski Sovyetler Birliği’nde önemli
B) they claimed to have pursued creative freedom değişiklikler yapmaya söz vererek 1985’te iktidarı
C) the beautiful side of life will have been expressed ele geçirmiştir.
with beautiful music
D) he has certainly breathed new life into this opera
E) opera used to be characterized by unnatural,
alienating conventions

15
38. Many social psychologists have turned their 41. Romantik akımın öncülerinden biri olan Jean-
attention to promoting health practices such as Jacques Rousseau, duyguyu akıldan daha yüksek
avoiding the abuse of alcohol, tobacco and other bir düzeye koymuştur.
substances.
A) Jean-Jacques Rousseau, one of the leaders of
A) Günümüz sosyal psikologları aşırı alkol, tütün ve the Romantic Movement, regarded emotion as
diğer maddelerden kaçınma gibi sağlık more important than reason.
uygulamalarını ilerletmeye çabalıyor. B) Jean-Jacques Rousseau, who was one of the
B) Birçok sosyal psikolog ilgisini aşırı alkol, tütün ve forerunners of the Romantic Movement, put
diğer maddelerden kaçınma gibi sağlık emotion at a higher level than reason.
uygulamalarını ilerletmeye yöneltti. C) Jean-Jacques Rousseau was a forerunner of the
C) Günümüzde, sosyal psikologların çoğu alkol, Romantic Movement and so he rated emotion
tütün ve diğer maddelerden kaçınma gibi sağlık higher than reason.
uygulamalarını ilerletmeyle ilgilenmeye D) Another forerunner of the Romantic Movement
başladılar. was Jean-Jacques Rousseau, and he rated
D) Aşırı alkol, tütün ve diğer maddelerden kaçınma emotion higher than reason.
gibi sağlık uygulamalarını ilerletmeyle ilgilenen E) Since he regarded emotion as higher than reason
birçok sosyal psikolog var. Jean-Jacques Rousseau is recognized as a
E) Aşırı alkol, tütün ve diğer maddelerden kaçınma forerunner of the Romantic Movement.
gibi sağlık uygulamalarını ilerletmekle ilgilenen
sosyal psikolog sayısı artıyor.
42. – 46. sorularda, boş bırakılan yere, parçanın
anlam bütünlüğünü sağlamak için getirilebilecek
39. – 41. sorularda, verilen Türkçe cümleye cümleyi bulunuz.
anlamca en yakın Đngilizce cümleyi bulunuz.
42. There is something vital, even disturbing;
39. Değişik sektörlerden büyük Avrupa ve Kore about the buildings he has designed. His serene
şirketleri Brezilya’nın daha yoksul olan kuzeydoğu spaces are charged with emotion. ----. And for all
bölgesine yatırım yapmaya karar verdi. its subtlety and silence, his architecture is
dynamic in unaccountable ways – hard as granite,
A) The poorer north eastern region of Brazil is fluid as water and airy as sunlight.
attracting the attention of large European and
Korean companies of various sectors. A) Here there is a shift to a grander and more
B) Some of the large European and Korean transparent space
companies from various sectors are eager to B) All detractors can say is that he is too 20th century
invest in the poor north eastern region of Brazil. C) They blanket your mood the way a great abstract
C) Large European and Korean companies from painting does
various sectors have decided to invest in the D) Above the main gallery, the colours are brighter
poorer north eastern region of Brazil. E) You employ stone, wood and concrete, and with
D) The poor north eastern area of Brazil is attracting these materials you build houses and palaces
investors from various large companies in Europe
and Korea.
E) The poor north eastern part of Brazil would 43. New and exciting links are fast being
benefit from investments made by European and developed between the cities of Europe. In
Korean large companies involved in a variety of France, high-speed trains are providing the links.
fields. ----. These breathe life into regional towns which
have now, for the first time, become accessible in
terms of time and money to millions of other
40. Eğer Asya’daki diğer dev ekonomilerle Europeans.
karşılaştırırsak, Çin’in büyüme hızı istisnai
değildir. A) The river-bank highways south of the river Seine
have been converted into a giant beach
A) Even compared with other giant economies in B) As in the Renaissance-era urban boom, one of
Asia, China’s growth rate hasn’t been the driving forces behind this flowering is art
exceptional. C) The new energy is not confined to Europe’s
B) When we compare China’s growth rate with other capitals
giant economies in Asia, it’s obvious that her D) More far-reaching is the extraordinary network of
economic growth isn’t exceptional. low-cost airlines that have suddenly come into
C) Only when it is compared with that of other giant being
Asian economies does China’s economic growth E) These growing links will accelerate the
rate appear exceptional. disappearance of national boundaries
D) China’s economic growth rate remains
exceptional even when we compare it with that of
other powerful Asian economies.
E) China’s economic growth rate isn’t exceptional if
we compare it with other giant economies in Asia.

16
44. “If you want one year of prosperity, grow 47. – 51. sorularda, karşılıklı konuşmanın boş
grain. If you want ten years of prosperity, grow bırakılan kısmını tamamlayabilecek ifadeyi
trees. If you want 100 years of prosperity, grow bulunuz.
people.” ----. It sums up how the entry of China’s
massive labour force into the global economy is 47. Mary: - We ought to go and see this exhibition
bringing prosperity to the nation. of rugs from a village in southern Turkey.
Helen: - Yes; the reviews about them are
A) China certainly has a huge, cheap workforce astonishingly positive.
B) Indeed, China has effectively doubled the global Mary: - ----
labour force Helen: - In a way, that’s true. Certainly all the rugs
C) Actually, China’s growth rate is steadily have been made in the same village.
increasing
D) China is having a dramatic effect on the world A) I don’t really think the exhibition will be as good
economy as they say.
E) This is an old Chinese proverb B) It’s the colours that seem to impress most people,
and they all come from natural dyes.
C) One even says it’s as if there is a whole village of
45. Over the past decade almost everyone tuned artists.
into American popular culture has heard the term D) Many of the designs are traditional, but there are
emotional intelligence. ----. It has been the subject quite a lot of new designs.
of several books, including a best-seller, and of E) They should hold more exhibitions of this kind.
seminars for schools and organizations.

A) Since this is a new concept, it has attracted much 48. Barry: - Have you heard about the fish, black
attention bass, which are killing off Japan’s native fish?
B) What is your EQ? Reg: - No; tell me more.
C) Some of the controversy arises from the fact that Barry: - ----
popular and scientific definitions of emotional Reg: - I can believe it. There are many similar
intelligence differ sharply stories.
D) Despite these difficulties research on emotional
intelligence has continued A) During the 1970s, game fishing became popular
E) As early as the 1930s, psychometricians in Japan.
recognized the possibility that people might have B) They are still hoping to restore the ecological
a social intelligence order.
C) No one knows how they got into the moats of the
palace, but they are there now.
46. There are seven euro banknote D) Well, someone imported some in 1925, and they
denominations, which can be recognized easily by flourished and now they are everywhere.
their look and feel. ----. On the other, they feature E) A large majority were in favour of this large-scale
bridges, signifying co-operation between the effort to exterminate the species.
nations of Europe and the rest of the world.

A) It is generally agreed that the euro has proved 49. Madge: - I see ski-resort operators are growing
beneficial to businesses active in combating global warming.
B) The eight euro coins have a common side and a Derek: - And so they should be; shorter winters
national side and less snow will hit them hard.
C) Indeed, on January 1st 2002, twelve national Madge: - ----
currencies made way for just one Derek: - I suppose they didn’t want to give the
D) On one side, the bank notes show windows and impression that the sport is endangered.
gateways, symbolizing a spirit of openness
E) Euro banknotes and coins have had a profound A) Then why have they waited so long before taking
impact outside the participating countries any action?
B) But what can they do about it?
C) It seems they are using wind power to run the
lifts.
D) But they are not the only people to be adversely
affected by global warming!
E) The amount of snow we’ve had this winter
doesn’t suggest any global warming!

17
50. Karen: - The one-child-only ruling for couples 53. (I) Archaeologists have to bear in mind some
in China must have brought family life to an end. points when working with early historical
Jennifer: - ---- chronologies. (II) This system can be confirmed and
Karen: - Just think about it. There are no brothers refined using astronomy. (III) The chronological
and sisters, so there are no aunts and uncles, and system requires careful reconstruction, and any list of
no cousins. rulers or kings needs to be reasonably complete. (IV)
Jennifer: - Yes; you’re right. I hadn’t thought about The list, although it may reliably record the number of
the implications. years in each reign, has still to be linked with our own
calendar if it is not to remain merely a “floating
A) Why not? chronology”. (V) The artefacts, features, or structures
B) I don’t see why. to be dated at a particular site have somehow to be
C) And a good thing too. related to the historical chronology, perhaps by their
D) Was the population really growing very fast? association with an inscription referring to the ruler of
E) They can play with the neighbour’s children. the time.

A) I B) II C) III D) IV E) V
51. Fred: - Who translated this poem?
James: - I don’t know. It just appeared on my desk.
Why do you ask? 54. (I) Transport yourself back to the early 1960s
Fred: - ---- before the now-famous television series Star Trek first
James: - Then in that case, forget about it. We appeared. (II) At that time, only visionaries would
certainly won’t publish it. have dared imagine that people of the 23rd century
would be learning about the world on huge flat-panel
A) We include a poem most weeks so a lot of video screens and talking to one another across the
people send them to me. width of the planet using wireless devices. (III) Future
B) Because whoever’s done it has missed the point weapons will dispense entirely with the clumsy darts
of the poem completely. and wires. (IV) Yet today these scenes are
C) It’s not so much a translation as an adaptation. I commonplace. (V) So it is fitting that new high-tech
really like it. devices also have similarities with fictional technology
D) It’s not your translation then? as it was first presented on that famous science-
E) I don’t really know. I was interested, that’s all. fiction series.

A) I B) II C) III D) IV E) V
52. – 56. sorularda, cümleler sırasıyla
okunduğunda parçanın anlam bütünlüğünü bozan
cümleyi bulunuz. 55. (I) America, more than any other country, needs a
properly functioning airline system. (II) So it could be
52. (I) The Romantic Age is a term used to describe in the public interest to use taxpayers’ money to help
life and literature in England in the late eighteenth and make the airlines function properly again. (III) The
early nineteenth centuries. (II) Many of the most government had shut down American airspace for
important English writers of the period turned away four days at enormous cost to the carriers. (IV) But
from the values and ideas characteristic of the Age of such support cannot continue for ever. (V) In fact,
Reason toward what they perceived as a more daring, taxpayers are already beginning to complain.
individual and imaginative approach to both literature
and life. (III) In general, they placed the individual A) I B) II C) III D) IV E) V
rather than society, at the centre of their vision. (IV)
The Industrial Revolution helped make England
prosperous and powerful, but it involved exploitation 56. (I) Writing, at its best, is a lonely life. (II)
of the workers. (V) They tended to be optimists who Organizations for writers may ease the writer’s
believed in the possibility of progress and loneliness, but rarely help him to improve his writing.
improvement for humanity as well as for individuals. (III) Once a writer sheds his loneliness, he may grow
in public stature, but his work often deteriorates. (IV)
A) I B) II C) III D) IV E) V The writer should always try for something that has
never been done or that others have tried to do and
failed. (V) This is because he needs to do his work
alone, and if he is a good writer, he must face
eternity, or the lack of it, each day.

A) I B) II C) III D) IV E) V

18
57. – 60. soruları aşağıdaki parçaya göre 60. It is clear from the passage that the US
cevaplayınız. judiciary system ----.

The US Supreme Court is not a radical institution, nor A) reflects the opinions of the president
is it likely to become one as a result of any particular B) is well-protected against any kind of extremism
presidential election. The risks for the judiciary in C) consists of the Supreme Court and the various
presidential elections are a lot lower than many lower courts and all act independently of each
people imagine. This is not because there are no other
significant ideological or methodological differences D) takes its character, not from the Supreme Court,
among judges. Differences do exist, and they display but from the federal courts
party affiliation to some extent. And they matter – not E) faces pressure from many quarters
just on public issues such as abortion rights and racial
discrimination but also in those procedures that
actually guide the way lower courts handle a large
variety of legal cases. That said, the courts have
pretty strong institutional defences against radicalism
of any kind. For one thing, the judiciary’s power is
spread among more than 800 federal judges, no one
of whose views matter all that much in the broad
scheme of things. Even on the Supreme Court the
idiosyncrasies or ideological extremism of any one
judge can have only a limited effect. Without four
likeminded judges, his or her views are just noise.

57. It is pointed out in the passage that though


there are differences of opinion within the
judiciary, ----.

A) they play only a very small role in their


deliberations
B) they are far fewer than they used to be
C) these in no way concern party politics
D) it is almost impossible to avoid radicalism
E) this only becomes apparent at election times

58. According to the passage, the Supreme Court


of the US ----.

A) is feared by the lower courts


B) is a breeding ground for radicalism
C) is cut off from the lower courts of justice
D) avoids, as far as possible, public issues like
abortion
E) can only be slightly affected by a presidential
election

59. According to the passage, all rulings of the


Supreme Court ----.

A) are reconsidered after an election


B) can be influenced by the federal judges
C) must have had the support of at least five judges
D) aim at preventing ideological extremism
E) are, to a very large extent, influenced by party
affiliation

19
61. – 64. soruları aşağıdaki parçaya gore 64. It is clear from the passage that the writer ----.
cevaplayınız.
A) is urging schools to assign more homework to
American schools need more time if they are to teach students of all grades
efficiently. The school year is fixed at or below 180 B) is doubtful about the benefits of homework for
days in all but a handful of states – down from more lower-grade students
than 190 in the late nineteenth century, when C) believes that the school day should be extended
Saturday-morning sessions were common. The D) is opposed to reducing the school year from 190
instructional day is only about six hours, of which to 180 days
much is taken up with non-academic matters. In 1994, E) is convinced of the need for more electives
a national commission calculated that in four years of including physical education
high school a typical American student puts in less
than half as much time on academic subjects as do
students in Japan, France and Germany. Extending
the school day or the school year can get expensive
and complicated, and reducing non-academic
electives and physical education brings complaints
from parents and students alike. But there is one quite
cheap and uncomplicated way to increase study time:
add more homework. You may not be surprised to
learn that homework raises student achievement, at
least in the higher grades. For young children
homework appears not to be particularly helpful. Even
among older students it is hard to be sure of the
extent to which more homework may lead to higher
achievement.

61. We understand from the passage that school


programmes in America ----.

A) are of little concern to the parents, and so they


tend to ignore them
B) are at present being reviewed by a national
commission
C) do not put much emphasis on academic learning
D) are run on similar lines to those in the rest of the
world
E) have been extensively revised since 1994

62. It is pointed out in the passage that in some


countries, like Japan, France and Germany, ----.

A) the school curricula allow roughly equal time for


academic and non-academic subjects
B) the school year is far too long and this makes it
unproductive
C) students are given less homework than their
American counterparts
D) achievement correlates well with the length of the
school day
E) the amount of time students spend on academic
learning far exceeds that spent by American
students

63. According to the passage, any extension of


the instructional day in American schools ----.

A) is not to be recommended on account of the


expenses involved
B) would arouse much discontent among parents
and students
C) needs to be reviewed by a national commission
D) should aim to bring them up to the level of
Japanese schools
E) would have to have the approval of all the states

20
65. – 68. soruları aşağıdaki parçaya göre 68. The point is made in the passage that, for
cevaplayınız. Leonardo, ----.

During the Renaissance, especially in the sixteenth A) painting provided a greater capacity for artistic
century, it was customary to debate the pre-eminence creativity
of the arts, particularly as between painting and B) Cellini’s understanding of the arts was a grossly
sculpture. The more commonly accepted opinion is distorted one
represented by Benvenuto Cellini, who thought that C) painting came easily, but sculpture offered many
sculpture is eight times as great as any other art challenges
based on drawing, because a statue has eight views D) the practice of any art requires a great deal of
and they must all be equally good. A painting, he said, invention and imagination
is nothing better than the image of a tree, man, or E) the effects of a statue are far more subtle than
other object. In fact, the difference between painting those of a painting
and sculpture is as great as between a shadow and
the object casting it. Leonardo, on the other hand,
thought that painting is superior to sculpture because
it is more intellectual. By this he meant that as a
technique it is infinitely more subtle in the effects that
it can produce, and infinitely wider in the scope it
offers to invention or imagination. Michelangelo, when
the question was referred to him, in his wise and
direct way said that things which have the same end
are themselves the same, and that therefore there
could be no difference between painting and sculpture
except differences due to better judgment and harder
work.

65. We understand from the passage that


Michelangelo’s view concerning the relative
merits of painting and sculpture ----.

A) does not reflect his own position as an artist


B) can be regarded as a humourous attempt to bring
about a peace between Cellini and Leonardo
C) is a light-hearted attempt to avoid the issue
D) is unbiased and favours neither
E) has no relevance outside the Renaissance period

66. As pointed out in the passage, according to


Cellini ----.

A) the generally held belief on the pre-eminence of


the arts was totally unfounded
B) Leonardo’s skills as a painter did not exceed
those of Michelangelo
C) the Renaissance debate on the pre-eminence of
the arts should not be taken seriously
D) the art of the sculptor is less demanding than that
of the painter
E) a painting is inferior to a work of sculpture
because it has no solidity

67. It is clear from the passage that, during the 16th


century, ----.

A) the art of Leonardo was more highly regarded


than that of Cellini
B) there was much discussion as to the hierarchy of
the arts
C) sculptors and painters liked to cooperate on
major projects
D) most artists were both sculptors and painters
E) Leonardo and Michelangelo were keen rivals but
each admired the work of the other

21
69. – 72. soruları aşağıdaki parçaya göre 72. It is clear from the passage that, although a
cevaplayınız. very large number of innovations have been
made, ----.
Governments have learned to value innovation these
days for good reason. Far from being simply some A) only a few innovators have received awards for
missing factor in the growth equation, innovation is their work
now recognized as the single most important B) the majority of them have turned out to be
ingredient in any modern economy. It actually economically unfeasible
accounts for more than half of economic growth in C) they have had no significant impact on the world
America and Britain. In short, it is innovation, more economy
than the application of capital or labour that keeps the D) none of them have received any special
world economy going. As a result, economists have recognition
decided that the innovators of the world are due some E) there has been no noticeable improvement in the
special recognition. It is not possible to recognize all human condition anywhere
the countless innovations that have helped to spread
wealth, health and human happiness around the
world. But a handful of people who have made the
biggest contribution to the wealth-creation process in
their own fields over the past few years, have been
nominated for awards.

69. One point made in the passage is that, due to


innumerable innovations, ----.

A) the world economy has acquired a certain level of


uniformity
B) the American economy has under-performed
C) capital has now returned to the fore in economic
policies
D) the quality of human life on earth has greatly
improved
E) countries like Britain have fewer labour problems
than formerly

70. It is pointed out in the passage that


government economic policies ----.

A) rely more and more on the management of the


labour force
B) now take into account the importance of
innovation
C) regard the wealth-creation process as the main
target
D) in Britain have undergone very little change over
many decades
E) throughout the world are undergoing many
changes

71. It is pointed out in the passage that the


American and British economies ----.

A) have been in the forefront in the creation of


wealth
B) have tended to ignore innovations
C) have grown largely on account of innovation
D) have had an adverse effect upon the world’s
growth equation
E) have always been primarily concerned with the
prosperity of their citizens

22
73. – 76. soruları aşağıdaki parçaya göre 76. According to the passage, a nonprofessional
cevaplayınız. mother’s working conditions ----.

A nonprofessional-class working mother, who has A) are far from satisfactory, and she enjoys no
been forced unwillingly into the labour market, is benefits
oppressed by various unique forces. She is B) are being reviewed with the aim of improving
oppressed by the fact that her work is oftentimes them
physically exhausting, ill-paid, and devoid of benefits C) are no worse than those of other workers
such as health insurance and paid sick leave. She is D) have only recently become difficult
oppressed by the fact that it is impossible to put a E) have received a great of public attention
small child in reliable day-care if you make only a
minimum wage, and she is oppressed by the terrible
child-care options that are available at an inexpensive
rate. She is oppressed by the fact that she has
nothing to fall back on. If she is out of work, and her
child needs a visit to the doctor and antibiotics, she
may not be able to afford those things and will have to
treat her sick child with unprescribed medications,
which themselves are far from cheap.

73. We understand from the passage that a


working mother, without a career, ----.

A) works so that her child can enjoy good day-care


B) is usually granted several fringe benefits
C) rarely stays in her job for a long period
D) faces a very hard life
E) has a great deal of choice in the kind of work she
does

74. It is clear from the passage that, when a


nonprofessional working mother loses her job
----.

A) she may neglect the child but not herself


B) there are always opportunities available on the
labour market
C) and her child gets ill, she probably cannot get
proper medical help
D) she has to be prepared to accept a lower-paid
one
E) she invariably has a great deal of trouble finding
a new one

75. We understand from the passage that very


many nonprofessional working mothers ----.

A) enjoy health insurance which also covers their


children
B) are, on the whole, satisfied with their jobs
C) feel their children are being suitably cared for
D) are paid far more than the recognized minimum
wage
E) only work because they have to work

23
77. – 80. soruları aşağıdaki parçaya göre 80. Clearly, the passage is mainly concerned with
cevaplayınız. ----.

Because a play presents its action through actors, its A) the techniques a director makes most use of in
impact is direct, immediate, and heightened by the the staging of a play
actor’s skills. Instead of responding to words on a B) the sense of immediacy and the intensity that a
printed page, the spectator sees what is done and well-staged play offers
hears what is said. The experience of the play is C) the role of imagination in prose fiction and drama
registered directly upon his senses. It may therefore D) the question of how a play can best be performed
be fuller and more compact. Where the work of prose E) the relationship between the actors and the writer
fiction may tell us what a character looks like in one of a play
paragraph, how he moves or speaks in a second,
what he says in a third, and how his auditors respond TEST BĐTTĐ.
in a fourth, the acted play presents this material all at CEVAPLARINIZI KONTROL EDĐNĐZ.
once. Simultaneous impressions are not separated.
Moreover, this experience is interpreted by actors
who may be highly skilled in rendering nuances of
meaning and strong emotion. Through facial
expression, gesture, speech rhythm, and intonation,
they may be able to make a speaker’s words more
expressive than can the reader’s unaided imagination.
Thus, the performance of a play by skilled actors,
expertly directed, gives the playwright a tremendous
source of power.

77. It is clear from the passage that unlike a


staged play, a work of prose fiction ----.

A) mainly focuses on character and action


B) is very effective in arousing the reader’s emotions
C) allows no possibility of multiple interpretation
D) makes much use of various literary devices
E) makes its impact slowly

78. It is emphasized in the passage that the


effectiveness of a play’s action ----.

A) disappears as soon as the performance is over


B) is maintained only temporarily depending on the
audience
C) is largely created through facial expression and
the playwright’s skill
D) is increased both through skilled performance
and through professional direction
E) can be further strengthened through character
analysis

79. It is pointed out in the passage that the


performance of a play on the stage ----.

A) enables the audience to be more closely involved


B) has the same impact on the audience as that of
the printed text
C) depends much more on the director than on the
actors for its success
D) can best be accomplished through close attention
to the playwright’s instructions
E) should not guide the way the audience feels

24
1. – 18. sorularda, cümlede boş bırakılan yerlere 9. Any drug development effort ----
uygun düşen sözcük ya da ifadeyi bulunuz. neurodegenerative disorders ---- carefully any
possible side effects.
1. In England, the general public’s ---- of medical
advice from the government stems from the fact A) combating / would have examined
that, in the past, such information has often B) to combat / will have to examine
proved vastly inaccurate. C) to have combated / will examine
D) having combated / had examined
A) approval B) inadequacy C) mistrust E) to be combated / would have to examine
D) distraction E) preference
10. Foot massages ---- those who ---- balance
2. Many researchers now focus on poverty and problems.
poor education as explanations for ---- mortality.
A) may have helped / are having
A) probable B) intense C) preventive B) will help / had
D) occasional E) excess C) could help / have
D) have helped / would have
E) would have helped / may have
3. Heart disease, stroke and lung cancer ----
constitute 35 per cent of all deaths in the US.
11. Until quite recently, no one ---- the deadly
A) together B) readily C) exactly germ that causes anthrax ---- outside a living host.
D) fairly E) well
A) had thought / would have thrived
B) thinks / is thriving
4. Controlled diet studies ---- that saturated fat C) has thought / will thrive
increases cholesterol levels. D) would think / had thrived
E) thought / could thrive
A) prescribed B) confirmed C) disturbed
D) regretted E) deduced
12. Bleeding after oral surgery can usually be
stopped ---- keeping steady pressure ---- the
5. The worst fires firemen have to ---- are those surgical site for the first hour.
that emit stifling smoke and noxious gases.
A) for / to B) in / to C) with / up
A) take over B) put over C) fill out D) by / on E) from / of
D) cope with E) pull through
13. As regards breast changes, our definition ----
6. The correct time to start a baby on solid food “normal” varies ---- age and experience.
---- its needs and readiness.
A) of / with B) for / from C) at / over
A) builds up B) depends on C) cares for D) in / for E) with / by
D) puts forward E) slows down
14. Children’s appetites begin to diminish around
7. Most of the world’s population ---- on natural one year, ---- the slowing of growth.
exposure to sunlight ---- adequate vitamin D
nutrition. A) regardless of “ B) in spite of
C) in case of D) consistent with
A) relies / to maintain E) except for
B) has relied / to have maintained
C) used to rely / would have maintained
D) will rely / having maintained 15. Ten per cent of the population is allergic to
E) is relying / to be maintaining thimerosol, ---- the health of millions of babies and
children worldwide is being compromised.

8. Shanghai ---- curb pollution or its citizens ----


rapidly increasing ill health. A) if B) so C) while
D) unless E) which
A) needs to / have faced
B) had to / had faced
C) has to / will have faced 16. In developing countries, ---- nutritional
D) will have to / would face concerns override the risk of HIV transmission,
E) must / will face breastfeeding may still be desirable.

A) where B) what C) that


D) which E) whether

25
17. ---- repairing nerve damage, glia may also be 25. ---- so that future encounters with the same
critical to learning and to forming memories. pathogen are dealt with swiftly.

A) Contrary to B) As regards C) Just as A) Effective vaccines can be prepared in a number


D) As well as E) In spite of of ways
B) Through immunization the body launches an
immune response, and develops memory cells
18. Women are ---- likely ---- men to discuss C) Most vaccines consist of the entire pathogen or
mental health problems with their general practice of a protein from the pathogen
physician. D) Tetanus and botulism vaccines are made from
toxins secreted by the respective pathogens
A) so / much B) also / as C) more / than E) Most persons contract measles or chickenpox
D) too / for E) as / that only once

19. – 23. sorularda, aşağıdaki parçada 26. ---- until James Watson and Francis Crick
numaralanmış yerlere uygun düşen sözcük ya da proposed a model for its structure that had
ifadeyi bulunuz. extraordinary explanatory power.

For the purpose of diagnosis, analysis and A) Many genes encode proteins that are not
experimentation, academic physicians tend to focus enzymes
on disease at a particular point in time. But disease B) A great deal was known about the physical and
needs (19) ---- as a process that evolves over time chemical properties of DNA
through the (20) ---- of genetic, environmental and C) DNA was not widely accepted as the genetic
lifestyle factors. This view puts a premium (21) ---- material
understanding the complex history of a patient, and it D) DNA is made of two polynucleotide chains
(22) ---- that most disease cannot be tied to a (23) ---- intertwined to form a double helix
cause. E) The idea that genes and enzymes are related in
some way was first clearly stated in 1908
19.
A) to have been treated B) to have treated
C) to treat D) being treated 27. When a coronary artery becomes narrowed,
E) to be treated ----.

A) one of the main jobs of the circulation is to bring


20. oxygen to all the cells of the body
A) availability B) discovery C) interaction B) ischemic heart disease can develop
D) compatibility E) reliability C) arteries that branch off from the aorta conduct
blood to all regions of the body
D) hormones are involved in regulating blood
21. pressure
A) against B) to C) over E) blood vessels carrying oxygen-rich blood are red
D) towards E) on
28. ---- that the proportion of glia to neurons
22. increases greatly as animals move up the
A) conducts B) rejects C) disputes evolutionary ladder.
D) acknowledges E) denies
A) Comparisons of brains reveal
B) This hypothesis has still to be tested
23. C) Neuroscientists are pursuing the enquiry
A) single B) multiple C) reliable D) The capacity for learning is being investigated
D) subsequent E) persistent E) The problem has not been adequately
researched

24. – 35. sorularda, verilen cümleyi uygunşekilde


29. Although nitrous oxide is still in common use
tamamlayan ifadeyi bulunuz.
as a general anesthetic, ----.
24. If born at 24 weeks, ----.
A) the operating room became known as the Ether
Dome
A) the fetus grows rapidly during the final trimester B) nitrous oxide and di-ethyl ether are not the same
B) several hormones are involved in initiating the C) ether has not taken its place
birth process D) its reliability is sometimes questioned
C) the brain would begin to send impulses that E) the term “ether” was often used indiscriminately
regulate the functions of some organs during the 19th century
D) the fetus has only about a 50% chance of
surviving
E) the average full-term baby weighs about 3,000
grams and measures about 52 centimetres

26
30. ----, it picks up glucose, amino acids and other 36. – 38. sorularda, verilen Đngilizce cümleye
nutrients. anlamca en yakın Türkçe cümleyi bulunuz.

A) As blood flows through capillaries within the wall 36. A striking feature of pregnancy is that blood
of the intestine pressure and peripheral vascular resistance fall
B) Because oxygen-rich blood is supplied to the liver soon after conception.
by the hepatic artery
C) Since the hepatic portal vein delivers nutrients to A) Döllenme sonrası kan basıncının ve peripheral
the liver vasküler direncin düşmesi, gebeliğin diğer bir
D) In case the thrombus blocks a sizable branch of a çarpıcı özelliğidir.
coronary artery B) Döllenmeyi takiben, kan basıncı ve peripheral
E) While liver sinuses merge to form hepatic veins vasküler direncin düşmesi, gebelikle ilgili önemli
bir özelliktir.
C) Gebeliğin bir diğer önemli özelliği, döllenme
31. The “liquid protein” diet, ----, caused deaths in sonucunda, kan basıncının ve periferal vasküler
many users. direncin düşmesidir.
D) Gebeliğin çarpıcı bir özelliği, kan basıncı ve
A) since people like to take supplements peripheral vasküler direncin, döllenmeden hemen
B) which was advocated some years ago for weight sonar düşmesidir.
loss E) Gebeliğin bir başka çarpıcı belirtisi, döllenme olur
C) as athletes require a well-balanced diet olmaz, kan basıncının ve periferal vasküler
D) that the diet is perfectly adequate direncin düşmeye başlamasıdır.
E) unless there had been regular medical
supervision
37. Most people realize the importance of calcium
for bone health, but they underestimate the
32. Studies have shown that people ---- have a importance of vitamin D, which fosters calcium
lower risk of coronary heart disease. absorption.

A) that had developed diabetes A) Đnsanlar kalsiyumun kemik sağlığı için önemini
B) whose intake of unsaturated fats was high bilmelerine rağmen kalsiyumun emilimini
C) since they exercise regularly kolaylaştıran D vitaminini göz ardı ederler.
D) if there is no history of heart disease in the family B) Pek çok insan kalsiyumun kemik sağlığı
E) who replace red meat with chicken and fish üzerindeki etkisinin farkındadır ama kalsiyumun
emilimini güçlendiren D vitamini genellikle
bilinmez.
33. Certain therapies are effective for certain C) Kalsiyumun kemik sağlığı için önemi bilinir, ancak
disorders ----. kalsiyumun emilimini sağlayan D vitamini
önemsenmez.
A) that individuals receiving therapy actually did D) Đnsanlar kalsiyumun kemik sağlığındaki önemini
improve bilmekle beraber kalsiyumun emilimini D
B) which treatment is effective for which problem vitamininin kolaylaştırdığını bilmezler.
C) since one school of therapy emphasizes insight E) Pek çok insan kemik sağlığı için kalsiyumun
D) but are relatively ineffective for others öneminin farkındadır fakat kalsiyum emilimini
E) as these were the untreated control-group güçlendiren D vitamininin önemini küçümser.
patients

38. Recent studies found no link between the risk


34. It now seems that glial cells, ----, may be nearly of developing brain tumour and the amount of
as important as neurons are in the thinking time one has spent using a mobile phone.
process.
A) Son çalışmalarda bir kişide beyin tümörü
A) which have been overlooked for half a century gelişmesi riskiyle, cep telefonu kullanma süresi
B) if the evidence is quite convincing arasındaki ilişki kanıtlanamadı.
C) that they communicate among themselves B) Beyin tümörü gelişmesiyle ilgili son çalışmalarda
D) as research into these cells was abandoned kişilerin cep telefonu kullanması riskli bulunmadı
E) since this affects how the brain performs C) Son çalışmalar beyin tümörü gelişmesi riskiyle,
kişinin cep telefonu kullanarak geçirdiği zaman
arasında bir ilişki bulmadı.
35. The disease is rarely fatal, ----. D) Son araştırmalar cep telefonu kullanılarak
geçirilen zamanın beyin tümörü gelişimine katkısı
A) if it had been promptly diagnosed olmadığı sonucunu buldu.
B) though it may result in paralysis E) Kişinin beyin tümörü geliştirme riskiyle cep
C) unless there was a history of allergies telefonu kullanarak geçirdiği zaman arasındaki
D) since the patient responded well to the treatment ilişki, son araştırma sonuçlarına göre oldukça
E) until proper medical care could be given zayıftır.

27
39. – 41. sorularda, verilen Türkçe cümleye 42. – 46. sorularda, boş bırakılan yere, parçanın
anlamca en yakın Đngilizce cümleyi bulunuz. anlam bütünlüğünü sağlamak için getirilebilecek
cümleyi bulunuz.
39. Her ne kadar hücreler çok farklı gibi görünse
de temel özellikleri dikkate değer şekilde 42. Opioid analgesics are very effective in
benzerdir. controlling pain but have many side effects. ----. In
addition, before a long-term use of opioid
A) Cells seem to be extremely diverse, but they all analgesics can be stopped, the dose must be
have similar features. gradually reduced to minimize the development of
B) Although cells may appear to be very diverse, the withdrawal symptoms.
fundamental features are remarkably similar.
C) Even though cells display a great deal of variety, A) Moreover, people with severe pain shouldn’t
fundamentally, they have certain similarities. avoid opioids
D) Cells appear to have much diversity even if B) With time a person using them may need higher
fundamentally they are the same. doses
E) Despite their great variety, cells do have C) AIDS can cause pain as severe and unrelenting
fundamentally similar features. as that of cancer
D) The sustained-release form provides relief for 8
to 12 hours
40. Enzim bozukluklarını içeren yüzlerce insan E) Several types of analgesics can help alleviate
hastalığına gen değişimlerinin neden olduğu pain
anlaşılmıştır.

A) Hundreds of human diseases involving enzyme 43. An organism must divide its energy between
defects have been found to be caused by genetic maintenance, repair and reproduction. ----. As a
mutations. result, organisms face a tough problem: What is
B) It has been established that genetic mutations the best allocation of finite metabolic energy to
have caused hundreds of human diseases maximize reproduction and repair?
including enzyme defects.
C) Genetic mutations have been found to be the A) Actually, some organisms do have unlimited
cause of hundreds of human diseases as well as energy
enzyme defects. B) No creatures are capable of living indefinitely
D) Hundred of human diseases have been found to C) The reproductive life of an organism may be even
result from genetic mutations and enzyme shorter
defects. D) As an organism ages, the problems become less
E) As it is known, hundreds of human diseases have acute
been caused by genetic mutations involving E) Even a well-fed organism has to cope with
enzyme defects. energy limitations

41. Kan basıncı ölçümleri gebelik boyunca normal 44. Breast tumours are usually composed of more
tutulması koşuluyla, yüksek tansiyonlu kadınlar, than one type of cancer. ----. Although scientists
artmış pre-eklampsi riski taşımazlar. know about this phenomenon, it has been difficult
to quantify because pathologists use differing
A) A proper control of blood pressure during diagnostic criteria.
pregnancy enables women with hypertension to
resist any increased risk of pre-eclampsia. A) The latter is much easier to diagnose
B) So long as blood pressure is kept within normal B) In some centres a single pathologist reviews all
limits during pregnancy, women suffering from patient samples
hypertension are immune from the high risk of C) It is hoped that this will enhance patient care
pre-eclampsia. D) This is a problem when the cancers do not all
C) Women, who have hypertension, can overcome respond to the same treatment
the increased risk of pre-eclampsia so long as E) As a result, the stage of the cancer also needs to
their blood pressure measurements remain be considered
stable.
D) Women whose blood pressure is properly
controlled during pregnancy, do not suffer from
hypertension and have no risk of pre-eclampsia.
E) Provided blood pressure measurements are
within normal limits throughout pregnancy,
women with hypertension do not have the
increased risk of pre-eclampsia.

28
45. A patient with Korsakoff’s syndrome reports 48. Alison: - Is it really necessary to give infants so
that he spent the weekend at the beach, when in many vaccines during their first year?
fact he was in the hospital. ----. However, he Pat: - ----
neither notices his amnesia nor questions his own Alison: - Why is that?
story; concerning his own past he doesn’t know Pat: - Because once a child is a year old, many
that he doesn’t know. parents feel that regular visits to a doctor are no
longer necessary.
A) His memory is impaired but he has replaced his
lost history with a story A) I suppose it depends on where you live.
B) Such sincere claims cannot be argued away B) No. But while they are small, they do not object.
C) There is no point in trying to contradict such an C) I think it’s vital. But a lot of pediatricians would
assertion disagree.
D) Similarly, a patient with Anton’s syndrome will D) Let’s hope they don’t introduce any more!
deny his own blindness E) It’s not essential. But it is usually advisable.
E) Actually, the syndrome is a very rare one

49. Amy: - Pat has always wanted to be a nurse


46. Although social and medical interventions and I’m sure she’ll make an excellent one.
have helped people live longer, none of the Lucy: - I’m sure she will. But it’s not an easy
techniques have affected the aging process. ----. profession.
But, there is a change: there are far more 65-year- Amy: - ----
olds today than there used to be because the past Lucy: - Yes; there’s no doubt about that.
century’s efforts reduced early mortality.
A) She says she’d prefer to work in a children’s
A) Fewer young people now die of infectious hospital.
diseases as sanitary conditions have improved B) What does her father think about it?
greatly C) At her age I wanted to be a nurse, didn’t you?
B) This is because people are living longer D) Of course it’s not. But it offers a great deal of job
C) A healthy 65-year-old in 1900 would be physically satisfaction.
indistinguishable from his or her counterpart in E) Even so, I’m sure I’d find the hours quite
2000 unbearable!
D) Improved working conditions have also
contributed, to a noticeable extent, to an
increased life expectancy 50. Paul: - Do you think Clive will agree to have
E) Attention to diet also plays an important role this operation?
Edith: - He already has agreed. We’re both
convinced it is the best course to take.
47. – 51. sorularda, karşılıklı konuşmanın boş Paul: - ----
bırakılan kısmını tamamlayabilecek ifadeyi Edith: - Reasonably good. After all he is basically a
bulunuz. very healthy person.

47. Amy: - I suppose Parkinson’s is on the A) Has the surgeon discussed the risks with him?
increase because people are living so much B) What are the chances of its being successful?
longer. C) Does he realize how risky it is?
David: - It’s not as simple as that. Fifty per cent of D) Have you thought about this seriously?
patients acquire it before they are sixty. E) What about getting a second opinion?
Amy: - ----
David: - No, it’s not. Environmental factors seem
to play quite an important role. 51. Mary: - Did you say your daughter was doing
the interior design for a hospital?
A) Do they really? So it’s not simply a condition of Brenda: - Yes, that’s right. They now give a great
old age. deal of importance to the colour schemes and the
B) I didn’t know that. Is there any cure? general use of space in a hospital.
C) Are you sure the condition is reversible? Mary: - ----
D) Among younger patients, is surgical intervention Brenda: - I don’t think it is. The morale of the
recommended? patients has an important role to play in their
E) Let’s hope they’ll soon find new ways of treating recovery.
the condition.
A) Isn’t that rather a waste of public money?
B) Still, most people want to get away as soon as
possible.
C) Yes; I remember reading something to that effect.
D) I’ll be glad when they give up all those white walls,
won’t you?
E) I’ve noticed an improvement in the children’s
wards.

29
52. – 56. sorularda, cümleler sırasıyla
okunduğunda parçanın anlam bütünlüğünü bozan
cümleyi bulunuz.

52. (I) The statistics are staggering. (II) Since 1981,


an estimated 28 million people have died of AIDS (III)
Today, 42 million men, women and children are
believed to be living with HIV. (IV) What is even more
disturbing, 5 million new infections are occurring each
year. (V) Indeed, vaccines have helped to eradicate
some of the worst diseases of the 20th century.

A) I B) II C) III D) IV E) V

53. (I) Dependence on sleep aids and anti-anxiety


drugs decreases alertness and results in slurred
speech, poor coordination, confusion and slowed
breathing. (II) These drugs may make a person
alternately depressed and anxious. (III) Prescription
drugs that can cause dependency are subject to
restrictions (IV) Some people experience memory
loss, faulty judgment, and sudden shifts in their
emotions. (V) Furthermore, older people may even
appear demented.

A) I B) II C) III D) IV E) V

54. (I) For a small child it is not obvious which shoe


goes on which foot. (II) The problem of fitting
molecules into biological systems presents a similar
dilemma. (III) In life, and especially in developing
pharmaceuticals, shape matters. (IV) Making pure
left-handed or right-handed drugs is therefore hard.
(V) Using the wrong-shaped molecule to treat a
disease is about as effective as using the wrong key
to get into your house.

A) I B) II C) III D) IV E) V

55. (I) Stress can affect our health by leading us to


engage in types of behaviours that undermine the
body’s ability to fight off disease. (II) When we are
feeling stressed, we often do not take proper care of
ourselves. (III) Thus, stressful situations may affect
immune system functioning. (IV) Students, for
instance, who are taking exams, may stay up all night
for several nights in a row. (V) Furthermore, they may
skip meals and snack on junk food.

A) I B) II C) III D) IV E) V

56. (I) In any school, in any week of the year a


dyslexic child experiences a huge amount of failure.
(II) Without self-confidence no real progress is
possible. (III) With sequencing difficulties, any form of
writing or maths is going to present severe problems.
(IV) The dyslexic child cannot fail to notice that almost
all the other children can do the work fairly easily. (V)
He therefore concludes that he must be stupid and his
confidence goes.

A) I B) II C) III D) IV E) V

30
57. – 60. soruları aşağıdaki parçaya göre 60. This passage is essentially concerned with ----.
cevaplayınız.
A) various causes of aging
Aging involves multiple harmful biological events that B) the damaging effects induced by aging
accumulate in different tissues over time and C) the significance of biomarkers in measuring the
gradually reduce an organism’s state of maintenance process of aging
and function. Calendar time, however, serves as an D) the role a person’s genes have in the process of
imperfect measurement of the physiological aging
processes involved in aging. We all know individuals E) the need to carry out more research into aging
who are the same chronological age but appear to be
very different when it comes to physiological age.
Rather than counting years — or gray hairs, for that
matter — modern gerontologists turn to biological
markers, or biomarkers, of aging. These physiological
parameters indicate an individual’s functional level
and some biomarkers, such as insulin levels, correlate
with mortality. The presence of such biomarkers
depends indirectly on patterns of gene expression,
which are induced by a variety of internal or external
stimuli.

57. It is pointed out in the passage that the


process of aging ----.

A) can best be determined through biological


markers
B) has a pattern which varies very little among
individuals
C) cannot be clearly understood without taking into
account the graying of a person’s hair
D) is best indicated by the amount of years a person
has lived
E) has an adverse effect on very few functions of an
individual’s body

58. We understand from the passage that


biomarkers ----.

A) do not always produce reliable results in


measuring a person’s biological age
B) are closely related to insulin levels
C) and calendar time must be taken into account in
finding out about a person’s health
D) are of primary importance for gerontologists in
learning about a person’s physical condition
E) can undermine an organism’s state of
maintenance and function

59. It is clear from the passage that as one ages


----.
A) one’s genetic defects become more obvious
B) the body’s ability to maintain itself starts to
deteriorate
C) the physiological processes accurately reflect
chronological time
D) there are functional changes in the body but
these are hard to detect
E) one’s insulin level remains stable

31
61. – 64. soruları aşağıdaki parçaya göre 64. It can be understood from the passage that not
cevaplayınız. every patient with congestive heart failure ----.

Cardiac transplantation, once considered an A) reports to a cardiologist for treatment


experimental procedure, has emerged as the therapy B) agrees to undergo cardiac transplantation
of choice for many appropriately selected patients C) can qualify for cardiac transplantation
with life-threatening irremediable heart disease. D) is admitted to hospital for diagnosis and
Congestive heart failure (CHF), the primary indication treatment
for cardiac transplantation, is the most commonly E) realizes just how serious the condition is
reported reason for hospital admission. A knowledge
of cardiac transplantation medicine is therefore
important for all physicians, as transplantation should
be considered a therapeutic option for many of these
patients. In the past, post-cardiac transplant care was
largely performed by specialized transplant
physicians, primarily cardiologists and cardiovascular
surgeons. As survival after cardiac transplantation
has improved markedly over the last decade, the
population of patients who are long-term survivors
after heart transplantation has grown. Primary care
physicians, as well as cardiologists not based at
cardiac transplant centres, often assist in the care of
these patients, most often in consultation with cardiac
transplant physicians. In addition, a physician may be
called on to assist in the management and evaluation
of a potential cardiac donor.

61. According to the passage, in recent years ----.

A) almost every patient with heart disease has come


to recognize cardiac transplantation as an
indispensable therapeutic option
B) more and more patients with CHF have chosen to
undergo cardiac transplantation
C) the number of cardiac transplant centres in the
world has increased due to advances in cardiac
transplantation
D) there has been a noticeable improvement in the
survival of post-cardiac transplant patients
E) cardiologists and cardiovascular surgeons have
been engaged in extensive research for the
improvement of post-cardiac transplant care

62. We learn from the passage that, to start with,


cardiac transplantation ----.

A) was an experimental procedure rather than a


therapy
B) was carried out on every patient with CHF
C) had a very low success-rate indeed
D) could only be practiced at special cardiac
transplant centres
E) was generally regarded with disapproval in the
medical world

63. It is pointed out in the passage that cardiac


transplant physicians ----.

A) are no longer involved in post-cardiac transplant


care
B) are often assisted in post-cardiac transplant care
by primary care physicians
C) are the only ones qualified to select cardiac
donors
D) are still experimenting in developing an improved
method of cardiac transplantation
E) are primarily affiliated with cardiac transplant
centres outside hospitals

32
65. – 68. soruları aşağıdaki parçaya göre 68. In this passage, emphasis is put on the fact
cevaplayınız. that ----.

When a patient sees a doctor, the patient is seeking A) some patients are harder to please than others
help –- to regain or retain health. The physician’s task B) a physician is very rarely able to tackle all of the
is to work for the patient’s health. The doctor does so “the five D’s” with any degree of success
by treating disease, by relieving discomfort, by C) a patient’s expectations can very rarely be met
assisting the patient with any disability, by preventing D) a patient’s state of mind can detract from his/her
premature death, and by maximizing contentment. recovery
(Some have summarized these activities as tackling E) a physician’s duties to a patient are wide in range
“the five D’s” of health – disease, discomfort,
disability, death, and dissatisfaction). Often there is
success in all these areas. In the best of
circumstances, the doctor is able to prevent disease
and help the patient remain healthy. In other cases,
disease and death defeat us. In some cases none of
the goals are achieved, but even that outcome must
not stop us from trying. By focusing on the health of
the patient, the doctor tests the myriad activities of
clinical medicine against the health outcome of the
patient.

65. According to the passage, as regards “the five


D’s” of health, ----.

A) a doctor should never admit to failure


B) it is, unfortunately, discomfort that is most
frequently overlooked
C) a doctor’s priority must be the accurate diagnosis
of the disease
D) the psychology of the patient is of paramount
importance
E) a doctor is not likely to achieve success in all
areas, all the time

66. It is stressed in the passage that a patient


seeks medical help ----.

A) either to recover from disease or to maintain


health
B) only when he/she is in acute pain
C) only after a condition has become chronic
D) because friends or family have urged him/her to
do so
E) if there seems to be a serious medical problem
calling for clinical tests

67. We understand from the passage that, in


offering medical help to a patient, the physician
----.

A) needs to know something about the patient’s


medical history
B) also seeks professional self-satisfaction
C) does contribute, to a significant extent, to the
patient’s well-being
D) often feels it is advisable to consult a colleague
and get a second opinion
E) has to take into account the psychological needs
of the patient

33
69. – 72. soruları aşağıdaki parçaya göre 71. We learn from the passage that, among the
cevaplayınız. different cancers, lung cancer ----.

Cancer describes a class of diseases characterized A) shows a downward trend in the United States
by the uncontrolled growth of aberrant cells. Cancers B) is the one that most needs to be dealt with
kill by the destructive invasion of normal organs immediately
through direct extension and spread to distant sites C) is the one that most readily yields to treatment
via the blood, lymph, or serosal surfaces. The D) has appeared with increasing frequency over the
abnormal clinical behaviour of cancer cells is often years
mirrored by biologic aberrations such as genetic E) affects as many non-smokers as smokers
mutations, chromosomal translocations, expression of
fetal or other discordant ontologic characteristics, and
the inappropriate secretion of hormones or enzymes. 72. We learn from the passage that cancer cells
All cancers invade or metastasize but each specific ----.
type has unique biologic and clinical features that
must be appreciated for proper diagnosis, treatment A) all present the same clinical features
and study. About 1.2 million new cases of invasive B) can be carried by the blood and spread to distant
cancer are diagnosed each year in the United States, organs
and about 500,000 people die annually of the C) are not necessarily invasive
disease. Cancer is the second most deadly disease D) rarely lead to genetic mutations
and is expected to surpass heart disease early in the E) have not been linked to biologic aberrations
twenty-first century to top that vicious list. Over the
past half century, the frequency of most cancers has
been stable, but some dramatic changes have taken
place. Steady declines in stomach and uterine cancer
have occurred, the latter undoubtedly due to routine
cytologic screening for cervical cancer. The cause of
the decline in stomach cancer is unknown. The most
striking change has been the increases in lung cancer
in both men and women, undoubtedly related to
smoking.

69. It is pointed out in the passage that, over the


next few decades, mortality due to cancer ----.

A) will start to decline as new methods of treatment


are introduced
B) will be higher than the mortality caused by heart
disease
C) will start to threaten younger people
D) will be greatly reduced through an increase in
early diagnosis
E) will increase at an alarming rate as very many
more people are starting to smoke

70. As it is pointed out in the passage, cancer ----.

A) has been on a steady increase since the mid-


twentieth century
B) has caused much higher mortality in the United
States than anywhere else in the world
C) is a disease with a great deal of variety, which
requires different treatments and studies
D) is still a disease much deadlier than heart
disease
E) of the stomach is the one that causes the most
problems

34
73. – 76. soruları aşağıdaki parçaya göre 75. We understand from the passage that
cevaplayınız. biomedical science ----.

Medicine is not a science, but a profession that A) is quite unrelated to patient-care


encompasses medical science learning as well as B) has only come to be appreciated in recent
personal, humanistic, and professional attributes. decades
Nonetheless, the delivery of Western medicine C) is only fully valued by those involved in serious
depends totally on science and the scientific method. research
Since Flexner issued his famous report on the subject D) is of great importance in really all of a physician’s
in 1910, American medical education has striven to activities
develop a strong scientific base as an integral part of E) bears little relation to other areas of science
medical education at every level: premedical, medical,
residency and continuing medical education.
Biomedical science is fundamental to understanding 76. It is clear that the main aim of the passage is
disease, making diagnoses, applying new therapies to ----.
and appreciating the complexities and opportunities of
new technologies. The process of becoming a A) emphasize that a thorough grounding in science
physician and being committed to lifelong learning is the basis of a good medical education
requires that one possess the scientific base not only B) establish the fact that the acquisition of new
to acquire and appreciate new knowledge but to see knowledge is not as important as finding new
new ways for applying it to patient care as well. The ways to apply it
physician must be able to understand reports of C) explain the controversial uses of biomedical
current research in the medical literature in order to science
grasp and evaluate the newest and latest D) encourage physicians to familiarize themselves
approaches, no matter how complicated the field may with the opportunities offered by new
become. technologies
E) introduce scientists to current research in medical
73. We understand from the passage that, for a literature
physician, ----.

A) it is almost impossible to keep up with recent


developments in medicine
B) it is what he learned during residency that most
influences his approach to his patients
C) medical education is not limited to medical school
and residency, but is actually a process of
lifelong learning
D) new developments outside his own field of
specialization can be ignored
E) the scientific base of his medical knowledge is far
more important than any new technologies and
approaches

74. It is clear from the passage that, as a result of


Flexner’s report, ----.

A) the humanistic aspect of the medical profession


has received much more attention
B) residency has come to the fore in American
medical education
C) the whole system of patient care in American
hospitals has been restructured
D) specialization rather than general medical
practice has become widespread in the United
States
E) medical education in the United States has
undergone a process of transformation

35
77. – 80. soruları aşağıdaki parçaya göre 79. It is pointed out in the passage that, a
cevaplayınız. reduction of lead in paint and petrol, ----.

In the past, lead poisoning was thought to arise from A) was recommended decades ago but never
pica (abnormal ingestion) among children living in old, enforced
broken-down houses with peeling layers of lead- B) will do little to overcome environmental
based paints. In the past two decades, lead contamination
intoxication has occurred with decreasing frequency. C) will bring substantial benefits to house painters
This may in part be related to less use of lead in paint and those repairing old houses, but to no one
and leaded gasoline; several studies relate else
environmental lead contamination to traffic density D) has produced as many negative results as
patterns. In the United States, hundreds of positive
occupations involve potentially significant exposure. It E) may have contributed to the decrease in cases of
is estimated that more than 800,000 American lead poisoning observed in recent years
workers have potentially significant lead exposure.
Lead and other metal workers or miners, storage
battery workers, and pottery makers are particularly 80. We learn from the passage that, despite its
heavily exposed. Workers in auto manufacturing, ship adverse effects, lead ----.
building, paint manufacture and printing industries are
also at substantial risk, as are house painters and A) is not generally regarded as a dangerous
those who repair old houses. In past centuries lead substance
was added to wine to sweeten it, a deception that was B) is not nearly as poisonous as several other
eventually made punishable by death. Recently, metals
adding lead to various herbal and folk medicines has C) still remains one of the most valuable and
resulted in poisoning. Bullets left in the body can versatile of earth’s metals
result in lead poisoning, especially if a joint is D) is still being used in a great many industries
involved, because synovial fluid appears to be a good E) is only potentially dangerous to children, not to
solvent for lead. The interval between the bullet adults
getting embedded in the body and clinical evidence of
lead poisoning has ranged from 2 days to 40 years. TEST BĐTTĐ.
CEVAPLARINIZI KONTROL EDĐNĐZ.

77. It is stated in the passage that lead exposure


----.

A) has been recognized by many employers as a


deadly threat to their work force
B) has been increasing at an alarming rate in many
industries in the United States
C) is a very serious concern in various occupations
and industries
D) has led to the introduction of various preventive
measures in certain occupations and industries
E) and its consequences have only recently become
a major concern of clinical research

78. According to the passage, there is scientific


evidence to suggest that ----.

A) a bullet induces lead poisoning as soon as it is


lodged in the body
B) heavy traffic causes lead contamination in the
environment
C) most cases of lead poisoning have resulted from
lead-based paints
D) the risk of lead poisoning is equally likely to occur
in all the different metal industries and
occupations
E) the quality of wines can be improved substantially
through the use of lead chemicals

36
1. – 18. sorularda, cümlede boş bırakılan yerlere 9. Individuals who ---- that animals ---- feelings are
uygun düşen sözcük ya da ifadeyi bulunuz. usually accused of anthropomorphism, or
ascribing human traits to nonhuman beings.
1. Copper is replacing aluminium in the metal
interconnections on some chips to improve ----. A) had claimed / had
B) claimed / will have
A) creativity B) credibility C) sustainability C) are claiming / would have
D) conductivity E) respectability D) claim / have
E) would claim / must have had

2. It is hoped that these ---- projects will lead to a


better understanding of typhoons and improve 10. The reason why the moon doesn’t orbit the
short-term weather forecasting. Sun is because the Earth is also ---- towards the
Sun, and so the two ---- through space together.
A) defensive B) excessive C) comprehensive
D) regrettable E) forceful A) going to pull / will have been moving
B) having been pulled / moved
C) pulling / were moving
3. Wouldn’t it be wonderful if science and D) to pull / move
scientists were taken more ---- in the political E) being pulled / are moving
process?

A) sullenly B) seriously C) satisfactorily 11. The recovery and identification of plant


D) ingeniously E) pretentiously remains from archaeological contexts are merely
the first steps in a wide-ranging series of research
4. The Sun’s gravitational pull on the moon is issues that ---- up paleoethnobotany, also ---- as
more than twice that ---- by the Earth. archaeobotany.

A) attempted B) undertaken C) magnified A) made / knowing


D) replaced E) exerted B) will make / is known
C) make / known
D) has made / having known
5. A mystery virus has ---- more than 90 per cent E) had made / is to be known
of some bird species in India.

A) found out B) broken through 12. Exploration of the Arctic began with the
C) turned up D) wiped out search ---- the Northwest Passage as a short cut
E) put off ---- the Far East.

A) for / to B) in / in C) by / for
6. The report emphasizes that, due to serious D) on / with E) from / of
acidification in the coastal waters, many marine
organisms have ----.
13. Roger Revelle’s calculations about what
A) died out B) taken off C) used up happens to the carbon dioxide released ---- the
D) run down E) ended up burning of fossil fuels were correct ---- showing
that much of it would end up in the sea.

7. The Proctor Prize ---- annually since 1950 to an A) from / with B) by / in C) to / by


outstanding scientist who ---- known for effective D) in / for E) through / about
communication of complex ideas.

A) was being presented / is being 14. Electromagnetic traps for atomic ions work
B) was presented / had been well for experiments using a small number of ions
C) would be presented / will be ---- they are completely impractical for large-scale
D) had been presented / has been systems.
E) has been presented / is
A) so long as B) now that C) although
D) in case E) whether
8. In their quest to build a computer that ----
advantage of the weirdness of quantum
mechanics, physicists ---- a number of disparate 15. The trend of increasing carbon dioxide in the
technologies. atmosphere is documented by the examination of
air bubbles trapped in glacial ice ---- by direct
A) is taking / will be testing measurements of the atmosphere.
B) will take / are testing
C) must take / had tested A) as well as B) unless C) because of
D) took / have had to test D) also E) whereas
E) had taken / have tested

37
16. Being a scientist does not prevent one from 24. – 35. sorularda, verilen cümleyi uygun şekilde
participating in other fields of human endeavour, tamamlayan ifadeyi bulunuz.
---- being an artist does not prevent one from
practising science. 24. Though there were daunting technical
obstacles about the Channel Tunnel to be
A) so far as B) rather than C) so that overcome, ----.
D) as well as E) just as
A) the bridge has never been completed
B) these are not nearly as worrying as the costs
17. The book adheres ---- closely to a standard involved
chemistry curriculum. C) England and France were eventually linked by an
under-sea railway
A) seldom B) such C) as D) an army of engineers is involved in the project
D) fairly E) enough E) doubts concerning its safety were still being
expressed
18. It was not obvious to scientists what the
solution would be to the cosmic radiation 25. As ships use less fuel than any other form of
astronauts are exposed to; ---- was it obvious that transport, ----.
there would be any solution at all.
A) shipping might have increased rapidly in this
A) either B) so C) and period
D) but E) nor B) the main environmental impact was on marine life
C) shipping companies have adopted new strategies
to reduce fuel costs
19. – 23. sorularda, aşağıdaki parçada D) ships take on a ballast of water in one port
numaralanmış yerlere uygun düşen sözcük ya da E) they are often regarded as environmentally
ifadeyi bulunuz. friendly

We can certainly hear external sounds while we are


dreaming. Otherwise, a dreamer couldn’t be (19) ---- 26. ---- that seemingly obsolete methods can still
by shouting. Around 40 to 50 per cent (20) ---- dreams work well.
also contain sounds, while touch, smell, taste and
pain are present in a (21) ---- smaller percentage of A) The discovery delighted them
dreams. Sounds occurring near a sleeper (22) ---- is B) The procedures are used for fabricating
already dreaming can be incorporated into the dream. electronic devices
However, the sounds (23) ---- will not cause the C) The problem with copper was
sleeper to dream. D) The lesson to be learned from this positive result
is
19. E) An ingenious solution to the problem emerged
A) ensured B) awakened C) heard
D) embarrassed E) calmed
27. If astronauts have nothing meaningful to do,
----.
20.
A) by B) to C) for A) the two programmes should have been kept quite
D) of E) in separate
B) the programme clearly suffered from
mismanagement
21. C) there is no point in sending them into space
A) too B) more C) much D) they exceeded the estimated cost by several
D) very E) most billion dollars
E) newer proposals may cut the price by half

22.
A) who B) what C) where 28. Once the wind had reached the critical
D) when E) how threshold of 94 miles per hour, ----.

A) the anchor-bolt systems have already weakened


23. B) it took only about 30 seconds for the bridge to
A) which B) themselves C) of whom collapse
D) whatever E) itself C) a basic problem is that of corrosion
D) but the order of collapse was related to the
complex and changing wind directions
E) it is possible to protect structures against the
force of an F-1 tornado

38
29. Unless all countries in the developed world 34. The early Arctic explorers could locate
reduce their fossil fuel consumption drastically, themselves by looking at the stars ----.
----.
A) for the purpose of using the most accurate
A) problems relating to global warming are far too compasses available
numerous B) that they worked out the latitude by using
B) this was not enough to improve fuel efficiency sextants
C) the price of petrol rises steadily C) since longitude is difficult to determine
D) there have been major innovations in personal D) as long as they could determine the exact time
transportation vehicles E) despite matching celestial observations to certain
E) the negative consequences of global warming will points in time
increase

35. Much of the scientific literature on amphibian


30. The search for patterns in the history of life declines focuses on decreases in tropical
builds on the work of generations of countries, ----.
palaeontologists ----.
A) unless larger numbers were involved
A) who went out into the field to dig up fossils B) where losses have been more dramatic
B) that the tools and data are now widely available C) when the imbalance will have to be corrected
C) but the mathematics required was not too D) that organisms may suffer in unpredictable ways
daunting E) which were notorious for fluctuating widely
D) so the database would include 36,000 genera of
marine organisms
E) which caused the extinction of the dinosaurs 36. – 38. sorularda, verilen Đngilizce cümleye
anlamca en yakın Türkçe cümleyi bulunuz.

31. ----, scientists can trace their common 36. Chemistry, as a field of study based on
ancestral genetic connections. scientific principles, came into being in the latter
part of the eighteenth century.
A) It is only in the past 20 years, however
B) Whatever tools and cognitive skills the emigrants A) Bilimsel ilkelere dayalı bir çalışma alanı olarak
had taken with them kimya, on sekizinci yüzyılın son bölümünde
C) Though she was not the only woman alive at the ortaya çıkmıştır.
time B) Çok öncelerden beri var olan kimya, on sekizinci
D) When many different populations are studied yüzyılın son bölümünde bilimsel ilkelere dayalı bir
through a comparison of genetic markers çalışma alanı olarak gelişmiştir.
E) Since the only clues were the sparsely scattered C) On sekizinci yüzyılın son bölümüne doğru ortaya
bones and artefacts our ancestors left behind çıkan kimya, bilimsel ilkelere dayalı bir çalışma
alanıdır.
D) Bilimsel ilkelere dayalı bir çalışma alanı olan
32. The discovery of planets outside our solar kimya, on sekizinci yüzyılın son bölümünde
system is of great importance, ----. gelişmesini tamamlamıştır.
E) On sekizinci yüzyılın son bölümünde ortaya çıkan
A) as individual dust particles are so small bilimsel ilkelere dayalı çalışma alanlarından biri
B) because the triangle of light stretches along the kimyadır.
Sun’s path in the sky
C) since it is the first tangible clue that we may not 37. Certain species of bees and ants exist as
be alone in the universe colonies made up of several different individuals,
D) if one can communicate with extraterrestrial each adapted for some particular function.
beings
E) whether similar collections of worlds surround A) Arıların ve karıncaların belirli türleri, çeşitli
other stars in the galaxy bireyleri içeren koloniler oluşturur ve her koloninin
farklı bir göreve uyum sağladığı bilinmektedir.
B) Koloniler halinde yaşayan bazı arı ve karınca
33. Many species of fish are poisonous to eat, ----. türleri, belli bir göreve uyum sağlamış birkaç
bireyden oluşur.
A) which have a fatally poisonous toxin called C) Farklılaşmış bireylerin oluşturduğu koloniler
tetrodoxin halinde varlıklarını sürdüren bazı arı ve karınca
B) but the most poisonous of all are some kinds of türleri, belli işlevlere uyum sağlamıştır.
fish in the Red Sea and Indo-Pacific region D) Belirli özel işlevlere uyum sağlamış olan arıların
C) whereas the fish’s ovaries, intestines and skin ve karıncaların bazı türleri, çok değişik
contain the poison bireylerden oluşan koloniler halinde yaşar.
D) even if less than 0.1 g of the poison is enough to E) Arıların ve karıncaların bazı türleri, her biri belli
kill an adult in as little as 20 minutes bir işleve uyum sağlamış birtakım değişik
E) since some people think they are also delicious bireylerden oluşan koloniler halinde varlıklarını
sürdürür.

39
38. Contrary to popular belief, it is not Earth’s 41. En eski çağlardan beri kullanılmalarına rağmen
magnetic field that shields people on the ground alaşımlar, modern teknolojide hâlâ vazgeçilmez
from cosmic rays, but rather the bulk of the bir yere sahiptir ve bilim adamları özel nitelikli
tmosphere. yeni alaşımlar geliştirmeye devam etmektedir.

A) Yeryüzündeki insanları kozmik ışınlardan A) Although alloys have been used since the earliest
atmosferin değil daha çok yerkürenin manyetik times, they still have an indispensable place in
alanının koruduğu, yaygın bir yanlış inançtır. modern technology, and scientists continue to
B) Halkın inandığından farklı olarak, yeryüzündeki develop new alloys with special properties.
insanları yalnız atmosferin kalınlığı değil B) Alloys have been in use ever since the earliest
yerkürenin manyetik alanı da kozmik ışınlardan times and are still essential for modern
korumaktadır. technology, but scientists are hard at work to
C) Yaygın inancın tersine, yerküre kozmik ışınlardan develop new alloys with special properties.
kendi manyetik alanından çok atmosferin kalınlığı C) Although the use of alloys goes back to the
sayesinde korunabilmektedir. earliest times, scientists still continue to develop
D) Yaygın inancın tersine, yeryüzündeki insanları new alloys with special properties, which have an
kozmik ışınlardan koruyan, yerkürenin manyetik indispensable place in modern technology.
alanı değil daha çok atmosferin kalınlığıdır. D) Even though alloys were used in ancient times, it
E) Yeryüzünü çevreleyen kalın atmosferin yanı sıra is in modern technology that they have been
yerin manyetik alanının da insanları kozmik most indispensable and, therefore, scientists
ışınlardan koruduğuna yaygın olarak continue to work for the development of new
inanılmaktadır. alloys with special properties.
E) Alloys have continuously been in use since
ancient times and are still of essential importance
39. – 41. sorularda, verilen Türkçe cümleye for modern technology despite the fact that
anlamca en yakın Đngilizce cümleyi bulunuz. 39. scientists continue to develop new alloys with
Ateşin keşfinden sonra, insanlar, yüksek special properties.
sıcaklıklara mâruz kalan belirli kayalar ve
minerallerde değişiklikler fark etmeye başladılar.
42. – 46. sorularda, boş bırakılan yere, parçanın
A) Following the discovery of fire, people noticed anlam bütünlüğünü sağlamak için getirilebilecek
that, under high temperatures, certain changes in cümleyi bulunuz.
rocks and minerals began.
B) After the discovery of fire, people began to notice 42. Engineers are problem solvers. ----. A child
changes in certain rocks and minerals exposed to playing with building blocks who learns how to
high temperatures. construct a taller structure is doing engineering. A
C) When fire was discovered, it was noticed that, secretary who stabilizes a wobbly desk by
because of high temperature, certain changes inserting a piece of cardboard under the short leg
began to take place in rocks and minerals. has engineered a solution to the problem.
D) With the discovery of fire, man became aware of
certain changes which began to take place in A) Certainly, engineers benefit from scientific theory
rocks and minerals due to high temperatures. B) Early in human history, there were no formal
E) After fire was discovered, human beings became schools to teach engineering
aware that, due to high temperatures, certain C) This approach resulted in some remarkable
rocks and minerals began to change. accomplishments
D) In a sense, all humans are engineers
E) Sometimes a solution is required before the
40. Bilgisayarların enerji ihtiyacını azaltmaya theory can catch up to the practice
yönelik stratejilerin bazıları, otomobillerde yakıt
tasarrufunu sağlamak için alınan önlemlere
benzemektedir. 43. Gecko lizards can run up a wall or across a
ceiling with ease because of their remarkable
A) In order to provide fuel efficiency in automobiles, toes. But gecko toes aren’t sticky in the usual
some of the solutions recommended for reducing way, like duct tape or Post-it notes. ----.
the energy needs of computers are being
implemented. A) In spite of this, the ability of geckos to stick to
B) One of the best strategies for reducing the surfaces has attracted scientific scrutiny since the
energy needs of computers has been taken from time of Aristotle
the measures often used to ensure fuel efficiency B) Instead, gecko toes have a combination of
in automobiles. structures that act together as a smarter adhesive
C) Some of the strategies for reducing the energy C) Hence, it is not surprising that scientists are trying
demands of computers are similar to measures to create artificial, geckolike adhesives
taken to ensure the fuel economy of automobiles. D) The theory that gecko toe pads act as suction
D) In order to reduce the energy needs of cups has since been disproven
computers, certain strategies resembling the E) A gecko can stop itself by re-attaching its toes to
measures recommended to provide fuel passing leaves or branches
efficiency in automobiles are being used.
E) The energy needs of computers can easily be
reduced by implementing some of the strategies
recommended for providing fuel efficiency in
automobiles.
40
44. ----. The dimension of length may be described 47. – 51. sorularda, karşılıklı konuşmanın boş
by units of metres, feet, inches, and so forth. bırakılan kısmını tamamlayabilecek ifadeyi
Thus, dimension is an abstract idea, whereas unit bulunuz.
is more specific.
47.
A) The metre is currently defined by the distance Michelle: - It says here that the Russian Space
light traverses in a given length of time Agency has developed a new alternative to
B) Any measuring system must establish base units NASA’s space shuttle.
from which all other units are derived Don: - ----
C) For units of measure to be useful, they must be Michelle: - Kliper, and it seems that it has gained a
standardized so that business transactions are lot of interest from the European Space Agency
unambiguous and Japan.
D) The metre was first defined in 1793 by dividing Don: - Well, let’s hope they get enough money to
the “quadrant of meridian” into 10 million parts get it off the ground.
E) The distinction between dimension and unit is
best understood by example A) Well it’s high time somebody did so.
B) Oh? What’s it called? Has it drawn any scientific
attention?
45. If you have ever burned your finger on a metal C) I wonder if it will be reliable.
pot while waiting for the water in it to boil, you D) Is it as complex as the space shuttle?
know that water heats up much more slowly than E) Really? Will it be able to be re-launched like the
metal. In fact, because of hydrogen bonding, shuttle is?
water has a better ability to resist temperature
change than most other substances. ----.
48.
A) Because of this property, Earth’s giant water Andrew: - This book is about the early history of
supply moderates temperatures, keeping them the computer and the Internet.
within limits that permit life Mark: - ----
B) Temperature and heat are related, but different Andrew: - Actually it is. It places them firmly into
C) Another way water moderates temperatures is by the social background of the period.
evaporative cooling
D) At 66% of your body weight, water helps A) Weren’t early computers more or less
moderate your internal temperature typewriters?
E) Water must absorb an unusually large amount of B) Obviously, much research has gone into it.
heat in order to vaporize because its hydrogen C) All I know about early computers is that they were
bonds tend to hold the molecules in place incredibly large.
D) That doesn’t sound very interesting to me!
E) It’s hard to imagine life without either of them,
46. Replication is not the only way to improve isn’t it?
accuracy in scientific experimentation. ----.
Blocking is a method of experimental design that
reduces the effects of chance errors; modelling, 49.
on the other hand, is much less familiar to Pam: - I can’t understand how anyone could ever
practicing scientists. dream of constructing a bridge to join so distant
an island to the mainland.
A) Accordingly, most scientists try to develop new Sarah: - ----
and more reliable methods Pam: - Really? What?
B) Scientific data always contain a mixture of signal Sarah: - One day, roughly 150 children were
and noise; the scientist’s job is to find the signal drowned when the boat taking them to school was
C) Two other strategies, called blocking and wrecked by storms.
modelling, can provide at least one replication’s
worth of accuracy at almost no cost A) It must have cost those who designed it a lot of
D) Replication is one of the finest ideas in the history sleepless nights!
of science, but it faces a severe law of B) The length is one problem; the weight a more
diminishing returns serious one.
E) Scientists prefer an average of two replicates to a C) It makes one wonder if anything is impossible!
single unreplicated observation because the D) It’s an amazing engineering achievement!
former is likely to be more accurate E) They had a very compelling reason for doing so.

41
50. 54. (I) In his article “The Future Doesn’t Need Us”, the
Alan: - From music sets to cell phones they’re scientist Bill Joy describes advances in three fields:
making everything smaller and smaller. But how? genetic engineering, nanotechnology and robotics. (II)
Joe: - It’s partly due to miniaturized electronics, The first has created the possibility of gene therapy
but they’re making the motors smaller, too. that could bring diseases like cancer under control.
Alan: - ---- (III) These technological advances carry a strong
Joe: - No; the physics principles remain the same. potential for improving our quality of life in the not-too-
The key is design and manufacturing ingenuity. distant future. (IV) The second refers to technologies
that manipulate matter on the extremely small scale of
A) Are the new, smaller motors very different from nanometres, allowing the creation of novel plant
earlier ones? species or new viruses. (V) Finally, robotics will
B) Is it true that MP3 players usually have two eventually raise the possibility of intelligent and self-
motors? replicating machines that are barely distinguishable
C) Do they still turn on small ball or cylinder from humans.
bearings?
D) Well, what’s happening to the prices? A) I B) II C) III D) IV E) V
E) Everything is becoming so small that we shall
soon be unable to find anything!
55. (I) With shipping predicted to increase threefold
within the next 30 years, there are plans for a
51. zeroemissions ferry. (II) It will catch the wind through
Hector: - This article talks about a double-blind computer-controlled sails covered by solar cells to
test for new medication. generate extra electricity. (III) The vessel will have a
Val: - ---- main hull surrounded by four side hulls, cutting drag.
Hector: - Well, it refers to a type of scientific (IV) This will also eliminate the need for ballast water,
testing in which neither the subjects nor the which can have a negative environmental impact. (V)
experimenters know the makeup of the test and Shipping is one of the cheaper ways of transporting
control group during the actual course of the goods across the vast oceans of our planet.
experiments.
Val: - I guess that’s the best way to prevent A) I B) II C) III D) IV E) V
anyone affecting the outcome of the experiment.

A) I’ve already read it. 56. (I) The narwhale has an eight-foot-long spiraled
B) Did you enjoy reading it? tooth that makes it resemble a unicorn of the sea. (II)
C) What kind of medication? Some thought that the whale used it to break arctic
D) I think all medication should be thoroughly tested ice; others theorized that it served as a weapon in
before doctors prescribe it. male fights. (III) Narwhales typically live for 40 to 50
E) What does that mean? years, and seldom leave their arctic habitat. (IV) The
tooth, in fact, may be a giant sensor for navigating
and hunting. (V) It appears capable of detecting
52. – 56. sorularda, cümleler sırasıyla changes in water temperature, pressure, and particle
okunduğunda parçanın anlam bütünlüğünü bozan gradients linked with salinity and prey.
cümleyi bulunuz.
A) I B) II C) III D) IV E) V
52. (I) With the advent of relativity theory, the physicist
Max Born was the first to develop a relativistic theory
of the rigid electron. (II) The theory brought him into
contact with Albert Einstein, first in 1909 and later
during World War I. (III) He and Einstein were to
remain close friends. (IV) Studies in nuclear physics
have had a pattern of staggering progress. (V) Their
correspondence is one of the treasures of 20thcentury
history.

A) I B) II C) III D) IV E) V

53. (I) A compound is a substance containing two or


more elements in a fixed ratio. (II) The smallest unit of
an element having all the characteristics of that
element is an atom. (III) They are much more
common than pure elements in nature. (IV) In fact,
few elements exist in a pure state in nature. (V) Many
compounds consist of only two elements; for nstance,
table salt (sodium chloride, NaCl) has an equal
number of parts of the elements sodium and chlorine.

A) I B) II C) III D) IV E) V

42
57. – 60. soruları aşağıdaki parçaya göre 60. When humans and fruit flies are compared
cevaplayınız. genetically, it is seen that ----.

Over billions of years, life has evolved into a A) there are no similarities between them at all
spectacular diversity of forms – more than a million B) there are many differences which still require
species presently exist. For each, the source of its further explanation
uniqueness is the particular combination of proteins C) human disease genes do not affect fruit flies
found within its cells. Yet in the midst of this diversity, when implanted into them
the similarities between living things are profound. For D) fruit flies are much more prone to suffer from
example, although the fruit fly genome encodes about disease
14,000 different proteins, and humans have two to E) several of their proteins display the same pattern
three times that number, many proteins are still of arrangement and function
recognizably similar in sequence and task, reflecting
their common ancestry. In fact, when scientists have
put human disease genes into flies, they often cause
the same symptoms in the insects as they do in
people. Furthermore, addition of a normal human
gene can sometimes compensate for the deletion of
the same gene from the fly.

57. It is understood from the passage that ----.

A) the proteins encoded in the human genome and


that of the fruit fly bear many similarities
B) all human genes cause disease symptoms in flies
C) humans have the same number of proteins in
their genome as fruit flies
D) humans and fruit flies have no similarities in their
genomes
E) humans and fruit flies are the only species with
innumerable proteins in their genomes

58. It is pointed out in the passage that ----.

A) there are no similarities at all between different


species
B) the combinations of proteins in living beings have
yet to be fully identified
C) the cause of the variety between species cannot
be understood
D) the evolution of life on earth has taken a very,
very long period of time
E) the various species do not share a common
origin

59. It is emphasized in the passage that ----.

A) the huge diversity between the species is rarely


due to the process of evolution
B) species on earth show a great deal of diversity,
but, at the same time, remarkable genetic
resemblance
C) the idea that human genes can be implanted into
flies has aroused much controversy among
scientists
D) the symptoms caused by disease genes can
hardly be specified at first glance
E) the variety of proteins in the fruit fly genome still
needs to be thoroughly explored

43
61. – 64. soruları aşağıdaki parçaya göre 64. One can conclude from the passage that
cevaplayınız. carbon dioxide ----.

Carbon dioxide (CO2), like water and most other pure A) changes into a series of states only when it is
substances, exists in solid, liquid, and gaseous states subjected to unusual levels of pressure
and can undergo changes from one state to another. B) is the most common substance used in the
Solid CO2, however, has an interesting property: at production of dry ice
normal pressures, it passes directly to the gaseous C) requires very high pressure in order to change
state without first melting to the liquid state. This from one state to another
property, together with the fact that this change D) is a substance which, similar to water, can be
occurs at -78°C, makes solid CO2 useful for keeping found in three different states
materials very cold. Because solid CO2 cools other E) must be stored and used at very low
objects and does not leave a liquid residue, it is called temperatures
“dry ice”. As for liquid CO2, it is obtained by putting
carbon dioxide gas under pressure. When liquid CO2
evaporates, it absorbs large quantities of heat, cooling
as low as -57°C. Because of this property, it is often
used as a refrigerant. If the compressed gas from the
evaporating CO2 liquid is allowed to expand through a
valve, the rapidly cooled vapour forms solid carbon
dioxide “snow”. This CO2 snow is compacted into
blocks and is the source of dry ice.

61. It is understood from the passage that liquid


carbon dioxide ----.

A) does not exist at normal pressures, but becomes


available by pressurizing CO2 gas
B) changes into the gaseous state at temperatures
ranging from -57°C to -78°C
C) does not have as many different uses as the
other states of CO2 have
D) is used widely in obtaining solid carbon dioxide
under high pressure
E) never stays stable but soon changes into the
solid state

62. It is clear from the passage that solid carbon


dioxide “snow” ----.

A) is the only form that solid CO2 usually takes under


normal pressures
B) is very effective in refrigeration if it is used in
large quantities
C) has almost the same properties as dry ice
although it leaves liquid residue on objects,
making it a poor regrigerant
D) is formed when the compressed gas obtained
from the evaporation of liquid CO2 expands and,
hence, rapidly cools
E) turns into liquid CO2 when it absorbs heat and,
consequently, melts

63. According to the passage, solid carbon


dioxide ----.

A) differs from liquid carbon dioxide in that it has a


less significant cooling effect
B) changes right away into the gaseous state at
-78°C, without first melting into the liquid form
C) absorbs more heat than the liquid and gaseous
forms
D) is the most common form carbon dioxide takes as
a substance
E) has a wide range of properties that make it
suitable for various uses

44
65. – 68. soruları aşağıdaki parçaya göre 68. It is clear from the passage that ----.
cevaplayınız.
A) the food stored in seeds can serve to germinate
The primary means of reproduction and dispersal for plants, but not to nourish animals
Earth’s most successful plants is seeds, which B) seeds and seed plants have been intimately
develop from the female gametophyte and its connected with the development of human
associated tissues. Seed plants show the greatest civilization
evolutionary complexity in the plant kingdom and are C) plants coming from seeds are more commonly
the dominant plants in most terrestrial environments. found than those coming from spores
Seeds are reproductively superior to spores for three D) flowering plants are extremely diverse
main reasons. First, a seed contains a multicellular, E) the ovules contained in some seeds are
well-developed young plant with embryonic root, protected while those in others are not
stem, and leaves already formed, whereas a spore is
a single cell. Second, a seed contains a food supply.
After germination, the plant embryo is nourished by
food stored in the seed until it becomes selfsufficient.
Because a spore is a single cell, few food reserves
exist for the plant that develops from a spore. Third, a
seed is protected by a resistant seed coat. Like
spores, seeds can live for extended periods of time at
reduced rates of metabolism, germinating when
conditions become favourable.

65. It can be understood from the passage that


----.

A) seeds cannot be dispersed as easily as spores


B) spores, like seeds, develop from a plant’s female
gametophyte and its associated tissues
C) spores contain an adequate food supply within
their single cell
D) spores are a better method of plant reproduction
than seeds
E) seeds are much more complex in structure than
spores

66. The passage, as a whole, ----.

A) shows the superiority of seeds to spores as a


reproductive method for plants
B) focuses on spores and their advantages as a
reproductive method for plants
C) describes the evolutionary complexity of seeds
D) stresses the similarities between seeds and
spores
E) explains the differences in nourishment between
seeds and spores

67. One similarity between spores and seeds


pointed out in the passage is that ----.

A) both of them are self-sufficient


B) they can both stay alive for a very long time,
waiting for a good time to germinate
C) their plants both thrive in terrestrial environments
D) they are both protected by a hard covering
E) they both have multicellular structures

45
69. – 72. soruları aşağıdaki parçaya göre 72. It is clear from the passage that any
cevaplayınız. information physics reveals about our universe
cannot be valid ----.
The most common view among scientists is that
mathematics and physics are quite different. Physics A) as it is impossible for every scientist to agree to it
describes the universe and depends on experiment B) so long as it is not explained mathematically
and observation. The particular laws that govern our C) since it is not always proved logically
universe, such as Newton’s laws of motion, must be D) unless it is confirmed through experiment and
determined empirically and then asserted like axioms observation
that cannot be logically proved, merely verified. E) because the methods used for verification are
Mathematics, on the other hand, is somehow often controversial
independent of the universe. Results and theorems,
such as the properties of the integers and real
numbers, do not depend in any way on the particular
nature of reality in which we find ourselves.
Mathematical truths would be true in any universe.

69. It is suggested in the passage that, unlike


mathematics, physics ----.

A) makes much use of logic in order to reach a


conclusion
B) formulates laws that need not be verified by
experimentation
C) has undergone much development since
Newton’s time
D) is essentially concerned with the world of matter
E) states facts about the universe that are taken for
granted

70. We understand from the passage that, for


most scientists, ----.

A) logical reasoning is as essential as experiment


and observation in any scientific study
B) mathematics and physics are the two fields of
science which have similar scientific concerns
and are, hence, interdependent
C) mathematics, like physics, is also indispensable
for a scientific study of the universe
D) the Newtonian laws have completely altered
man’s perception of the universe
E) physics is essentially empirical, whereas
mathematics is not

71. As pointed out in the passage, the idea that


mathematics and physics differ from each other
----.

A) has often been queried and debated since


Newton
B) is accepted by most scientists
C) has only recently been accepted by the scientific
community
D) is evidence of a prevailing prejudice among
mathematicians and physicists
E) was originally put forward by Newton after he
formulated his laws of motion

46
73. – 76. soruları aşağıdaki parçaya göre 76. The passage makes it clear that NASA wants
cevaplayınız. to return astronauts to the moon ----.

The entire future of human space exploration rests on A) to protect it from further damage from comet and
a patch of lunar ice. For the past two years NASA has meteorite impacts
focused on designing a new crew vehicle and launch B) for a full exploration of atomic hydrogen in the
system that could return astronauts to the moon by lunar soil
2018. The agency’s ultimate goal is to establish a C) in order to build a permanent base there for
permanent lunar base and use it for a human mission space exploration
to Mars. But the grand plan depends on a risky D) so that they can bring the ice found there back to
prediction that NASA will find water ice in a Earth
permanently shadowed crater basin at one of the E) despite the fact that the lunar surface has frigid
moon’s poles. Plentiful ice deposits would be an asset temperatures
for lunar colonists, who could use the water for life
support or convert it to hydrogen and oxygen rocket
fuel. And two orbiters sent to the moon in the 1990s,
Clementine and Lunar Prospector, found evidence of
ice in perpetually shadowed polar areas where
consistently frigid temperatures would preserve the
water carried to the moon by comet and meteorite
impacts. But some scientists have disputed
Clementine’s radar data, and the anomalous neutron
emissions observed by Lunar Prospector could have
been caused by atomic hydrogen in the lunar soil
instead of ice.

73. It can be understood from the passage that


some scientists ----.

A) think that human space exploration should not


continue
B) want to send two orbiters to the moon, called
Clementine and Lunar Prospector
C) disagree with the evidence that seems to show
the existence of water ice on the moon
D) do not believe that comets and meteorites could
possibly have carried water to the moon
E) maintain that a human mission to Mars could not
be successfully launched from the moon

74. According to the passage, in order for humans


to live permanently on the moon, ----.

A) the frigid polar areas would first need to be


artificially heated
B) NASA needs to first prove the existence of water
ice there
C) NASA must first prepare a human mission to
Mars
D) NASA must remove the plentiful ice deposits at
the poles
E) water must be carried there by the two orbiters,
Clementine and Lunar Prospector

75. It is pointed out in the passage that


Clementine and Lunar Prospector ----.

A) were used to establish a permanent lunar base


B) tried to preserve the water carried to the moon by
comet and meteorite impacts
C) will be used as crew vehicles to transport
astronauts to the moon
D) returned faulty data during their exploration of the
moon’s polar regions
E) have, according to some scientists, discovered
traces of ice in the polar areas of the moon

47
77. – 80. soruları aşağıdaki parçaya göre 80. It is clear from the passage that the
cevaplayınız. researchers from Duke University ----.

Despite bacteria’s presence in all parts of the planet, A) do not plan to do any more research into
their diversity in the world’s soils is poorly understood. bacterial diversity
To better understand what makes the organisms B) were more interested in the American Southwest
thrive, Duke University researchers trekked far and than in the Amazon basin
wide to collect a few centimetres of dirt as samples C) have largely focused on the types of bacteria
from 98 locations across North and South America, found in acidic soil
then analyzed each sample for genetic variation. To D) have carried out their fieldwork to throw light
their surprise, the strongest predictor of high diversity upon the causes of bacterial diversity
was neutral pH. The acidic soil of the Peruvian E) had difficulty in trekking during their search
Amazon, for example, harboured far fewer bacterial
species than did the neutral dirt of the arid American
Southwest. “There are a lot of variables that didn’t TEST BĐTTĐ.
turn out to be very important,” says the researcher CEVAPLARINIZI KONTROL EDĐNĐZ.
Robert Jackson, who adds that a more complete
search for different habitats might turn up other
stimulators of diversity, such as carbon abundance.

77. According to the passage, scientists were


surprised that ----.

A) they had to trek to so many different areas to


conduct their research into bacterial species
diversity
B) carbon abundance was revealed to be the most
important predictor of diversity of bacterial
species
C) bacteria is present in all parts of the planet
D) the arid American Southwest is home to many
more species of bacteria than the lush Peruvian
Amazon
E) they would have to perform more complete
research in the future

78. It is understood from the passage that further


research must be carried out ----.

A) because the previous research was not


conducted properly
B) in order to find other indicators of diversity in
bacterial species
C) so that all the bacterial species of North and
South America can be identified
D) to determine exactly the genetic variations of
bacterial species
E) so that scientists can increase the diversity of
bacterial species

79. The passage points out that the best


conditions for diversity of bacteria species ----.

A) so far seem to exist in soil having a neutral pH


B) were found in the Peruvian Amazon region
C) have not yet been discovered
D) have now been fully researched
E) are found in a few centimetres of dirt

48
1. – 18. sorularda, cümlede boş bırakılan yerlere 8. It ---- true that property prices in Ireland and
uygun düşen sözcük ya da ifadeyi bulunuz. Spain ---- by 208 and 150 per cent, respectively,
since 1997.
1. In the 1990s the Belgian government was
involved in numerous scandals that contaminated A) may be / would increase
it with a reputation for incompetence and ----. B) has been / had increased
C) was / increased
A) participation B) despair C) corruption D) could be / would have increased
D) dislike E) certainty E) is / have increased

2. Queen Mary’s attempts to restore Catholicism 9. In Mozambique, the 1992 peace accord that ----
to England during her reign (1553-1558) resulted 15 years of civil war ---- a blanket amnesty for all
in ---- turmoil and much bloodshed. those who had committed war crimes.

A) internal B) reasonable C) stable A) has ended / had mandated


D) arrogant E) versatile B) had ended / would have mandated
C) ended / mandated
D) ends / will mandate
3. In spite of the widespread effects of Christianity E) could have ended / has mandated
on the Anglo-Saxons, they clung ---- to many of
the superstitions and customs from their pagan
past. 10. Many observers predict that as China ---- to
open itself, state control ----.
A) hardly B) firmly C) rapidly
D) fairly E) urgently A) has continued / had eased
B) continues / will ease
C) continued / could have eased
4. In 1968, Bermuda, which used to be a British D) will continue / has eased
colony, was ---- a new constitution and autonomy E) had continued / may have eased
except for foreign relations, defence and internal
security.
11. Although archaeological exploration of Tibet
A) leased B) exploited C) appropriated ----, evidence of civilization in the region ---- back
D) granted E) abolished to at least 4000 B.C.

A) was limited / will have to date


5. The trouble with golf as a hobby is that it ---- too B) is limited / had dated
much of one’s time. C) had been limited / has dated
D) has been limited / dates
A) turns over B) looks for C) gives up E) will be limited / must have dated
D) puts out E) takes up
12. Economists take pride ---- the sophisticated
6. The Council of Europe, the continent’s oldest statistical techniques ---- which they rely to
political organization, was ---- in 1949 to defend, analyze phenomena such as growth rates,
among other things, human rights and the rule of inflation, unemployment, trade and fiscal
law. practices.

A) set up B) pulled through C) set aside A) at / for B) about / from


D) put up E) made out C) over / by D) out of / through
E) in / on

7. The EU ---- that the establishment of the


International Criminal Court ---- a milestone 13. Much can be learned from a study ---- two
achievement in global human rights protection. Medicare bills that are currently working their way
---- Congress.
A) believed / may have represented
B) had believed / has represented A) in / at B) for / into C) to / over
C) has believed / represented D) of / through E) from / for
D) believes / represents
E) would have believed / had represented
14. ---- demand grows for graduate business
degrees, schools are offering their students more
options regarding the length and content of their
programmes.

A) If B) As C) Unless
D) Once E) Whenever

49
15. For more than a decade, economists have 19. – 23. sorularda, aşağıdaki parçada
maintained ---- the dollar was too expensive and numaralanmış yerlere uygun düşen sözcük ya da
its devaluation was unavoidable. ifadeyi bulunuz.

A) whereas B) because C) that The European Parliament is comprised of 626


D) since E) as members. It (19) ---- significantly stronger since its
inception. (20) ----, the Parliament was simply an
advisory body, but its responsibilities were widened
16. Designed as a palace and fortress for the (21) ---- the Single European Act and Treaty of the
Moorish monarchs of Granada, the Alhambra is European Union of 1993. Three major responsibilities
surrounded by a heavily fortified wall ---- a mile in of the Parliament are legislative power, control over
perimetre. the budget and supervision of executive (22) ----. The
European Commission (23) ---- community legislation
A) except for B) apart from C) just as to the Parliament. The Parliament must approve the
D) so long as E) more than legislation before submitting it to the Council for
adoption.
17. In Renaissance England, though the European 19.
influence was strong ---- poetry was concerned, A) became B) had become
the native drama continued to develop and gain C) has become D) would become
popularity. E) becomes
A) as well as B) in that C) so that
D) as far as E) in case 20.
A) Further B) Consequently C) Occasionally
D) Initially E) Accordingly
18. The Symbolists explored the subtle changes in
the human psyche and conveyed them through
symbol and metaphor ---- by direct statement. 21.
A) on B) at C) over
A) including B) rather than C) not so much as D) in E) through
D) such as E) as if

22.
A) decisions B) reasons C) obstacles
D) results E) commodities

23.
A) determines B) requires C) presents
D) recognizes E) approves

24. – 35. sorularda, verilen cümleyi uygun şekilde


tamamlayan ifadeyi bulunuz.

24. As he grew older, ----.

A) he grew less active and less dogmatic


B) he’ll start to interfere less with the affairs of the
company
C) he’s not grown any wiser
D) he now seems determined to assert his authority
E) the more he plays the role of the big
businessman

25. Although Russia’s space technology is


simpler than that of NASA in the US, ----.

A) the Shuttle is grounded for repairs


B) it stil manages to work reliably
C) subsequent flights ran smoothly
D) more than one flight had ended in disaster
E) another one has flown more than 100 missions

50
26. ---- as long as the economy was good and the 31. The European Central Bank can’t raise rates
US remained strong abroad. ----.

A) The American people seem to have ignored A) just as they would have benefited Europe’s
Clinton’s weaknesses in character during his largest economies
presidency B) because the Eurozone’s two largest economies,
B) Clinton became the second president in France and Germany, need lower interest rates
American history to be impeached to spur growth
C) Clinton made history by becoming the first US C) since the European countries were particularly
president to testify in front of a grand jury scared of an approaching crash
D) In the second year of his presidency, Clinton D) while, in Spain, interest rates on mortgages were
faced persistent troubles on the domestic front almost zero
E) Clinton’s overall popularity among Americans E) so long as real estate in Ireland and Spain had
remained high been overvalued by 15 and 13 per cent
respectively

27. ---- that the Celts once dominated the breadth


of Europe from the Black Sea to the Atlantic. 32. ----, Afghanistan is still a country on edge.

A) They will consult little known historians A) As the UN Security Council passed two
B) Some archaeological clues had been discovered resolutions in 1999 and 2000, demanding the
C) The whole question may have sparked an Taliban cease their support for terrorism
academic debate B) Unless President Hamid Karzai had an army of
D) Most of us are unaware of the fact 20,000
E) Most historians will have denied C) Since 50 per cent of voters braved threats of
insurgent attacks to vote in the September
parliamentary elections
28. ---- since the break-up of the Soviet Union. D) Ever since the Taliban seized control of Kabul in
September 1996
A) Millions of Ukrainians have gone abroad in E) Four years after the US and its allies ousted the
search of a living Taliban from power
B) Ukrainian migrants often took on menial jobs
C) Many Ukrainian women had been tempted
overseas by promises of glamorous careers 33. ----, he was well-read in both Latin and Greek,
D) There were Ukrainian immigrants in Western and excelled in swimming and boxing.
Europe working on farms
E) From time to time Ukraine felt threatened A) When Lord Byron entered Cambridge at the age
politically of 17
B) However eager Lord Byron may have been to
fight for the oppressed
29. ---- where you can read a book or meet with C) Since Lord Byron had left England in 1816, never
friends. to return
D) Because, to his contemporaries, Lord Byron
A) Security at the new art museum is provided by seemed more a colourful and scandalous
young artists personality than he was a poet
B) In Paris’ new contemporary art space, the Palais E) As Lord Byron was a fiery rebel, an idealist and a
de Tokyo, there is a salon conventional aristocrat
C) The new contemporary museum is being built for
the French collector François Pinault
D) None of the French contemporary artists enjoys 34. The aid package offered by the G8 may be
international prominence wasted ----.
E) France never paid much attention to its own
contemporary artists, beginning with the A) so the G8 summit in 2005 promised 100 per cent
19thcentury impressionists debt relief to an initial group of 14 countries
B) if the rich world’s governments made expansive
promises about fostering development in Africa
30. The UN insists ----. C) although it had not brought about a significant
transfer of resources
A) even though China’s economic growth must have D) unless there are improvements in the
had a perverse effect on democratization management of public spending in sub-Saharan
B) because China had underinvested in crucial Africa
social services, especially education and public E) but most aid is now conditional on good
health governance and structural reform in sub-
C) that China is under an unconditional obligation to Saharan Africa
prohibit torture and ill-treatment
D) as Korea’s political system is more likely to
experience decay than democracy
E) since civil wars have devastated African
economies, leaving millions dead and millions
more displaced

51
35. Market research worldwide shows that football 38. Undoubtedly, some intelligence services in the
has attracted millions in some countries ----. world know where every single high-ranking
member of the largest terrorist groups is.
A) that there is also a desire for football in China
and Japan A) Dünyadaki en büyük istihbarat örgütleri, bazı
B) where until a few years ago people were not terörist gruplarının üst düzey üyelerinin nerede
interested in it olduğunu kuşkusuz biliyordur.
C) which football is starting to compete with baseball B) Kuşkusuz, bazı istihbarat örgütleri en büyük
in the USA terrorist gruplarının her bir üst düzey üyesinin
D) who will be able to enjoy a fascinating battle in dünyanın neresinde olduğunu bilmek istiyordur.
Europe for the Champions’ League C) Kuşkusuz, en büyük terörist gruplarının üst düzey
E) unless football enters people’s homes through üyelerinin bulunduğu yer dünyadaki bütün
different media, but above all through television istihbarat örgütlerince bilinmektedir.
D) Kuşkusuz, dünyada, bazı terörist gruplarının her
bir üst düzey üyesinin nerede olduğu büyük
36. – 38. sorularda, verilen Đngilizce cümleye istihbarat örgütlerince bilinmektedir.
anlamca en yakın Türkçe cümleyi bulunuz. 36. E) Kuşkusuz, dünyadaki bazı istihbarat örgütleri en
Before privatization, Russia’s oil and steel büyük terörist gruplarının her bir üst düzey
companies were thoroughly criminalized, and üyesinin nerede olduğunu bilmektedir.
output fell day by day.

A) Özelleştirme öncesi, Rusya’da yasadışı işlere 39. – 41. sorularda, verilen Türkçe cümleye
bulaşan petrol ve çelik şirketleri üretimin her gün anlamca en yakın Đngilizce cümleyi bulunuz. 39.
biraz daha düşmesine yol açtı. Önemli olan, siyasi liderlerin uygun gördükleri
B) Özelleştirmeden önce, Rus petrol ve çelik gibi harcama yapmada özgür olup olmadıkları
şirketlerinin tümünde pek çok kanunsuzluk vardı değil, yetkilerine ilişkin sınırların var olup
ve günlük üretimleri giderek düşüyordu. olmadığıdır.
C) Özelleştirmeden önce, Rusya’nın petrol ve çelik
şirketleri tamamen yasadışı işlere giriştiler ve A) The important thing is whether political leaders
üretim günden güne düştü. have freedom to spend as they wish, not whether
D) Rusya’nın petrol ve çelik şirketleri, özelleştirme their powers are limited.
öncesi üretimlerini düşürmüş ve pek çok yasadışı B) Important political leaders are free to spend as
uygulamalarda bulunmuştu. they think fit, but there are limits to their powers.
E) Rus petrol ve çelik şirketlerinde özelleştirmeden C) It is whether political leaders have limited powers,
önce üretim giderek düşüyor ve pek çok not whether they are able to spend money on
kanunsuzluk oluyordu. their own projects, which is important.
D) What is important is not whether political leaders
are free to spend as they see fit, but whether
37. Speaking for 25 nations and bringing together limits exist on their powers.
their considerable voice and influence, the E) Whether political leaders are important or not and
European Union is a leader in global efforts to whether they are free or not to spend as they see
protect human rights. fit, they must be subject to limitations regarding
their powers.
A) Avrupa Birliği, 25 ulus adına konuşmaktadır ve
onların güçlü sesini ve nüfuzunu birleştirerek
insane haklarını korumaya yönelik küresel 40. Çoğu Fransız siyasetçi, Avrupa Birliği
girişimlerin lideri durumuna gelmiştir. Anayasası’nın halk tarafından ezici bir şekilde
B) 25 ulus adına konuşan ve onların güçlü sesini ve reddedileceğini sanmıyordu.
nüfuzunu birleştiren Avrupa Birliği, insan haklarını
koruma amaçlı küresel çabaların bir lideridir. A) The politicians, most of whom were French, were
C) Đnsan haklarını korumaya yönelik küresel not expecting the European Union Constitution to
girişimlerin bir lideri olan Avrupa Birliği, 25 ulus be so overwhelmingly rejected by the people.
adına konuşmaktadır ve onların güçlü sesi ile B) Most French politicians did not think that the
nüfuzunu birleştirmektedir. European Union Constitution would be
D) 25 ulus adına konuşan Avrupa Birliği, onların overwhelmingly rejected by the people.
güçlü sesini ve nüfuzunu birleştirdiği için insane C) The fact that the European Union Constitution
haklarını korumaya yönelik küresel çabalarda was overwhelmingly rejected by the people
lider rolü oynayabilmektedir. surprised most French politicians.
E) Avrupa Birliği, 25 ulus adına konuşma yetkisiyle D) Some French politicians supposed that the
onların güçlü sesini ve nüfuzunu birleştirerek, European Union Constitution would be
insane haklarını korumayı amaçlayan küresel overwhelmingly rejected by the people.
çabalarda bir lider olmuştur. E) The firm rejection of the European Union
Constitution by French politicians came as a
surprise to the people.

52
41. Ukrayna, anayasaya dayalı demokrasi kurma 44. A few years ago the price of a set of
yönünde sağladığı ilerlemeye karşılık olarak, Encyclopaedia Britannica was $1,600. ----. Why did
1995’te Avrupa Konseyi’ne kabul edildi. the price drop to such a great extent?

A) Ukraine, having made considerable progress A) An encyclopaedia is an information good, and its
towards establishing a constitutionally-based production involves collecting information and
democracy, was accepted into the Council of packaging it for use by consumers
Europe in 1995. B) Now you can get a CD version of the
B) Because it had made tremendous progress on its encyclopaedia, along with a dictionary, thesaurus
path to a constitutionally-based democracy, and world atlas, for as little as $80
Ukraine was accepted into the Council of Europe C) The cost of compiling the information for the first
in 1995. copy of an encyclopaedia is huge
C) Ukraine made tremendous progress towards D) The move from book-form encyclopaedias to
establishing a constitutionally-based democracy digital ones decreased the cost of production
after being accepted into the Council of Europe in E) The cost of reproducing the encyclopaedia in
1995. digital format is small
D) Before its acceptance into the Council of Europe
in 1995, Ukraine had already made recognizable
progress towards establishing a 45. Many emergencies begin ambiguously, and it
constitutionallybased democracy. is difficult to understand the difference between,
E) Ukraine was accepted into the Council of Europe for example, a man who is drunk and one who is
in 1995, in recognition of the progress the country ill. ----. What you are likely to see, of course, are
had made towards establishing a constitutionally- other people who, for the same reasons, are also
based democracy. acting as if nothing is wrong.

A) One common way to deal with such dilemmas is


42. – 46. sorularda, boş bırakılan yere, parçanın to postpone action, to act as if nothing is wrong,
anlam bütünlüğünü sağlamak için getirilebilecek and to look around to see how others are
cümleyi bulunuz. reacting
B) We have all heard about crowds panicking
42. At any point in time, some regions of a country because each person leads everybody else to
may experience difficulties while others prosper. overreact
For example, high oil prices will simultaneously C) It was also difficult to tell whether a woman was
benefit oil producers in Texas, but hurt being threatened by a stranger or arguing with
businesses and consumers in northern states like her husband
Vermont that rely heavily on natural gas for D) Despite this fact, we cannot tell at first glance
heating. ----. whether what we see is smoke from a fire or just
steam pouring out of the window
A) Likewise, recessions can affect different parts of E) We would have been embarrassed if we had
the country in different ways reacted as if the situation were an emergency
B) When unemployment is low, firms compete for when it actually was not
workers and wages rise sharply
C) Low employment and high employment have
somewhat different effects on wages 46. Psychologists argue over whether language
D) As a consequence, the greater the differences in influences how people think, but it could affect
unemployment across regions, the higher the half of what they see. The view from the right eye
natural rate of unemployment will be in the is processed in the brain’s left hemisphere, which
country as a whole also seems to handle language. Researchers have
E) What this means is that even if the total found that native English speakers, who have
unemployment rate in the country is fairly normal, separate words for blue and green, are faster at
inflation can still occur distinguishing between these colours when they
appear within their right visual field, in contrast to
people whose language uses the same word to
43. It is straightforward, in principle, to determine indicate either colour. ----.
who is employed: ----. What is more difficult is to
distinguish between those who are unemployed A) Scientists planned to continue the research on
and those who are not in the labour force. these same lines using different colours
B) Most of the world’s languages use a single word
A) These two groups have very different to mean both blue and green
characteristics C) Investigators tested how well the right and left
B) The unemployed are those individuals who do fields of view distinguish between the colours
not currently have a job, but who are actively known in English as blue and green
looking for work D) This leads scientists to question the reason why,
C) The unemployed and employed comprise the in some languages, there is no differentiation
labour force between the two colours
D) Just count the people who are working E) This suggests that for English speakers,
E) That person is classified as not being in the language influences the visual discrimination
labour force between the colours blue and green

53
47. – 51. sorularda, karşılıklı konuşmanın boş 50.
bırakılan kısmını tamamlayabilecek ifadeyi Robert: - Have you seen the French documentary
bulunuz. film March of the Penguins?
Cherie: - ----
47. Robert: - Yes, and I’ve read an article that says that
Publisher: - Academics like to write about it was also really difficult to film because of the
Shakespeare’s plays, you know. rough terrain and the harsh climate.
Bookseller: - ---- Cherie: - Shall we go see it this evening?
Publisher: - What is the public interested in reading
then? A) No, I haven’t.
Bookseller: - Biographies. They want to learn about B) Yes, I have.
his life. C) Not yet, but I’ve heard it’s really fascinating.
D) How much does it cost these days to see a film?
A) That makes sense. Tell me more. E) I don’t think it’s playing in cinemas any longer.
B) I’m sure they do. It earns them a lot of money.
C) Why is that? They don’t sell well, do they?
D) I know they do. But that’s not what the public 51.
wants. John: - It says here that there are some really good
E) Do you publish many of them? plays and musicals on Broadway this season.
Daniel: - ----
John: - Well, instead of mostly revivals of old plays
48. and Andrew Lloyd Webber musicals, they’ve got
Ron: - Why don’t we take a trip this summer? some really great new dramas and musical
Susan: - Oh yes, I’d very much like to visit my comedies.
friend Colleen in Chicago. Daniel: - Let me have a look at that article – maybe
Ron: - ---- we’ll find some that we want to see.
Susan: - So there’ll be plenty for you to do while
Colleen and I are chatting and catching up on old A) Oh? What’s the difference between this season
times. and last season?
B) How long have you been so interested in
A) Have you really? Why didn’t you say so sooner? Broadway theatre, anyway?
B) And I know that the city has become a great C) Too bad we don’t live near New York City.
place for cultural activities. D) Let’s call our friends and see if they want to go
C) Oh, no. Not Colleen! and see something.
D) But there’s nothing to do or see in Chicago! E) I like films better than theatre.
E) Let’s call and book a flight right away! And what
about dates? Shall we say mid-July onwards?
52. – 56. sorularda, cümleler sırasıyla
okunduğunda parçanın anlam bütünlüğünü bozan
49. cümleyi bulunuz.
Mary: - Have you heard the news that Colombia’s
cocaine production has been reduced by onethird 52. (I) The face of education is changing rapidly as a
to one-half? direct result of innovative computer technology. (II)
Roy: - No. How did they manage to do that? Gone are the days of studying repetitive grammar
Mary: - The Colombian government used airplanes exercises from an old copy of A First Aid In English,
to spray weed-killer on areas they suspected were while chalk dust floats in the air. (III) Teachers are
being used to grow coca plants. becoming weary of teaching the same subjects in the
Roy: - ---- same way year after year. (IV) The students of today
are more likely to find themselves in front of a
A) Wow, that should reduce the amount of cocaine computer screen than a black board. (V) As the tools
available on the market, and raise the price as of education change, so does the nature of learning
well. and acquisition of knowledge.
B) Who is the president of Colombia?
C) Would you like to visit Colombia? A) I B) II C) III D) IV E) V
D) Did you know that Colombia has a large wildlife
refuge that has many species of animals and
plants not found anywhere else on earth? 53. (I) Because a play is written to be performed, it
E) They are still having trouble with guerrilla forces uses certain conventions you do not encounter in
and land mines in Colombia, according to this short stories. (II) It contains stage directions that tell
article. the actors how to speak and how to move upon the
stage. (III) Most of the story is presented through
dialogue, the words the characters speak. (IV) When
you read a play do you try to imagine how it would
appear on stage? (V) In addition, it is divided into
short units of action called “scenes” and larger ones
called “acts”.

A) I B) II C) III D) IV E) V

54
54. (I) Labour unions arose in the late 1800s and
early 1900s largely in response to the awful working
conditions in factories. (II) Unions try to increase the
wages of their members in three ways. (III) In garment
factories, iron plants and textile mills, labourers
worked about 14 hours per day, seven days a week.
(IV) The long workweek was not new to those who
had worked on farms, but the working conditions
were. (V) Men, women and children as young as 5
operated clattering machinery so dangerous that
many workers lost their sight, hearing and limbs.

A) I B) II C) III D) IV E) V

55. (I) After World War II, rock music quickly


captivated audiences, especially young audiences,
around the world. (II) It articulated an entire
generation’s frustrations with the traditions and values
of the society of that day. (III) It even influenced hair
styles, dress, language and political and social
behaviour. (IV) Rock is believed to have got its name
from the fifties blues song “There is Good Rocking
Tonight”. (V) Further, it created international heroes
like the Beatles, inspired numerous films and
mushroomed into a multibillion-dollar industry.

A) I B) II C) III D) IV E) V

56. (I) In November 1859, British biologist Charles R.


Darwin published one of the most important and
controversial books ever written. (II) Entitled On the
Origin of Species by Means of Natural Selection,
Darwin’s book was an immediate bestseller. (III)
Darwin’s second point was to propose a mechanism
for evolution. (IV) This book soon made his name
almost synonymous with the concept of evolution. (V)
For this reason, Darwin stands out in history with
people like Newton and Einstein, scientists who
synthesized ideas with great explanatory power.

A) I B) II C) III D) IV E) V

55
57. – 60. soruları aşağıdaki parçaya göre 60. According to the passage, the reason that 5-
cevaplayınız. yearolds say that there are more checkers in a
straight row than in a cluster with the same
The young child’s reliance on visual impressions is number is that they ----.
made clear by an experiment on the conservation of
number. If two rows of checkers are matched one for A) are trying to confuse the experimenter
one against each other, young children will say, B) become confused when the experimenter moves
correctly, that the rows have the same number of the checkers
checkers. If the checkers in one row are brought C) have a counting ability on the same level with
closer together to form a cluster, 5-year-olds say there that of older children
are now more checkers in the straight row – even D) depend on the outward appearance of things to
though no checkers have been removed. The visual understand them
impression of a long row of checkers overrides the E) are frustrated with the experiment and unwilling
numerical equality that was obvious when the to carry on
checkers appeared in matching rows. In contrast, 7-
year-olds assume that if the number of objects was
equal before, it must remain equal. At this age,
numerical equality has become more significant than
visual impression.

57. The passage is mainly concerned with ----.

A) differences in counting ability between 5- and 7-


year-old children
B) the game of checkers
C) the fact that older children are better at playing
checkers than younger children
D) an incorrect assumption made by most 7-
yearolds
E) differences in perception between older and
younger children

58. It can be inferred from the passage that, while


very small children depend more on what they see
to form a concept of their environment, older
children ----.

A) depend more on their intellect


B) never make a mistake when counting objects
C) know that the experimenters are trying to trick
them
D) are more interested in their friends
E) have a better understanding of games

59. By the words “conservation of number” in the


passage is meant ----.

A) numbers should be used sparingly so as not to


run out of them
B) even when the members of a group move their
places, the number of the members remains
constant
C) every child should learn how to play checkers
D) when the members of a group move their places,
the number of members may change
E) the visual impression of a long row of checkers
taking precedence over numerical equality, at
least for very young children

56
61. – 64. soruları aşağıdaki parçaya göre 64. It can be understood from the passage that
cevaplayınız. although Kitty Genovese cried out for help when
she was attacked, ----.
In 1964, Kitty Genovese was murdered outside her
home in New York City late at night. She fought back, A) the police arrived too late to help
and the murder took over half an hour. At least 38 B) the American public disregarded the incident
neighbours heard her screams for help, but nobody C) none of her neighbours helped her
came to her aid. No one even called the police. The D) her neighbours put themselves in danger trying to
American public was horrified by this incident, and help her
social psychologists began to investigate the causes E) her neighbours had to appear in court because
of what at first was termed “bystander apathy”. Their they did nothing to help her
work showed that “apathy” was not a very accurate
term, however. It is not simple indifference that
prevents bystanders from intervening in emergencies.
First, there are realistic deterrents such as physical
danger. Second, getting involved may mean lengthy
court appearances or other entanglements. Third,
emergencies are unpredictable and require quick,
unplanned action; few of us are prepared for such
situations. Finally, one risks making a fool of oneself
by misinterpreting a situation as an emergency when
it is not. Researchers concluded that “the bystander to
an emergency situation is in an unenviable position. It
is perhaps surprising that anyone should intervene at
all”.

61. According to the passage, Kitty Genovese’s


murder is an example of what was at first called
----.

A) a realistic deterrent
B) quick, unplanned action
C) court entanglements
D) misinterpretation of a situation
E) bystander apathy

62. It is clear from the passage that ----.

A) whenever a person tries to offer help in an


emergency, other bystanders will think he is
foolish
B) all people intervening in emergencies must testify
in court
C) the American public likes to get involved in
emergencies
D) there are so many obstacles to intervening in an
emergency that most people are unwilling to do
so
E) simple indifference prevents most bystanders
from intervening in emergencies

63. According to the passage, one thing that


prevents witnesses from getting involved in
emergencies is ----.

A) the fact that they are too busy


B) their distrust of the legal system
C) their surprise and horror at the incident
D) their fear of the police
E) the possibility of physical harm

57
65. – 68. soruları aşağıdaki parçaya göre 67. It is clear from the passage that the new towns
cevaplayınız. and residential areas built after World War II in
Europe ----.
During the “hunger winter” of 1944 in Amsterdam,
over 20,000 people died of starvation. Many of the A) were the work of a very talented group of young
city’s trees were cut down, and the interiors of architects
abandoned buildings broken up for fuel. When peace B) immediately became popular and have remained
came, this once most beautiful and urbane of cities so to this day
was in urgent need of large-scale reconstruction. In C) became the most beautiful areas in and around
the years following the end of World War II in Europe, the cities
modern architecture had an unprecedented D) failed to provide a sense of community for the
opportunity to demonstrate a socially minded, urban residents
style. The consensus today is that in most places it E) had playgrounds designed by van Eyck
failed. The young Dutch architect Aldo van Eyck was
one of the earliest critics of the mechanistic approach
taken by his modernist colleagues to urban 68. According to the passage, Aldo van Eyck ----.
reconstruction. The failure of architecture and
planning to recreate forms of urban community and A) approved of the mechanistic design approach of
solidarity has become a problem in post-war Europe, his modernist colleagues
as so many acclaimed housing estates, new towns, or B) thought that many post-war residential areas
newly designed urban quarters, around Europe, have deserved acclaim
been troubled by vandalism, disrepair and C) viewed the post-war period as an opportunity to
abandonment. Van Eyck saw this coming. In 1947 at display his design ability
the age of 28, he went to work for the Office for Public D) thought the post-war period was a good
Works in Amsterdam and, as his first project, built a opportunity to destroy older buildings and design
small playground. This was in line with his belief that modern cities
by promoting and shaping the daily “encounter” or E) believed that urban architecture had the power to
“inbetween-ness” of social space, architecture could create a sense of solidarity and trust in a city’s
humanize cities and create public trust. residents

65. We understand from the passage that, in the


late 1940s, Europe was in need of massive
reconstruction due to ----.

A) the destruction caused by World War II


B) the mechanistic approach taken by post-war
architects to urban reconstruction
C) vandalism, disrepair and abandonment of
property
D) the unprecedented opportunity to demonstrate a
socially minded, urban style
E) the failure of modern architecture

66. It is stated in the passage that although ----, it


was mostly unable to do so.

A) a small playground was the first project designed


by van Eyck while at the Office for Public Works
in Amsterdam
B) modern European architecture had a chance
after World War II to create an attractive new
style in cities
C) post-war architecture in Europe was greatly
influenced by the ideas of Aldo van Eyck
D) Amsterdam suffered major destruction during
World War II
E) post-World War II architecture in Europe took a
mechanistic approach

58
69. – 72. soruları aşağıdaki parçaya gore 72. As is understood from the passage, in the
cevaplayınız. past, football was not a good example of
globalization because ----.
Not long ago, football was not a good example of
globalization. The labour market in international club A) there weren’t many national football clubs playing
football was highly protected. National leagues like internationally
Italy’s Serie A and Spain’s La Liga imposed quotas on B) most players didn’t want to leave their native
their teams, allowing them to import only a limited country
number of players. Some teams could have only two C) most teams weren’t allowed to have many foreign
foreign players on the field. This arrangement, players
however, began to crumble in 1995, when the D) the number of clubs throughout Europe was
European court ruled that the difference of treatment limited
of nationals from other EU countries was E) foreign players could not cross the borders to
anticonstitutional. This permitted players to move play in other countries
freely within the EU, and made the club teams much
more multi-national. Now it is not unusual for a
majority of the players on a successful league team to
be foreign nationals.

69. It can be understood from the passage that the


quotas imposed in the past by national leagues
----.
A) pushed their teams toward a more global
approach
B) allowed many foreign nationals to enter the
labour market
C) created a closed labour market in international
club football
D) were approved by the European court
E) made the club teams quite multi-national

70. An example of football’s globalization given in


the passage is ----.

A) the large number of foreign players on many


teams today
B) the ruling by the European court in 1995
C) the quotas that many national leagues still
impose on their teams
D) that the Italian teams exchange their players
more frequently than the other teams do
E) that the number of football teams in the EU
countries has increased since 1995

71. According to the passage, the European court


ruling of 1995 ----.

A) created a new constitution


B) resulted in a complete reorganisation of the
European football leagues
C) restricted the number of foreign players a team
could import
D) made mandatory the inclusion of at least two
foreign players from outside the EU on every
national team
E) made it illegal for EU teams to refuse to employ
foreign players simply because they were foreign

59
73. – 76. soruları aşağıdaki parçaya göre 76. It can be inferred from the passage that Beers’
cevaplayınız. desire to educate people about mental health
resulted from ----.
Despite various scientific advances, in the early
1900s the public still did not understand mental illness A) the improvements he saw taking place in mental
and viewed mental hospitals and their inmates with hospitals
fear and horror. Clifford Beers undertook the task of B) his three-year confinement in mental hospitals
educating the public about mental health. As a young C) his having been chained for long periods of time
man, Beers developed a bipolar disorder and was D) his reading of the book A Mind That Found Itself
confined for 3 years in several private and state E) various scientific advances
hospitals. Although chains and other methods of
torture had been abandoned long before, the
straitjacket was still widely used to restrain excited
patients. Lack of funds made the average state
mental hospital – with its overcrowded wards, poor
food, and unsympathetic attendants – a far from
pleasant place to live. After his recovery, Beers wrote
about his experiences in the now-famous book A
Mind That Found Itself (1908), which aroused
considerable public interest. Beers worked
ceaselessly to educate the public about mental illness
and helped to organize the National Committee for
Mental Hygiene. In 1950, this organization joined with
two related groups to form the National Association
for Mental Health. The mental hygiene movement
played an invaluable role in stimulating the
organization of child-guidance clinics and community
mental health centres to aid in the prevention and
treatment of mental disorders.

73. According to the passage, Clifford Beers’


book, A Mind That Found Itself, ----.

A) became popular when it was published, but is not


popular anymore
B) concentrates on the better aspects of life in a
mental hospital
C) did not gain much popularity among the public
D) did little to inform the public about what went on
inside mental hospitals
E) made people more interested in mental health
than they had previously been

74. It can be understood from the passage that, in


the early 1900s, people feared mental hospitals
and mental patients due to ----.

A) their lack of education about mental health


B) the torture and chains that were still in constant
use to cure mental illness
C) the book Clifford Beers had written about his
experiences in mental hospitals
D) the work of the National Committee for Mental
Hygiene
E) the inmates who had escaped from the hospitals

75. It is pointed out in the passage that the part


played by the mental hygiene movement in
founding centres for child guidance and mental
health was ----.

A) unwanted
B) minimal
C) not at all helpful
D) extremely important
E) short-lived

60
77. – 80. soruları aşağıdaki parçaya göre 80. This passage is mainly concerned with ----.
cevaplayınız.
A) the illegal construction of a new archaeological
Each year in the touristic town of Agrigento, Sicily, park in Agrigento, Sicily
hundreds of illegally-built houses are bulldozed by the B) the beautiful ancient temples found in Agrigento,
local government. New construction in Agrigento, Sicily
home to many ancient temples which tourists come to C) the pollution of the archaeological park in
see, has been banned since 1968. In spite of this, Agrigento, Sicily, which began in 1968
hundreds of new and half-built houses can be seen in D) the reasons why illegal construction is still
the hills surrounding the archaeological park. Not only continuing in Agrigento, Sicily
do these buildings spoil the landscape, but many are E) the problem of illegally-built houses in Agrigento,
also unsafe and unsanitary. Some of the people living Sicily
in these buildings pour sewage into the sea and pile
garbage on roadsides since their houses are illegal TEST BĐTTĐ.
and they aren’t allowed to use the city sewage system CEVAPLARINIZI KONTROL EDĐNĐZ.
and garbage service. Several of these houses are
also built on dangerous cliffs, sites that would never
be allowed by Italy’s strict building codes.

77. It is made clear in the passage that pouring


sewage into the sea and piling garbage on
roadsides are examples of ----.

A) the reason these houses are built in the hills


B) compliance with Italy’s strict building codes
C) why the illegally-built houses in Agrigento are
unsanitary
D) the beautification of Agrigento’s archaeological
park
E) a protest against the Italian government

78. According to the passage, the building


prohibition in Agrigento, Sicily, ----.

A) has meant that no new houses have been built


there since 1968
B) has not put a stop to the building of new houses
C) has meant that there are no houses on the
surrounding hills
D) has caused the destruction of many ancient
temples
E) has led to the strict control of new construction

79. It can be inferred from the passage that the


local government in Agrigento, Sicily, does not
want new houses to be built near the
archaeological park because ----.

A) the local government wants to use the land to


build houses for its officials
B) the local government cannot collect taxes from
the owners of these houses as they are built on
public land
C) there is no water, sewage or garbage service
available near the temples
D) new houses spoil the landscape around the
ancient temples
E) the local government is planning to build several
new hotels for tourists on the land

61
1. – 18. sorularda, cümlede boş bırakılan yerlere 9. Rising health insurance costs and harsh social
uygun düşen sözcük ya da ifadeyi bulunuz. stigmas ---- many employers to overlook workers’
mental health needs, but ignoring the problem ----
1. The ---- of wasting in people with AIDS is similar more than addressing it.
to that seen in people who die from starvation.
A) will cause / had cost
A) degree B) solution C) cure B) cause / will have cost
D) reversal E) relief C) are causing / may have cost
D) caused / has cost
E) have caused / should have cost
2. Only a few foods supply ---- amounts of vitamin
D, notably those derived from animals.
10. Various agencies ---- to keep pesticide use
A) decisive B) significant C) compulsive within safe limits, and most consumers feel they
D) previous E) alien ---- on them.

A) were set up / have depended


3. By protecting the global community from B) have been set up / can depend
infection before it strikes, one can ---- save C) would be set up / were depending
millions of lives. D) are being set up / must have depended
E) would have been set up / must depend
A) precisely B) crucially C) potentially
D) preferably E) forcefully
11. Doctors who had been studying longevity ----
an assortment of genes that ---- life span in
4. Normal dietary iron intake cannot usually ---- for different organisms.
iron loss from chronic bleeding.
A) will identify / had influenced
A) compensate B) account C) request B) have identified / will be influencing
D) ensure E) exchange C) identified / will have influenced
D) had identified / could influence
E) identify / could have influenced
5. Recent findings answer key questions about
how tumour cells ---- residence in other parts of
the body. 12. An intracerebral haemorrhage begins abruptly
---- a headache, followed ---- signs of steadily
A) cut down B) pull up C) slow down increasing neurologic losses.
D) wait for E) take up
A) at / through B) from / in C) after / on
D) in / up E) with / by
6. When a tissue is injured, a rapid chain of events
---- the production of fibrin, a stringy, insoluble
mass of protein fibers that forms a clot from liquid 13. ---- anxiety, depression is the most common
blood. psychiatric disorder and typically begins ---- the
20s, 30s or 40s.
A) holds up B) makes up C) breaks down
D) leads to E) puts out A) In / through B) Over / during C) After / in
D) Through / by E) By / for

7. Migraine-patients who go untreated for too long


---- structural changes in their brains, so they ---- 14. Elastin and collagen are proteins ---- form the
proper therapy. spongy material that lies under our skin and gives
it bounce and flexibility.
A) are incurring / insisted on
B) incurred / would insist on A) in which B) which C) whose
C) incur / used to insist on D) whichever E) whatever
D) must incur / would have insisted on
E) may incur / should insist on
15. ---- our bodies are exposed to cold, their first
rule of survival is to maintain the brain and
8. Japanese researchers ---- that tomato juice ---- internal organs at a temperature of 37°C.
prevent emphysema, a smoking-related lung
disease. A) Since B) Although C) When
D) Just as E) Until
A) will say / may have helped
B) said / will have helped
C) say / had helped 16. Any injury forceful ---- to fracture the jaw may
D) have said / could help also injure the cervical vertebrae.
E) had said / will help
A) so as B) as if C) even
D) too E) enough
62
17. According to Vietnamese researchers, the bird 24. – 35. sorularda, verilen cümleyi uygun şekilde
flu virus may have affected hundreds of people, tamamlayan ifadeyi bulunuz.
who have not ---- been diagnosed.
24. Whereas extra training may not accelerate
A) also B) or C) so children’s motor development, ----.
D) yet E) just
A) some of them were then given extra stimulation
B) infants deprived of physical stimulation would
18. Is yawning contagious and, if so, are some of have delayed motor development
us ---- prone to “catching” a yawn ---- others? C) these are the ones who do not have the
opportunity to move about
A) more / than B) not only / but also D) some amount of practice is necessary for
C) either / or D) so / that development to occur normally
E) both / and E) there had been several techniques for comparing
the effects of maturation and learning

19. – 23. sorularda, aşağıdaki parçada


numaralanmış yerlere uygun düşen sözcük ya da 25. ----, which is why it is vital to ensure that it is
ifadeyi bulunuz. healthy.

Light exercise helps to keep brains healthy, at least in A) The liver is located under the ribs on the
rats. University of Florida scientists (19) ---- active and righthand side of the body
sedentary rats, then examined samples (20) ---- their B) Formerly, the liver received little attention
brain tissue. The active animals had less of the C) Liver surgery today has advanced enormously
oxidative damage that had been thought to result from D) The truth is that the liver, as a hard-working
aging and to cause (21) ---- types of mental illness. organ, has always received a great deal of
Mild exercise was enough; the healthier rats had medical attention
access to a spinning wheel, which they used (22) ---- E) The liver is involved in more than 500 body
every day, although they were not forced to do so. functions, from detoxification to hormone balance
The sedentary rats (23) ---- no source of exercise.

19. 26. Many people think that if they have a good


A) tracking B) tracked night’s sleep, ----.
C) will be tracking D) will have tracked
E) track A) they will wake up without an alarm and feel
rested and refreshed
B) sleeping seven hours a night is associated with
20. the lowest mortality risk
A) of B) to C) for C) they are usually drowsy early in the morning even
D) through E) by after a full night’s sleep
D) they would have had no trouble remaining awake
during the day
21. E) medications for insomnia can’t be effective,
A) applicable B) supplementary compared with nonpharmacological approaches
C) healthy D) reasonable
E) various
27. When glucose or fatty acids are limited, ----.

22. A) each antibody is designed to destroy just one


A) hardly B) occasionally C) completely invader
D) abominably E) enviously B) cells are forced to use amino acids for energy
and glucose
C) plant proteins are of lower quality than animal
23. proteins
A) having B) have C) had D) the body would have no storage site for extra
D) have had E) will have had amino acids
E) in a normal, healthy individual, most diseases
never have a chance to get started

63
28. Although some studies have found 33. People with anorexia nervosa imagine that
lowcarbohydrate diets to be effective, ----. they are themselves fat, ----.

A) the obesity epidemic has fueled explosive growth A) even when they are dangerously underweight
in the weight loss industry B) but among female athletes, anorexia nervosa
B) Americans spend as much as $15 billion a year may be as high as 66 per cent
on low-carbohydrate aids and foods C) because family conflicts often play an important
C) few doctors will have recommended role in the development of anorexia nervosa
lowcarbohydrate diets as a healthy way to weight D) as their first dietary objective is to stop weight
loss loss
D) others have found that they offer only short-lived E) although starvation brings other physical
benefits problems such as anaemia and impaired immune
E) weight loss schemes have focused on reduced response
intake of carbohydrates

34. Psychophysiological disorders are physical


29. ---- since it can have an overall negative disorders ----.
impact on a person’s life.
A) since doctors estimate that emotional stress
A) There was a feeling of inadequacy among the plays an important role in some medical problems
team members B) but research in psychophysiology has focused on
B) AIDS may cause depression indirectly such illnesses as asthma and hypertension
C) He began to feel the work was too stressful C) which refer to relationships between specific
D) She put off going to a doctor about the lump illnesses and characteristic attitudes
E) Some people are more ambitious than others D) though researchers have focused on ulcers,
colitis and rheumatoid arthritis
E) in which emotions are believed to play a central
30. Laxatives provide an example ----. role

A) of how drugs can interfere with nutrient 35. Haematocrit is commonly used to diagnose
absorption iron deficiency ----.
B) if they are taken daily over a long period of time
C) when the acidity of the digestive tract was A) in case anemia is characterized by a reduced
affected number of red blood cells
D) since the rate of absorption was excessive B) even though it is an inconclusive measure of iron
E) that stimulate the secretion of digestive juices status
C) even if low haemoglobin values signal depleted
iron stores
31. Symptoms, ----, are similar in viral and D) so long as the body cannot synthesize
bacterial pharyngitis. haemoglobin
E) in view of the fact that other tests are more
A) when gargling can relieve throat discomfort specific in detecting early deficiencies
B) since the mucous membrane may be inflamed
C) if there was a high white blood cell count
D) though the cause is usually viral 36. – 38. sorularda, verilen Đngilizce cümleye
E) which include a sore throat and pain in anlamca en yakın Türkçe cümleyi bulunuz.
swallowing
36. Owing to infertility treatments, the percentage
of twin births in the US has nearly doubled in the
32. Anthropometric surveys in the US and Europe past 20 years.
have shown that short people tend to
overestimate their stature ----. A) Son 20 yılda sayısı ikiye katlanan kısırlık
tedavileri nedeniyle, ABD’de ikiz doğum oranı çok
A) but body image is a person’s mental picture of arttı.
their physical appearance B) ABD’de kısırlık tedavisinde son 20 yılda görülen
B) which took more effort for their height and weight artış, ikiz doğum oranını iki kattan fazla yükseltti.
C) while heavy people often underestimate their C) Kısırlık tedavilerindeki hemen hemen 20 yıl süren
weight artış, ABD’de ikiz doğum oranının ikiye
D) so some individuals perceive themselves as katlanmasına neden oldu.
having lost almost no weight D) Kısırlık tedavilerinden dolayı, ABD’de ikiz doğum
E) since this mental image affects lifestyle behaviour oranı son 20 yılda hemen hemen ikiye katlandı.
E) ABD’de giderek yaygınlaşan kısırlık tedavileri,
son 20 yılda ikiz doğum oranının ikiye
katlanmasına neden oldu.

64
37. Alcohol was used for centuries as an 39. – 41. sorularda, verilen Türkçe cümleye
anaesthetic, but it was difficult to determine the anlamca en yakın Đngilizce cümleyi bulunuz.
correct dosage because doctors could never be
sure how much would be fatal. 39. Tek bir sperm yumurtaya girer girmez, ikinci
bir spermin girmesini engelleyen iki tepki oluşur.
A) Alkol yüzyıllar boyunca anestetik olarak
kullanılmıştır, ancak doğru dozu belirlemek zordu A) When a sperm gets into the egg, the entry of a
çünkü doktorlar ne kadarının öldürücü second sperm should be prevented through two
olacağından asla emin olamıyorlardı. reactions.
B) Yüzyıllar boyunca anestetik olarak kullanılan B) As soon as one sperm enters the egg, two
alkolün doğru dozunu belirlemek çok zordu çünkü reactions occur that prevent a second sperm from
doktorlar ne kadarının öldürücü olacağını asla entering.
bilemiyorlardı. C) Upon the entry of one sperm into the egg, two
C) Yüzyıllar boyunca doktorlar, ne kadarının reactions take place so that the entry of a second
öldürücü olacağını belirlemede zorlanmalarına sperm may be prevented.
rağmen alkolü anestetik olarak kullanmışlardır. D) Just as a sperm enters an egg, two reactions
D) Alkol doktorlar tarafından ne kadarının öldürücü usually occur which aim at preventing a second
olduğunu belirlemedeki zorluklara rağmen sperm from entering.
yüzyıllardır kullanılıyor. E) It is known that two reactions occur for the
E) Alkol yüzyıllar boyunca anestetik olarak prevention of a second sperm’s entry.
kullanılmıştır, fakat doktorların doğru dozu
belirlemesi çok zordu çünkü ne kadarının
öldürücü olabileceğini kimse asla bilemiyordu. 40. Çinko, çeşitli metabolik süreçlerde yer alan
madensel enzimler dahil vücuttaki pek çok
proteinin çalışmasını destekler.
38. Health information on product packages must
emphasize the importance of the total diet and not A) Zinc supports the work of numerous proteins in
exaggerate the role of a particular food in disease the body, including the metalloenzymes, which
prevention. are involved in a variety of metabolic processes.
B) The work of proteins in various parts of the body
A) Ürün paketlerinin üzerindeki sağlık bilgileri diyetin is supported by zinc which also affects the
tümünün önemini yeterince vurgulamıyor ve metalloenzymes that are related to metabolic
genellikle, belirgin bir gıdanın hastalık önleyici functions.
işlevi abartılıyor. C) The work of many proteins in the body and also
B) Ürün paketlerinin üzerindeki sağlık bilgileri ne the metalloenzymes supporting many different
diyetin tümünün önemini küçümsemeli ne de bir metabolic processes interact with zinc.
gıdanın hastalık önlemedeki rolünü abartmalıdır. D) Zinc supports not only the work of several
C) Ürün paketlerinin üzerindeki sağlık bilgileri diyetin proteins in the body but also the metalloenzymes
tümünün önemini vurgulamalı ve belirli bir which play a useful role in various metabolic
gıdanın hastalık önlemedeki rolünü actions.
abartmamalıdır. E) The work of all the proteins in the body as well as
D) Belirli bir gıdanın hastalık önlemedeki rolü the metalloenzymes associated with many
abartılmamalıdır ve ürün paketlerinin üzerinde different metabolic actions, are supported by
diyetin sağlık açısından bir bütün olarak önemli zinc.
olduğu vurgulanmalıdır.
E) Ürün paketlerinin üzerindeki hastalık önleme
konusundaki bilgiler diyetin tümünün önemini göz 41. Pek çok insan, egzersiz yapmanın daha çok
ardı etmemeli ve o ürünün sağlık açısından yemelerine neden olacağını sanır, oysa bu,
önemini abartmamalıdır. tümüyle doğru değildir.

A) Most people are convinced that, should they


exercise, they will eat more, but this is not true at
all.
B) Many people think that exercising will make them
eat more, but this is not entirely true.
C) A lot of people are of the opinion that, through
exercising, they will start to eat much more even
though this is not absolutely correct.
D) A number of people claim that exercising will
make them eat more although this is not the
case.
E) Several people imagine that too much exercising
will lead to too much eating, but this is not quite
correct.

65
42. – 46. sorularda, boş bırakılan yere, parçanın 45. Most researchers believe sleep has some sort
anlam bütünlüğünü sağlamak için getirilebilecek of critical maintenance or restorative effect on
cümleyi bulunuz. neural tissue. ----. It has been found that in some
birds, therefore, only half the brain rests at once.
42. Health food stores and popular magazine The other half stays alert, and the eye it controls
articles advertise a variety of protein stays open against potential risk.
supplements, and people take these supplements
for many different reasons. ----. Dieters take them A) No one is sure why animals sleep
to maintain their bodies’ protein while losing B) Humans are frequently subjected to situations
weight. Women take them to strengthen their where they need to decide when and how much
fingernails. to sleep
C) But this unconscious state has a cost: it makes
A) The “liquid protein” diet, advocated some years animals vulnerable to predators
ago for weight loss, caused deaths in many users D) The first two or three hours of deep sleep seem
B) Protein and amino acid supplements don’t work to be the most vital for humans
these miracles E) The list includes pigeons, ducks, domestic
C) Athletes do not need protein supplements chickens and a few other birds
because muscle work builds muscle
D) Protein supplements are expensive and less
completely digested than protein-rich foods 46. Broken spirits, not bones, may be the worst
E) Athletes take them to build muscle result of falling down. Many older people fall, but
for some, the experience makes them so afraid of
toppling again that their mind impairs their ability
43. Short-term memory actually stores material to walk without trembling or losing balance. ----.
that is needed for short time periods, and it serves
as a work space for mental computation. ----. That A) For example, physicians should encourage
is, information may reside in short-term memory patients who have fallen to walk again
while it is being encoded or transferred into B) Physicians are encouraged to ask patients about
longterm memory. recent falls
C) Similarly, physicians should not assume that
A) Another possible function is that short-term every older person who falls has a neurological
memory may be a way-station to long-term problem
memory D) Thus, they quickly make themselves dependent
B) Some of the best support for these ideas comes on canes or wheelchairs
from experiments on free recall E) This woman was misdiagnosed and was
C) People have other, more effective ways of prescribed medication for Parkinson’s disease
encoding information in long-term memory
D) At the time of recall the last few words presented
are still likely to be in short-term memory 47. – 51. sorularda, karşılıklı konuşmanın boş
E) However, the special memory for language bırakılan kısmını tamamlayabilecek ifadeyi
seems limited to relatively simple sentences bulunuz.

47.
44. The chronic overarousal caused by chronic Alice: - What did you think of the Pelé film?
stressors may contribute to coronary heart Polly: - What struck me was that when he joined
disease (CHD). CHD occurs when the blood his first team he was 17 but looked like a 14-year
vessels that supply the heart muscles are old.
narrowed or closed, blocking the flow of oxygen Alice: - ----
and nutrients to the heart. ----. When oxygen to the Polly: - And that of parasites too.
heart is completely blocked, it can cause a
myocardial infarction. A) In fact, he always was small.
B) That was the result of malnutrition, poor kid.
A) CHD is a leading cause of death and chronic C) It shocked me to learn that until that time he had
illness in the US never had a pair of sports shoes.
B) This can lead to pain, called angina pectoris, that D) Right. And had various health problems,
radiates across the chest and arm including malnutrition.
C) People in high-stress jobs are at increased risk E) I hated the way other teams turned against him.
for CHD
D) High family demands, in addition to a stressful
job, can adversely affect a woman’s
cardiovascular health
E) The physiological response that the body
experiences in the face of a stressor may have a
direct, negative effect on physical health

66
48. 51.
Bill: - Have you read this study linking restless leg Dr. Raymond: - The survey indicates that, in many
syndrome in children to iron deficiency? countries around the world, a great majority of
Debbie: - ---- women breastfeed their newborns.
Bill: - That’s right; many of the children suffering Dr. Simpson: - That may be true; however, in other
from the disease have a parent affected with it, parts of the world, formula-feeding is more
usually the mother. popular than breastfeeding.
Dr. Raymond: - ----
A) Yes, I do. I think it’s a very important study, don’t Dr. Simpson: - No doubt about it. Yet, on the other
you agree? hand, there are valid reasons for not reastfeeding.
B) But I hear that it’s the first study of the disease
involving such a large group of children as A) Actually, I attach as much importance to
subjects. formulafeeding as to breastfeeding.
C) Sure I have; but I think the conclusions presented B) I certainly believe that parents in today’s society
about the genetic factor may be premature. have to coordinate work and family.
D) Not yet; what does it say? C) It is a fact that women throughout the world know
E) Of course I have; the study also mentions that very little about breastfeeding, and they ought to
family history of the disease may be a factor. be advised by health care professionals.
D) I know; still, breastfeeding offers many health
benefits to both mother and infant.
49. E) Clearly, educating fathers could change attitudes
Annie: - I’m really having a problem with my and promote both breastfeeding and
exercise program. After I finish exercising, I have formulafeeding.
trouble breathing and my chest gets tight.
Sue: - Why don’t you see an allergist? It sounds
like asthma. 52. – 56. sorularda, cümleler sırasıyla
Annie: - ---- okunduğunda parçanın anlam bütünlüğünü bozan
Sue: - Still, you should see a doctor to get proper cümleyi bulunuz.
diagnosis and treatment.
52. (I) At one time, scientists believed aging to be not
A) What should I do if I have an asthma attack? just deterioration but an active continuation of an
B) Are you sure? What are the symptoms of organism’s genetically programmed development. (II)
asthma? Once an individual achieved maturity, “aging genes”
C) Asthma? What a relief. I thought I had a heart began to direct its progress toward the grave. (III) If
problem. they remain active long enough, they can also
D) I went to the doctor yesterday to get it checked dramatically enhance the organism’s health and
out. extend its life span. (IV) This idea has been
E) Do you know any websites where I can get more discredited, and conventional wisdom now holds that
information? aging really is just wearing out over time because the
body’s normal maintenance and repair mechanisms
simply wane. (V) Evolutionary natural selection, the
50. logic goes, has no reason to keep these mechanisms
Nancy: - My doctor just told me that he wants me working once an organism has passed its
to have a bronchoscope! Doesn’t that involve a reproductive age.
biopsy?
Chuck: - Not necessarily. It may just mean that he A) I B) II C) III D) IV E) V
needs to have a look inside the airways in your
lungs.
Nancy: - ---- 53. (I) All children are naturally active. (II) If a child is
Chuck: - Well, maybe he wants to finally determine inactive for a long period of time, he or she must be
the cause of your recurring cough and breathing checked by a doctor to determine whether there is a
problem. problem. (III) However, many become overly active
when, for instance, they are anticipating a birthday
A) Actually, in recent weeks, my husband has also party. (IV) Still, such behaviour is markedly different
been coughing a great deal. from true hyperactivity. (V) Hyperactive children have
B) What for? trouble sleeping, they cannot sit still and they act
C) Well I don’t want to have it done. impulsively.
D) Have you ever had the procedure?
E) Will it hurt? A) I B) II C) III D) IV E) V

67
54. (I) Why do some people eat sweet or fatty foods 57. According to the passage, one of the problems
when they’ve heard bad news or feel depressed? (II) caused by Woo Suk Hwang’s false
A study of the link between chronic stress and over- announcements of cloning embryonic stem cells
ating has found that long-term stress overstimulates is that ----.
the adrenal system. (III) The consumption of sugar
and fat counters that response by producing a feeling A) the public may no longer trust scientists to
of calm. (IV) Most people enjoy eating a meal conduct ESC research in an ethical manner
together with friends or family. (V) Besides the B) ESC research has become less promising today
immediate effects of fats and sugar, there is a feeling than it was in the past
of comfort caused by eating many of these foods. C) political problems with ESC research now exist
where none had existed previously
A) I B) II C) III D) IV E) V D) ESCs will now probably never be derived from
cloned human embryos
E) scientists in other fields than ESC research may
55. (I) In half of the 23 developing countries, citizens feel that they, too, can falsify their research
and officials interviewed considered corruption in the findings to gain recognition
health sector to be a major problem. (II) Many even
identified health as the most corrupt sector in the
government. (III) The solution is to enforce existing 58. It can be concluded from the passage that,
rules and hold managers more accountable. (IV) during the past two years, ----.
Systematic audits, clear contracting rules, and
adequate oversight can help prevent corruption in the A) Woo Suk Hwang has done much to further the
sector. (V) In the developing world, going to public effort of ESC research
health clinics means long waits and large gaps B) no further ESC research has been conducted
between diagnosis and treatment. C) ESC research has taken a lot of time and money
D) many revolutionary treatments have been
A) I B) II C) III D) IV E) V developed as a result of ESC research
E) it has been thought that ESC research had made
further progress than it actually had
56. (I) Combat stress may arise when an event,
situation or condition in a fighting zone requires a
soldier to alter his or her behaviour in response to 59. It can be inferred from the passage that when
new demand. (II) As a result, certain situations could Woo Suk Hwang announced that he had cloned
have placed so much strain on an individual that he or human embryonic stem cells, ----.
she could not maintain a normal level of functioning.
(III) This change in behaviour typically presents A) it caused much excitement in the scientific world
cognitive, physiological and emotional challenges. B) he became a much-respected scientist, and
(IV) Such stress is a normal and expected experience remains so to this day
for deployed personnel, and the vast majority of C) the results were immediately discredited by other
soldiers manage it effectively. (V) Many actually scientists
perform better under reasonable levels of stress. D) ESCs had already been derived from cloned
human embryos
A) I B) II C) III D) IV E) V E) his findings were independently confirmed by
other researchers

57. – 60. soruları aşağıdaki parçaya göre 60. We understand from the passage that the
cevaplayınız. majority of scientists in the field of embryonic
stem cell research ----.
Few recent papers in biology have received as much
praise as Woo Suk Hwang’s 2004 and 2005 A) did not feel professionally embarrassed by Woo
announcements of cloning human embryonic stem Suk Hwang’s lie
cells – or fallen as fast into disrepute with the B) were surprised when Woo Suk Hwang’s findings
discovery that they were rank fakes. Embryonic stem were proven to be fake
cell (ESC) research is no less promising today than it C) think that Woo Suk Hwang’s false claims prove
was before Hwang’s deceit was revealed; most that ESC research can never be conducted
investigators continue to believe that it will eventually ethically
yield revolutionary medical treatments. The fact that D) believe that they must deliver results quickly,
no one has yet derived ESCs from cloned human even if they have to falsify them to do so
embryos simply means that the science is less E) are of the belief that, in the end, ESC research
advanced than has been supposed over the past two will provide surprising new medical therapies
years. Still, Hwang has badly undermined the
reputation of a field that already has more than its
share of political and public relations problems. Some
longtime opponents of ESC research will undoubtedly
argue that Hwang’s lies only prove that the
investigators cannot be trusted to conduct their work
ethically, and the public may believe them.

68
61. – 64. soruları aşağıdaki parçaya göre 64. It can be understood from the passage that
cevaplayınız. some mental health professionals believe that ----.

The increasing visibility of homeless mentally ill A) mentally ill individuals should be moved out of
individuals, particularly in large cities, has aroused large cities into the countryside
public concern and prompted a move toward B) the legal system should be changed to allow
reinstitutionalization. However, an important ethical preventive detention of homeless mentally ill
issue is involved. If such people are not readjusting to individuals
society, should they be involuntarily committed to a C) the best way to protect society is to help the
mental hospital? One of the most cherished civil rights mentally ill readjust to mental hospitals
in a democratic society is the right to liberty. It is D) mentally ill individuals should be
essential that any action toward commitment reinstitutionalized only if there is a potential for
safeguard this right. Some experts believe that legal them to harm others
action is warranted only if a person is potentially E) all mentally ill individuals should be in institutions,
dangerous to others. The rare, but highly publicized, even if they may not commit a dangerous act
occasions when a mentally ill person experiencing a
psychotic episode attacks an innocent bystander have
generated fears for public safety. But dangerousness
is difficult to predict. Studies have shown that mental
health professionals are poor at predicting whether a
person will commit a dangerous act. Moreover, the
legal system is designed to protect people from
preventive detention.

61. The passage is mainly concerned with ----.

A) the ethical question of whether it is right to move


homeless mentally ill people into mental hospitals
against their will
B) the dangers of releasing mental patients back
into society
C) the difficulty that mental health professionals
have in predicting violent behaviour in mentally ill
patients
D) the fact that the legal system is designed to
protect people from preventive detention
E) the psychotic episodes experienced by some
mentally ill individuals

62. It is pointed out in the passage that it may be


difficult to involuntarily reinstitutionalize mental
patients who have been released into society
because ----.

A) they may suffer a psychotic episode


B) they may be dangerous to themselves or others
C) the legal system was made to protect people’s
freedom
D) they are not able to readjust to living in society
E) it is first necessary to apprehend and sedate
them

63. The passage makes it clear that the public,


particularly in large cities, ----.

A) believes that homeless mentally ill individuals


should be reinstitutionalized only if they have
previously attacked someone
B) fears being attacked by a homeless mentally ill
person having a psychotic episode
C) wants to help homeless mentally ill individuals
readjust to society
D) fears that they themselves may become mentally
ill if they are approached by a mentally ill
individual
E) thinks that it is wrong to hospitalize any person
against his will

69
65. – 68. soruları aşağıdaki parçaya göre 67. One example given in the passage of the type
cevaplayınız. of drug that could be developed using Riek’s fibril
model is ----.
Researchers are a step closer to understanding how
Alzheimer’s disease takes shape – literally. A sign of A) a drug that destroys the amyloid protein in the
Alzheimer’s is the presence of protein aggregates in final stages of becoming a fibril
the brain known as plaques. They are made up of B) one containing molecules that bond with proteins
various lengths and conformations of the beta amyloid in the brain, thus preventing fibril formation
protein. The proteins link end to end, forming long, C) one that makes the molecules in the brain’s
threadlike structures called fibrils. Now biologist protein aggregates extremely sticky
Roland Riek and his colleagues have constructed a D) a drug that will show scientists the structure of
three-dimensional model of the fibrils based on their other variations of the amyloid protein
own experiments and earlier data published by E) one that will encourage protein bonding in the
others. Riek says the model will help investigators to brain, and therefore promote fibril formation
understand protein structure, which could lead to
better targeted drugs. For example, molecules could
be engineered to act as protein binding partners, thus 68. It is pointed out in the passage that the
interfering with fibril formation. Such a sticky molecule existence of plaques in the brain ----.
could also be used to diagnose the disease early. The
model work might lend insight to other neurological A) is an indicator of Alzheimer’s disease
disorders that involve fibril formation, such as B) results from the conditions created by
Parkinson’s disease. Riek says his group will extend Parkinson’s disease
the three-dimensional work to other variations of the C) shows that no further research into protein
amyloid protein, because it undergoes many aggregates in the brain is necessary
conformational changes on its way to forming a fibril. D) has been proven by the research of Riek and his
“We need to try to trap them in these intermediate colleagues
states”, he explains. E) comes from a drug that interferes with fibril
formation
65. We understand from the passage that Roland
Riek’s model of fibrils ----.

A) has already been extended to include other


variations of the amyloid protein
B) has brought scientists no nearer to a cure for
Alzheimer’s disease
C) may be used to better understand other
neurological disorders involving fibril formation
D) has made Alzheimer’s a fully curable disease
E) is based entirely on research undertaken by his
colleagues

66. According to the passage, Riek and his


colleagues plan to do more research on the
amyloid protein because ----.

A) they want to determine the changes that take


place during fibril formation
B) their model does not fully represent fibrils as they
exist in a patient’s brain
C) there are questions about their previous research
D) it is such a sticky molecule that it is difficult to
work with
E) it is made up of various lengths and
conformations of plaques

70
69. – 72. soruları aşağıdaki parçaya göre 71. The main idea of the passage is that ----.
cevaplayınız.
A) most people fear bats as they are nocturnal
Bats are creatures of the night that are commonly animals
held in fear. At first glance, those fears might seem to B) bats cannot be infected with a virus without
have some medical justification. Long known as immediately suffering any symptoms
vectors for rabies, it is now thought that bats may be C) SARS and other deadly new viruses may have
the origin of some of the most deadly emerging their origins in bats
viruses, including SARS. From research with other D) bats are often kept nearby other animals in Asian
viruses, virologist Linfa Wang of the Australian Animal live-animal markets
Health Laboratory knew bats could get chronic E) virologist Linfa Wang did research on 408 wild
infections from viruses while not getting sick, making bats from China
them ideal carriers for disease. Bats, civets and a
menagerie of other animals were often found caged
near one another in live-animal markets in Asia. So 72. The passage points out that although bats
Wang hypothesized that bats might harbour SARS as have long been understood to be carriers of
well. Wang and his colleagues analyzed blood, throat rabies, ----.
and faecal swabs from 408 wild bats from China.
Genetic analysis revealed five bats, which A) the ones having close contact with humans do
represented three of nine species of horseshoe bats not carry the disease
tested, possessed viruses closely related to SARS. B) there is not much risk of transfer of this disease
They reported last September that the genetic to humans
variation within those coronaviruses was far greater C) they were tested for the disease anyway
than that seen in human or civet SARS. Therefore, D) they have not until recently been thought to be
bats, probably having lived longer with the diseases, carriers of SARS
may be the origin of the coronaviruses seen in other E) they have never undergone genetic analysis to
species. reveal other diseases in their bodies

69. We understand from the passage that since


bats can get chronic infections from viruses
without getting sick, ----.

A) they are not thought to be dangerous to humans


and other animals
B) they must have a well-developed immune system
C) people have a great fear of becoming infected
from them
D) they are perfect hosts for disease
E) their blood and faeces carry no sign of infection,
either

70. According to the passage, Linfa Wang’s study


of wild Chinese bats ----.

A) showed that the coronaviruses carried by some


of the bats had a much greater genetic variation
than seen in human SARS
B) revealed that the bats, while chronically infected
with viruses other than SARS, did not become ill
or suffer from any symptoms
C) has aroused a great deal of controversy among
flu specialists
D) proved that the SARS virus carried by the bats
was the same as that found in civets
E) sparked a movement to reform the live-animal
markets in Asia

71
73. – 76. soruları aşağıdaki parçaya göre 76. We learn from the passage that, as Lustig
cevaplayınız. discovered during her study, unlike the elderly,
the young adults ----.
Much work on aging brains has focused on their
failings, but a new study looks at how they succeed. A) used certain regions of their brain for certain
In a University of Michigan at Ann Arbor report on tasks
which brain regions respond to challenging tasks, B) always tended to avoid challenging situations
researchers found that aging brains function C) showed no great deal of difference between
differently than young brains. Cindy Lustig of Ann tackling easy and difficult mental tasks
Arbor used functional magnetic resonance imaging to D) used all the parts of their brain in responding to
observe the brains of young adults (aged 18 to 30) every mental task
and seniors (65 to 92) as they tackled simple and E) were more interested in the performance of their
difficult mental exercises. For the easy tasks, brain brain than in the causes of mental failure
activity was very similar, but tougher challenges
prompted differences. The seniors activated several
frontal brain regions that the young adults did not. In
addition, the younger people “turned off” parts of the
brain not used during the tasks, but the elders kept
those regions active. Lustig concludes that “older
adults’ brains can indeed rise to the challenge, at
least in some situations, but they may do so
differently”.

73. According to the passage, Lustig’s study


shows that ----.

A) young and aging brains show little difference in


function when they are dealing with simple tasks
B) aging brains cannot cope with a wide range of
tasks, whether simple or challenging
C) young people use their brain capacity much more
fully than elders
D) the reasons why aging brains fail in their activities
are varied and need to be specified
E) various mental exercises can delay the aging of
the brain

74. It is clear from the passage that Lustig’s


principal aim in conducting the study was ----.

A) to determine whether elderly people show the


same reaction when they are faced with a difficult
or an easy situation
B) to carry out her research on a group of young
and elderly people associated with the University
of Michigan at Ann Arbor
C) to understand the difference in how young and
old people use their brain to deal with both simple
and difficult tasks
D) to formulate a series of mental exercises suitable
for her use of magnetic resonance imaging
E) to discover the causes of aging brain failures

75. It is pointed out in the passage that elderly


people ----.

A) perform far better than young people in all kinds


of mental activities
B) use more of the different parts of their brain to
deal with difficult mental tasks than young people
do
C) ought to be always encouraged to do mental
exercises
D) should try hard to improve their mental capacity
and do well in challenging situations
E) have aroused Lustig’s interest and fully
collaborated with her in her research

72
77. – 80. soruları aşağıdaki parçaya göre 79. We learn from the passage that a patient with a
cevaplayınız. score in the high 20s on the Hamilton Depression
Rating Scale ----.
Treating depression could change significantly
following the results of a small Canadian clinical trial A) can easily be treated through medication or talk
that ended in 2005. The procedure used in the study therapy
freed several patients from heavy depression that had B) is rated as the most serious case of depression
resisted medication, talk therapy and even C) experiences negligible activity in the subgenual
electroconvulsive (shock) treatment. Study co-author cingulate
Helen S. Mayberg cautions that any trial so small D) is not considered suitable for the new clinical trial
– just six patients – must be considered provisional. E) usually resists medication and, hence, becomes
Yet four of the six subjects felt dramatic and lasting a hopeless case
effects. University of Toronto neurosurgeon Andres
Lozano implanted battery-powered, pacemakerlike
devices underneath a patient’s clavicle, then ran 80. One can see from the passage that the clinical
flexible, hair-thin electrodes to the subgenual trial undertaken by the Mayberg team ----.
cingulate, a well-buried cortical area that Mayberg
had previously found active in depressive or sad A) is only experimental and needs to be further
states. The electrodes delivered pulses of four volts, tested
130 times a second. Mayberg hypothesized that in B) has completely revealed the secrets of the
badly depressed patients the subgenual cingulated subgenual cingulate
acts like a switch left open, allowing depressive C) has so far received minimal attention in Canada
circuits to fire more than is necessary. Her results D) did not produce positive results for the subjects of
suggest that the regular stimulation might moderate the trial
that activity. In 2005, after a year of living with the E) has been going on for over a decade, producing
continuous impulses, the four patients had lowered its positive results only recently
their scores on the Hamilton Depression Rating Scale
from the soul-deadening high 20s to between one and
eight – quite healthy.
TEST BĐTTĐ.
77. As one understands from the passage, with CEVAPLARINIZI KONTROL EDĐNĐZ.
their clinical trial, Mayberg and her colleagues ----.

A) have concluded that stimulation of the subgenual


cingulate has no effect on depressed patients
B) have conclusively established that depressed
patients can best be treated through the use of
pacemakerlike devices
C) have demonstrated that electroconvulsive
treatment of depressed patients is still the most
effective method
D) have shown that the Hamilton Depression Rating
Scale needs to be re-arranged
E) seem to have made a promising breakthrough in
the therapy of depression

78. It is clear from the passage that, for Mayberg,


----.

A) in a state of depression, the subgenual


cingulated in the brain is affected
B) the results of the clinical trial have aroused much
controversy in the medical world
C) the experiment carried out on four patients has
already transformed the treatment of depression
D) Lozano’s work involves a very complicated
process and needs to be further improved
E) the University of Toronto neurosurgeons, like
Lozano, have always contributed significantly to
the search for effective methods of treatment in
cases of depression

73
1. – 18. sorularda, cümlede boş bırakılan yerlere 7. Some evolutionary biologists argue that if the
uygun düşen sözcük ya da ifadeyi bulunuz. clock of evolution ---- to the beginning and
allowed to run again to the present day, the
1. Marine biodiversity ensures that ecosystems resulting animals on Earth ---- very different from
recover relatively quickly after an accidental or the ones we know now.
natural ----.
A) has been rewound / would have been
A) disturbance B) hesitation B) is rewound / will be
C) encouragement D) dedication C) might be rewound / will have been
E) spectacle D) had been rewound / had been
E) could be rewound / might be

2. According to kinetic theory, the absolute


temperature of a gas is directly ---- to the average 8. Scientists who ---- alert the world to the
kinetic energy of the molecules. existence of a hole in the stratospheric ozone
layer recently reported that this feature of the
A) experimental B) fundamental atmosphere ---- widening soon.
C) negligible D) proportional
E) exceptional A) help / would stop
B) have helped / might have stopped
C) helped / may stop
3. At times during the last Ice Age the North D) will help / might stop
Atlantic thermohaline circulation was ---- weaker E) had helped / has stopped
than it is today.

A) pleasantly B) rarely 9. Meteorites ---- the best available record of the


C) considerably D) directly chemical and physical processes that ---- during
E) fully the first million years of our solar system’s
history.

4. In the 1940s, computer pioneer Konrad Zuse A) provide / occurred


began to ---- that the universe might be nothing B) are providing / have occurred
but a giant computer continually executing formal C) had provided / occurred
rules to compute its own evolution. D) could provide / would occur
E) provided / might occur
A) denounce B) pressurize
C) empower D) evade
E) speculate 10. Today one third of the carbon dioxide (CO )
2
given off by burning fossil fuels ---- the oceans,
thus ---- their naturally alkaline pH.
5. In recent years, carbon dioxide (CO ), a
2
naturally occurring greenhouse gas, has been ---- A) is entering / reduces
as a result of activities such as the burning of B) enters / reducing
fossil fuels and deforestation. C) had entered / will reduce
D) will enter / reduced
A) setting out B) building up E) would enter / having reduced
C) going out D) coming in
E) reaching up
11. Until recently, some scientists ---- that many
individuals of the same species ---- specific tasks
6. The movement of electrons within better than the same number of individuals from
electromagnetic waves ---- some of the wave’s different species.
energy, affecting the properties of the wave and
how it travels. A) will think / are performing
B) were thinking / will perform
A) tells off B) puts in C) think / ought to perform
C) finds out D) uses up D) had thought / would be performing
E) goes around E) thought / could perform

12. Archaeological records show evidence ----


local plants being used as medicine ---- ancient
Egyptian and Stone Age times.

A) about / at B) of / in
C) with / by D) from / for
E) on / to

74
13. The Weddell seal can swim under the ice ---- a 19. – 23. sorularda, aşağıdaki parçada
depth of 500 metres ---- more than an hour without numaralanmış yerlere uygun düşen sözcük ya da
coming up for air. ifadeyi bulunuz.
A) to / between B) in / during
C) at / for D) on / through Among the earliest events in fruit fly development are
E) with / about those that determine which end of the egg cell will
become the head and which end will become the tail.
14. ---- providing energy, proteins provide the raw These events (19) ---- in the ovaries of the mother fly
materials for building the body’s tissues and and involve communication between an unfertilized
regulating its many activities. egg cell and the cells next to it. One of the first genes
activated in the egg cell produces a protein that
A) According to B) As regards leaves the egg cell and signals neighbouring follicle
C) Despite D) In addition to cells. Then these follicle cells (20) ---- to turn on
E) Contrary to genes for other proteins, which signal back to the egg
cell. One of the egg cell’s responses is to localize a
specific type of mRNA at one end of the cell. This
15. The theory posits that Earth’s climate changes RNA marks the end of the egg (21) ---- the fly’s head
---- cyclic variations in the way it orbits the sun. will develop, and thus defines the fly’s head-to-tail
axis. (22) ----, other egg cell genes direct the
A) in place of B) in case of positioning (23) ---- the top-to-bottom and side-to-side
C) so as to D) in view of axes.
E) as a result of

19.
16. Life on Earth would be impossible without A) instruct B) dispel C) embrace
water, ---- all life forms, from bacteria to plants D) identify E) occur
and animals, contain it.
A) since B) even so
C) unless D) that 20.
E) when A) will be stimulated B) stimulate
C) are stimulated D) have been stimulated
E) are stimulating
17. Obtaining nutrients is of ---- vital importance
---- both individual organisms and ecosystems are
structured around the central theme of nutrition, 21.
the process of taking in and using food. A) who B) what C) whom
D) where E) how
A) more / than B) such / that
C) so / as D) much / like
E) either / or 22.
A) On the contrary B) Similarly
C) Nevertheless D) Despite this
18. Butterflies have some characteristics that are E) As a result
---- for professional scientists to understand ----
amateur enthusiasts.
23.
A) easier / than B) the easiest / as A) behind B) to C) about
C) as easy / so D) so easy / that D) of E) at
E) easiest / like

24. – 35. sorularda, verilen cümleyi uygun şekilde


tamamlayan ifadeyi bulunuz.

24. Long before Linnaeus established his system


th
for naming plants in the 18 century, ----.

A) around the world, orchids have long been


symbols of fertility and potency as in the Greek
legend of Orchis
B) the common names of flowers should be highly
evocative or imaginative
C) early attempts at growing orchids had consisted
of placing plants in pots filled with a thick mixture
of rotting wood and leaves
D) many Amazonian orchids are referred to locally
as “monkey love-potions”
E) people throughout the world called plants by their
own inventive names

75
25. While air quality may improve with increased 29. ---- because at these speeds they can propel
biofuel use, ----. the car without using engine power.

A) water quality can suffer due to over-use of A) The two-mode hybrid systems contain two
fertilizers and overdrawn water supplies electric motors surrounding two planetary gear
B) worldwide ethanol demand has pushed up the sets
cost of corn by 25% and sugar by 100% B) At higher velocities, engine power is required
C) meanwhile, fuel crops had increased in value C) The systems can deliver continuous power in the
D) the energy balance of today’s ethanol is positive required amounts
E) 75 million gallons of biodiesel and 4 billion D) Single-mode hybrid automobile systems are more
gallons of ethanol were made last year fuel-efficient at lower speeds
E) Two-mode systems switch between modes
without the driver realizing it
26. When sunlight hits a raindrop, ----.

A) a ray of sunlight actually consists of a mixture of 30. Biological psychology is interdisciplinary by


differently-coloured light nature ----.
B) a typical raindrop is spherical in shape
C) the rainbow is actually a circle which is centered A) now that about half the people who have
on the point that is directly opposite the sun from advanced degrees in psychology will work in
the observer colleges and universities
D) there is a reduction in its speed and this causes B) if our treatment of consciousness reflected both
the light to bend the biological and cognitive perspectives
E) refraction is the bending of light as it passes from C) since it seeks to establish relationships between
one medium to another psychological processes and biological ones
D) just as biological researchers have often
attempted to explain psychological principles in
27. Having taken in more carbohydrates than it terms of biological ones
needs, ----. E) and so cognitive science and cultural psychology
are further examples of this phenomenon
A) sugar can contribute to nutrient deficiencies only
by displacing nutrients
B) the body uses glucose to meet its energy 31. Most viruses cannot survive very long outside
requirements, fills its glycogen stores to capacity, a living host cell, ----.
and may still have some left over
C) researchers agree that unusually high doses of A) although temperate bacteriophages do not
refined sugar can alter blood lipids to favour heart always destroy their hosts
disease B) but the type of attachment proteins on the
D) high-fibre foods not only add bulk to the diet, but surface of a virus determines what type of cell it
are economical and nutritious can infect
E) a high-fat diet raises the risks of heart disease, C) since viruses have several ways to penetrate
some types of cancer, hypertension, diabetes animal cells
and obesity D) so their survival depends to a great extent on
their being transmitted from animal to animal
E) yet under a microscope, most bacteria appear
28. ----, yet the “software programmes”, or genes, similar in size and form
inside our bodies have not changed much in
thousands of years.
32. ----, it is now actually quite a simple matter to
A) Computer software has come down in price by make electrons oppose the “push” of applied
half annually electric and magnetic fields.
B) There is a gene that tells fat cells to hold on to
every calorie in order to protect the body during A) Although this process might have seemed
periods of starvation impossible in the past
C) Scientists are researching new methods to B) Rather than the wave reacting to an individual
overcome the difficulties of gene therapy molecule
D) A human gene is composed of two sets of 23 C) Whether there is a collective response of milions
chromosomes of molecules
E) Our computers and other electronic devices D) Because one wants to understand how negative
typically have their software updated every few refraction can arise
months E) Since much remains to be done to turn such
visions into reality

76
33. Physical activity limits the rise in blood 37. The fact that the majority of the scientific
glucose that would normally occur after a meal world subscribes to a particular view does not
----. make it absolutely right.

A) but aerobic exercise is typically recommended for A) Belirli bir görüş mutlak doğru olmasa da bilim
people who want to lose weight dünyasında çoğunluk tarafından benimsenmiş
B) since research is ongoing in this area olabilir.
C) whereas it takes weeks to months of aerobic B) Bilim dünyasında çoğunluğun aynı görüşe sahip
training to improve physical fitness olması, bu görüşün mutlaka doğru olduğu
D) that it will be required only under certain specific anlamına gelmez.
conditions C) Bilim dünyasının çoğunluğu belirli bir görüşü
E) by making insulin work better in moving glucose mutlak doğru kabul etmiş olsa da gerçek bunun
into muscle tam tersi olabilir.
D) Bilim dünyasının çoğunluğunun belirli bir görüşü
kabul etmesi, bu görüşü mutlak doğru kılmaz.
34. ----, astronomers want to take pictures of E) Belirli bir görüş bilim dünyasında çoğunlukla
galaxies of various ages from infancy to maturity. benimsenmiş olsa bile, mutlaka doğru
olmayabilir.
A) Because Hubble has taken long exposures of
small patches of sky
B) In order to get an idea of what the Milky Way 38. Even if all fossil-fuel power stations worldwide
might have looked like in the past were switched off tomorrow, global temperatures
C) Even though old galaxies were smaller in size would continue to rise for another fifty years.
and more irregular in shape than modern ones
D) As one would expect, if today’s galaxies formed A) Fosil yakıta dayanan enerji santralleri tüm
from the union of several smaller ones dünyada durdurulsa bile küresel sıcaklıkların
E) If the rate of star formation reached its peak hızla artması sorunu en az bir elli yıl daha
around seven billion years ago çözülemeyecektir.
B) Tüm dünyadaki fosil yakıtla çalışan enerji
santralleri yarın kapatılsa bile, bu durum küresel
35. RNA interference, ----, can turn specific genes sıcaklığın artmasını elli yıl daha durduramaz.
off. C) Eğer fosil yakıtlı enerji santralleri tüm dünyada
hemen kapatılabilse, küresel sıcaklık ancak elli yıl
A) that a new technology could be developed daha yükselmeye devam eder.
B) whose ability to understand the brain was D) Fosil yakıt kullanan enerji santralleri yarın tüm
accelerating dünyada kapatılsaydı, küresel sıcaklığın artması
C) just as hypertension in animals is common sadece elli yıl sürerdi.
D) in that nanoparticles can latch onto cancer cells E) Dünyadaki tüm fosil yakıtlı enerji santralleri yarın
E) which scientists have only recently begun to devreden çıkarılsa bile, küresel sıcaklıklar bir elli
understand yıl daha yükselmeye devam edecektir.

36. – 38. sorularda, verilen Đngilizce cümleye 39. – 41. sorularda, verilen Türkçe cümleye
anlamca en yakın Türkçe cümleyi bulunuz. anlamca en yakın Đngilizce cümleyi bulunuz.

36. Most scientists are of the opinion that 39. Darwin’in ileri sürmüş olduğu başlıca
hurricane Katrina, which caused colossal damage düşünceler, bilimdeki pek çok kavram gibi, eski
to the coastal areas of Louisiana in America, was Yunanlılara kadar izlenebilir.
clearly linked with global warming.
A) The main ideas Darwin advanced, like many
A) Pek çok bilim adamı, Amerika’da Louisiana sahil concepts in science, can be traced back to the
bölgelerinde büyük tahribata yol açan Katrina ancient Greeks.
kasırgasının, küresel ısınmanın bir sonucu B) Similar to many concepts in science, the majority
olduğu görüşünü benimsemektedir. of ideas put forward by Darwin are often
B) Çoğu bilim adamı, Amerika’da Louisiana’nın attributed to the ancient Greeks.
C) sahil bölgelerinde çok büyük hasara neden olan C) Like a number of scientific concepts, most of the
Katrina kasırgasının, küresel ısınmayla açıkça ideas suggested by Darwin may have been
bağlantılı olduğu görüşündedir. derived from the ancient Greeks.
D) Pek çok bilim adamına göre küresel ısınmayla D) Many ideas advanced by Darwin can, like the
açıkça bağlantılı olan Katrina kasırgası, majority of concepts in science, be related to the
Amerika’da Louisiana kıyılarında çok büyük ancient Greeks.
tahribata yol açmıştır. E) Like a lot of ideas in science, a great majority of
E) Çoğu bilim adamına göre küresel ısınmayla kesin concepts developed by Darwin are referred to in
ilişkisi olan Katrina kasırgası, en korkunç etkisini the works of the ancient Greeks.
Amerika’nın Louisiana sahillerinde göstermiştir.
F) Birçok bilim adamı, küresel ısınmadan
kaynaklanan kasırgaların Amerika’da
Louisiana’nın sahil bölgelerini vuran Katrina gibi,
büyük hasarlara neden olacağını düşünmektedir.

77
40. Tür olarak varlığımızı sürdürmemiz toprağa 42. – 46. sorularda, boş bırakılan yere, parça-da
bağlıdır; ancak erozyon ve kimyasal kirlilik, bu anlam bütünlüğünü sağlamak için getirile-bilecek
yaşamsal kaynağı tüm dünyada tehdit etmektedir. cümleyi bulunuz.

A) Erosion and chemical pollution throughout the 42. Hippopotamuses can be irritable and
world threaten our survival as a species, which aggressive when it comes to defending their
depends on soil as a vital resource. territory and their young. ----. They have trampled
B) Soil is indispensable for our survival, and yet this or gored people who came too near, dragged
resource of vital importance is threatened by them into lakes, tipped over their boats, and bitten
erosion and chemical pollution worldwide. off their heads.
C) For our survival as a species, we especially
depend on soil, and yet this important resource is A) Hippos are led by dominant males, which can
threatened worldwide by erosion and chemical weigh 6,000 pounds or more
pollution. B) Agricultural irrigation systems and other
D) Our survival as a species depends on soil, and developments have depleted the hippos’ wetland,
yet erosion and chemical pollution threaten this river and lake habitats
vital resource throughout the world. C) Although hippos occasionally fight with
E) Throughout the world, erosion and chemical crocodiles, a growing number of their attacks are
pollution threaten soil, which, as a vital resource, on humans
is indispensable for our survival. D) A decade ago there were about 160,000 hippos
in Africa, but the population has dwindled to
between 125,000 and 148,000 today
41. Đki galaksinin çarpışması, evrenin kütlesine E) In countries beset by civil unrest, where people
hükmettiği sanılan görünmez kara maddenin are hungry and desperate, hippos are hunted for
bugüne kadar elde edilen en iyi kanıtını sağlar. their meat
A) Following the collision of two galaxies, there
appears the best evidence so far known of the 43. The historian G. Sarton said that the
invisible dark matter which is believed to pervade development of mathematics is unknown to the
the mass of the universe. general public. ----. Cayley’s seminal
B) The collision of two galaxies provides the best investigations of matrix algebra were crucial for
evidence yet obtained of the invisible dark matter the development of linear algebra. The terms
assumed to dominate the mass of the universe. matrix, determinant and Jacobian, familiar to most
C) The only evidence so far of the invisible dark science students, were invented by Slyvester.
matter thought to penetrate the mass of the
universe is provided by the collision of two A) Cayley was a Trinity College fellow at Cambridge
galaxies. for a few years until he married
D) It is from the collision of two galaxies that the best B) It isn’t clear when they met, but by 1847 they
evidence yet of the invisible dark matter which is were corresponding to share thoughts about
assumed to hold together the mass of the mathematics
universe has been obtained. C) Each had triumphed on the University of
E) The invisible dark matter which is thought to Cambridge’s fearsome Tripos examinations
dominate the mass of the universe is best D) Certainly very few have ever heard of A. Cayley
understood through the evidence provided by the or J.J. Slyvester, two of the most prolific
collision of two galaxies. mathematicians of the Victorian era
E) J.J. Slyvester was not only a mathematician but
also an enthusiastic poet who called himself the
“mathematical Adam”

44. ----? The answer to that question can range


from days to months to decades on the one
extreme and from centuries to millenia, and
possibly even longer depending on such diverse
and interrelated factors as design, construction
and maintenance.

A) How old is the world-famous Brooklyn Bridge


B) Can a bridge possibly be designed to last a
century
C) How long did London’s Millennium Bridge stay
open
D) The Tacoma Narrows Bridge lasted only four
months before it fell to the wind, didn’t it
E) How long can a bridge last

78
45. Detecting a virus on any nanosize particle 48.
usually means fixing it to a substrate or attaching Tim: - Did you know that NASA is going to send
a fluorescent probe to it, neither of which is another manned mission to upgrade and repair
practical for detecting particles in real time. ----. the Hubble space telescope?
The system splits a laser beam in two, sending Max: - Oh? I thought that, after the 2003 Columbia
one half to a sample. When the light hits a small shuttle disaster, they were going to send manned
particle, it is reflected back and recombined with spacecraft only to the International Space Station.
the reserved half of the laser beam, producing a Tim: - ----
detectable interference pattern only when a Max: - I hope NASA’s taking the proper
moving particle is present. precautions this time.

A) The method works because it relies on the light’s A) The space telescope is deteriorating because of
amplitude rather than its intensity dust and radiation.
B) The investigators have so far detected single B) Well, NASA changed its mind because a robotic
particles as small as seven nanometres across mission has turned out to be impossible.
C) Now physicists have assembled a simple system C) Hubble was first launched into space in 1990. Did
for doing just that you know that?
D) A substrate is a substance that reacts when it D) I learned from this article that Edwin Hubble was
comes into contact with a particular enzyme the first astronomer to describe the expansion of
E) Amplitude is the square root of intensity the universe.
E) The Hubble telescope has sent back thousands
of valuable images. I think it’s worth the mission,
46. Why do young chameleons prefer to stay don’t you?
close to the ground? In a recent study published
in Behavioral Ecology and Sociobiology,
biologists argue that cannibalism in the common 49.
chameleon has resulted in a habitat shift. ----. Mary: - I watched a news report this evening about
Juvenile chameleons tend to stay in low grasses, a man who had started many forest fires.
whereas adults make better use of their Investigators were able to find him by examining
anatomical gifts by living primarily in trees. the areas where the fires had started.
Paul: - ----
A) That is, as individuals develop, their choice of Mary: - No; they looked very carefully, sometimes
habitat changes with a magnifying glass or metal detector, to find
B) With its prehensile tail and strong, opposing toes, the match or other agent that had been used to
the common chameleon is a natural climber set the fire, and then they traced it back to the
C) Young chameleons showed little change in person. It almost always works.
behaviour when with other juveniles
D) The biologists placed a one-way mirror between A) Have you ever been near a forest fire when it
an adult and a juvenile, so that the adult could was burning?
see the juvenile but not the other way round B) A fire last August nearly burnt up my aunt’s home
E) Whether an attack was likely when there was in California. I hope they catch whoever set that
close contact between the generations was also fire, too.
tested C) How could they possibly have done that? Weren’t
all the clues burnt up in the fire?
D) How could they find the place where the fire had
47. – 51. sorularda, karşılıklı konuşmanın boş started?
bırakılan kısmını tamamlayabilecek ifadeyi E) I think people should be very careful with
bulunuz. matches or cigarettes when they are in the forest.

47.
Maeve: - l learned today that there are actually two 50.
types of synapses in an animal’s nervous system. Carol: - Do you know what makes birds’ vision
Charles: - ---- better than ours?
Maeve: - Which type transmits signals faster? Mike: - ----
Charles: - The second, because it sends signals Carol: - Why do they have that ability when
directly, without using a neurotransmitter. humans don’t?
Mike: - I think it’s because early mammals were
A) Oh, really? I only know of one type. active at night, when there’s no ultraviolet light
B) Most people have only heard of chemical from the sun, and so they lost the ability, but birds
synapses. didn’t.
C) Electrical synapses were first found in crayfish in
1957. A) It’s partly because they can see ultraviolet light
D) Yes, chemical and electrical synapses. wavelengths, while humans can’t.
E) Synapses send information from the nervous B) They need to see better in order to determine the
system to the brain, and vice versa. health of a potential mate.
C) It’s impossible for humans to know what birds’
perception of colours is actually like.
D) I think their vision is always strengthened by
ultraviolet light.
E) Insects can also see ultraviolet wavelengths.

79
51. 55. (I) Plant biologists estimate that 25-50% of all
Brenda: - Have you heard of the new Internet plant species are polyploids, that is, having three or
technology that allows people to conduct a search more sets of chromosomes. (II) Hybridization between
for information by entering a photo taken with a two species accounts for most of this polyploidy,
mobile telephone into the search engine? perhaps because the unusually diverse assortment of
Ryan: - ---- genes a hybrid inherits from parents of different
Brenda: - Well, for example, sending a photo of a species can be advantageous. (III) Many of the plants
nearby landmark building might give you a street we grow for food are polyploids, including oats,
map of the area. potatoes, bananas, plums, apples and wheat. (IV)
Ryan: - That would be useful if you were lost in a Cotton, also a polyploid, is the source of one of the
foreign city. world’s most popular clothing fibres. (V) Cotton thread
is made from the long white plumes that extend from
A) I can barely use my mobile to call someone, let the seeds of the plant.
alone to send a picture over the Internet!
B) Who told you that? A) I B) II C) III D) IV E) V
C) What good would that be?
D) Oh, another new technology.
E) Don’t believe everything you read or see on the 56. (I) Migration is a very precise evolutionary
television. adaptation to seasonal changes, but the benefits of
migration are not without cost. (II) Many weeks may
be spent each year on energy-demanding journeys.
52. – 56. sorularda, cümleler sırasıyla (III) Some animals may become lost or die along the
okunduğunda parçanın anlam bütünlüğünü bozan way. (IV) Green turtles migrate more than 2,000
cümleyi bulunuz. kilometres across open ocean between their feeding
area off the coast of Brazil and their nesting place on
52. (I) Are humans the only primates that cry? (II) The Ascension Island. (V) And migrating individuals are
answer depends on how you define “crying”. (III) If it often at greater risk from predators in unfamiliar
is defined as tears coming from the eyes, then the areas.
answer is yes. (IV) Others take a conservative stance
and say that it is too difficult to tell whether or not non- A) I B) II C) III D) IV E) V
human primates have feelings. (V) However, if crying
is vocalization that occurs under the conditions of
distress, then you can find crying in almost all
primates.

A) I B) II C) III D) IV E) V

53. (I) Vertebrate skeletons must be both rigid and


strong. (II) However, there are disadvantages to
having grossly under- or overbuilt bones. (III) Animals
have to balance the needs for strength and stability
against the cost of producing, maintaining and
manufacturing a heavier skeleton. (IV) Consequently,
skeletal size tends to match mechanical requirements
closely. (V) Indeed, limb-bone fractures are relatively
rare.

A) I B) II C) III D) IV E) V

54. (I) The world’s coral reefs are in trouble. (II)


According to an international consortium of scientists
and volunteers, only 30 per cent of reefs are healthy
now. (III) Modern coral reefs as we know them have
been accumulating since the Holocene Epoch 10,000
years ago. (IV) US government agencies,
conservation organizations and other scientists echo
the point. (V) A few go so far as to say that coral reefs
in some areas may be doomed.

A) I B) II C) III D) IV E) V

80
57. – 60. soruları aşağıdaki parçaya göre 59. According to the passage, the lack of
cevaplayınız. subsequent chromosomal damage in mice
brought into the Chernobyl Exclusion Zone from
During our visit in the summer of 1994 to the radiation-free areas proves that ----.
Chernobyl Exclusion Zone, a region within a 30 km
radius of the Chernobyl Nuclear Power Plant, we A) mammals can suffer the effects of radiation and
were amazed by the diversity of mammals living in the still carry a normal embryo
shadow of the ruined reactor only eight years after the B) the radiation found in the mice native to the
meltdown. During our excursion through the woods, Exclusion Zone had compounded with each new
we trapped some of the local mice for examination in generation
a makeshift laboratory. We were surprised to find that, C) the mice native to the Exclusion Zone had,
although each mouse registered unprecedented actually, not developed their immunity to radiation
levels of radiation in its bones and muscles, all the after the explosion occurred
animals seemed physically normal, and many of the D) unprecedented levels of radiation in an animal’s
females were carrying normal-looking embryos. We tissues always signal extensive chromosomal
found that the mice did not have any obvious damage
chromosomal damage. We wondered whether the E) trapping animals is a difficult task, best left to
absence of injury could be explained by some sort of hunters native to the area
adaptive change, perhaps a more efficient DNA-repair
mechanism, after many prior generations had been
exposed to radiation. But when we transplanted wild 60. We understand from the passage that, on their
mice from uncontaminated regions into cages in the visit to the Chernobyl Exclusion Zone, the
Exclusion Zone and then examined their scientists ----.
chromosomes, they were likewise unaffected by the
radiation. In at least this respect, the mice seemed to A) did not expect to find animals that were physically
have a natural “immunity” to harm from radiation. normal
B) themselves began to suffer from exposure to high
57. We see from the passage that the scientists levels of radiation
who visited the Chernobyl Exclusion Zone in 1994 C) mainly wanted to observe the effects of the
concluded that ----. reactor’s meltdown on the surrounding plant life
D) transported mice from the Exclusion Zone to an
A) all mice appear to have inborn protection against uncontaminated area to see if their radiation
the harmful effects of radiation levels would decrease
B) only the mice born in the Exclusion Zone were E) were surprised to find that animals in the
immune to the chromosomal damage caused by Exclusion Zone did not look the same as animals
high levels of radiation from outside the Exclusion Zone
C) mice certainly have better-developed DNA-repair
mechanisms than other animals
D) the meltdown of the nuclear reactor at Chernobyl
caused greater than usual diversity among the
mammals living nearby
E) their makeshift laboratory did not produce valid
results for their experiments with the mice

58. It is clear from the passage that the mice


native to the Chernobyl Exclusion Zone ----.

A) had suffered extensive chromosomal damage


B) were found to have very high radiation levels in
their bodies
C) were not affected by the radiation as much as the
mice which had been brought in from outside the
Exclusion Zone
D) were not put in cages by the scientists studying
them
E) showed less genetic diversity than mice from
other areas

81
61. – 64. soruları aşağıdaki parçaya göre 63. It is pointed out in the passage that, since
cevaplayınız. there may be more ice on one part of the moon’s
surface than on another, ----.
In an attempt to settle the question of whether ice
exists on the moon, NASA plans to launch the Lunar A) a battery-powered vehicle is an essential part of
Reconnaissance Orbiter (LRO) in 2008. Travelling in the probe
a polar orbit only 50 kilometres above the moon’s B) facilities which will examine the ice must be built
surface, the probe will focus a high-resolution neutron near larger ice patches
sensor on the suspected ice deposits to determine C) the search there for ice is expensive and
their precise locations. But because the ice is controversial
probably buried and mixed with lunar dirt, NASA will D) it is essential to test for ice in several different
also need to land a probe to dig up and analyze soil areas
samples. This mission, scheduled for 2011, is a E) it will not be possible to use the ice for future
challenging one because instruments operating in space exploration
shadowed areas cannot use solar power. The craft
could land at a sunlit site and send a battery-powered
vehicle into a dark crater, but the batteries would 64. We see from the passage that the main
quickly die. A radioisotope thermal generator could problem of landing a probe on the moon to test
provide electricity using heat from plutonium decay, for ice in shadowed areas is ----.
but NASA is leaning against this option because it is
expensive and controversial. Another idea under A) that the public is not interested in the project
consideration is sending a probe that could hop from B) the hard, rocky surface of the moon
place to place on the lunar surface by restarting its C) lack of government funding for the project
landing rockets, lifting the craft to 100 metres above D) the extremely cold temperatures the probe would
its original landing site and moving it to another spot have to work in
in the crater basin to hunt for ice. Investigating more E) that it would not be able to use solar power
than one site is crucial because the ice may be
unevenly distributed. Yet another alternative would be
to fire ground-penetrating instruments at several
places in the shadowed basin, either from a lander at
the crater’s rim or from an orbiting craft.

61. It is clear from the passage that ----.

A) firing ground-penetrating instruments at the moon


could upset the balance of its surface
B) there are several options for producing a probe
that could work in the shadowed areas of the
moon
C) NASA will use plutonium decay to provide power
for its newest landing probe
D) the spacecraft that NASA wants to send to the
moon will probably never actually be
manufactured
E) NASA plans only to send a probe to orbit the
moon, not to land on it

62. We understand from the passage that, as part


of an effort to prove the existence of ice on the
moon, NASA ----.

A) will make no use of high-resolution radio


telescopes
B) is currently observing the moon from Earth
C) is planning to send one spacecraft to orbit the
moon and another to land there
D) is going to send a landing craft that will rely solely
on solar power
E) has already sent a spacecraft there to take
pictures

82
65. – 68. soruları aşağıdaki parçaya göre 68. lt is understood from the passage that adult
cevaplayınız. stem cells ----.

Stem cells, unlike all other cells in the body, can copy A) have been manipulated by scientists in order to
themselves indefinitely. So-called adult stem cells are produce new organs
found in many parts of the body, constantly B) are not as well-understood as other types of cells
rejuvenating the brain, remodeling arteries so blood in our bodies
can bypass clogs, and growing new skin to heal C) are always actively engaged in our bodies
wounds. However, adult stem cells have more limited D) will someday be used to regenerate whole
power than embryonic stem cells, which can turn into organs
any type of cell in the body. Indeed, scientists are E) can turn into any other cell type
hoping that embryonic stem cells could be turned into
neurons to fix damaged brains, cardiac cells to repair
damaged hearts, or pancreatic cells to create insulin
for people with diabetes. Maybe they could even be
used to regenerate whole organs. To date, scientists
worldwide have made more than 100 different human
embryonic cell lines. Still, the existing lines have
serious limitations. Most have been grown on a lattice
of mouse embryonic skin cells for support.
Consequently, the human embryonic cells are
contaminated by mouse cells, and though they’re still
useful for research, they cannot at present be used to
develop therapies for humans.

65. According to the passage, the main problem


with the currently existing embryonic stem cell
lines is ----.

A) the fact that they could be turned into neurons


B) the lack of diversity between the different lines
C) that there are not enough of them to develop
therapies useful for treating human diseases
D) that they are contaminated by the mouse cells
upon which they have been grown
E) that they do not produce reliable research results

66. As regards the therapeutic possibilities, the


passage emphasizes the advantages of ----.

A) developing human embryonic stem cells based


on mouse cells
B) embryonic stem cells over adult stem cells
C) human embryonic stem cells over mouse
embryonic skin cells
D) man-made embryonic stem cell lines
E) adult stem cells when used to rejuvenate the
blood

67. We see from the passage that embryonic stem


cells ----.

A) are far less versatile than adult stem cells


B) hold no possibility of being used to cure disease
C) in the past were able to treat illnesses, but cannot
be used for this purpose today
D) might, in the future, be used to treat humans with
damaged brains or hearts
E) cannot reproduce themselves, unlike adult stem
cells

83
69. – 72. soruları aşağıdaki parçaya göre 72. It can be inferred from the passage that ----.
cevaplayınız.
A) atmospheric methane is produced only by human
The concentrations of methane (CH4) and carbon activity
B) the greenhouse effect of methane is not as
dioxide (CO2) gases in the atmosphere have both widely-known as that of carbon dioxide
risen dramatically since the start of the Industrial C) scientists expect atmospheric methane levels to
Revolution. However, unlike its more familiar continue rising
greenhouse-gas cousin, atmospheric methane has D) deforestation contributes to increasing
recently stopped increasing in abundance. This atmospheric methane levels
development wasn’t entirely unanticipated, given that E) carbon dioxide is not as important as methane in
the rate of increase has been slowing for at least a terms of causing global warming
quarter-century. The recent stabilization of methane
levels is something that some scientists are trying
very hard to explain. Methane has many sources.
Some are natural, such as wetlands and plants, and
some are the consequences of modern society, such
as landfills and wastewater treatment. Methane is
destroyed principally by its reaction with the hydroxyl
radical (OH) in the lower atmosphere. One theory
about the stabilization of methane levels is that
deforestation has reduced the number of plants
contributing to atmospheric methane. Another idea is
that an increase in the prevalence of tropical
thunderstorms may have raised the amounts of the
various nitrogen oxides high in the atmosphere.
There, these gases have the side effect of boosting
the production of OH, which in turn acts to destroy
methane.

69. It is pointed out in the passage that methane in


the atmosphere is destroyed primarily by ----.

A) the interventions of scientists


B) the presence of carbon dioxide gas
C) wetlands and plants
D) contact with OH, the hydroxyl radical
E) the Industrial Revolution

70. According to the passage, although


atmospheric carbon dioxide levels continue to
rise, ----.

A) atmospheric methane levels are no longer rising


B) an increase in tropical thunderstorms may reduce
these carbon dioxide levels
C) this rise is expected to level out some time in the
next quarter-century
D) scientists are trying very hard to explain this
increase
E) they are not evenly distributed

71. We understand from the passage that landfills


and wastewater treatment facilities are examples
of ----.

A) natural sources of carbon dioxide


B) ways to boost the production of OH in the
atmosphere
C) man-made sources of methane
D) high levels of atmospheric methane
E) the recent stabilization of methane levels

84
73. – 76. soruları aşağıdaki parçaya göre 76. According to the passage, the main proof
cevaplayınız. given by Luís and Walter Alvarez of a giant meteor
impact that could have destroyed the dinasours
In 1980, the physicist Luís Alvarez and his son Walter was ----.
advanced a startling theory about the demise of the
dinosaurs: that it was caused by forces that came A) the fossil-rich rock of the late Cretaceous period
from beyond this world. They hypothesized that B) a great cloud of dust surrounding Earth
perhaps a meteor impact had ended the age of the C) the 112-mile-wide crater they had discovered
dinosaurs. The primary evidence was that in soil core D) an enormous fireball in the stratosphere
samples taken in locations around the globe, iridium, E) the presence of iridium in soil all over the world
a substance very rare on Earth but prevalent on
asteroids, had been found in a thin layer of clay
separating the fossil-rich rock of the late Cretaceous
period (the end of the dinosaur age) and the sparsely
fossiled rock of the Tertiary period that followed. The
Alvarezes hypothesized that a very large
extraterrestrial object had slammed into the planet,
sending an enormous fireball into the stratosphere,
along with vast amounts of debris. A great cloud of
dust enshrouded Earth, blocking sunlight for months,
even years, and plants and animals perished in the
ensuing cold and dark. When the dust finally settled
back to Earth, it formed the telltale worldwide layer of
iridium in the clay. The scientific world was not
impressed by the theory. Indeed, some scientists
scoffed at the Alvarezes’ hypothesis, but in 1990
scientists realized that a crater of 112 miles in
diametre in Mexico and dated at 65 million years old
might be evidence that the dinosaurs had indeed died
out due to the effects of a giant meteor.

73. It is clear from the passage that, when the


Alvarezes advanced their meteor-impact theory,
----.

A) their focus was mostly on the Tertiary period


B) it was not a surprising idea
C) they didn’t make use of core samples
D) few scientists believed them
E) there were vast amounts of debris in the
stratosphere

74. It is pointed out in the passage that the cloud


of dust caused by the supposed meteor impact
----.

A) caused the fossils of that period to be particularly


easy to extract
B) poisoned the plants and animals living on Earth
at that time
C) made the Earth dark and cold for a very long
time, causing plants and animals to die
D) did not contain iridium
E) formed a very large crater in Mexico when it
settled

75. We understand from the passage that, by the


time of the Tertiary period, ----.

A) the dinosaurs had died out


B) forces from beyond this world had invaded the
planet
C) the dust from the meteor impact had still not
settled
D) fossils were well-preserved
E) the age of the dinosaurs was thriving

85
77. – 80. soruları aşağıdaki parçaya göre 80. It is clear from the passage that ----.
cevaplayınız.
A) Mount Vesuvius is a dying volcano which will
Mount Vesuvius in southern Italy is actually a volcano someday cease to erupt
inside the exploded skeleton of an older volcano. B) the Mediterranean Sea is part of the European
Looked at from above, the remaining ridge of a much continental plate
larger volcano can be seen on the north side. This C) the European continental plate will one day
older volcano had probably erupted violently long completely cover the African one
before human settlement. Southern Italy is unstable D) 3.5 million Italians lost their lives in Vesuvius’s
ground. The African continental plate, on which most last eruption
of the Mediterranean Sea rests, is actually diving E) there was once a much larger volcano where
beneath the European plate. That kind of Mount Vesuvius is today
underground collision produces molten rock, or
magma, rich in volatile gases such as sulfur dioxide.
Under pressure underground, these gases stay
dissolved. But when the magma rises to the surface,
the gases are released. Accordingly, when volcanoes
like Vesuvius erupt, they tend to erupt explosively. To
this day, in fact, Vesuvius remains one of the world’s
most dangerous volcanoes; some 3.5 million Italians
live in its shadow. Although monitoring devices are in
place to warn of the volcano’s activity, if there were a
major eruption with little warning, there could be a
tremendous loss of life.

77. We see from the passage that although Mount


Vesuvius is a very dangerous volcano ----.

A) it is safe to live nearby because of the monitoring


devices that warn of the volcano’s activity
B) many people still live nearby
C) it is more dangerous than the older volcano that
used to be in its place
D) it does not result from an underground collision of
continental plates
E) its eruption would never result in people’s deaths

78. We can understand from the passage that the


pushing of the African continental plate beneath
the European continental plate ----.

A) does not create magma containing sulfur dioxide


and other unstable gases
B) is the result of volcanic activity such as we see in
Southern Italy
C) makes Southern Italy a region prone to volcanic
eruptions
D) has made Northern Africa a “hot spot” for
volcanic activity
E) means that the Mediterranean Sea is slowly
widening

79. We understand from the passage that Mount


Vesuvius’s eruptions are usually very explosive
because of ----.

A) the exploded skeleton of an older volcano within


which it is located
B) the strong skeletal structure of the volcano
C) its proximity to a large body of water
D) the unstable gases released when the volcano’s
magma reaches the surface of the Earth
E) the monitoring devices placed near the volcano

86
th
1. – 18. sorularda, cümlede boş bırakılan yerlere 9. It ---- until the 17 century that military leaders
uygun düşen sözcük ya da ifadeyi bulunuz. began to realize that stress on soldiers ---- a
profound influence on the success of military
1. In their ---- to overcome the anti-social effects of operations.
modern architecture, architects have directed
their attention to more informal settlements. A) had not been / is having
B) is not / has
A) avoidance B) condition C) attempt C) was not / could have
D) involvement E) development D) has not been / had
E) may not be / must have

2. Much of our knowledge of the ---- lives of the


ancient Romans has been derived from the 10. A deeply hypnotized subject ---- to initiate
excavations at Pompeii and nearby Herculaneum. activity and would rather wait for the hypnotist ----
something to do.
A) pretentious B) daily C) complete
D) convenient E) extensive A) does not like / to suggest
B) had not liked / suggesting
C) did not like / should suggest
3. The growing closeness between China and the D) may not like / has suggested
Gulf nations has not gone unnoticed in the rest of E) will not like / to be suggesting
the world, most ---- in the US.

A) similarly B) vaguely C) relatively 11. Ever since James R. Flynn ---- his startling
D) equally E) notably results, psychologists and educators ---- to figure
out whether people really are getting smarter.

4. Language learning can ---- in interesting ways A) has published / had struggled
across different societies and cultural settings. B) published / have struggled
C) had published / will struggle
A) divide B) distract C) vary D) was publishing / had been struggling
D) tend E) dismay E) publishes / are struggling

5. As a family we are used to moving from one 12. Two factors that contribute ---- the Eastern
part of the country to another, and we usually ---- Sierra’s wildflower diversity are its local habitat
pretty quickly in each new home. diversity and the fact that it is the meeting point ---
- three floristic regions.
A) run through B) move round C) turn down
D) come through E) settle down A) over / from B) for / in C) into / at
D) on / about E) to / for

6. The EU recognizes that progress on human


rights around the world ---- the cooperation and 13. ---- the same time as modern humans pushed
collaboration of many groups and individuals. into Europe, some of the same group that had
paused ---- the Middle East spread east into
A) gets into B) makes up C) takes off Central Asia.
D) depends on E) puts off
A) Of / for B) About / in C) With / at
D) During / to E) On / through
7. If they ---- us their plans at the beginning, these
problems ---- us now.
14. Research evidence suggests that, in their
A) showed / would not have been worrying games, girls show preference for home-centered
B) had shown / would not be worrying interests ---- boys are drawn to more naughty and
C) show / will not be worrying dangerous themes and plots.
D) have shown / could not be worrying
E) will show / may not be worrying A) unless B) when C) as though
D) whereas E) now that

8. Most physical anthropologists ---- that modern


human abilities ---- present since the emergence 15. ---- winemaking in France dates back to pre-
of Homo sapiens some 40,000-100,000 years ago. Roman times it was the Romans who spread the
practice.
A) will agree / would be A) Although B) Until C) If
B) could agree / are D) Whether E) In that
C) have agreed / were
D) agree / have been
E) had agreed / must be

87
16. ---- regulate the life of a society, general and 24. – 35. sorularda, verilen cümleyi uygun şekilde
legal rules are set down in written form by the tamamlayan ifadeyi bulunuz.
highest legislative authority of a country.
24. Although the pace of progress in Latin
A) Due to B) In order to America and the Caribbean over the past two
C) With reference to D) Contrary to decades has been impressive, ----.
E) With regard to
A) there remain tremendous development needs in
the region, resulting from poverty and inequality
17. The Mann-Whitney test is a procedure used in B) the EU works with Latin American and Caribbean
nonparametric statistics to determine ---- the nations to advance these goals
means of two populations are equal. C) since 2000, the European Investment Bank has
financed €1.3 billion in projects in the region
A) so far as B) so long as C) while D) Europe and Latin America share historic and
D) since E) whether cultural ties stretching back over 500 years
E) the European Commission Humanitarian Aid
Office has funded disaster relief operations in the
18. France is a large country and, although it has region
over 56 million inhabitants, is ---- densely
populated ---- most of its western European
neighbours. 25. Once rarely found in newsrooms, ----.

A) so / that B) more / as C) less / than A) each radio and television station and daily
D) either / or E) not only / but newspaper will have subscribed to one or more
wire services
B) the news editor will also assign headline sizes to
19. – 23. sorularda, aşağıdaki parçada be written on the various stories as they are
numaralanmış yerlere uygun düşen sözcük ya da edited throughout the night
ifadeyi bulunuz. C) a very small newspaper or radio station may have
a reporting staff consisting of one or two persons
A market economy provides plenty of opportunities to D) the reporter’s task could have been to present
people, but there are risks. Your level of success in a information, not to pass judgment on it
market economy – how much income you earn and E) women now comprise about half of the news-
how much wealth you accumulate – will depend on editorial staffs of America’s daily newspapers
your innate intelligence (19) ---- your efforts. But there
is also an element of luck: Your fate (20) ---- affected
by where you were born, what occupation you 26. As most of the archaeological and
choose, and your genetic makeup and health. There paleontological records consist of bones, ----.
are also chance events, such as natural disasters and
human accidents, that can affect your (21) ----. Given A) people often think of the human skeleton as a
the uncertainty of market economics, most symbol of death
governments have a “social safety net” (22) ---- B) skeletal remains form the basis for most of what
provides for citizens who do not succeed in the we know about human ancestors and our
market economy. The safety net includes evolution
programmes that redistribute income from the rich to C) the pace of skeletal weakening has accelerated
the poor and (23) ---- programmes of support. over the past few millenia
D) it is undoubtedly true that bone resists
19. decomposition better than flesh
A) owing to B) in case of C) by means of E) the bones of modern humans are, on average,
D) as well as E) on behalf of more slender than those of our ancestors

20.
A) was B) is 27. While some of the tapestries at the exhibition
C) had been D) would have been are representations of local life, ----.
E) should have been
A) they were created by individuals without any
21. formal art training
A) account B) demand C) prosperity B) few observers appreciated the colour or the
D) requirement E) schedule creativity of the abstract designs
C) others depict fruit, flowers, trees and the like
22. D) one wonders whether they have taken up to 18
A) what B) of which C) when months to complete
D) that E) wherever E) the exhibition itself will continue until the end of
the year
23.
A) one another B) other C) another
D) each other E) every other

88
28. ---- if it is faced with American-inspired 32. In Britain last year, there were about 5.5
international sanctions. million residents born outside the country, ----.

A) Iran has threatened to disrupt oil supplies A) whatever one means by Britishness, citizenship
B) Tehran would probably have found considerable and integration
foreign support B) since the government denies migrants free
C) Iran’s use of the “oil weapon” in the past had English lessons
serious economic consequences around the C) of whom only about 60 per cent were citizens
globe D) although many migrants work in situations that
D) The Iranians have developed a petro-euro only require their native language
system for oil trading E) while British customs and institutions help people
E) The likelihood of Tehran taking action to to integrate
undermine the US economy has grown stronger

33. Classical musical literacy in much of Europe


29. Though there has been significant today is in decline ----.
improvement in the growth of Africa’s gross
domestic product in recent years, ----. A) because its theory remains far behind the
realities of contemporary music practice
A) inflation eased to single digits, from 10.6% to B) whether the musical world has changed
8.4%, in the previous year C) even though new technology has also affected
B) macroeconomic conditions in the region the music itself
continued to improve in 2006 D) when hundreds of thousands of musicians form
C) the report suggests concrete approaches for amateur symphony orchestras and chamber
employment-based poverty reduction music groups
programmes E) though many countries were giving considerable
D) poverty remains higher there than in other encouragement to young conductors and
developing regions composers
E) West Africa has the lowest unemployment rate of
all
34. The public may be interested in whom
ministers have lunch with, ----
30. ----, the actual pace of change in these
countries has been disappointingly slow. A) regardless of the fact that civil servants are
always at the heart of public decision-making.
A) Since most of the sub-Saharan African countries B) even though the contribution of civil servants to
have good governance policy-making was decreasing in recent years.
B) Even though many sub-Saharan African C) and what are the benefits of greater openness for
governments say they support structural reforms a democratic political system?
C) Unless firmer action is taken against corruption in D) but is it in the public interest for the day-to-day
sub-Saharan Africa activities of ministers to be fully disclosed?
D) In order to improve the management of public E) even if most senior civil servants get paid more
spending in sub-Saharan African countries than academics.
E) In spite of the IMF’s demand that countries at a
similar stage of development should receive
equal treatment 35. It is generally felt that members of the white
working class in Britain express racist sentiments
----.
31. France’s biggest problem is mass
unemployment, ----. A) or there may be disagreements and tensions
between different groups of people
A) since students are protesting against a new, B) since they are the people most directly affected
more flexible job contract by Asian immigration
B) if employers transform permanent jobs into less C) if the class hierarchy is internalized by each
secure ones member of society
C) which had been outside the control of the D) unless they want to be respected on their own
government for at least two decades terms
D) as long as the government continues to support E) whether or not they have control over their own
the heavily-protected high-level workers lives
E) which has continued for more than two decades

89
38. Although the British gained control of the
36. – 38. sorularda, verilen Đngilizce cümleye Alabama region in 1763 with the Treaty of Paris,
anlamca en yakın Türkçe cümleyi bulunuz. they had to cede almost all of the region to the US
and Spain after the American Revolution.
36. With the exception of its southwest, a large
part of Afghanistan is covered by high snow- A) 1763 Paris Antlaşması’yla Alabama bölgesinin
capped mountains and divided by deep valleys. kontrolü Đngilizlere geçti; ancak, Amerikan
Devrimi’nden sonra bölgenin tümü Birleşik
A) Afganistan’ın güneybatısı hariç büyük bir bölü- Devletlere ve Đspanya’ya bırakıldı.
mü, zirveleri karla örtülü yüksek dağlarla kaplıdır B) Đngilizler, 1763 Paris Antlaşması’yla kontrolünü
ve derin vadilerle bölünmüştür. ele geçirdikleri Alabama bölgesinin hâkimiyetini,
B) Afganistan’ın birçok bölgesinde olduğu gibi Amerikan Devrimi’nden sonra, Birleşik Devletlere
güneybatısında da karla kaplı yüksek dağlar ve ve Đspanya’ya bırakmak zorunda kaldılar.
derin vadiler büyük bir yer tutar. C) 1763 Paris Antlaşması’ndan sonra Birleşik
C) Afganistan’da güneybatı kesimler dışındaki Devletler ile Đspanya’nın hakimiyeti altında olan
bölgelerin çoğunu, karlı yüksek dağlar kaplar ve Alabama bölgesinin kontrolü daha önceden
derin vadiler böler. Đngilizlerin elindeydi; ancak, bu durum sadece
D) Karlı yüksek dağlar ve derin vadiler, güneybatısı Amerikan Devrimi’ne kadar sürdü.
dışında Afganistan’ın büyük bir bölümünü kaplar. D) Alabama bölgesinin kontrolü 1763’te imzalanan
E) Afganistan, güneybatısının büyük bir bölümü Paris Antlaşması’yla tümüyle Đngilizlere geçmiştir;
dışında, tepeleri karlı yüksek dağlarla ve boydan fakat bölgeye Amerikan Devrimi sonrasında
boya uzanan derin vadilerle kaplıdır. Birleşik Devletler ve Đspanya hakim olmuştur.
E) Đngilizler 1763’te Paris Antlaşması’yla Alabama
bölgesinin kontrolünü ele geçirmelerine rağmen,
37. The writers of the Romantic movement Amerikan Devrimi’nden sonra bölgenin hemen
generally regarded humans as inherently good hemen tümünü Birleşik Devletlere ve Đspanya’ya
but corrupted by society and its institutions. bırakmak zorunda kaldılar.

A) Romantik akımın yazarları, çoğunlukla, doğuştan


iyi olan insanın toplumdaki kurumlar tarafından 39. – 41. sorularda, verilen Türkçe cümleye
bozulduğunu kabul ederler. anlamca en yakın Đngilizce cümleyi bulunuz.
B) Romantik akımın yazarları, insanların doğuştan
iyi olduğunu ancak toplumun kurumlarının 39. 2000 seçimlerinden sonra Başkan George W.
genellikle onları değiştirdiğini benimsemişlerdir. Bush her ne kadar ülke içi sorunlara odaklanmış
C) Romantik akım yazarlarının, tüm insanların olarak Beyaz Saray’a gelmişse de kısa sürede,
doğuştan genellikle iyi olmalarına rağmen, toplum dikkatini dış konulara çevirmek zorunda kaldı.
ve kurumları yüzünden bozulduklarını kabul
ettiklerine inanılmıştır. A) Although, after the 2000 election, President
D) Romantik akım yazarları, genellikle, insanları George W. Bush came into the White House
doğuştan iyi fakat toplum ve kurumları tarafından focused on domestic issues, he was soon forced
bozulmuş olarak görmüşlerdir. to turn his attention to foreign affairs.
E) Romantik akım yazarlarına göre, insanlar doğuş- B) Following the 2000 election, President George
tan iyidir, fakat genellikle toplum ve kurumları ta- W. Bush came into the White House with a full
rafından bozulmuştur. awareness of domestic problems but, within a
short time, he had to focus his attention on
international issues.
C) After the 2000 election, when President George
W. Bush entered the White House primarily
concerned with domestic matters, his attention
was immediately focused on foreign affairs.
D) Even if, after the 2000 election, President George
W. Bush began his tenure in the White House
thoroughly aware of domestic affairs, his
attention soon turned to international matters.
E) A short time after the 2000 election, President
George W. Bush began to focus his attention on
foreign issues, although he had come into the
White House mainly concerned with domestic
affairs.

90
40. 100 yaş ve üzerinde olan insanlar, Amerikan 42. – 46. sorularda, boş bırakılan yere, parçada
nüfusunun en hızlı artan yaş gruplarından birini anlam bütünlüğünü sağlamak için getirilebilecek
temsil etmektedir. cümleyi bulunuz.

A) The fastest-growing age group of the American 42. Isaac Newton and Francis Bacon were
population is represented by people living to age considered by their contemporaries to be
100 and beyond. ornaments of the English humanities, and many
B) People living to age 100 and beyond is known as whom we now call scientists were called “natural
the fastest-growing age group of the American philosophers” in their day. ----. According to the
population. Oxford English Dictionary, the earliest use of the
C) It is known that people living to age 100 and term was in 1840.
beyond are one of the fastest-growing age
groups of the American population. A) Inspired by the example of the University of
D) It is in the American population that the group of Berlin, other institutions of learning were
people living to age 100 and beyond is growing beginning to detach themselves from theology
the fastest. and classics and devote themselves to research
E) People living to age 100 and beyond represent B) Sigmund Freud considered himself a “biologist of
one of the fastest-growing age groups of the the mind”
American population. C) Charles Darwin was a man driven to explain his
ideas in rational terms
th
41. Vietnam ile Amerika Birleşik Devletleri D) The term “scientist” was only invented in the 19
arasındaki savaşta en vahşi çarpışma, 1968 century as a kind of counterpart to the term
başlarında, Tet olarak bilinen Vietnam Yeni Yılı “artist”
sırasında ol-muştur. E) Human life was illuminated by being compared to
a chariot pulled by two horses of different
A) The bloodiest fighting in the Vietnam-United temperaments, a flowing stream, or the task of
States of America war took place during the pushing a stone up a hill
Vietnamese New Year known as Tet, in 1968.
B) During the war between Vietnam and the United
43. The low-tax, high-income countries are mostly
States of America, it was early in 1968, the time
English-speaking ones that share a direct,
of the Vietnamese New Year known as Tet, that th
the fiercest battle was fought. historical lineage with 19 century Britain. ----. The
C) In the war between Vietnam and the United high-tax, high-income states are the Nordic social
States of America, the most savage fighting democracies, notably Denmark, Finland, Sweden
occurred early in 1968, during the Vietnamese and Norway, which have been governed by social
New Year, known as Tet. democratic parties for much or all of the post-
D) Throughout the war between Vietnam and the World War II era. They combine a healthy respect
United States of America, the most dangerous of for market forces with a strong commitment to
the battles fought was the one early in 1968, anti-poverty programmes.
during the Vietnamese New Year known as Tet.
E) In the war between Vietnam and the United A) Unemployment rates are roughly the same in
States of America, the fighting occurring early in both groups
1968, during the Vietnamese New Year known as B) These countries include Australia, Canada,
Tet, was quite terrible. Ireland, New Zealand, the UK and the US
C) All of them, but especially Sweden and Finland,
have taken to the sweeping revolution in
information and communications technology
D) One of the great challenges of sustainable
development is to combine society’s desire for
economic prosperity and social security
E) The US spends less than most other rich
countries on social services for the poor and
disabled

91
44. As you may have seen in a museum, men and 47. – 51. sorularda, karşılıklı konuşmanın boş
women have grown taller and heavier in the last bırakılan kısmını tamamlayabilecek ifadeyi
300 years. ----. Body weights are also substantially bulunuz.
higher today. The average weight of English
males in their thirties was about 60 kg in 1790 – 47.
20% below today’s average. Benjamin: - Have you ever heard of a “cargo cult”?
Adam: - ----
A) Not only did lower food supplies lead to smaller Benjamin: - It happens when a person from an
physical stature, but they also led to a higher economically-developed country goes to a place
incidence of chronic disease that is completely isolated from the outside world.
B) A typical Frenchman in his thirties at that time The people there see all the things the foreigner
weighed only 50 kg brings, and start to treat the foreigner as a god.
C) As an example, an average male adult today is at
approximately 1.75 m tall, which is nearly 12 cm A) No. You tell me.
taller than the typical Englishman in the late B) No, is it something about world travel?
eighteenth century C) What did you say?
D) Robert Fogel from the University of Chicago D) Who came up with that term?
estimated that the chronic malnutrition caused by E) I don’t want to hear about it.
limited food supplies at those times limited labour
productivity
E) Economic growth increased food supplies, 48.
enabling workers to become more productive and Frank: - Do you know which country has the most
increase gross domestic product even more tourists each year?
Tom: - ----
Frank: - No, it’s France, but the US takes in more
45. A number of globalization issues involving money from tourism than any other country.
children require our thoughtful consideration and
action. ----. These children and their families A) No, I don’t.
represent a challenge to educators because of B) I should think it must be Italy.
cultural and language barriers. C) There is fierce competition among countries, isn’t
there?
A) A great challenge in the twenty-first century is D) Where do you like to travel?
how to enrich or give positive content to the E) Portugal is the country I am most interested in.
process of globalization
B) The effects of globalization are seen over a wide
spectrum of our lives, including children’s play 49.
and people’s beliefs and attitudes about it Barry: - It says in this newspaper article that, in the
C) One can judge the quality of a nation by the year 2000, almost 40% of all traffic fatalities in the
willingness of its citizens to care about other US were alcohol-related.
people’s children Jean: - ----
D) An educational consequence of globalization Barry: - Yes. The percentage has dropped from
facing many teachers of young children in the US 51% in 1987.
and elsewhere is the growing number of recent Jean: - That’s an improvement, then.
immigrant children in classrooms
E) Conditions are being created so that more and A) What do you think about this topic?
more people will come to have both a global B) People shouldn’t drink alcohol and then drive.
identity and a local identity (one’s own cultural C) I think they need better public transportation
reference group) in the twenty-first century systems in US cities.
D) Is that a change from previous years?
E) Were intoxicated pedestrians also included in the
46. After earning his degree, John M. Keynes survey?
became a civil servant, taking a job with the India
Office in Whitehall, London. ----. With the onset of
World War I, Keynes returned to government
employment, this time in the Treasury.

A) Between the wars, Keynes wrote his most


famous work: The General Theory of
Employment, Interest and Money
B) After World War I, he attended the Versailles
Peace Conference
C) His book, The Economic Consequences of the
Peace, condemned the Versailles Peace Treaty
and its negotiators
D) His wife, a Russian ballerina, was very active in
promoting the arts
E) After a while, he returned to Cambridge, where
he taught economics at the University

92
50. 54. (I) In the last decade, improvements in the
Alex: - This article claims that, within the next ten effectiveness of organ transplants have increased the
years, the economies of developing countries will demand for used human organs. (II) Each year,
grow extensively. thousands of Americans will die waiting for
Jeannette: - Yes, I’ve read it. I don’t think the replacement kidneys, hearts and lungs. (III) Because
economies will grow only because of foreign or the supply has not increased along with demand,
domestic investment, however. however, there are shortages of transplantable
Alex: - ---- organs. (IV) In a normal market, the price would rise
Jeannette: - They’ll grow because most of the poor to eliminate the shortage, but because it is illegal to
people in those countries will finally have enough buy and sell human organs, there is no pricing
money to start spending it. mechanism to close the gap between the quantity
supplied and the quantity demanded. (V) This gap in
A) How do you know? the organ-transplant system has led Nobel-winning
B) Then what is it that will cause the growth? economist Gary Becker to suggest monetary
C) I don’t agree with the article, do you? incentives for organ donors.
D) Who wrote this article, anyway?
E) What do you think will happen? A) I B) II C) III D) IV E) V

51. 55. (I) Oil wealth in Venezuela has given rise to grand
Anna: - With global warming seeming to become a aspirations ever since 1922, when a blowout of oil
reality, tourist destinations are changing. sprayed “black rain” over the small town of Cabimas.
Peter: - ---- (II) By 1928, Venezuela had become the world’s
Anna: - Well, the most popular places formerly largest oil exporter, with Venezuelans of all classes
have become too hot for most tourists, so they’re acquiring costly Yanqui tastes. (III) In recent surveys,
choosing cooler spots. a majority of Venezuelans said they had benefitted
Peter: - Then the traditional tourist spots probably from government spending on food and health care
stand to lose income. as well as on education. (IV) In 1976, the government
nationalized its subsoil wealth, and high oil prices and
A) What can we do to stop the trend? stable politics increased the national living standard.
B) Oh, are people going to Antarctica to see the ice (V) But by 1980, oil prices began to fall, and hard
before it melts? times followed, making life difficult for most
C) I don’t believe that the earth is really warming up. Venezuelans.
D) Do you think this will affect Turkey’s economy?
E) Really? In what way? A) I B) II C) III D) IV E) V

52. – 56. sorularda, cümleler sırasıyla 56. (I) The 26.2-mile footrace known as the Last
okunduğunda parçanın anlam bütünlüğünü bozan Marathon takes place every other February on King
cümleyi bulunuz. George Island, about 2,000 miles from the South
Pole. (II) Some 600 scientists and support workers
52. (I) During the 1990s, the Japanese economy was live here in the summer conducting meteorological
in a prolonged recession. (II) Economists and and wildlife studies. (III) The race was founded in
journalists put forward many different ideas to try to 1955 by Thom Gilligan, a marathoner from Boston.
jump-start the economy. (III) This is an example, (IV) The participants, ranging in age from 18 to 71
although an unusual one, of government fiscal policy. years old and hailing from 15 countries, are
(IV) One suggestion was that the Japanese competitive runners as well as casual joggers. (V) All
government should issue everyone a certificate the participants, however, possess the “three D’s”
entitling each person to the equivalent, in yen, of some runners jokingly say are necessary to complete
$200. (V) However, these yen certificates would only a marathon: desire, discipline and dementia.
be valid for purchases for one month.
A) I B) II C) III D) IV E) V
A) I B) II C) III D) IV E) V

53. (I) Folk ballads were songs sung by the common


people of England. (II) Their origins remain a mystery.
(III) Most seem to have been composed between
1200 and 1500, and while there has been much
argument, no one is exactly sure how they were
created. (IV) The ballad form does not allow elaborate
detail in plot, setting or character. (V) Present-day
theories suggest that many were invented by local
minstrels, descendants of the Anglo-Saxons, who
entertained the humble people of a village by making
up songs.

A) I B) II C) III D) IV E) V

93
57. – 60. soruları aşağıdaki parçaya göre 59. One understands from the passage that
cevaplayınız. Pompeii ----.

The Roman city of Pompeii in A.D. 79 was a thriving A) was the only Roman city famous for its taverns
provincial centre, a few miles from the Bay of Naples, and shopping centres
with a population of between 10,000 and 20,000 B) was one of the Roman centres for the slave trade
people. Its narrow streets, made narrower by street C) had a very efficient water system
vendors and shops with cloth awnings for shade, D) was the second largest city in the Roman Empire
were full of shoppers, tavern-goers, slaves, and E) had been destroyed by volcanic eruptions several
vacationers from the North. A huge new aqueduct times before A.D. 79
supplied running water from the Lower Apennine
mountains, which flowed from fountains throughout
the city, even in private homes. But the key to 60. According to the passage, what geologist
Pompeii’s prosperity, and that of smaller settlements Philip Janey is actually saying in the part quoted
nearby like Oplontis and Terzigna, was the region’s is that ----.
rich black earth provided by Mount Vesuvius’ volcanic
eruptions. “One of the ironies of volcanoes is that they A) the city of Pompeii should have been founded on
tend to produce very fertile soils, and that tends to the other side of the Bay of Naples
tempt people to live around them”, says geologist B) volcanic terrain is most suitable for people to
Philip Janey. Had Roman knowledge in the summer settle and live on
of A.D. 79 been less mythological and more C) people always prefer to settle in volcanic areas
geological, the Pompeiians might have recognized the since they believe the land there is more fertile
danger signs from Mount Vesuvius and escaped the D) the Pompeiians knew that the area around Mount
volcanic eruption that was to follow. Vesuvius was not a safe place to settle in, but
they settled there anyway
57. According to the passage, in A.D. 79, there E) people attracted by the fertile lands around
had been some geological indications that ----. volcanoes prefer to live there, ignoring the
dangers of a volcanic eruption
A) Mount Vesuvius was about to erupt, but the
people of Pompeii failed to understand them
B) the Bay of Naples posed a serious danger to the
city of Pompeii, but it was ignored by the
Pompeiians
C) the area in which the city of Pompeii was situated
was becoming less and less fertile
D) the water resources in the Lower Apennine
mountains were no longer adequate to supply
water to the city of Pompeii
E) Oplontis and Terzigna, the settlements near
Pompeii, were not safe to live in and, therefore,
had to be evacuated

58. It is emphasized in the passage that the


economic well-being of Pompeii ----.

A) reached its climax in A.D. 79, the year in which


there was a sharp increase in its population
B) primarily depended on the commercial activities
of its people as well as holidaymakers from the
North
C) attracted all kinds of people with money, who
crowded its streets and led a carefree life
D) was essentially related to the fertility of its land,
which was due to the volcanic eruptions of Mount
Vesuvius
E) was the outcome of its exploitation of the
settlements around it such as Oplontis and
Terzigna

94
61. – 64. soruları aşağıdaki parçaya göre 64. It is clear from the passage that, in his
cevaplayınız. paintings, Cézanne ----.

Artist Paul Cézanne wanted to make paint “bleed”. A) developed a completely new style which was
The old masters, he said, painted warmblooded flesh imitated by his contemporaries
and made the trees look warm and alive, and he B) was influenced, to a great extent, by the French
would too. He wanted to capture “the green odour” of Impressionists
his Provence fields and “the perfume of marble from C) used daring techniques to produce his likenesses
Saint-Victoire”, the mountain that was the subject of of the fields of Provence
so many of his paintings. He was bold, spreading and D) attached more importance to the depiction of the
slapping paint onto his still-lifes with a palette knife. “I human body than the natural landscape
will astonish Paris with an apple”, he boasted. In the E) almost always depicted Paris and its
years when his friends Manet, Monet, Pissarro and surroundings
Renoir were finally gaining acceptance, Cézanne
worked furiously and mostly in isolation, ridiculed by
critics and mocked by the public, sometimes tearing
up his own canvases. He wanted more than the quick
impressions of the Impressionists, and devoted
himself to studying the natural world. He called
himself a “slave to nature”, but he knew that he could
never completely capture the natural landscape on
canvas.

61. It is stated in the passage that, as a painter,


Cézanne ----.

A) wasn’t able to depict nature in his works as fully


or as vividly as he would have liked to
B) achieved far more popularity in art circles than his
contemporaries
C) dismissed the old masters as only artists whose
style was old-fashioned
D) was interested more in the representation of still
life than in the depiction of landscape and natural
scenes
E) was noted for his quiet personality and refined
manners

62. According to the passage, Cézanne’s style of


painting ----.

A) was widely acclaimed by art critics and the public


B) mainly derived from the views held by his friends
Manet, Monet, Pissarro and Renoir
C) was achieved by copying the works of the old
masters whom he studied
D) differed significantly from that of his fellow artists
E) had a great influence on his contemporaries and
their work

63. It is pointed out in the passage that the world


of nature ----.

A) appealed to Cézanne so powerfully that it


became the main focus of his art
B) depicted by Manet and the other Impressionists
failed to appeal to the public
C) was most successfully represented by Cézanne
in all of his works
D) meant for Cézanne only different shades of
colours and nothing else
E) was understood by Cézanne as a reference to
environmental issues

95
65. – 68. soruları aşağıdaki parçaya göre 67. It is pointed out in the passage that the
cevaplayınız. Angolan government ----.

Angola’s emergence as a serious player in the global A) is working on multi-billion dollar deepwater
oil sector has been underlined by the publication of its investment programmes to be implemented in
latest production figures. After several years of slow 2008
output growth, the fruits of the multi-billion dollar B) feels that its 2008 oil production target may not
deepwater investment programme are finally feeding be realized due to the high costs of deepwater
through. Thus, production reached 1.3 million barrels investments
a day during the final quarter of 2005. With the new C) is resolved to challenge only Libya for oil
deepwater fields now coming into production, the leadership in Africa
government’s target of producing 2 million barrels a D) has invested huge sums for the production of oil
day by 2008 now seems eminently within reach. In from its deepwater fields
the meantime, official Angolan government figures E) announced that it has published its oil figures in
indicate that oil production averaged 1.25 million order to prove to the world that it is a serious
barrels a day during the course of 2005, a steep jump player in the global oil sector
on the year before. In addition, the government’s
estimate of proven oil reserves has finally been
increased from 5.4 billion barrels to 12.4 billion 68. It is pointed out in the passage that, according
barrels. This shows that, as sub-Saharan Africa’s to the Angolan government, ----.
second biggest oil producer after Nigeria, Angola is in
an excellent position to overtake countries such as A) the country’s verified oil reserves are more than
Libya and Algeria in the table of oil powers on the twice the original estimate
continent as a whole. B) the country owns the largest and most productive
deepwater oil fields in sub-Saharan Africa
65. It is suggested in the passage that, with its C) Angola has already become the largest oil
rich oil reserves and rapidly increasing oil producer in all of Africa
production, Angola ----. D) after 2008, Nigeria and other oil producing
countries in Africa will no longer be major players
A) has become one of the major oil powers in Africa in the global oil sector
B) has already completely left behind such major oil E) the country’s deepwater oil reserves have finally
producers as Nigeria, Libya and Algeria been verified but are too costly to be exploited
C) has emerged as a threat in sub-Saharan Africa
as well as the global oil sector
D) can be regarded as a long-time constructive
player in the African oil league
E) has always been in fierce competition with
Nigeria and other sub-Saharan African countries

66. One understands from the passage that


Angola’s previously poor oil output ----.

A) can only be improved through the exploitation of


its deepwater reserves
B) has improved dramatically and reached a record
level in 2005
C) is related to Nigeria’s dominant position in the
sub-Saharan oil sector
D) prevents it from competing efficiently with other
oil producing countries
E) does not make it a current major player in the
global oil sector

96
69. – 72. soruları aşağıdaki parçaya göre 71. It is stated in the passage that, a case in an
cevaplayınız. international criminal court today ----.

Supporters of today’s international criminal tribunals A) may not always be conducted within the context
say that their work builds on the post-World War II of civilian law
tribunals in Nuremberg and, to a lesser degree, B) has far-reaching positive political and social
Tokyo. As a matter of legal doctrine, that is true. The consequences
category of “crimes against humanity”, for example, C) is not necessarily concerned with crimes against
was developed at Nuremberg and is now a central humanity
element in many prosecutions. But there is a critical D) can continue for many years before being
difference between now and then. The courts in finalized
Nuremberg and Tokyo were part of a broader political E) can be conducted at a lower cost
project that aimed to rehabilitate Germany and Japan,
respectively, both socially and economically, not
simply to try guilt or innocence or hand out harsh 72. One understands from the passage that
punishments. These were military courts that people put on trial at an international criminal
operated with military efficiency, and the Allies could court today ----.
then focus fully on the reconstruction of these
countries. Yet, the international courts for the former A) usually complain about protracted proceedings
Yugoslavia, Rwanda, and the new International and excessive expenses
Criminal Court (ICC) in the Hague, on the other hand, B) always deny that they have committed crimes
operate under civilian law and provide generous against humanity
protection to defendants. The result is a ballooning of C) are given extensive rights to defend themselves
the court timelines and costs. For instance, it took the D) are invariably found guilty and, therefore, given
International Criminal Tribunal for Rwanda (ICTR) 10 harsh punishments
years to complete the same number of trials that E) only remain on trial for, at most, one year
Nuremberg conducted in less than a year. Indeed, it is
clear that, because of their protracted proceedings
and excessive costs, today’s war crimes trials cannot
serve the decisive political and social function that
Nuremberg did.

69. It is argued in the passage that today’s


international criminal courts ----.

A) were originally established in Nuremberg and


Tokyo for the trial of war crimes committed during
World War II
B) have no correlation whatsoever with the
principles of the Nuremberg trials
C) follow a punitive policy and recognize no right of
appeal for defendants
D) are costly, time-consuming and lacking in
efficiency
E) have been set up only for the trial of specific
crimes and are to be abolished in due course

70. It is stressed in the passage that the


Nuremberg and Tokyo trials ----.

A) were not interested in what is today termed


“crimes against humanity”
B) were conducted in accordance with the
provisions of both military and civilian law
C) involved both prolonged and extremely
complicated proceedings
D) were mainly confined to war crimes with the
single aim of handing out harsh punishments
E) made up the first stage of a broader policy which
aimed for the reconstruction of Germany and
Japan

97
73. – 76. soruları aşağıdaki parçaya göre 76. It is pointed out in the passage that girls ----.
cevaplayınız.
A) never fail in their determination to excel over
The ability to deal with numbers and mathematical boys in mathematics
concepts reveals an interesting pattern of differences B) begin to use numbers before boys do
between girls and boys. Girls usually begin counting C) are relatively more conscious than boys as
before boys. Throughout the primary-school years regards their sex differences
and middle school, girls are better at computational D) always use mathematical concepts as efficiently
problems, whereas boys do better with math- as boys in solving problems
reasoning problems. During this period, girls also tend E) in the long run perform much better than boys in
to get higher grades. By high school, however, boys all aspects of mathematics
begin to perform better, especially at the higher levels
of ability. Some psychologists believe this advantage
is genetic, but others think that it may derive, in part,
from males’ use of more effective strategies and their
lower level of anxiety when approaching mathematics
problems. It has also been suggested that the sex
differences come about to some degree because girls
view math as a male activity (and, therefore, have
less interest in it) and because some parents and
teachers offer greater encouragement to males in this
area. Some studies support this analysis, but others
do not. In this area, too, biological and socialization
factors probably combine to produce the observed
differences.

73. The author thinks that differences between


girls and boys ----.

A) can be observed only through their approach to


mathematics
B) have concerned psychologists far more than their
performance at school
C) is a subject that has attracted a great deal of
attention in recent years from psychologists
D) remain constant throughout their school years
E) may be attributed to the joint effect of their
biological nature and social environment

74. It is suggested in the passage that boys ----.

A) are never influenced by their social environment


in their attitude towards mathematics
B) are always discouraged when they are faced with
a mathematical problem
C) are genetically far more motivated towards
mathematics than girls
D) are more successful than girls at primary level in
dealing with mathematical problems which
require reasoning
E) usually rely on their parents and teachers in
solving all kinds of mathematical problems

75. One learns from the passage that, according


to one view, the sex difference between boys and
girls ----.

A) has been categorically ruled out by all


psychologists as an important factor in learning
mathematics
B) ought to be taken into consideration by parents
and teachers
C) is negligible at the high school level
D) cannot be a significant factor in their performance
at high school
E) does have some effect on their mathematical
performance

98
77. – 80. soruları aşağıdaki parçaya göre 80. It is pointed out in the passage that, after the
cevaplayınız. introduction of the printing press, a reader’s easy
access to books ----.
The printing press was first introduced into England
by William Caxton in the last quarter of the fifteenth A) contributed enormously to the development of
century. During his earlier travels in Europe, he had different types of literature in fifteenth-century
seen the newly invented system of printing from England
moveable type in Germany. He set up his own press B) was hindered by the fact that books were hardly
in London in 1476. This initiated a major change in affordable
English literature. Now books did not have to be C) was what the London book publishers primarily
laboriously copied by hand. Soon, they would become cared about
relatively cheap. With books easily obtainable, more D) made storytelling obsolete and, hence, angered
people could learn to read, and more books would be the storytellers of the time
produced. The experience of literature would soon E) enabled the individual to share the thoughts and
shift from the breathless group of listeners gathered in experiences of another person through the
a hall or around a fire, hearing an old tale told once printed page
more, to the solitary individual, alone with the
thoughts and feelings of another person speaking TEST BĐTTĐ.
from the printed page. CEVAPLARINIZI KONTROL EDĐNĐZ.

77. It is asserted in the passage that the


introduction of the printing press into England ----.

A) made book production easy but caused a major


increase in their prices
B) caused the complete disappearance of oral
literature
C) was too late to have any constructive effect on
social and literary life
D) aroused a great deal of objection from copyists
and booksellers
E) had a very significant impact on people’s
relationship with literature

78. One understands from the passage that, with


the printing press, ----.

A) Caxton soon caught the attention of the general


public and was much respected
B) Caxton was able to produce a lot of books and
make a huge profit from their sale
C) London came to be a major centre of book
production in the fifteenth century
D) the number of books produced increased greatly
and to the benefit of the reading public
E) booksellers in London began to compete with
each other fiercely

79. It is clear from the passage that Caxton’s


interest in the printing press ----.

A) was originally aroused when he was visiting


Germany
B) had always been strong ever since his early
years
C) was essentially related to his desire to make a lot
of profit from book production
D) was the main motive for his journey to Germany
E) became stronger as more and more people were
learning to read

99
1. – 18. sorularda, cümlede boş bırakılan yerlere 8. Alcohol ---- every organ of the body, but the
uygun düşen sözcük ya da ifadeyi bulunuz. most dramatic evidence of its disruptive
behaviour ---- in the liver.
1. The ---- of lactose intolerance varies widely
among ethnic groups, indicating that the trait is A) affected / has appeared
genetically determined. B) affects / appears
C) is affecting / appeared
A) conversion B) prevalence C) recession D) had affected / would appear
D) notice E) supply E) has affected / had appeared

2. Some evidence suggests that weight training 9. Although constipation usually ---- lifestyle
can raise HDL if undertaken regularly, but habits, in some cases it may be a side effect of
frequent and sustained aerobic activity may be medication or may reflect a medical problem such
more ---- in lowering LDL and raising HDL. as tumours that ---- the passage of waste.

A) tentative B) irrelevant C) factual A) had reflected / obstruct


D) effective E) protective B) has reflected / obstructed
C) reflected / had obstructed
D) will reflect / have obstructed
3. The army psychologists ---- initiated treatment, E) reflects / are obstructing
and so he soon regained his confidence as a
soldier. 10. Indeed, some studies ---- that taking the
A) promptly B) vaguely C) obviously glycemic effect into account in meal planning ----
D) reluctantly E) imminently a practical way to improve glucose control.

A) have shown / is B) show / was


4. Red blood cells contain haemoglobin, which C) had shown / had been D) showed / will be
enables them to carry oxygen from the lungs and - E) could show / has been
--- it to all parts of the body.
11. Of every 10,000 children born in the US,
A) contribute B) organize C) deliver almost 7 ---- from health problems because their
D) transform E) guide mothers ---- alcohol during pregnancy.

A) suffered / have consumed


5. When someone chokes on food, it is because B) were suffering / consume
the food has slipped into the air passage and ---- C) had suffered / were consuming
breathing. D) will have suffered / will consume
E) suffer / consumed
A) given in B) kept on C) set out
D) switched off E) cut off
12. People who wish to improve their athletic
performance need to be aware that it can be
6. The vitamin portion of a coenzyme allows a achieved ---- physical conditioning combined ----
chemical reaction to ----, while the remaining high energy food intakes.
portion of the coenzyme binds to the enzyme.
A) by / with B) with / for C) in / by
A) put on B) get off C) catch up D) through / to E) of / into
D) take place E) settle down
13. Almost 2 million people die ---- tuberculosis
7. In a study carried out over a period of six (TB) each year, mostly in developing countries
months, researchers ---- that smoking ---- far more lacking access ---- fast, accurate testing
heart attacks than haemochromatosis. technology.

A) have found / had caused A) from / at B) about / with C) of / to


B) had found / has caused D) by / in E) with / through
C) found / caused
D) find / could have caused
E) would have found / causes 14. ---- the body has manufactured antibodies
against a particular antigen (such as the measles
virus), it remembers how to make them the next
time it is attacked by the same antigen.

A) Until B) Once C) Unless


D) Even though E) Whereas

100
15. Some researchers believe schizophrenia is a 19. – 23. sorularda, aşağıdaki parçada
single disorder, ---- others believe it is a syndrome numaralanmış yerlere uygun düşen sözcük ya da
based on numerous underlying diseases. ifadeyi bulunuz.

A) if B) since C) even so Older people suffer more broken bones than younger
D) while E) unless people because the mass and strength of bones
decrease with age. There is no single reason why this
occurs, or why some individuals and populations are
16. Anyone who wonders ---- the newborn baby more (19) ---- than others. Like other complex traits,
senses touch or experiences pain should watch age-related changes in bones result (20) ----
the baby’s reaction to a heel prick for a blood interactions between environmental and genetic
sample. factors. Scientists have linked changes in bone
strength to (21) ---- in physical activity, the levels of
A) whereby B) whether C) until dietary calcium and vitamin D, and alcohol and
D) in case E) what tobacco use. However, among (22) ----, physical
activity is the variable (23) ---- likely to account for the
geographic heterogeneity in the incidence of
17. Water, indispensable and abundant, provides fractures.
the environment ---- nearly all the body’s internal
activities are conducted. 19.
A) considerate B) equal C) sensible
A) in which B) whose C) wherever D) negligible E) vulnerable
D) by what E) however
20.
A) to B) of C) from
18. Certain plants can help some conditions more D) over E) at
effectively than medical drugs, and with
significantly ---- side-effects. 21.
A) circulations B) settlements C) variations
A) little B) more C) much D) tendencies E) similarities
D) fewer E) any
22.
A) these B) that C) which
D) whom E) this

23.
A) how B) as C) such
D) so E) most

24. – 35. sorularda, verilen cümleyi uygun şekilde


tamamlayan ifadeyi bulunuz.

24. Although AIDS still has no cure, ----.

A) the devastating effects of HIV infection seemed


unstoppable
B) remarkable progress has been made in
understanding and treating HIV infection
C) a specific dietary strategy for the treatment of the
disease has not been devised
D) the severity of wasting may determine the
duration of survival
E) the causes of malnutrition and wasting in HIV
infection are related to the disease itself

25. If a child has the potential for cardiovascular


disease, ----.

A) this article focuses on efforts to prevent childhood


obesity and cardiovascular disease
B) most people consider cardiovascular disease to
be an adult disease
C) obesity in children affects these changes
D) poor health choices, such as poor diet, will cause
the disease itself to develop
E) questions arose about the extent to which
genetics is involved in the risk of cardiovascular
disease

101
26. By comparing urinary creatinine excretion to 30. Imitation is a very important means ----.
standards for sex and height, ---- if muscle mass
is adequate or depleted. A) that the mirror neuron system serves as a bridge
in this process
A) it is concluded B) though Iacoboni and his group used functional
B) it has been understood magnetic resonance imaging to observe the
C) one can determine brains of human subjects
D) the physician can assume C) in which both activities triggered the inferior
E) we are convinced frontal gyrus of the brain
D) from which scientists can derive evidence about
gene function
27. Just as the human mind allows a person to E) by which we learn and transmit skills, language
develop a concept of intellectual self, ----. and culture

A) each cell in a person’s body has molecules on its


surface that are unique to that specific person 31. Most researchers find the new method of
B) the function of the immune system is to defend generating human embryonic stem cells
the body against invaders promising, ----.
C) virtually any organ can be attacked by the
immune system, including the kidneys, lungs, A) even if they held the possibility of regenerating
heart and brain body parts
D) the immune system provides a concept of B) as it might lead to new and possibly better stem-
biological self cell lines
E) the immune system is composed of cells and C) so that stem cells can become any other kind of
soluble substances cell
D) if this had led to healthier children
E) since this procedure was introduced more than a
28. Once people have been diagnosed with decade ago
cancer, ----.

A) therapists could have used relaxation methods to 32. ----, analysts had to rely on a more primitive
assist them in coping with some of the negative urine screen for this purpose.
aspects of cancer treatments
B) they can affect their survival time by adapting a A) Unless drugs opened the way to a better system
“fighting spirit”, having strong emotional and B) Although an athlete could beat the test by
social support or attending group counselling ingesting artificial testosterone throughout the
sessions testing period
C) attempts by their partners to protect them from C) If later screens for testosterone levels matched
the reality of their illness were not helpful the preliminary results
D) at least half of those treated with chemotherapy D) Only if the body were able to make testosterone
had experienced nausea, fatigue, depression, from an artificial compound
sleep problems and loss of appetite E) Before the carbon isotope radio (CIR) urine test
E) family history and ethnic background are factors became standard protocol for testing athletes for
in many types of cancer doping

29. Because the visual system is not well- 33. The vast majority of smoking-related cancer
developed at birth, ----. deaths are from lung cancer ----.

A) newborn babies’ ability to change focus is limited A) although smoking may also be responsible for
and they are very nearsighted deaths from cancers of the lip, pharynx,
B) a nearsighted adult with 20/30 vision will be able pancreas, trachea and kidney
to see at 20 feet B) because the rate of smoking in the US has slowly
C) newborn infants can distinguish the sound of the declined since the mid-1960s, especially among
human voice from other sounds men
D) developmental psychologists would have C) but weight gain would have been quite modest
designed some ingenious procedures to study for most people who had stopped smoking
the mental capacities of young infants D) if group counselling had been successful in
E) parents noticed with delight that the baby had helping people to quit smoking
begun to make eye contact E) even if many nonsmokers find the smoke of
others to be a nuisance and even irritating to their
eyes and nose

102
34. A person with leukemia has an unusually high 37. People who do not consume milk products or
number of leukocytes, ----. other calcium-rich foods that provide even half of
the recommended calcium amount may benefit
A) since it is usually fatal unless treated from calcium supplements.
B) although cancerous cells grow uncontrollably
C) most of which do not function normally A) Süt ürünleri ya da kalsiyum yönünden zengin
D) that is, cancer of the white blood cells diğer besinleri tüketmeyenler önerilen kalsiyum
E) whereas, in some cases, they do not respond to miktarının ancak yarısını alabildiklerinden
the standard treatments kalsiyum desteğinden yararlanabilirler.
B) Tavsiye edilen kalsiyum miktarının yarısını bile
karşılayacak miktarda süt ürünleri ya da kalsiyum
35. Hypnosis was first used as a surgical bakımından zengin diğer besinleri tüketme-yen
anaesthetic in India in 1845, ----. insanlar kalsiyum desteğinden yararlanabilirler.
C) Tavsiye edilen kalsiyum miktarının yarısını
A) if doctors had used hypnosis for patients who are karşılayacak kadar kalsiyum desteğinden
allergic to anesthetics yararlanan insanlar süt ürünleri ve kalsiyum
B) since hypnotized patients will recover more yönünden zengin diğer besinleri tüketmeyebilirler.
quickly, with less pain and with fewer side effects D) Önerilen kalsiyum miktarının yarısını karşılasa da
C) although hypnosis as an alternative to sedation is kalsiyum desteğinden yararlanan insanlar süt
making a comeback in the operating room ürünleri ve kalsiyum bakımından zengin diğer
D) but it was quickly abandoned with the besinleri de tüketmelidirler.
introduction of ether the following year E) Kalsiyum desteğinden yararlanan insanlar,
E) even if studies using advanced scanning tavsiye edilen kalsiyum miktarının ancak yarısını
technology have shed new light on how hypnosis karşılayabilecek miktarda süt ürünleri ya da
works to block pain kalsiyum bakımından zengin diğer besinleri
tüketenlerdir.

36. – 38. sorularda, verilen Đngilizce cümleye


anlamca en yakın Türkçe cümleyi bulunuz. 38. If a person has a family history of a certain
disease, it is a powerful indicator of his/her
36. Although many forms of therapy have been tendency to contract that disease.
tried, none has been able to prevent the disease
from progressing. A) Bir hastalığın aile içindeki öyküsü, kişinin bu
hastalığa yakalanma eğilimini ortaya koyan en
A) Pek çok tedavi biçimi denenmiş olmasına güçlü belirtidir.
rağmen, hiçbiri hastalığın ilerlemesini B) Bir ailedeki bireylerin hastalık öyküleri, o ailedeki
engelleyememiştir. bireylerin hastalığa yakalanma eğilimini ortaya
B) Uygulanan tedavi yöntemlerinden hiçbiri hasta- koyar.
lığın ağırlaşmasını engelleyemedi. C) Kişinin belirli bir hastalığa yakalanma eğilimi, en
C) Pek çok tedavi yöntemi var, ancak hiçbiri hasta- doğru biçimde, ailesinin hastalık öyküsünden
lığın ilerlemesini tam olarak durduramıyor. belirlenebilir.
D) Farklı bir tedavi yöntemi bulunsa da hastalığın D) Kişinin belirli bir hastalığa yakalanma eğiliminin
yayılması hiçbir zaman engellenemez. en güçlü belirtisi, o hastalığın aile öyküsüdür.
E) Hastalığın ilerlemesini durdurmak için bazı tedavi E) Kişinin belirli bir hastalıkla ilgili aile öyküsü varsa
biçimleri denenmesine rağmen başarılı bu, onun o hastalığa yakalanma eğiliminin güçlü
olunamadı. bir göstergesidir.

103
39. – 41. sorularda, verilen Türkçe cümleye 41. Mikroskop altında incelenmek üzere iğne ile
anlamca en yakın Đngilizce cümleyi bulunuz. alınmış karaciğer dokusu örneği içeren karaciğer
biyopsisi, vakaların yalnızca % 75’i kadarında
39. Yıllarca, gelişmiş pek çok ülkede başlıca ölüm tanıyı doğrular.
nedeni, genellikle kardiyovasküler hastalıklar
olarak bilinen kalp ve damar hastalıkları olmuştur. A) A liver biopsy, which is carried out by the removal
with a needle of a sample of liver tissue for
A) Over the years, it has generally been known that microscopic examination, is indispensable for a
among the major causes of death in most reliable diagnosis in 75% of cases.
developed countries is cardiovascular disease, B) About 75% of cases can be definitively
which refers to diseases of the heart and blood diagnosed through a liver biopsy, which involves
vessels. the removal by needle of a sample of liver tissue
B) Generally, diseases of the heart and blood for examination under a microscope.
vessels, which for many years were known as C) The diagnosis of about 75% of cases can be
cardiovascular disease, have been among the confirmed through a liver biopsy, which is
main causes of death in various developed performed with the removal of a sample of liver
countries. tissue for microscopic examination.
C) Cardiovascular disease, which as a whole D) A liver biopsy, in which a sample of liver tissue is
includes the major diseases of the heart and removed by needle for examination under a
blood vessels, has over the years been the usual microscope, confirms the diagnosis in only about
cause of death in a number of developed 75% of cases.
countries. E) In about 75% of cases, the diagnosis is
D) For years, the major cause of death in many confirmed by means of a liver biopsy, which is
developed countries has been diseases of the performed by removing with a needle a sample of
heart and blood vessels, generally known as liver tissue for examination.
cardiovascular disease.
E) Numerous diseases of the heart and blood
vessels, which are as a whole called 42. – 46. sorularda, boş bırakılan yere, parçada
cardiovascular disease, have for years been the anlam bütünlüğünü sağlamak için getirile-bilecek
most common cause of death in developed cümleyi bulunuz.
countries.
42. Alcohol is rich in energy (7 kcalories per
gram), but, as in pure sugar or fat, the kcalories
40. Yiyeceklerdeki iyot miktarı değişkendir ve are empty of nutrients. ----. This means that the
genellikle, bitkilerin yetiştiği veya hayvanların more alcohol people drink, the less likely it is that
üzerinde otladığı topraktaki miktarı yansıtır. they will eat enough food to obtain adequate
nutrients.
A) The amount of iodine contained in foods varies
significantly and depends on the amount in the A) Thus, the more kcalories provided by alcohol, the
soil, needed for the growth of plants or the less nutritious food is normally eaten
grazing of animals. B) Alcohol in heavy doses is not efficiently
B) Foods contain various amounts of iodine, and metabolized, generating more heat than fat
these are usually related to the amount in the soil C) Most dramatic, however, is alcohol’s effect on the
used for plant cultivation and animal grazing. B vitamin folate
C) The amount of iodine in foods is variable and D) The combination of poor folate status and alcohol
generally reflects the amount in the soil in which consumption has, as a result, been implicated in
plants are grown or on which animals graze. promoting colorectal cancer
D) The soil in which plants are grown or on which E) Generally, moderate drinkers consume alcohol as
animals graze usually contains the same amount added energy – on top of their normal food intake
of iodine as found in various foods.
E) The amount of iodine which is contained in the
soil used for the cultivation of plants or the 43. The pharmaceutical industry is particularly
grazing of animals is, on the whole, reflected by interested in new substances that could mask
the amount of iodine in foods. unpleasant flavours. ----. And chronically ill
people, such as AIDS patients, who have to force
down many unpleasant tablets daily, would
greatly appreciate more agreeable formulations.

A) Such a quality might prove undesirable for


sweets or soda
B) The idea of a bitter-blocker has still to be tested
C) Flavour-optimized drugs would be especially
valuable for treating children, who often refuse to
swallow nasty-tasting syrups
D) Nutritionists want to know how much these
choices could affect the health of individuals
E) Widespread use could possibly undermine a
natural protective function

104
44. The main diagnostic signs of autism are social 47. – 51. sorularda, karşılıklı konuşmanın boş
isolation, lack of eye contact, poor language bırakılan kısmını tamamlayabilecek ifadeyi
capacity and absence of empathy. ----. Many bulunuz.
people with autism, for instance, have trouble
understanding metaphors; they also have
difficulty mimicking other people’s actions. 47.
Margaret: - Have you read this article about the
A) It would be premature to conclude that damage 1987 nuclear contamination incident in Goiânia,
to the cerebellum is the sole cause of the Brazil?
disorder Henry: - ----
B) Other, less well-known symptoms, however, are Margaret: - Well, apparently some people found an
also commonly evident abandoned radiation-therapy machine and opened
C) Some researchers posit that the main it up. Many people were contaminated and 4
abnormality in autism is a deficit in the ability to eventually died.
construct “a theory of other minds” Henry: - That’s terrible! The machine should have
D) Certain brain cells are often referred to as motor- been disposed of safely.
command neurons
E) Still, children with autism do have characteristic A) Why did you read the article?
abnormalities in the cerebellum B) Not yet. Is it worth reading?
C) Yes, it’s terrible, isn’t it?
D) What do you think about it?
45. A pregnant woman should participate in “low- E) No, what happened?
impact” activities and avoid sports in which she
might fall or be hit by other people or objects. ----.
Swimming is also ideal because it allows the body 48.
to remain cool and move freely with the water’s Bill: - I read today about a new type of stent used
support. to widen a patient’s artery.
Jen: - ----
A) It also maintains the habits that help a woman Bill: - It releases a drug into the artery over several
lose excess weight and get back into shape after weeks which prevents excess build-up of fibrous
the birth tissue, reducing the further narrowing of the
B) In general, the more weight a woman gains artery.
beyond what she needs for pregnancy, the more Jen: - That sounds like a promising development.
she will retain
C) Women who begin their pregnancy at a healthy A) I think many doctors are too quick to insert stents.
weight need to gain about 30 pounds, which They should first try medication to reduce
covers the growth and development of the clogging in the patient’s arteries.
placenta, uterus, blood, breasts and infant B) How does it differ from other stents?
D) Several of these guidelines have been aimed at C) Doctors say that coronary stents have
preventing dehydration revolutionized patient care.
E) For example, a daily walk is always beneficial D) Patients with clogged coronary arteries always
used to undergo bypass surgery.
E) Have you ever recommended this procedure?
46. The liver has primary responsibility for
detoxifying alcohol. ----. Cancer is more likely to
occur in cirrhotic livers than in healthy ones, but 49.
alcohol abusers are likely to die of a variety of Tracy: - Here’s an article about a drug trial that
causes before liver cancer develops. went badly wrong. After testing a drug on
monkeys and obtaining the desired result,
A) An excessive intake of alcohol often leads to researchers went ahead and did a trial on six
cirrhosis of the liver, a degenerative disease that human volunteers. Now they’re all in intensive
limits the organ’s effectiveness care, and one of them is in a coma!
B) This evidence indicates that high levels of alcohol June: - ----
consumption are related to various kinds of Tracy: - I think it’s because the humans’ cells
cancer reacted differently with the drug than the
C) For cancers of all sites, alcohol is not as strong a monkeys’ cells.
risk factor as either smoking or an imprudent diet
D) Recent research has found a synergistic effect A) The researchers should release the clinical data
between alcohol and tobacco so that other researchers can benefit from their
E) Pancreatic cancer has a special affinity to alcohol mistake.
consumption B) Researchers should be more careful when
testing new drugs on humans.
C) What do you think will happen to him?
D) I wonder why the drug didn’t work in the
volunteers in the same way it had worked in the
monkeys?
E) Researchers hoped that the drug would be
capable of activating immune cells other
antibody-drugs could not activate.

105
50. 54. (I) Most people believe that some drugs are
Fran: - ---- acceptable and even desirable because of the
Jim: - Why are these studies so important? medical benefits they confer. (II) But all psychoactive
Fran: - Well, scientists are comparing drugs – drugs that cross the blood-brain barrier and
monozygotic (identical) twins with dizygotic alter mental functioning – are potentially harmful to
(fraternal) twins to see if their traits or behaviour health. (III) Even drugs that are not psychoactive have
are influenced more by their genes or their the potential for unpleasant side effects. (IV) For a
environment. time, cocaine was used as an anaesthetic for surgery,
especially eye surgery. (V) For example,
A) According to a recent study, fraternal twins are antihistamines may cause confusion, dizziness, dry
no more similar than any two biological siblings. mouth and blurred vision.
B) In the study, it has been suggested that the IQ
scores of two identical twins are usually more A) I B) II C) III D) IV E) V
closely related than those of two fraternal twins.
C) There are lots of studies recently being carried
out on twins. 55. (I) Antibiotics are drugs that disable or kill
D) One can conclude from this study that fraternal infectious microorganisms. (II) However, for nearly
twins share an environment as similar as that of every antibiotic that has been developed, a resistant
identical twins. strain of bacterium has appeared within a few
E) That particular study concentrated on identical decades. (III) Penicillin, for example, was originally
twins. isolated from a mold and has been widely prescribed
since the 1940s. (IV) A revolution in human health
rapidly followed its introduction, rendering many
51. previously fatal diseases easily curable (such as strep
Art: - Did you know that as many as 235,000 throat and surgical infections). (V) During the 1950s,
hospital patients in the US die unneccessarily some doctors even predicted the end of human
each year? infectious diseases altogether.
Ed: - ----
Art: - About 45 per cent of the deaths are due to A) I B) II C) III D) IV E) V
medical errors, and the rest are because of
adverse reactions to medication.
56. (I) Of all the things we feel, none captures our
A) When are you going to begin your residency? attention like pain. (II) We may sometimes be
B) I don’t believe it. Where did you read that? unconcerned about other experiences but it is hard to
C) That must result in a lot of malpractice lawsuits. ignore the experience of pain. (III) Yet for all the
D) I think there must be some sort of mistake in that discomfort it causes, we would be at risk if we had no
article. sense of pain. (IV) For instance, it would be difficult
E) That’s a staggering number! What causes them? for children to learn not to touch a hot stove, or to stop
chewing their tongues. (V) These receptors are
neurons with specialized free nerve endings.
52. – 56. sorularda, cümleler sırasıyla
okunduğunda parçanın anlam bütünlüğünü bozan A) I B) II C) III D) IV E) V
cümleyi bulunuz.

52. (I) According to a recent study, depressed white


girls are more likely to grow out of their problems than
black girls are. (II) White teens become less
depressed as they approach early adulthood. (III)
Factors such as social class and parental separation
should not be overlooked. (IV) The depression rate
among blacks, however, remains steady. (V) This
appears to be largely the result of poorer health care
and lack of insurance.

A) I B) II C) III D) IV E) V

53. (I) According to recent data, doctors sometimes


misdiagnose migraines as “sinus headaches”. (II)
Migraines are an inherited form of recurring
headaches. (III) This confusion in diagnosis occurs
because neurons in the brain stem can activate the
sinuses during a migraine, causing them to secrete a
clear fluid. (IV) Yet this discharge differs from the
cloudy fluid produced by a sinus infection. (V)
Therefore, it is advisable for doctors to consider
migraines more seriously and to look for clear nasal
secretions as symptoms.

A) I B) II C) III D) IV E) V

106
57. – 60. soruları aşağıdaki parçaya göre 60. It is clearly emphasized in the passage that
cevaplayınız. several important bodily functions ----.

The most common cause of cyanide poisoning is A) are severely impaired as a result of cyanide
smoke inhalation. A source to the public is acetonitrile poisoning
in the form of artificial fingernail remover. Cyanide B) can be weakened through intravenous
poisoning produces cellular hypoxia by binding with administration of sodium thiosulphate
the ferric iron of mitochondrial cytochrome oxidase, C) can be undermined by rhodanese unless an
disrupting the electron transport chain and the ability antidote is administered
of cells to use oxygen. Patients who inhale cyanide D) depend, to a large extent, on the presence of
may rapidly develop coma, shock, seizures, lactic thiocyanate in the blood
acidosis, and respiratory and cardiac arrest. Mild E) are not affected by cellular hypoxia or the
exposures following smoke inhalation are now being disruption of the electron transport chain
described. Diagnosis may be difficult in these
patients, and emergency administration of an antidote
may be lifesaving. Patients with smoke inhalation who
show evidence of lactic acidosis should be suspected
of cyanide poisoning. The body has a natural enzyme,
called “rhodanese”, which can complex cyanide and
sulphur to form mildly toxic thiocyanate. Intravenous
administration of sodium thiosulphate provides the
sulphur necessary to produce thiocyanate and is
relatively safe. Sodium nitrite may also be
administered, but its use is reserved for the most
critical cases only, because it causes hypertension
and methemoglobinemia.

57. It is pointed out in the passage that cyanide


poisoning ----.

A) results solely from the use of some cosmetics


such as artificial fingernail remover
B) can be diagnosed very easily, but its treatment
takes a long time
C) has the potential to quickly lead to some very
serious complications
D) has no effect whatsoever on cells and their
functions
E) is rarely related to smoke inhalation

58. According to the passage, in very severe


cases of cyanide poisoning, ----.

A) administration of sodium nitrite may be


recommended
B) hypertension and methemoglobinemia cannot be
avoided
C) various kinds of antidotes can be administered
but few are reliable
D) patients should be put under constant
observation to control lactic acidosis
E) the use of sodium thiosulphate should be
completely ruled out

59. It is suggested in the passage that lactic


acidosis in patients suffering from smoke
inhalation ----.

A) is the most common cause of cyanide poisoning


B) can be eliminated through the enzyme called
“rhodanese”
C) can cause the formation of toxic thiocyanate in
the body
D) is one of the indicators of cyanide poisoning
E) should not be taken as evidence of cyanide
poisoning

107
61. – 64. soruları aşağıdaki parçaya göre 64. We see from the passage that meningitis may
cevaplayınız. be caused by ----.

When doctors suspect meningitis, they must quickly A) different antibiotics


determine whether it is from a bacterial, viral, fungal, B) cerebrospinal fluid
or other type of infection or from irritation caused by C) a spinal tap
something other than an infection (for example, a D) the blood-sugar level
chemical). The possible causes are many, and the E) an infection or an irritation
treatment differs for each. The test usually used to
diagnose meningitis and determine its cause is the
spinal tap, or lumbar puncture. A thin needle is
inserted between two bones in the lower spinal
column to withdraw a sample of cerebrospinal fluid
from an area just below the spinal cord. The doctor
then examines the fluid for bacteria under a
microscope and sends a sample of it to the laboratory
to be cultured and identified. The bacteria can be
tested for susceptibility to treatment with different
antibiotics. The sugar level, an increase in protein,
and the number and type of white blood cells in the
fluid also help determine the type of infection.

61. It is mentioned in the passage that the fluid


obtained from a lumbar puncture ----.

A) always contains elevated levels of protein and


white blood cells
B) is different from that obtained during a spinal tap
C) is examined carefully to help understand the
cause of the meningitis
D) is not helpful in deciding on methods of treatment
for meningitis
E) is not a common diagnostic test for meningitis

62. It is clear from the passage that, for testing


purposes, a small amount of cerebrospinal fluid
----.

A) can be removed from low down in the spinal


column
B) is not enough for an accurate diagnosis of
meningitis
C) can be extracted once meningitis has been
diagnosed
D) should be taken but this will have no influence on
the choice of antibiotic to be used
E) should be examined only if an increase in the
protein levels is suspected

63. It is pointed out in the passage that, since the


causes of meningitis are varied, ----.

A) any kind of antibiotic may be used to treat any


case of meningitis
B) the treatments must also vary
C) it is virtually impossible to track down the cause
in any one instance
D) irritation caused by a chemical is the most likely
E) the type of white blood cell determines the choice
of antibiotic

108
65. – 68. soruları aşağıdaki parçaya göre 68. It can be understood from the passage that, as
cevaplayınız. the concentration gradient between the blood and
dialyzing fluid is reduced, ----.
In dialysis with an artificial kidney, the rate of
movement of solute across the dialyzing membrane A) the maximum rate of transfer occurs
depends on four variables: the concentration gradient B) the membrane slowly becomes more permeable
of the solute between the blood and the dialyzing to the solute
fluid, the permeability of the membrane to the solute, C) the diffusion of solute across the membrane
the surface area of the membrane, and the length of slows down
time that the blood and fluid remain in contact with the D) the transfer of blood and fluid across the
membrane. Thus, the maximum rate of solute transfer membrane is optimized
occurs initially when the concentration gradient is E) an artificial kidney must be used to keep the
greatest (when dialysis is begun) and slows down as patient healthy
the concentration gradient is dissipated. In a flowing
system such as haemodialysis, in which blood and
dialysate fluid flow through an artificial kidney, the
dissipation of the concentration gradient can be
reduced, and diffusion of solute across the membrane
can be optimized by increasing the flow rate of either
or both the blood and dialyzing fluid.

65. According to the passage, solute transfer in


dialysis is faster ----.

A) than the natural kidney function


B) when the concentration gradient between the
blood and dialyzing fluid is dissipated
C) when the patient’s own kidney is used
D) at the beginning of the process
E) when dialysis is prolonged

66. We see from the passage that the permeability


of the membrane to the solute ----.

A) is the most important of the variables mentioned


B) is one of the variables that affects the speed of
dialysis
C) decreases as the surface area of the membrane
decreases
D) is not a factor in haemodialysis
E) is greater initially and then decreases as dialysis
continues

67. It is clear from the passage that increasing the


flow rate of blood through the artificial kidney is
----.

A) a good way to increase the length of time the


blood and fluid are in contact with the membrane
B) not a practical option in dialysis
C) only achieved when certain pre-conditions are
met
D) one of the ways in which dialysis can be made
more efficient
E) better than raising the rate of flow of the dialyzing
fluid

109
69. – 72. soruları aşağıdaki parçaya göre 72. We see from the passage that, when the lateral
cevaplayınız. hypothalamic area of an animal’s brain is
damaged, the animal ----.
Stimulation of several areas of the hypothalamus in
the brain causes an animal to experience extreme A) cannot control its activities
hunger, a voracious appetite, and an intense desire to B) may starve to death
search for food. The area most associated with C) becomes overactive
hunger is the lateral hypothalamic area. Damage to D) is in danger of also losing the ventromedial
this area sometimes causes the animal to lose desire nucleus
for food, sometimes causing lethal starvation. On the E) loses all sensitivity to stimulation of the brain
other hand, a centre in the hypothalamus that
opposes the desire for food, called the satiety centre,
is located in the ventromedial nucleus. When this
centre is stimulated electrically, an animal that is
eating food suddenly stops eating and shows
complete indifference to food. However, if this area is
destroyed bilaterally, the animal cannot be satiated;
instead, its hypothalamic hunger centres become
overactive, so that it has a voracious appetite,
resulting in tremendous obesity.

69. It is pointed out in the passage that the satiety


centre, when stimulated, causes an animal to ----.

A) lose interest in eating


B) have a voracious appetite
C) have an overactive hunger centre
D) have an intense desire to search for food
E) connect this stimulation directly with hunger

70. The topic of this passage is ----.

A) the effect of the destruction of the main areas of


the brain
B) the effect of brain damage on the choice of food
C) changes in the lateral hypothalamic area of the
brain resulting from starvation
D) the stimulation of different areas of the brain
E) the hypothalamus and its relation to appetite

71. According to the passage, obesity in animals


can be caused by ----.

A) stimulation of the ventromedial nucleus in the


hypothalamus
B) a damaged lateral hypothalamic area
C) irreparable bilateral damage to the ventromedial
nucleus
D) over-stimulation of the satiety centre
E) the easy availability of food

110
73. – 76. soruları aşağıdaki parçaya göre 76. It is mentioned in the passage that the alveoli,
cevaplayınız. alveolar sacs, alveolar ducts and respiratory
bronchioles ----.
The ultimate importance of the pulmonary ventilatory
system is to continually renew the air in the gas A) occasionally renew the air in the gas exchange
exchange areas of the lungs, where the air is in areas of the lungs
proximity to pulmonary blood. These areas include B) are the parts of the lungs that are closest to
the alveoli, alveolar sacs, alveolar ducts and pulmonary blood
respiratory bronchioles. However, during normal quiet C) do not receive any of the inspired air during quiet
respiration, the volume of air is only enough to fill the respiration
respiratory passageways down as far as the terminal D) are part of the dead space of the respiratory
bronchioles, with only a small portion of the inspired system
air actually flowing all the way to the alveoli. Some of E) are outside the gas exchange areas of the lungs
the molecules in the inspired air travel the final short
distance to the alveoli through diffusion, but most
never reach the gas exchange areas of the lungs,
instead going to fill respiratory passages where gas
exchange does not occur, such as the nose, pharynx
and trachea. This air is called dead space air because
it is not useful for the gas exchange process; the
respiratory passages where no gas exchange takes
place are called dead space. On expiration, the air in
the dead space is expired first, before any of the air
from the alveoli reaches the atmosphere. Therefore,
the dead space does not aid in removal of the
expiratory gases from the lungs.

73. It is clearly pointed out in the passage that


only a small amount of the air inspired during
normal respiration ----.

A) reaches the atmosphere


B) is expired before the next breath is taken
C) is called dead space air
D) stays in the nose, pharynx and trachea
E) ever reaches the gas exchange areas of the
lungs

74. According to the passage, dead space air ----.

A) aids the gas exchange process tremendously


B) is the last to reach the atmosphere during
expiration
C) flows all the way to the alveoli through diffusion
D) does not help in the expiration of gases from the
lungs
E) is of ultimate importance in the pulmonary
ventilatory system

75. It can be understood from the passage that the


main function of the pulmonary ventilatory system
is to provide ----.

A) clean air to the pulmonary blood


B) dead space air to the gas exchange areas of the
lungs
C) gas exchange in the dead space areas
D) atmospheric air during expiration
E) clean air to the nose, pharynx and trachea

111
77. – 80. soruları aşağıdaki parçaya göre 80. According to the passage, 40 to 100 million
cevaplayınız. people worldwide ----.

Virologist Robert Webster thinks that the H5N1 strain A) transmit flu viruses to other people every year
of the avian influenza virus poses the most serious B) have already died from the H5N1 strain
public health threat since the Spanish flu pandemic of C) are currently infected with the H5N1 avian virus
1918, which killed an estimated 40 million to 100 D) were threatened by the Spanish flu pandemic
million people worldwide. Although the H5N1 strain E) died from the Spanish flu pandemic of 1918
has so far shown no signs that it has acquired the
ability to transmit easily from person to person,
Webster says that it is only a matter of time before it TEST BĐTTĐ.
does. For that to happen, Webster and others believe CEVAPLARINIZI KONTROL EDĐNĐZ.
that a version of the human flu virus, which is easily
transmittable between people, and the H5N1 avian
virus would have to infect the same mammalian cell at
the same time and re-combine their DNA. If H5N1
picks up those genes from the human flu virus that
enable it to spread from person to person, Webster
says that virtually nobody will have immunity to it, and
many deaths may ensue.

77. We see from the passage that although the


H5N1 strain of the avian flu virus is not easily
transmitted from person to person at present, ----.

A) the human flu virus is also not easily


transmittable between people
B) neither was the Spanish flu virus in 1918
C) it will probably acquire the ability to do so in the
near future
D) most people already have immunity to it
E) it is not a serious public health threat

78. It is clear from the passage that if the human


flu virus and the H5N1 strain were to infect the
same mammalian cell at the same time, ----.

A) the H5N1 strain would probably gain the ability to


spread between people
B) mammals would become immune to both flu
strains
C) this could prove dangerous but will almost
certainly never happen
D) it would pose no danger to the public
E) the H5N1 strain would be destroyed

79. As we can infer from the passage, virologist


Robert Webster thinks that ----.

A) most humans are immune to the H5N1 avian


influenza strain as it exists at present
B) the H5N1 strain is unlikely ever to be transmitted
between humans
C) the H5N1 strain and the human flu virus could
never combine to create a stronger virus
D) the avian influenza virus will soon pose a huge
threat to humans
E) many human deaths could result from a
resurgence of the Spanish flu

112
8. Cosmologists are addressing some of the
1. – 18. sorularda, cümlede boş bırakılan yerlere fundamental questions that people ---- to resolve
uygun düşen sözcük ya da ifadeyi bulunuz. over the centuries through philosophical thinking,
but they ---- this based on systematic observation
1. The greatest ---- to the spread of nuclear and quantitative methodology.
technology and nuclear power reactors to
developing countries is that it will increase the A) would attempt / have done
risks of nuclear weapons proliferation. B) attempt / will do
C) may attempt / did
A) obligation B) contribution C) solution D) attempted / should do
D) condition E) objection E) have attempted / are doing

2. A combination of factors made the 1984


accident in a storage tank at a Union Carbide 9. The shuttle ---- the atmosphere at precisely 38°
plant in India almost ----. for heat shields below the fuselage and the wings
---- the craft from heat damage.
A) crucial B) inevitable C) vulnerable
D) bearable E) permanent A) must re-enter / to protect
B) has re-entered / having protected
C) re-entered / to have protected
3. In the opinion of most scientists, engineering D) re-enters / to be protecting
does not ---- offer universally acceptable E) should re-enter / to have been protecting
solutions.

A) randomly B) previously C) necessarily 10. From the year 1665, when Robert Hooke ----
D) excessively E) extremely cells, until the middle of the twentieth century,
biologists ---- only light microscopes for viewing
cells.
4. Scientists suggest that huge amounts of
greenhouse gases will be ---- into the atmosphere A) used to discover / could have had
if rising temperatures cause the Arctic permafrost B) had discovered / would have
to melt. C) discovered / had
D) has discovered / have had
A) produced B) accelerated C) disrupted E) could discover / have
D) released E) joined
11. Sea bindweed Calystegia soldanella ---- a
5. Several research groups have been racing to fleshyleaved cousin of the more widespread,
---- how to regenerate hair cells. whiteflowered hedge bindweed (C. sepium) that
---- fences and hedges everywhere in the summer.
A) figure out B) go for C) connect with
D) set up E) make up A) might be / had clothed
B) is / clothes
C) should be / has clothed
6. An important aspect of the application of D) could be / would have clothed
mathematics is that different ways of making E) was / can clothe
mathematical sense of everyday questions ----
different answers.
12. In February 1996, ---- a meeting in Bermuda,
A) keep up B) bring over C) lead to international partners in the Human Genome
D) show off E) find out Project agreed to formalize the conditions of data
access, including release of the sequence into
public databases ---- 24 hours.
7. Earthquake rupture ---- to occur by enlargement
of a crack, but more recent observations ---- a A) at / within B) from / in C) in / by
“pulse-like” mode of rupture enlargement. D) during / to E) on / through

A) had been thought / would be indicated


B) can be thought / had indicated 13. The United States government is about to start
C) was thought / will have indicated monitoring the air ---- major cities for biological
D) has been thought / indicate weapons ---- looking for bacteria and viruses in
E) must be thought / may have indicated the air filtres that now monitor pollution.

. A) of / on B) at / towards C) out of / with


D) in / by E) from / through

113
14. Comets are thought to have changed very little 19. – 23. sorularda, aşağıdaki parçada
over the last 4 billion years, ---- their composition numaralanmış yerlere uygun düşen sözcük ya da
should hold clues to the origin of the solar ifadeyi bulunuz.
system.
Names and numbers were causing trouble long
A) but B) whereas C) just as before the Internet age. Biology had a naming crisis in
D) in that E) so the 17th and 18th centuries. The problem wasn’t so
much a shortage of names but an excess of (19) ----.
Plants and animals (20) ---- by many different names
15. Robots will never be much good at household in different places. Then came the great reform of
tasks such as pouring coffee or polishing shoes Carolus Linnaeus and his system of Latin binomials,
---- they can calculate their position accurately. (21) ---- each organism by genus and species. The
new scheme revolutionized taxonomy, not because
A) since B) when C) unless there is any magic in Latin or in two-part names, but
D) so that E) in case because Linnaeus and his (22) ---- laboured to
preserve a strict one-to-one mapping between names
and organisms. Official codes of nomenclature
16. About half of all women over 65 years of age continue to enforce this rule – one name, one species
take some type of nutrition supplement, ---- only – although rooting out synonyms and homonyms is a
about one-fifth of older men do. (23) ---- struggle.

A) before B) because C) that is 19.


D) while E) despite A) them B) that C) theirs
D) those E) themselves
17. Coal produces ---- CO2 per energy unit ---- any
other fossil fuel. 20.
A) were to be known B) would be known
A) also / as B) more / than C) such / that C) are known D) were known
D) either / or E) so / as E) will be known

18. Every rock, ---- copper-veined, silver-clad, or 21.


black-glazed, tells a story about the Earth as a A) to have been identifying
whole. B) identified
C) to have identified
A) both B) also C) whether D) to be identifying
D) all E) that E) identifying

22.
A) participants B) followers C) occupants
D) suppliers E) practitioners

23.
A) constant B) primary C) rapid
D) similar E) partial

24. – 35. sorularda, verilen cümleyi uygun şekilde


tamamlayan ifadeyi bulunuz.

24. Shortly after the Golden Gate Bridge was


opened, ----.

A) some aesthetic and artistic concerns may have


dominated the visual design of the bridge
B) it has been an enormously successful bridge by
most aesthetic and functional criteria
C) its roadway proved to be overly flexible under
certain wind conditions
D) the design of the bridge’s towers was tested on
scale models, and construction of the bridge
started
E) a number of such structures were acting similarly

114
25. ----, yet relatively few have been identified in 30. Our eyes can detect photons, the smallest
modern organisms. quantum unit of an electromagnetic wave, ----.

A) Scientists hypothesize that the human hepatitis A) whose frequencies lie in the narrow visible range
delta virus (HDV) arose from a ribozyme B) in which the human retina has more “pixels” than
B) The CPEB3 ribozyme is structurally and a consumer digital camera
biochemically related to human hepatitis C) that it increases our knowledge of the structure of
deltavirus (HDV) ribozymes atoms
C) This ribozyme occurs exclusively in mammals D) because scientists have lacked a detector able to
D) The selection had yielded several ribozymes see an individual photon
E) Ribozymes are thought to have played a pivotal E) if a revolution in photon detection is now under
role in the early evolution of life way

26. Should the cabin pressure somehow be lost, 31. Abnormally heavy and early rainfall in the
----. Sudan caused the River Nile to overflow in 2007,
----.
A) the oxygen canisters located above the
passenger seats in a plane provide oxygen to the A) so global land surface temperatures in January
passengers through masks and April had reached the highest levels ever
B) the oxygen-generator canisters must be replaced recorded for those months
periodically to ensure that they will operate B) because the first documented tropical cyclone in
properly when needed the Arabian Sea hit Oman and Iran, causing 50
C) the oxygen canister contains a core of sodium deaths
chlorate, which is activated by a small explosive C) but in May, ocean waves up to 5 metres high
charge swamped parts of the Maldive Islands
D) the airline maintenance rules made it clear that a D) unless other extreme weather events include the
bright yellow safety cap must be installed on the summer heatweave in southeastern Europe
oxygen canisters E) while unusually heavy snowfall affected South
E) a small explosion was initiated when a passenger Africa and parts of South America
pulled the oxygen mask toward herself

32. ALH84001 is one of several meteorites that are


27. ---- when they are exposed to higher than generally acknowledged to have come from Mars
normal temperatures. ----.

A) Recent climate warming is associated with A) when it is named for the place in Antarctica (Allan
genetic change Hills) and the year (1984) it was found
B) Recent global warming might already be driving B) because they contain trapped gases that match
such changes the Martian atmosphere
C) Researchers compiled data on chromosomal C) while geologists at NASA discovered in the rock
polymorphism covering periods of 13 to 46 years a variety of surprising characteristics at a
D) Some organisms undergo genetic change microscopic level
E) Weather records for the same periods and D) where it turned out to be the oldest known rock
locations are studied from any planet
E) so that the findings were based on meticulous
studies and the papers of scientists at NASA
28. ---- as to why human mental capacities are so
much greater than those of chimpanzees.
33. Until fairly recently, we viewed the ocean as a
A) Two scientists recently reported bountiful, virtually limitless resource, ----.
B) Scientists have always suspected
C) Researchers have found a clue A) so many countries are also taking steps to
D) The newly adopted scheme also includes a third restore and conserve wetlands
category B) unless we are now seeing the effects of our
E) Such a decision was reached after days of disregard for marine communities
debate C) yet seafood would become less plentiful
D) and we have harvested the ocean heavily and
used it as a dumping ground for wastes
29. Although stem cells are found in many E) regardless of the fact that laws in many countries
tissues, ----. now prohibit disposal of sewage and other
wastes at sea
A) they have great potential to treat diseases
B) the most promising ones seem to be those in
bone marrow
C) specialists have been very enthusiastic
D) special staining techniques revealed that the cells
were indeed dividing
E) the results of that study have yet to be
announced

115
34. In the Pacific Ocean, the analogue of the Gulf 38. Meteorites provide the best available data
Stream Current in the Atlantic is the Kuroshio about the chemical and physical processes that
Current, ----. occurred during the first few million years of our
solar system’s history.
A) which flows north along the coast of Asia to the
east coast of Japan A) Güneş sistemimizin tarihinde, ilk birkaç milyon yıl
B) as it flows northeast across the Atlantic from its içinde ortaya çıkmış olan kimyasal ve fiziksel
source in the Gulf of Mexico oluşumlara ilişkin elde edilebilen en iyi veriler
C) so the Gulf Stream Current indeed contributes to göktaşlarında bulunmaktadır.
Europe’s warmth B) Güneş sistemimizin başlangıcındaki birkaç milyon
D) where it transports no heat to locations on the yıl içinde meydana gelmiş kimyasal ve fiziksel
eastern side of the Pacific süreçlerle ilgili kullanılabilir verilerin en iyileri
E) but ocean currents do little to warm the region göktaşlarından elde edilmiştir.
C) Göktaşlarının sağladığı veriler, güneş sistemimizin
tarihinin ilk birkaç milyon yıllık sürecindeki kimyasal ve
35. Researchers have found that the DNA in fiziksel oluşumları anlayabilmemize en büyük katkıyı
bacteria deteriorates sharply after about 1.1 sağlamıştır.
million years, ----. D) Güneş sistemimizin ilk birkaç milyon yıllık tarihinde
meydana gelen kimyasal ve fiziksel süreçlerle ilgili
A) whereas the DNA of the average bacterium has verilerin en güvenilir olanları göktaşlarından
about 3 million units sağlanmıştır.
B) which consisted of just 210 units linked together E) Göktaşları, güneş sistemimizin tarihinin ilk birkaç
C) after which the size of the DNA gets cut in half milyon yılında meydana gelmiş olan kimyasal ve
D) but older microorganisms didn’t perform as well fiziksel süreçler hakkında mevcut en iyi verileri sağlar.
E) and some of the oldest microorganisms were
watched for as long as a year 39. – 41. sorularda, verilen Türkçe cümleye
anlamca en yakın Đngilizce cümleyi bulunuz.

36. – 38. sorularda, verilen Đngilizce cümleye 39. Bilim adamlarına göre, atmosfere yayılmakta
anlamca en yakın Türkçe cümleyi bulunuz. olan atık gazlar nedeniyle dünyadaki iklimler daha
sıcak bir hale gelmiştir.
36. Scientists tell us that 90 per cent of all matter
in the universe consists of hydrogen. A) Scientists claim that so much waste gas has
been spread into the atmosphere that the
A) Bilim adamlarının bize söylediğine göre, hidrojen climates in the world have become increasingly
içeren maddeler evrendeki tüm maddelerin yüzde warm.
90’ını oluşturmaktadır. B) As scientists have pointed out, due to waste
B) Bilim adamları bize, evrendeki tüm maddelerin gases that pollute the atmosphere, the world’s
yüzde 90’ının hidrojen içerdiğini söylemektedir. climate has become much warmer.
C) Hidrojenin evrendeki tüm maddelerin yüzde 90’ını C) According to scientists, the climates in the world
oluşturduğu, bize bilim adamlarınca söylenen bir have become warmer because of waste gases
gerçektir. that are spreading into the atmosphere.
D) Bilim adamlarının bize söylediği gibi, hidrojen, D) For scientists, the atmosphere has been so
evrendeki maddelerin yüzde 90’ında polluted by waste gases that the climates in the
bulunmaktadır. world have already become much warmer.
E) Bilim adamları, hidrojenin, evrendeki tüm E) As far as scientists are concerned, the world’s
maddelerin yüzde 90’ını oluşturduğunu söylüyor. climate has become extremely warm owing to
waste gases in the atmosphere.

37. Inventions of modern science are no longer


the creations of a single person, as they were in
the past.

A) Çağdaş bilimin buluşları, artık, geçmişte olduğu


gibi tek bir kişinin yaratıları değildir.
B) Çağımızda bilimsel buluşlar, geçmişte olduğunun
tersine, bir kişinin yaratıcılığına
dayanmamaktadır.
C) Çağdaş bilimde artık, geçmişte olduğu gibi, tek
kişinin yaratıcılığına dayanan buluşlara
rastlanmamaktadır.
D) Çağdaş bilimle ortaya konan buluşlar, geçmişten
farklı olarak, birden çok kişinin ortaya koyduğu
yaratılardır.
E) Geçmişteki buluşlar, çağdaş bilimdekilerin
tersine, bir tek kişinin yaratılarıydı.

116
40. Evrenin sakinleri olarak, ışığın ilk kaynağının 42. – 46. sorularda, boş bırakılan yere, parçada
nasıl oluştuğunu, hayatın nasıl meydana geldiğini anlam bütünlüğünü sağlamak için getirilebilecek
ve bu çok büyük boşlukta akıllı varlıklar olarak cümleyi bulunuz.
bizim yalnız olup olmadığımızı merak etmekten
kendimizi alamayız. 42. Engineers are hired by clients (and employers)
specifically for their specialized expertise. ----.
A) How the first light was formed, how life started, Therefore, engineers have ethical obligations to
and whether we are the only intelligent beings in their clients, because the client often cannot
this huge emptiness are the questions that, as assess the quality of the engineer’s technical
inhabitants of the universe, we cannot keep advice. These obligations are part of engineering
ourselves from asking. ethics, the set of behavioural standards that all
B) As inhabitants of the universe, we cannot help engineers are expected to follow.
wondering how the first source of light formed,
how life came into existence and whether we are A) Civil engineering is generally considered the
alone as intelligent beings in this vast emptiness. oldest engineering discipline
C) As this universe’s only intelligent inhabitants, it is B) Successful teamwork results in accomplishments
up to us to ask questions such as how the first larger than those that can be produced by
light source was formed, how life on Earth individual team members
started, and how we came to inhabit a tiny planet C) Generally, the client knows less about the subject
in this vast emptiness. than the engineer
D) As inhabitants of this universe, we cannot help D) Biochemical engineers combine biological
asking such pressing questions as how the first processes with traditional chemical engineering
light source was formed, how life started, and to produce foods and pharmaceuticals and to
whether there are other intelligent beings living in treat wastes
this vast emptiness. E) An engineer does not need to have a licence to
E) Being inhabitants of the vast emptiness that is practise engineering, but those who do may have
our universe, we cannot help wondering how light more career opportunities
was formed, how life started, and whether we are
quite alone as intelligent beings in the universe.
43. An athlete’s body must be heavier for its
height than a nonathlete’s body because the
41. ABD Ticaret Bakanlığı, hassas teknolojik athlete’s bones and muscles are denser. ----.
bilgilere ulaşımı sınırlandırmaya ilişkin However, this is not true. Weight standards that
politikaların gözden geçirilmesi gerekip may be appropriate for others are inappropriate
gerekmediğini incelemek üzere bir grup uzmanı for athletes. Therefore, measures such as fatfold
görevlendirmiştir. tests yield more useful information about body
composition.
A) A group of experts commissioned by the US
Department of Commerce are examining how the A) When athletes consult standard weight-forheight
policies concerning limited access to sensitive tables and see that they are on the heavy side,
technological information should be reformulated. they may mistakenly believe that they are too fat
B) A group of experts have been appointed by the B) The increasing incidence of abnormal eating
US Department of Commerce to review the habits among athletes, especially young women,
policies that limit access to confidental is causing concern
technological information. C) Athletes are particularly likely to develop eating
C) The US Department of Commerce has recruited disorders
a group of experts to find out to what extent the D) They fail to realize that the loss of lean tissue that
policies for the limitation of access to delicate accompanies energy restriction actually impairs
technological information can be revised. their physical performance
D) The US Department of Commerce has E) Male athletes, especially wrestlers and gymnasts,
commissioned a group of experts to examine are affected by these disorders as well, but
whether policies on limiting access to sensitive research shows that females have a greater
technological information should be reviewed. tendency
E) The policies on limited access to confidental
technological information are being reviewed by a
group of experts, appointed by the US
Department of Commerce.

117
44. Thanks to their status as one of the world’s 46. Seismologists have struggled for years to find
top predators, great whites are among the best a reliable earthquake predictor. Could balls of
known sharks on Earth, yet essentially nothing is light in the sky preceding quakes hold the key?
known about their mating habits. That could soon The US Federal Emergency Management Agency
change, as researchers have discovered a remote (FEMA) has begun asking that very question. ----.
spot in the North Pacific Ocean that may be a Thus, they have funded NASA to study
mating ground for great whites, according to a earthquake lights using weather satellites and the
recent study. ----. But, as scientists have MODIS research satellite during the past few
explained, the theory that the area is a feeding years.
ground for great whites may be incorrect.
A) In 1999, floating balls of light in the sky were
A) It’s not an area that a shark would logically go to broadcast on Turkish television, reportedly
from California to find something to eat filmed the night before the earthquake in Đzmit
B) At first, scientists nicknamed the region, 1,553 B) In 1968, the first photographs of “earthquake
miles west of the Baja Peninsula, the “great white lights” were taken by Yutaka Yasui of the
café” because they suspected sharks could be Kakioka Magnetic Observatory
going there to feed C) The main problem facing FEMA is that
C) The sharks migrate long distances seasonally earthquake lights still don’t have an accepted
from the coast of California to Hawaii and to the scientific explanation
offshore area D) Most earthquakes occur at plate boundaries,
D) On average, the sharks dive every 10 minutes, where one plate slides beneath another
325 metres down, perhaps to sniff for mates, hundreds of kilometres below the Earth’s
whose scent could be detected at a certain level surface
of depth E) Mainstream geologists had dismissed these
E) Sharks gather at marine mammal habitats in earlier claims as coincidental
California during autumn and winter months,
feeding on the abundant elephant seals and
other prey before migrating to the offshore waters 47. – 51. sorularda, karşılıklı konuşmanın boş
bırakılan kısmını tamamlayabilecek ifadeyi
bulunuz.
45. The dark side of nanotechnology is the
nightmare possibility that “nano-robots” could be 47.
programmed to turn everything on Earth into Sarah: - Have you ever thought about how
more nano-robots. ----. Some researchers, paleontologists name the new fossils they find?
however, say that while they also have some They don’t only give the fossils a boring,
worries about nanotechnology, they don’t want it descriptive name in Latin.
banned because its benefits outweigh its risks. Laura: - ----
Sarah: - Well, in this article it says that one
A) R. Smalley discovered the three-dimensional paleontologist named a newly discovered type of
nanoscale carbon cages called fullerenes dinosaur fossil gojirasaurus after the original
B) E. Drexler says he invented the word Japanese name for Godzilla!
“nanotechnology” Laura: - That’s funny! I wonder if they’ll name any
C) The inventors of nanotechnology were awarded a fossils after King Kong.
Nobel Prize
D) In 1990, a team of scientists found they could use A) I have a cousin who’s studying paleontology; why
a scanning tunnelling microscope to drag don’t we ask him?
individual atoms of xenon over the surface of a B) Certainly. There are rules for how new species
crystal of nickel and fossils must be named.
E) There are other fears, such as nanoscale C) Well, they surely have a catalogue of names they
particles creating unforeseen toxic hazards can use for every fossil they discover.
D) Oh, really? What kinds of names do they give
them, then?
E) Paleontologists have a sense of humour, too!

118
48. 51. Keith: - It looks like more and more countries in
Lisa: - What are you reading? the EU are turning to wind power for their energy.
Andy: - A book about caterpillars in the Costa Cherie: - ----
Rican tropical forests. Did you know that there’s Keith: - Actually it’s not, because sometimes the
one type of caterpillar that looks like a snake’s wind turbines are built without proper planning,
head? and this affects the surrounding environment
Lisa: - ---- negatively.
Andy: - I should imagine, only to scare away Cherie: - Oh, I wasn’t aware of that.
predators.
A) I think a combination of wind and solar power
A) Looking at caterpillars always makes me feel like would be best.
I’ve got one crawling up my arm! B) I think that’s very good. Yes?
B) How strange. I wonder why? C) What’s your opinion of wind power?
C) I don’t know why this type of caterpillar lives in a D) I thought wind turbines couldn’t generate enough
tropical forest. power to make a difference.
D) How interesting. Does the book mention other E) Do you think wind power will help reduce carbon
types? emissions?
E) Why did the writer choose to focus on only the
forests in Costa Rica?
52. – 56. sorularda, cümleler sırasıyla
okunduğunda parçanın anlam bütünlüğünü bozan
49. cümleyi bulunuz.
Kathy: - I went to a lecture yesterday given by a
microbiologist. He focused on bacteria caught in 52. (I) When their nuclear fuel is exhausted, stars die,
Antarctic ice millions of years ago, and stressed and the residual iron core collapses on itself. (II) The
their importance in understanding how life on outcome of a star’s death depends on mass,
Earth works over long periods of time. however. (III) Stars with between 10 and 20 times the
Bruce: - ---- mass of the Sun collapse in a spectacular explosion
Kathy: - He said that life on Earth consists mostly known as a supernova, leaving behind a neutron star,
of microbes, and they can adapt to every possible whereas those larger than 20 solar masses implode
environment. to form black holes in a hypernova. (IV) In both cases,
Bruce: - That sounds interesting. Did he suggest copious bursts of neutrinos are released along with
any reading material on the subject? optical, x-ray and gamma radiation. (V) Recent
studies indicate that some massive stars may be
A) What is the lecturer’s special area of study within rotating only slowly or not at all.
the field of microbiology?
B) Did you ask him any questions after the lecture? A) I B) II C) III D) IV E) V
C) I find microbiology incredibly interesting; don’t
you?
D) Who else attended the lecture besides you? 53. (I) Since the cloned-sheep Dolly’s birth,
E) How can anything so tiny have an influnce on the researchers have cloned many other mammals,
planet as a whole? Amazing. Anything else? including mice, cats, horses, cows and pigs. (II) Such
reproductive cloning has many potential applications.
(III) Their most advanced embryo, however, had
50. stopped growing at about six cells, and their intention
Terry: - Did you know that scientists have found was solely to harvest embryonic stem cells. (IV) By
perfectly preserved comet dust in the ice in using genetically engineered donor nuclei, geneticists
Antarctica? can study the effects of changing single genes or
Lynne: - ---- combinations of genes. (V) And in the future,
Terry: - Yes, it is. The samples found previously in biologists may routinely produce genetically identical
Antarctica and in Greenland had been compacted animals for experimentation, a potential benefit to
and changed by the ice around them, but these genetics research.
new samples haven’t.
Lynne: - Then their larger size and good condition A) I B) II C) III D) IV E) V
must make them easier to analyse.

A) That’s nothing new! Don’t you think?


B) Are the dust samples taken from a comet’s tail by
spacecraft similar to this?
C) Where in Antarctica was the dust discovered?
D) It must have been difficult for the scientists to
locate the dust.
E) That was Jean Duprat’s study, wasn’t it?

119
54. (I) The algal ancestors of plants may have 57. As one understands from the passage, when
carpeted moist fringes of lakes or coastal salt the ultimate version of the weedkilling robot
marshes over 500 million years ago. (II) Both groups comes into use, ----.
have similar microscopic structures for making their
cellulose cell walls and a similar mechanism for A) there will be no need for the use of herbicides
forming the cell plate that divides the cytoplasm since weeds will simply be pulled out by the robot
during cell division. (III) These shallow-water habitats B) herbicides will no longer be used anywhere in the
were subject to occasional drying, and natural world
selection would have favoured algae that could C) the American type of weedkilling robots will still
survive periodic droughts. (IV) Some species be used since they are very efficient
accumulated adaptations that enabled them to live D) farmers will be able to upgrade their spraying
permanently above the water line. (V) The modern- techniques in order to kill the weeds among their
day green alga Coleochaete may resemble an early crops
plant ancestor, and it grows at the edges of lakes as E) many of the environmental problems farmers face
disk-like, multicellular colonies. will be solved much more efficiently

A) I B) II C) III D) IV E) V
58. It is clear from the passage that robots used in
the United States for weedkilling ----.
55. (I) Humans have a long history of studying,
appreciating, and using animal diversity, but A) are a technological challenge that farmers in
classifying a new animal isn’t always easy. (II) Denmark and the United States face
Imagine you were the first European zoologist to B) were first invented and widely used by farmers in
encounter a strange animal in Australia that has the the United States
following physical features. (III) It has a bill and C) will never be useful for improving traditional
webbed feet similar to a duck’s, but the rest of its furry spraying techniques
body looks very much like that of a muskrat or other D) are convenient for use only on railways and
aquatic rodent, and it lays eggs. (IV) Unlike the rest of airport runways rather than in farming areas
the world, Australia has relatively few placental E) are being developed and tested in Denmark
mammals. (V) How would you classify it?

A) I B) II C) III D) IV E) V 59. It is implied in the passage that herbicides


used for weedkilling ----.

56. (I) Reefs are under attack from all sides. (II) Coral A) have certainly done much damage to the
reefs are one of the oldest and most diverse environment in Denmark
ecosystems on Earth. (III) Hurricanes and tsunamis B) can be most effective if they are sprayed along
can cause injuries that take decades for a reef to with fertilizer
repair naturally. (IV) Meanwhile, destructive fishing C) are more commonly preferred in the United
practices, pollution, ships running aground and States than in Denmark
climate change pose an even more serious threat. (V) D) cause no environmental damage if applied only in
A report issued by the UN Environment Programme small amounts
warned that 30 per cent of the world’s coral reefs are E) are poisonous and, therefore, threaten the
either already dead or seriously damaged. environment

A) I B) II C) III D) IV E) V
60. As is clear from the passage, the type of robot
currently under trial in Denmark ----.
57. – 60. soruları aşağıdaki parçaya göre
cevaplayınız. A) is actually adaptable to all kinds of agricultural
purposes
Robots make unlikely green warriors, but they could B) has been regarded by farmers as a major step
soon be doing their bit for the environment. Trials of a forward in agricultural technology
Danish robot that maps the position of weeds growing C) identifies the weeds among crops but does not
among crops suggest that herbicide use could be pluck them
reduced by 70 per cent if farmers used it to adopt D) has been copied from the type which is used in
more selective spraying techniques. Actually, the the United States and is far more efficient
robot drives across fields scanning the ground for any E) is not able to tell the difference between different
weeds and noting their positions. A later version will plant species
be able to kill the weeds too by applying a few drops
of herbicide. But the longer-term goal is to avoid
herbicides altogether by having the robot pluck the
weeds out of the ground rather than poisoning them.
Although weedkilling robots have already been put to
work in the United States, they cannot be used for
agricultural purposes because they do not distinguish
between plant species and tend to treat anything
green as a weed. Instead, they are used to clear
unwanted plants from railways and airport runways.

120
61. – 64. soruları aşağıdaki parçaya göre 64. It is clear from the passage that Seversky’s
cevaplayınız. original plan for in-flight refuelling ----.

The first documented scheme for in-flight refueling A) was perceived as a threat by the Russian
came from a young Russian aviator named Alexander government
de Seversky. His father owned a plane and taught B) involved the transfer of fuel from one airborne
him to fly when he was in his early teens. In 1917, aircraft to another by means of a hose
when he was 23, Seversky proposed a method for C) gave him a clear advantage when he was
extending flight: One plane could carry extra fuel and applying for work in the United States
deliver it to another through a hose. After the Russian D) was inspired by his aeronautical studies in
Revolution, Russia’s new Bolshevik government sent America
him to the United States to study aircraft design, and E) was completely revised, following suggestions
he stayed there when political developments made from the US War Department
his return to Russia dangerous. He got a job as an
aeronautical engineer for the US War Department and
was awarded the world’s first patent for air-to-air
refuelling, in which large fuel tankers would supply
fuel to fighter aircraft while in flight. Seversky went on
to a distinguished career in airplane design and
achieved perhaps his greatest fame as the author of
the influential 1942 book Victory through Air Power.
He never put his refueling plan into action, though,
and other aviators later came up with ideas of their
own.

61. One understands from the passage that


although Alexander de Seversky was the owner of
the first patent for in-flight refuelling ----.

A) the US War Department was uninterested in his


plans
B) his true passion was always flying, which he
learned as a teenager
C) Russia’s Bolshevik government did not support
his schemes
D) his plans were never implemented
E) other inventors made use of his ideas on aircraft
design

62. We see from the passage that Seversky’s 1942


book Victory through Air Power ----.

A) made him more famous than his ideas for inflight


refuelling
B) was extremely unpopular in Bolshevik Russia
C) was used as a training manual by the US War
Department
D) extensively detailed his plans for in-flight
refuelling
E) remained unpublished at the time of his death

63. According to the passage, Seversky was


unable to return to Russia due to ----.

A) his employment as an aeronautical engineer for


the US War Department
B) the dangers of international travel at the time
C) changes in the political situation there
D) the fact that he was an extremely popular aircraft
designer
E) the political nature of the book he published in
1942

121
65. – 68. soruları aşağıdaki parçaya göre 67. As pointed out in the passage, the United
cevaplayınız. States ----.

One of the most pressing international priorities is to A) is so worried about the dissemination of nuclear
control the dissemination of nuclear materials that materials that it is making every effort to isolate
could be used in attacks by terrorists or rogue states. rogue states
Nuclear materials contain unstable isotopes, which B) has started taking technological precautions
emit x-rays and gamma rays. The characteristic along its Canadian and Mexican borders to
energies of these photons provide a fingerprint control the entry of nuclear materials into the
revealing which radioactive isotopes are present. country
Unfortunately, some isotopes that occur in benign C) is confident that the only type of atomic bomb
applications emit gamma rays with energies that are terrorists or rogue states can build will be no
very similar to those emitted by materials used in better than a Hiroshima-style one
weapons, which leads to ambiguous identifications D) strongly maintains that nuclear materials should
and false alarms. This problem has been worrying the be used only in useful and benign applications
United States, which is installing thousands of E) claims that the nuclear materials used in
radiation portal monitors to detect the gamma rays weapons are controlled strictly to prevent any
emitted by nuclear materials carried by vehicles leakage
crossing the Canadian and Mexican borders. One of
the worst fears of the authorities is that terrorists
might smuggle highly-enriched uranium into the 68. As one understands from the passage, the
country to build a crude Hiroshima-style atomic bomb. kind of radioactive isotopes present in nuclear
materials are ----.

65. The point is made in the passage that an issue A) more useful for weapons than for any other
of extremely compelling urgency in the world ----. purpose
B) only revealed when they are activated for any
A) is to ensure that nuclear materials are used not in use
weapons but in benign applications C) those which mostly emit a large amount of
B) has been the international threat posed against gamma rays
the United States by terrorists and rogue states D) identified by the energies typical of their nature
C) is to make sure that nuclear energy facilities are E) those that emit far more energy if uranium is
much safer and more reliable than ever highly enriched
D) has been to find out how rogue states have come
to possess highly-enriched uranium
E) is the prevention of the spread of nuclear
materials, serving the aims of terrorists and rogue
states

66. It is clearly pointed out in the passage that it is


very hard to ----.

A) reach an international understanding that the


dissemination of nuclear materials must be fully
controlled
B) trace how highly-enriched uranium can be
smuggled and marketed internationally
C) distinguish between gamma rays emitted by
nuclear materials used for constructive or
destructive purposes
D) force rogue states to give up their efforts to
develop nuclear weapons
E) identify all radioactive isotopes that emit x-rays
and gamma rays

122
69. – 72. soruları aşağıdaki parçaya göre 71. It is pointed out in the passage that as the
cevaplayınız. number of recyclings increases ----.

The world now recycles just over 50 per cent of the A) the dependence on wood fibres will no longer be
paper it uses. Reprocessing plants are being necessary
established in most countries. However, trees will B) less and less pure pulp will be used in the paper
never be fully spared because of the use of wood industry
fibres themselves. Pure pulp is rich in water, which C) the problem of adhesives becomes less and less
provides for ample hydrogen bonding that holds fibres serious and urgent
together when made into paper. But each time a fibre D) the cost of reprocessing is reduced to a viable
is cleaned, de-inked and dried in a reprocessing plant, level
only 80 per cent of the bonds are recovered. After E) the ability of a fibre to make strong bonds
four or five recyclings, a fibre can no longer make decreases
strong enough bonds. Engineers can do little that is
economically viable to overcome this physical
limitation, so they focus on reducing the cost of 72. It is pointed out in the passage that engineers
reprocessing fresher fibres. One main challenge is in the paper industry ----.
finding a better way to neutralize “stickies”, which is
the mess of adhesives from stamps, labels, seals, A) do not regard “stickies” in waste paper as a
tape, magazine spines and various other sources, serious challenge
that jam the machinery. The industry has been B) have made a breakthough in strengthening the
working for a decade to find a chemical process that bonding capacity of recycled fibres
will break down stickies, but no full solution has been C) are mostly interested in reducing the cost of
found yet. reprocessing the fresher fibres found in waste
paper
69. According to the passage, in the recycling of D) do not consider the recycling of waste paper to
waste paper, ----. be a viable way of making profit
E) maintain that wood fibres make better paper
A) engineers have developed a chemical process to although it can cost much more
get rid of adhesives
B) it is essential that, among other chemical
procedures, de-inking is first to be completed to
enable fibres to bond strongly
C) there is actually no need to depend on pure pulp
since the fibres are adequately bonded
D) various countries have developed new
techniques except for an effective solution for the
removal of “stickies”
E) one serious and costly handicap is the problem of
adhesives that get stuck in the machines

70. It is maintained in the passage that although


more than 50 per cent of paper used is recycled
----.

A) there is still much dependence in the paper


industry on the use of wood pulp
B) the world’s paper industry is still in its early
stages and needs to upgrade itself in terms of
efficiency and cost
C) most countries regard this as minimal and,
therefore, encourage the establishment of more
paper plants
D) this is not enough to save forests from total
destruction due to widespread exploitation
E) engineers are working hard to develop new
technologies in order to increase the amount to
80 per cent

123
73. – 76. soruları aşağıdaki parçaya göre 76. In this passage, the writer ----.
cevaplayınız.
A) gives an account of the benefits that convection
Mount Everest is the highest mountain on Earth streams provide to the Hawaiian Islands
above sea level, but it is not the world’s tallest. That B) describes in detail the movements and effects of
honour goes to the Hawaiian volcano Mauna Kea. the tectonic plates under the Pacific Ocean
When measured from its base on the Pacific Ocean C) gives information about how Mount Everest was
floor, it is about 1,000 metres taller than Mount formed
Everest. Mauna Kea is part of a 5,600-kilometre-long D) explains how the volcanic chain extending across
chain of volcanoes stretching westward from the main the Pacific Ocean was formed
Hawaiian island. This volcanic chain is formed by E) states that different types of volcanoes will
small convection streams called “hot spots”, just continue to form across the Pacific Ocean
below the Earth’s crust, where magma rises from the
hotter parts of the mantle, the region between the
crust and the core of the earth. These hot spots melt
sections of the tectonic plates moving above them,
causing magma and bits of the molten plate to erupt
onto the sea floor. Over time, the lava accumulates,
forming a mountain that rises above sea level. The
moving tectonic plates carry the newly-formed
mountain away from its original location, as newer
volcanoes continue to form in the same spot.

73. One understands from the passage that, as


much of Mauna Kea is below sea level, ----.

A) nobody knows how high it actually is since it


cannot be measured precisely
B) it looks lower than Mount Everest, but in fact, it is
not when measured from its bottom
C) its volcanic activity is actually much more
extensive than is generally thought
D) the process of its geological formation is only
now being revealed scientifically
E) the so-called “hot spots” on the ocean floor
continue to add lava to its base

74. It is clear from the passage that the tectonic


plates in the Pacific Ocean ----.

A) move constantly and, thus, undermine the


formation of a volcanic chain in the region
B) prevent the formation of convection streams that
cause eruptions on the ocean floor
C) are so thick that the so-called “hot spots” have no
physical effect on them
D) play a part in the formation of volcanic chains on
the ocean floor
E) cover the mantle so well that no eruption of
magma can take place on the ocean floor

75. According to the passage, the lava which


erupts onto the Pacific Ocean floor ----.

A) mainly consists of magma but also includes small


pieces of the molten tectonic plate
B) is constantly dispersed in all directions because
of the movements of the tectonic plates
C) soon forms a chain of mountains that are
relatively high but hardly rise above sea level
D) flows from newly-formed mountains such as the
Hawaiian volcano Mauna Kea
E) can cover a very large area that may extend for
thousands of kilometres in all directions

124
77. – 80. soruları aşağıdaki parçaya göre 80. As one learns from the passage, pallasites and
cevaplayınız. mesosiderites ----.

Meteorites offer glimpses of the earliest stages of A) have their origins in various disrupted asteroids
planetary formation. Stony-iron meterorites come in including the asteroid Vesta
two main classes: pallasites and mesosiderites, and it B) provide us full knowledge of how planets were
was previously thought they may have had similar formed in the early solar system
origins. A new study, however, has revealed that their C) are the two major groups of meteorites that have
oxygen isotope properties differ and that they come a stony-iron nature
from distinct places. Accordingly, the characteristics of D) played a formative role in planetary enlargement
mesosiderites suggest they came from the third in the early solar system
largest asteroid, Vesta, which is the target of the E) have always remained a scientific mystery, which
NASA Dawn Mission. On the other hand, pallasites NASA is trying to unravel
are made of mixed core-mantle material from a
disrupted asteroid, indicating that extensive asteroid
deformation was an integral part of planetary TEST BĐTTĐ.
enlargement in the early solar system. CEVAPLARINIZI KONTROL EDĐNĐZ.

77. According to the passage, while scientists


think they know the asteroid from which
mesosiderites came, ----.

A) its oxygen isotope properties need to be fully


studied and explained
B) the original asteroid with which pallasites are
associated is not named
C) its characteristics are only now being revealed in
a series of new studies
D) there are conflicting views among scientists as
regards pallasites
E) the NASA Dawn Mission has not yet established
its position in the solar system

78. As one understands from the passage,


meteorites ----.

A) are scientifically useful because through them is


partially revealed the very early development of
planets
B) have been the major target of the NASA Dawn
Mission, which is primarily concerned with
planetary formation
C) are made up of the material that has come from
disrupted asteroids such as the asteroid called
Vesta
D) are known as either pallasites or mesosiderites,
both of which have the same physical properties
E) clearly show that, in the early solar system, every
asteroid underwent a process of structural
disruption

79. As is pointed out in the passage, the growth of


planets in the early solar system ----.

A) can only be understood through a close study of


mesosiderites rather than pallasites
B) was mainly due to the oxygen isotope properties
of certain meteorites
C) depended on mixed core-mantle material from
disrupted asteroids
D) was essentially influenced by Vesta, which is the
third largest asteroid
E) was closely connected with the large-scale
deformation of asteroids

125
1. – 18. sorularda, cümlede boş bırakılan yerlere 8. In 1972, two collective burials ---- under an
uygun düşen sözcük ya da ifadeyi bulunuz. overhanging rock at Qilakitsoq, a small Inuit
settlement on the west coast of Greenland, ---- to
1. Foreshadowing is an established literary ---- about A.D. 1475.
that adds layers of meaning to events or details in
films and literary works. A) are discovered / dated
B) were discovered / dating
A) reference B) indication C) device C) have been discovered / having dated
D) deception E) appreciation D) had been discovered / having been dated
E) were being discovered / having to be dated

2. Due to its geographical position, ---- historical


heritage, and strategic importance, Strasbourg 9. In 1989, Jennifer Johnson of Sanford, Florida,
was chosen as a seat for various European ---- the first woman ---- of transferring cocaine to
institutions. her unborn baby through the umbilical cord.

A) familiar B) rich C) fluent A) was becoming / having been convicted


D) elective E) subsequent B) has become / to convict
C) would become / to be convicting
D) became / to be convicted
3. The endlessly flat landscape and thousands of E) had become / convicting
miles of well-maintained cycle tracks make
cycling an ---- popular activity in the Netherlands.
10. Smoking ---- in almost all segments of the
A) additionally B) effectively C) ultimately American population, so that, in various polls, 60
D) extremely E) accurately to 65% of Americans ---- non-smokers today.

A) declines / may have been


4. The search for truth ---- rational guidelines, and, B) has declined / are
though some of our subjects may lie on the outer C) had declined / would be
limits of scientific research, we examine them D) could have declined / will be
through science’s lens. E) declined / were

A) demands B) relates C) reaches


D) orders E) establishes 11. If a new environmental or safety rule ---- in the
auto industry, executives warn that the company
---- money.
5. The term “geography” ---- the English language
in the 16th century from Latin and ultimately A) were to be proposed / had lost
Greek, at the time when European navigators were B) had been proposed / would have lost
beginning to explore Africa and the New World. C) will be proposed / is going to lose
D) were proposed / should lose
A) broke into B) turned out C) carried away E) is proposed / will lose
D) went on E) came into
12. Tibetan monks, who are experts ----
6. If this business goes on in such an awful way, it meditation, rid themselves of negativity ----
will ---- hundreds of unemployed. augmenting the brain’s gamma waves, which can
be measured in a lab.
A) do without B) hang around with
C) end up with D) go up against A) of / from B) on / about C) for / through
E) make out for D) in / by E) at / towards

7. Although there ---- some growth in the non-oil 13. One can see ---- a glance why small children
sector, Nigeria ---- dangerously reliant on are immediately attracted ---- these noisy,
hydrocarbon revenues. colourful toys.

A) has been / remains A) at / to B) in / by C) with / for


B) had been / would remain D) by / with E) of / to
C) was / would have remained
D) must be / has remained
E) is / had remained 14. Children can benefit from their experiences ----
they retain information from these experiences
over time.

A) that B) even though C) so long as


D) so E) while

126
15. ---- fiction gives something more than 19. – 23. sorularda, aşağıdaki parçada
pleasure, it hardly justifies itself as a subject of numaralanmış yerlere uygun düşen sözcük ya da
study. ifadeyi bulunuz.

A) Because B) Unless C) When Statutes are usually framed in more or less general
D) If E) After terms, and, in order to apply these general terms to
particular cases, interpretation becomes necessary.
(19) ---- the famous Byzantine Emperor Justinian but
16. ---- the average education level of the women also the framers of the Prussian and Napoleonic
in the United States is higher than that of their Codes (20) ---- to take away this power of
male counterparts, they are highly concentrated in interpretation from the judiciary. But these efforts and
underpaid and menial jobs. similar ones all failed because interpretation is (21) ---
whenever a written formula has to be applied to a
A) Although B) Now that C) Until particular case. When interpreting the meaning of a
D) As if E) In case legal rule, the judge must first discover the true facts
of the case before him, and secondly, he must find
out (22) ---- the legislator intended him to do (23) ----
17. Despite the passage of a decade since its the existing circumstances.
opening, the Maritime Museum in Istanbul
remains undeservedly little-known today ---- a lack 19.
of publicity. A) Not only B) Even if C) The more
D) Both E) As
A) instead of B) in spite of C) contrary to
D) as to E) owing to
20.
A) may have attempted
18. ---- in France, where it was a pleasure strictly B) attempted
limited to the aristocracy, in the British Isles C) are attempting
drinking chocolate was made available to the D) have been attempting
middle classes from the outset. E) could have been attempted
A) Except B) Rather C) Unlike
D) Just as E) Besides 21.
A) influential B) incomparable C) impressive
D) plausible E) unavoidable

22.
A) that B) which C) why
D) what E) whoever

23.
A) after B) towards C) on
D) over E) under

24. – 35. sorularda, verilen cümleyi uygun şekilde


tamamlayan ifadeyi bulunuz.

24. ----, but he had a reputation for blurring the


line between fact and fiction.

A) In the early twentieth century, some Americans


were still hunting whales much as they had in
Herman Melville’s day
B) For “Moby-Dick”, Herman Melville drew on
scientific, historical, and journalistic accounts of
whales
C) Once a whale washed ashore, it was bound to
end up as someone’s property
D) Whales entered early American law through the
question of who owned them when
E) There is no shortage of whaling histories for a
Melville aficionado to turn to

127
25. Whenever attitude researchers ask 29. As most businesses in Southeast Asia have
participants questions, ----. remained as family enterprises and not attained a
global dimension, ----.
A) researchers would have probably needed to
demonstrate that the scientific benefits of the A) in the wake of the 1997 financial crisis, Southeast
research outweighed the possible ethical costs Asia had been overtaken by China and India
B) this is especially so when a person’s attitude runs B) some are being left behind by foreign businesses
counter to a prevailing norm C) across Southeast Asia, the impact of the 1997
C) researchers have devised several techniques to crisis was closely related with the degree of
overcome such problems corruption in the banking system
D) the techniques often raised questions about D) today, South Korea and Taiwan are four times
research ethics, especially if participants did not richer than Malaysia and ten times richer than
know their attitudes were being measured Indonesia
E) there is the possibility that participants will be E) compared with other regions of the world, Asia’s
reluctant to reveal their true feelings income gap is slowly decreasing

26. Thanks to the popularity of nature 30. In a recently published paper, it is pointed out
documentaries on TV, ----. that China has produced much of the world’s rice
for many decades, ----.
A) many observers have noted high biological
productivity around seamounts and islands A) but in these rice paddies, nitrogen-based fertilizer
B) biologists know much less about the migration of has, to a large extent, replaced animal manure
marine species B) so it is another change in agricultural practice
C) the healthy populations of pelagic fish are in a that has the unintended side benefit of reducing
worldwide decline because of over-harvesting methane emissions
D) there has been an increase of environmental C) what is more, these rice farmers are using less
awareness among people from all walks of life water than they did before
E) in Africa every year, hundreds of thousands of D) yet for the past 30 years, the area devoted to rice
wildebeests and zebras leave their traditional agriculture there has fallen from about 37 million
habitats to avoid the dry season hectares to about 27 million
E) and this change in how rice is grown in China
reduces the amount of methane given off
27. ----, whose drums marked the hours of the
emperor’s day.
31. One of the first questions the archaeologist
A) Most Chinese emperors in the past led a must face ---- is to decide the cause of death.
prosperous life
B) In Beijing, during imperial times, no structure was A) since the area has been explored for remains
permitted to be taller than the Drum Tower B) whatever happens to bones between the time
C) The female members of the imperial family in they are deposited and dug up
China were not allowed to take an interest in the C) on which a great deal of work is currently
affairs of the State focussed
D) Of the numerous gardens within the Forbidden D) even though few scholars went further than
City, the Imperial Garden is the most accessible labelling animals as either wild or domestic
E) The Forbidden City in modern Beijing has been E) when he or she is interpreting animal remains
extensively renovated and, thus, made more
attractive for visitors
32. In 1993, the philanthropist Henry Buhl bought
a rare gelatin-silver print of a Stieglitz photograph
28. If teamwork were taught along with reading, of Georgia O’Keeffe’s hands, ----.
writing and mathematics, ----.
A) which was the beginning of a collection he
A) some of the research in this area would centre on continued to amass over the years
bad behaviours that degrade a team’s B) if he would have had over a thousand images of
performance hands
B) it is only after a great tragedy or triumph that the C) so the Guggenheim is exhibiting more than a
importance of teamwork is drawn into the hundred and seventy of them
spotlight D) when it is an interesting way to contemplate the
C) there would be enormous benefits to students history of photography, among other things
and society alike E) and they include a shot by Robert Capa and a
D) a crucial question that should be asked before portrait of Nusch Eluard by Dora Maar
putting a team together is whether you need one
at all
E) a successful team would deal with its members
who do not contribute anything to the group

128
33. Muğla has always preserved its traditional 36. – 38. sorularda, verilen Đngilizce cümleye
cultural fabric, ----. anlamca en yakın Türkçe cümleyi bulunuz.

A) if you’d like to take a journey through its history 36. Due to high fertility and the high number of
B) yet it constitutes a unique model for Turkish young people in the population, the proportion of
traditional architecture adolescents in the total global population is likely
C) although the luxury tourist centres that surround it to increase in the coming years.
have not
D) so it has links to resort towns such as Bodrum, A) Doğurganlık oranı ve genç nüfusun sayısı dikkate
Köyceğiz, and Marmaris alındığında, ergenlerin toplam küresel nüfus
E) because it started to urbanize rapidly with the içindeki oranının önümüzdeki yıllarda artma
founding of Muğla University olasılığı oldukça yüksektir.
B) Yüksek doğurganlık ve nüfus içindeki gençlerin
yüksek sayısı nedeniyle, toplam küresel nüfusta
34. Hierapolis was a famous spa in Roman times ergenlerin oranı, gelecek yıllarda muhtemelen
----. artacaktır.
C) Ergenlerin toplam küresel nüfus içindeki oranının
A) since the founder of the city was the emperor önümüzdeki yıllarda artması, yüksek
Antiochus ll of the Seleucid dynasty doğurganlığa ve nüfus içindeki genç sayısının
B) when it was founded in the second quarter of the yükselmesine bağlıdır.
3rd century B.C. D) Yüksek doğurganlığın ve nüfustaki gençlerin
C) because the only shrines that remain today are a yüksek sayısının sonucu olarak, gelecek yıllarda,
temple of Apollo and the cave-sanctuary of Pluto toplam küresel nüfus içinde ergen oranı giderek
D) just as Pamukkale is today yükselecektir.
E) even if these date from the Hellenistic period E) Yüksek doğurganlığa ve genç nüfusun artışına
bağlı olarak, yetişkinlerin toplam küresel nüfus
içindeki oranı, gelecek yıllarda kademeli bir
35. In the 1950s, particularly in the Unites States, şekilde yükselecektir.
logistics as a business concept began to gain
ground ----.
37. It has been estimated that the number of
A) whereas the need for expert logisticians became tourists visiting China will grow by the end of this
imperative decade to about 50 million.
B) that it was a branch of military science
C) because businesses expanded and reached out A) On yıl sonra, Çin’i ziyaret eden turist sayısında
both to far-flung markets and sources of meydana gelecek artışın 50 milyonun üstünde
materials, creating complex supply chains olacağı hesaplanmaktadır.
D) when the world of commerce as we know it today B) Önümüzdeki on yılın içinde, Çin’i ziyaret eden
would have been impossible without the logistics turist sayısındaki artışın aşağı yukarı 50 milyon
industry olacağı tahmin ediliyor.
E) in that logistics companies have become more C) Tahmin edileceği gibi, bu on yılın sonunda, Çin’I
sophisticated with today’s increasing ziyaret eden turist sayısı artarak yaklaşık 50
globalization milyonu bulacaktır.
D) Çin’i ziyaret eden turist sayısındaki artışın 50
milyona ulaşmasının yaklaşık on yılı bulacağı
tahmin ediliyor.
E) Çin’i ziyaret eden turist sayısının bu on yılın
sonuna kadar yaklaşık 50 milyona yükseleceği
tahmin edilmektedir.

129
38. All oil-producing African countries have 40. Đspanya’da on dokuzuncu yüzyıl sonları,
agreed in principle to set aside a certain yaygın siyasal yolsuzluklara bir tepki olarak
percentage of the revenues they earn from oil gelişen anarşizmle birlikte ulusal bir çöküş
exports in order to establish a common dönemiydi.
investment fund.
A) The later decades of the nineteenth century in
A) Petrol ihracatından sağladıkları gelirlerin belirli bir Spain were a period of national decline as well as
yüzdesini ayırarak ortak yatırım fonu kurmayı anarchism, which emerged as a reaction against
amaçlayan tüm petrol üreten Afrika ülkeleri ilke extensive political malpractice.
anlaşmasına varmışlardır. B) The late nineteenth century in Spain was a period
B) Ortak bir yatırım fonu oluşturmak için, petrol of national decline, along with anarchism
üreten Afrika ülkelerinin tümü, ilke olarak, petrol developing as a response to widespread political
ihracatından sağladıkları gelirlerle belirli bir corruption.
oranda katkı yapma konusunda anlaşmaya C) As a response to extreme political decadence,
varmışlardır. anarchism developed in Spain in the late
C) Belirli bir oranın üzerinde petrol üreten Afrika nineteenth century, which was a period of
ülkeleri, petrol ihracatından elde edilen gelirlerle national decline.
ortak yatırım fonu oluşturmak için ilke D) Anarchism in Spain came into being in the late
anlaşmasına varmış bulunuyorlar. nineteenth century, also known as a period of
D) Petrol üreten tüm Afrika ülkeleri, ortak bir yatırım national decline, as a response to widespread
fonu kurmak amacıyla, petrol ihracatından elde corruption in politics.
ettikleri gelirlerin belirli bir yüzdesini ayırmak için E) The end of the late nineteenth century in Spain
ilke olarak anlaşmışlardır. marked a period of national decline, which was
E) Petrol üreten Afrika ülkelerinin tümü, petrol also noted for anarchism developing as a
ihracatından elde ettikleri gelirlerin belirli bir struggle against extreme political corruption.
yüzdesini ayırarak ortak bir yatırım fonu
oluşturma ilkesinde de anlaştılar.
41. Öteki Avrupalılara kıyasla, Almanların daha
uzun yıllık tatilleri olduğu ve yurt dışı gezilere
39. – 41. sorularda, verilen Türkçe cümleye daha çok para harcadıkları bir gerçektir.
anlamca en yakın Đngilizce cümleyi bulunuz.
A) It is true that, compared with other Europeans,
39. 930’dan Norveç’le birleştiği 1262’ye kadar the Germans have longer annual holidays and
bağımsız bir cumhuriyet olan Đzlanda, spend more money on foreign travel.
bağımsızlığını 1944 yılında geri kazandı. B) It is admitted that, comparatively, the Germans
enjoy longer annual holidays than other
A) Though an independent republic since 930, Europeans and save a lot of money for their
Iceland was taken over by Norway in 1262 and foreign travels.
only regained its independence in 1944. C) The fact that, unlike other Europeans, the
B) Iceland, annexed by Norway in 1262, had been Germans are entitled to longer holidays each
an independent republic since 930, and became year and put aside more money for their travels
independent again only in 1944. abroad is undeniable.
C) Iceland had been an independent republic D) Truly, contrary to other Europeans, the German
between 930 and 1262 when it was invaded by people tend to have longer annual holidays and
Norway, becoming a republic again in 1944. are prepared to spend more money on their
D) Before it united with Norway in 1262, Iceland travels abroad.
had, as it is today and has been since 1944, E) When the Germans are compared with other
been an independent republic since 930. Europeans, it is clear that their annual holidays
E) Iceland, which was an independent republic from are longer and the money they spend for their
930 to 1262 when it joined with Norway, regained travels abroad is much more.
its independence in 1944.

130
42. – 46. sorularda, boş bırakılan yere, parçada 44. Photo interpretation in archaeology is the
anlam bütünlüğünü sağlamak için getirilebilecek process by which features photographed from the
cümleyi bulunuz. air, such as soil-marks, are analyzed in order to
deduce the types of archaeological structures
42. Unlike wine, whisky does not change after it causing them. ----. Such plans may guide
has been bottled. ----. The first rule to respect is to excavation to key points in a structure, place field-
keep whisky bottles upright so that the alcohol is collected data in context, or themselves be used
not in contact with the cork. It should also be kept as the starting point for new research.
away from all sources of heat in order to avoid
drying out the cork, which would allow air to get A) Archaeologists may use aircraft to search the
into the bottle. ground for traces of former sites in the landscape
B) Given that the visibility of features varies from
A) Keeping a young whisky in a bottle and putting year to year, photos taken over several years
an old whisky in a decanter combines aesthetic need to be studied to compile an accurate plan
pleasure with the joy of tasting C) The aerial photograph also highlights an
B) One question which is often asked, and over interesting earlier feature: a shallow Neolithic
which whisky lovers are divided, concerns ditch running across the middle of the fort
whether or not to use a decanter D) Soil marks may reveal the presence of varied
C) However, it is advisable to take some precautions ditches, banks or foundations
to assure its longevity E) Those who take and use aerial photographs must
D) The Classic Malts Collection owes its understand the means by which the evidence is
considerable success since its launch in 1988 as made visible in order to determine the type of
much to the quality of the whisky as to the feature that has been recorded
excellent design of the six bottles that make up
the range
E) Marketing and design have revolutionized the 45. Near-death experiences are relatively
whisky industry, especially the manufacture of stereotyped in format. ----. They often describe
containers sensations like “floating” or “flying”. These
experiences can also cause fear or a feeling of
detachment, but they are usually described as
43. The scientific study of facial expression has intensely “real” – not at all like a dream or a
largely focused on the way in which facial hallucination.
expressions communicate emotions. ----. In fact,
subsequent research generally identified six basic A) People who have had such experiences say that
emotions (happiness, surprise, sadness, fear, they seemed to be no longer in their own body
disgust and anger) and their related facial but outside it, looking down on themselves from
expressions, from which more complex or two or three metres above
blended emotions are derived. B) Survivors of these experiences may, weeks later,
experience a surge of emotionality, as if emotions
A) In this respect, Darwin suggested that there are of every sort were being stimulated or released
only a small number of universal emotions and C) Sometimes, near-death experiences are followed
that these are associated with universal facial by a sudden onset of musical or artistic interest
expressions D) Tony Cicoria reported seeing a bluish-white light
B) Similarly, tears are an acceptable display of that surrounded him and having a sense of
sadness at a funeral but not on hearing ecstacy
disappointing news in a business setting E) Similar near-death experiences have often been
C) These rules exist because of the important described by people who have been, or believed
communicative function of facial expressions themselves to be, in great danger
D) People from a variety of Western, Asian and tribal
cultures were remarkably accurate in identifying
the six emotions
E) For instance, the expression of emotion is
encouraged for women in Mediterranean cultures
but is discouraged for men in northern European
and Asian cultures

131
46. Adults who abuse children were almost always 49.
abused themselves, but not all people who were Della: - Did you know that Eugene O’Neill was the
abused during childhood become abusers. ----. son of a Shakespearean actor?
However, children who do not form such Hector: - ----
relationships are at high risk to become abusers Della: - That’s right. Instead, he started acting in
themselves. plays where he could make more money, and
Eugene spent his first seven years travelling with
A) Biological fathers are much more likely than him while he was acting in such a play.
mothers to abuse their children Hector: - I suppose that had an effect on the plays
B) These children often grow to adulthood that Eugene O’Neill would write in the future.
displaying aggressive behaviours, lacking the
ability to adequately express emotions, and A) No, I didn’t. Did Eugene O’Neill act as well?
having inadequate cognitive and problem-solving B) And he had a long and distinguished
skills Shakespearean career.
C) Older children and adolescents are even more C) Didn’t he have a brother who died at the age of
likely to be witnesses and victims of such two?
violence D) Yes, but didn’t his father give up acting
D) Many have experiences that compensate for the Shakespeare?
abuse, forming relationships that allow them to E) He won the Pulitzer Prize a year after his death,
feel valued and safe in some ways and grow up for Long Day’s Journey into Night.
to become good parents
E) School-age children and adolescents who are
abused may exhibit symptoms of 50.
psychopathology or behaviour disorders Marian: - Do you remember China’s “one child”
policy, where each family was allowed to produce
only one child in order to reduce the country’s
47. – 51. sorularda, karşılıklı konuşmanın boş population?
bırakılan kısmını tamamlayabilecek ifadeyi Helen: - Yes, of course I do. The policy has created
bulunuz. a huge imbalance in the male-to-female
population ratio, due to the fact that many couples
47. abort their female foetuses.
Steve: - It’s clear that, during his time in office, Marian: - ----
former US President John F. Kennedy inspired Helen: - It did, but only recently, and many couples
many people to enter public service. are still doing it even though it’s against the law.
Gary: - ----
Steve: - Actually, some people think that his A) I think that abortion should be allowed only in
extramarital affairs and his elegant style made him medical emergencies.
more interesting, so that people wanted to follow B) How can they bring themselves to do that?
his call to serve their country. C) Why don’t they want to have a girl?
Gary: - You may be right about that. D) Didn’t the couples think before deciding to have a
child?
A) What about his contribution to American civil E) But I thought the Chinese government made
rights? abortion illegal.
B) I’ve read he was a near-professional level golfer,
but didn’t want the public to know it.
C) I think the public loved his wife Jackie as much as 51.
the president himself, didn’t they? Geneveive: - Did you know that nuclear weapons
D) There’s no strong evidence that he was are regularly transported by truck to inspection
assassinated as the result of a conspiracy. facilities in the US and the UK?
E) But what about his reputation as a wealthy Carl: - ----
womanizer? Didn’t that work against him? Geneveive: - Yes; it’s extremely dangerous. If there
were a traffic accident and some of the weapons
were to explode, many people would die.
48. Carl: - I suppose the trucks would also be a target
Geri: - Did you know that there’s a new kind of test for terrorist attacks, wouldn’t they?
that shows us our true ideas about certain topics?
Alfred: - ---- A) Isn’t that a bit risky?
Geri: - It’s more than that; this test can show us B) Why do they do that?
opinions and prejudices that we didn’t even know C) What is your opinion of this practice?
we had! D) I don’t support the idea of nuclear weapons,
Alfred: - Well, it sounds interesting. Where did you anyway.
find this test? E) Can’t the weapons inspectors travel instead of
transporting the weapons?
A) What’s the test called?
B) Oh, you mean like a personality test?
C) How do you know that the test is reliable?
D) Have you taken the test yet? What were the
results like?
E) You’re really interested in this kind of thing, aren’t
you?

132
52. – 56. sorularda, cümleler sırasıyla 56. (I) In the film Amistad, Steven Spielberg
okunduğunda parçanın anlam bütünlüğünü bozan expresses his outrage at the institution of slavery. (II)
cümleyi bulunuz. That is, he makes something that enables the film
audience to feel the same kind of outrage toward
52. (I) Why do people pursue self-esteem? (II) Most slavery that he feels. (III) Clearly, the artist at the start
people would answer that having self-esteem makes felt deeply about slavery. (IV) He then set out to give
you feel good. (III) They found that participants who to his audience a similar feeling. (V) On the contrary,
had their self-esteem raised had lower psychological these configurations stimulate differing emotional
arousal and reported less anxiety. (IV) There is states in the audience.
probably a grain of truth here, but on the other hand,
there are causality issues to be addressed. (V) So A) I B) II C) III D) IV E) V
actually, rather than self-esteem producing
happiness, feeling happy may inflate self-esteem.

A) I B) II C) III D) IV E) V

53. (I) A longstanding issue in European prehistory is


that of the so-called megalithic monuments. (II)
Similarly, in the 19th century, megaliths were seen as
the work of a single group of people, who had
migrated to western Europe. (III) These are
impressive prehistoric structures built of large stones.
(IV) In general, the stones are arranged to form a
single chamber, buried under a mound of earth and
entered from one side, and the chambers may be
large with a long entrance passage. (V) Human
remains and artifacts are usually found within these
structures, and it is clear that most served as
collective burial chambers, i.e., tombs for several
people.

A) I B) II C) III D) IV E) V

54. (I) Why is it important for engineering students to


study engineering ethics? (II) Several notorious cases
that have received a great deal of media attention in
the past few years have led engineers to gain an
increased sense of their professional responsibilities.
(III) Most engineers are, on the whole, motivated by
the size and social impact of their projects. (IV) These
cases have led to an awareness of the importance of
ethics within the engineering profession as engineers
realize how their technical work has far-reaching
impacts on society. (V) Indeed, the work of engineers
has always affected public health and safety and
influenced business practices and even politics in
society.

A) I B) II C) III D) IV E) V

55. (I) Good reading involves criticism, for choice


necessitates judgement. (II) In a good story every
element works with every other element for the
accomplishment of the central purpose. (III) Yet there
are no easy rules for literary judgement. (IV) Such
judgement depends ultimately on our perceptivity,
intelligence, and experience. (V) It is a product of how
much and how alertly we have lived and how much
and how well we have read.

A) I B) II C) III D) IV E) V

133
57. – 60. soruları aşağıdaki parçaya göre 60. As can be understood from the passage, the
cevaplayınız. laws of a country, ----.

We commonly speak of both law and laws, and these A) though varied and numerous, embody a common
terms, though not used with precision, point to two goal for the country’s peace and prosperity
different aspects under which legal science may be B) which lawyers are expected to interpret precisely,
approached. The laws of a country are thought of as indicate the efficiency of the legal system
separate, distinct, individual rules; the law of a C) whether understood properly or misinterpreted,
country, however much we may analyse it into are necessary for the well-being of society
separate rules, is something more than the mere sum D) which can also be understood accurately by
of such rules. It is, rather, a whole, a system which laymen, are seldom related to each other
orders our conduct and in which the separate rules E) though separate and distinct from each other, are
have their place and their relation to each other and to essentially interrelated
the whole. Moreover, it is never completely exhausted
by any analysis, however far the analysis may be
pushed, and however much the analysis may be
necessary to our understanding of the whole. Thus,
each rule which we call a law is part of the whole we
call the law. Lawyers generally speak of law; laymen
more often of laws.

57. It is pointed out in the passage that, in legal


science, the term “law” ----.

A) is very ambiguous and cannot be clearly defined


due to the variety of meanings it has
B) signifies a whole which, as a system, embodies
all the separate rules referred to as laws
C) refers to the different aspects of a country’s legal
system and, hence, is difficult to explain
D) has always been controversial and, therefore,
cannot be interpreted accurately
E) has a limited meaning that does not deserve any
analysis or understanding

58. As regards the use of the terms “law” and


“laws”, the writer claims that ----.

A) there needs to be a common approach among


the lawyers of a country
B) lawyers and the people outside the legal
profession share the same view
C) each country has a different approach because of
its distinct legal system
D) it is not lawyers but laymen who need to be more
precise
E) there is not much clarity since both terms are
often confused

59. As is emphasized in the passage, there can be


no limit to ----.

A) the political laws that constitute the totality of a


country’s legal system
B) the number of the laws in a country that are
distinct from each other
C) an analysis of a country’s law, however extensive
the analysis may be
D) what meanings and interpretations laymen can
make of the laws of a country
E) the variety of laws that can be included within the
law of a country

134
61. – 64. soruları aşağıdaki parçaya göre 63. It is suggested in the passage that although
cevaplayınız. Barrett’s book was published four years after its
initial planning in 2003, ----.
Four years ago in 2003, when Paul Barrett first began
planning an ambitious book on Muslims in America, A) it has captured the attention of the reading public
who would have thought that the topic would still be of due to the events that took place during this
such urgent interest by the time it was published early period
this year? But, if anything, intervening events have B) the facts that it relayed about Muslims in America
made Barrett’s exploration of American Muslims more had already been revealed to the reading public
timely and important than ever. With the United States C) the war in Iraq was deliberately excluded from it
even more deeply embroiled in warfare in Iraq and because of the uncertainty about the outcome
Afghanistan, with sectarian conflict exploding within D) its predictions about the consequences of the
the Muslim world, with tensions high among Muslim American involvement in the Middle East were
populations in Europe, and with relations strained proven by the developments during this period
between some leaders of the Christian and Islamic E) it dwelt solely on a comparison of Christianity and
religions, readers need as much information as they Islam, leaving out a number of other important
can get about Islam and its adherents. In his book, current issues
Barrett notes that, despite being targets of suspicion
in the wake of the attacks of September 11, Muslims
in America, as a group, offer a perfect illustration of 64. According to the passage, there are several
old-fashioned American assimilation. Overall, they are reasons ----.
prosperous, well-educated, politically active, and
successful in business and the professions. A) why the relations between the Christian and
Muslim leaders in Europe have been broken
61. It is clear from the passage that Paul Barrett’s B) that can be put forward for the American
book ----. involvement in Iraq and Afghanistan
C) for the sectarian conflicts that are taking place in
A) presents a socially, politically and economically Iraq and elsewhere in the Muslim world
very positive picture of Muslims in America D) why readers should be attracted by any study,
B) focuses completely on the political achievements such as Barrett’s, related to Islam and Muslims
of Muslims in America E) for the war in Iraq, which has already increased
C) seems to present a somewhat cynical view of the tensions between the United States and Europe
way of life of American Muslims
D) is mainly concerned with how the wars in Iraq
and Afghanistan have affected American Muslims
E) has failed to arouse any significant interest
among American Muslim readers

62. As one understands from the passage, in the


aftermath of September 11, Muslims in America
----.

A) experienced a wide range of economic handicaps


and political setbacks
B) formed a group that ambitiously pursued their
professional objectives
C) began to resist full assimilation into American
society and culture
D) began, according to Barrett, to be regarded as
potentially dangerous
E) were involved in sectarian conflicts that divided
them politically

135
65. – 68. soruları aşağıdaki parçaya göre 68. The point is made in the passage that
cevaplayınız. Kierkegaard ----.

Kierkegaard was one of the most original thinkers of A) approved of certain metaphysical arguments
the nineteenth century in Europe. He wrote widely on concerning the relation of thought to existence
religious, philosophical, and literary themes. However, B) relied heavily on Kant and Hegel for the
his peculiar manner of presenting some of his leading development of his own philosophy
ideas initially obscured their fundamental significance. C) was a thinker, some of whose ideas were
He developed his views in strong opposition to unusual for his time
prevailing opinions, such as certain metaphysical D) clearly defined what existentialism meant for a full
claims about the relation of thought to existence. He understanding of human existence
reacted against the ethical and religious theories of E) benefited a great deal from the philosophical
Kant and Hegel. Moreover, he opposed the doctrines legacy of his predecessors
and ideas which were being advanced by some of his
contemporaries like Feuerbach and Marx. His
discussion of the human condition, which emphasizes
the significance of individual choice, has arguably
been his most striking philosophical legacy,
particularly for the growth of existentialism.

65. It is clear from the passage that, in view of his


focus on the question of individual choice,
Kierkegaard can be regarded as ----.

A) the only thinker in the nineteenth century who


was concerned with metaphysical matters
B) a follower of Kant and Hegel, whose ethical and
religious ideas inspired him greatly
C) the philosopher who led the way for the
development of existentialist philosophy
D) the philosopher who explained the religious and
philosophical aspects of human existence
E) a major thinker who influenced his
contemporaries, including Feuerbach and Marx

66. It is pointed out in the passage that much of


Kierkegaard’s most striking writings ----.

A) restate the views and theories which prevailed


and were commonly shared in his time
B) were based on the ethical and religious theories
that had been formulated during earlier centuries
C) were widely read and discussed by his
contemporaries in Europe in the nineteenth
century
D) are limited to topics related to religion and
philosophy
E) deal with the idea of individual choice

67. It is suggested in the passage that because


Kierkegaard expressed his major ideas in an
unusual personal way, ----.

A) their essential importance was not recognized at


first
B) he was widely appreciated and fully understood
by his contemporaries
C) he immediately became famous as a philosopher
in nineteenth-century Europe
D) they were very influential on other philosophers,
including Feuerbach and Marx
E) most thinkers in Europe sensed that he was
formulating a new philosophy

136
69. – 72. soruları aşağıdaki parçaya göre 72. The writer states that the customs, values, and
cevaplayınız. beliefs of a society ----.

We have all heard people from other countries A) may have some impact on its educational system
described in very general terms. For instance, it has B) are a poor guide to the nature of that society
been said that “Germans work hard” and “Americans C) should be excluded from its political system
are friendly”. Such generalizations or stereotypes are D) must always uphold the role of women in that
very crude, and common sense tells us that not all society
Germans work hard and not all Americans are E) are among the elements that make up its culture
friendly. At the same time, there appears to be some
truth in these generalizations since people from
different countries share different characteristics.
What these crude statements acknowledge, however,
is that people from different countries have distinctive
cultures and social customs. A society’s culture
includes its customs, values, beliefs, ideas and the
artifacts it produces. Attitudes towards such things as
work, leisure, wealth, the role of women, and the
value of education in one society’s culture might be
significantly different from the attitudes and values
found in another society’s culture. This is also the
case regarding attitudes found in different countries
towards politics and the political system.

69. It is stressed in the passage that each country


----.

A) attaches much importance to its own social


customs and educational system
B) has developed a political system which may differ
from its social and cultural values
C) can be identified through its people’s attitudes
towards other countries and peoples
D) has its own special characteristics and, hence, is
culturally and politically unlike the others
E) shares with other countries a wide variety of
values and attitudes

70. In the passage, the writer points out that while


on the one hand, describing other countries or
peoples in general terms may be misleading, on
the other, ----.

A) generalizations of this kind may be true to some


extent
B) one must make an effort to appreciate and
understand their culture
C) it is important for us to have a friendly attitude
towards them
D) we must do our best to ignore the differences
among them
E) such generalizations are useful especially in
understanding the value of education

71. The writer suggests that political attitudes ----.

A) in a country are fundamentally influenced by


education
B) vary greatly from country to country
C) in a society have nothing to do with its cultural
values
D) towards women in society must be constructive
E) in a country cannot be differentiated from the
attitudes towards work and leisure

137
73. – 76. soruları aşağıdaki parçaya göre 75. The writer asserts in the passage that the
cevaplayınız. demographic and territorial size of the British
Empire ----.
The way in which British people view Britain’s role in
the world is still influenced by its past. Today Britain is A) remained unchanged right into the 1960s, when
an important regional power, but in the recent past it some colonies began to gain their independence
was a world power. Until World War II, Britain ruled B) was the cause of much hostility from Germany
the largest empire that the world has ever known. and other European countries
Incredible as it may seem today, during the 1920s, C) led American leaders into thinking that Germany
almost one-fifth of the world’s population lived under was not a major force in world politics
British rule. But the empire disappeared rapidly during D) was an indication of the fact that Britain had a
the 1940s, 1950s and 1960s as the colonies that had large number of colonies seeking independence
made up the empire gained their independence. In E) was so vast that no other empire in world history
the aftermath of the empire, British leaders were not was ever a match for it
very successful in adapting to play a much smaller
part in world affairs. However, Britain’s allies made it
clear that they no longer saw Britain as a major force 76. In the passage, the writer ----.
in world politics. The “special relationship” which was
said to exist between Britain and the United States A) depicts a very distorted picture of Britain’s
weakened as other European countries, particularly colonies in their struggle for independence
Germany, recovered after World War II. So, whereas B) points to the fact that Britain is still an important
London had been recognized by American leaders as European power
the “capital” of Europe during the 1940s and the C) explains how American political leaders
1950s, Bonn was seen as the new capital of Europe deliberately undermined Britain’s relations with
during the 1960s. Germany
D) approves the political strategies formulated by
British leaders in the aftermath of World War II
73. The point is made in the passage that, during E) discusses the economic reasons for the decline
the period following World War II, ----. of the British empire

A) London and Bonn were equally regarded by the


world as the major political centres of Europe
B) the American attitude towards Britain was still
one of admiration and strong cooperation
C) the Americans followed a policy of neutrality
towards Britain and Germany
D) Britain lost the empire and so ceased to be a
leading power in the world
E) Britain and the United States formulated a set of
new policies to strengthen their relationship

74. The writer claims that British people ----.

A) do not share the policies adopted by other


European countries towards the United States
B) attach great importance to relations with the
United States
C) think of Britain’s place in world politics today in
terms of their imperial past
D) still regard the former colonies, along with the
United States, as their lasting and strongest allies
E) no longer think that the political legacy of their
empire has any significance whatsoever

138
77. – 80. soruları aşağıdaki parçaya göre 79. It is emphasized in the passage that although
cevaplayınız. investors are excited by the stock market
situation in emerging markets, they ----.
Developing markets, historically the domain of
hyperinflation and political manipulation, now enjoy A) are always prepared for a risk of hyperinflation
high surpluses, thanks to record commodity prices which could send stock prices tumbling
and severe fiscal discipline. Since 2001 these B) know that these markets now represent a quarter
economies have achieved three times the average of global output
annual per-capita economic growth of their developed C) must be aware of the fact that these markets can
counterparts and now represent a quarter of global be risky since they are potentially very fragile
output. Stocks in emerging markets are causing much D) feel that severe fiscal discipline introduced by
excitement among investors. However, too much governments puts their investments at high risk
excitement invites peril. Emerging markets have E) are very critical of governments’ meddling with
undeniably changed in the past decade, but lately companies and neighbouring countries
they are looking overgrown, and even a minor crisis
could send them tumbling. And while the potential
triggers for a fall have changed, they are still there. As 80. According to the passage, as the economic
economies in the developing world get stronger, strength of the developing world increases, one of
governments are getting more assertive and meddling the drawbacks this leads to is that ----.
with both companies and neighbouring countries,
increasing political risk. A) prices of commodities and stocks reach higher
levels, opening the way to hyperinflation and
77. It is clear from the passage that, over the last political manipulation
few years, ----. B) governments begin to interfere in the affairs of
their neighbours and, hence, may cause political
A) the economies of the developed countries have risk
had so much growth that its effects on emerging C) governments get into fierce competition with each
markets have been harmful other in order to double their share in global
B) stock prices in emerging markets have been output
relatively stable due to very harsh fiscal D) investors begin to be seriously concerned about
measures introduced by governments the future of stock and commodity prices
C) inflation rates in the developing world have E) the annual per-capita growth slows down owing
shown an upward trend because of political to high surpluses and record commodity prices
manipulation and poor economic performance
D) emerging economies have performed so well that
their growth has been much higher than the TEST BĐTTĐ.
growth achieved by the developed world CEVAPLARINIZI KONTROL EDĐNĐZ.
E) governments of developing economies have
taken every measure in order to avoid the kind of
political crisis that may trigger an economic
collapse

78. As one learns from the passage, despite their


current economic success, ----.

A) emerging markets are not trusted by companies


since there is always the possibility of political
manipulation
B) developing economies are still far from
overcoming hyperinflation
C) governments in the developing world have
maintained their fight against hyperinflation
D) developed economies remain unconcerned about
the potential of a severe crisis
E) the countries in the developing world have
traditionally suffered from hyperinflation

139
1. – 18. sorularda, cümlede boş bırakılan yerlere 8. During pregnancy, lead ingested by the mother
uygun düşen sözcük ya da ifadeyi bulunuz. ---- across the placenta, ---- severe damage on the
developing foetal nervous system.
1. Damage to the liver is the main health ---- for
long-term heavy drinkers. A) has moved / to have inflicted
B) moves / inflicting
A) consideration B) conclusion C) attentiveness C) had moved / having inflicted
D) examination E) regulation D) would move / to be inflicted
E) moved / having been inflicted

2. Health risks from pesticide exposure are


probably small for healthy adults, but children, the 9. The first evidence that there are genetic factors
elderly, and people with compromised immune in smoking ---- in the 1950s from studies which ----
systems may be ---- to some types of pesticide that identical twins tended to be more similar in
poisoning. their choice to smoke or not than did fraternal
twins.
A) hostile B) substantial C) severe
D) reversible E) vulnerable A) could appear / would indicate
B) has appeared / had indicated
C) had appeared / have indicated
3. The amount of alcoholic drinks a person can D) would appear / were indicating
consume safely is ---- individual, depending on E) appeared / indicated
genetics, health condition, sex, body composition,
age, and family history.
10. In an article published in 1990, scientists of
A) violently B) worthily C) offensively the National Institute of Medical Research ----
D) highly E) loudly more than 300 biological theories that ---- to
account for senescence – the progressive and
general deterioration that accompanies aging in
4. For people who have a family history of a humans.
genetic disorder but no symptoms, a predictive
test can help ---- a person’s risk for developing the A) were reviewing / attempted
disorder in the future. B) have reviewed / should attempt
C) reviewed / had attempted
A) sustain B) remove C) determine D) would review / have attempted
D) arouse E) represent E) may have reviewed / attempt

5. If adenosine triphosphate (ATP) supply does 11. If she ---- energy-spending activities into her
not ---- demand, muscle contraction ceases. daily routine when she was younger, she ---- a
weightcontrol programme today.
A) make up for B) keep up with C) account for
D) run out of E) give up A) could incorporate / doesn’t have to attend
B) incorporated / didn’t have to attend
C) would incorporate / wouldn’t have had to attend
6. A wise vegetarian does not solely ---- the D) had incorporated / wouldn’t have to attend
products made of textured vegetable protein, but E) was able to incorporate / hadn’t had to attend
learns to use a variety of whole foods instead.

A) set out B) take along C) draw up 12. ---- 1992 and 1998, total lung cancer deaths in
D) rely on E) make for the United States declined 1.6% per year, but
almost all of this decline was due to a decrease ---
- 2.7% annually for men.
7. Insomnia is a difficulty in falling or staying
asleep or a disturbance in sleep that ---- people A) During / in B) Through / at
feel as if they ---- insufficient sleep when they C) Over / about D) Among / under
awaken. E) Between / of

A) makes / have had


B) made / have 13. People with an avoidant personality are
C) has made / should have oversensitive ---- rejection, and they have a strong
D) had made / had had desire ---- affection and acceptance.
E) would make / had
A) with / in B) to / for C) about / over
D) through / with E) for / about

140
14. ---- the immune system is intricate, its basic 19. – 23. sorularda, aşağıdaki parçada
strategy is simple: to recognize the enemy, numaralanmış yerlere uygun düşen sözcük ya da
mobilize forces and attack. ifadeyi bulunuz.

A) Although B) If C) Because It has been nearly two years since the last cases of
D) In case E) When SARS were reported in China. (19) ----, a new
affliction has risen to take its place as a more deadly
pandemic – avian flu. Also known as H5N1, this
15. Scientists are developing various influenza virus is endemic to waterfowl and has
countertechnologies, from vaccines made of DNA shown a disturbing propensity (20) ---- the past nine
to proteins that prime our blood to attack the years to infect chickens and human beings. Avian flu
malaria parasite ---- it is still inside the mosquito. is the Ebola of the poultry world, a haemorrhagic fever
that (21) ---- much bleeding from every orifice of its
A) so long as B) as soon as C) while winged victims. It leads to extensive destruction of
D) just as E) whether these animals in (22) ---- two days. When the virus
jumps to humans, it (23) ---- very noticeable at first,
but in fact, has a fatality rate as high as 33 per cent.
16. According to geneticists, genetic testing (also
called genetic screening) is useful ---- it is 19.
presented in such a way that people can A) As a result B) Accordingly C) Moreover
understand what the limitations of the tests are D) Likewise E) Since then
and what the results mean.

A) whereas B) only if C) even if 20.


D) so that E) before A) over B) through C) at
D) by E) from
17. Most patients want to know even bad
prognoses, but ---- a physician tells a given 21.
patient should be determined primarily by the A) upholds B) spreads C) causes
patient, not the physician. D) implements E) consists
A) how long B) how many C) how often
D) how much E) how few 22.
A) just B) any C) most
D) several E) still
18. Excess vitamin A can cause toxicity, ---- it is
taken all at once (acute toxicity) ---- over a long
period of time (chronic toxicity). 23.
A) was not B) has not been C) must not be
A) whether / or B) not only / but also D) is not E) ought not to be
C) both / and D) neither / nor
E) so long as / and
24. – 35. sorularda, verilen cümleyi uygun şekilde
tamamlayan ifadeyi bulunuz.

24. Whenever doctors consider prescribing a


drug, ----.

A) infrequent headaches or muscle strains can be


relieved with over-the-counter drugs
B) they must weigh the possible risks against the
expected benefits
C) infants and very young children are at special risk
of adverse drug reactions
D) it’s necessary to accept a higher risk of a severe
drug reaction to treat a life-threatening disease
E) potential benefits and risks were seldom able to
be determined with mathematical precision

141
25. ----, but the question of its effect on health is 29. Even if tumours grow in and disrupt the brain,
less clear. ----.

A) Some early research indicated that cycles of A) cancers are named according to the organ or
weight loss and gain presented risks for blood tissue in which they originate
pressure and cardiovascular disease B) cancer cells may continue to grow until they kill
B) The very thinnest and the very heaviest people the organism
seem to be at greatest risk for all-cause mortality C) they can often be removed completely by surgery
C) Gaining weight is a risk for health D) certain brain diseases change the pattern of brain
D) Being severely obese places a person at an waves
elevated risk for several types of health problems E) symptoms result when brain tissue is destroyed
and premature death or pressure builds on the brain
E) Obesity is undesirable from a social point of view

30. Anaemia means a deficiency of red blood


26. Because adults dislike hearing babies cry, ----. cells, ----.

A) infants sometimes stop crying if they have A) whereas the body replaces the plasma within 1 to
interesting things to watch 3 days after rapid haemorrhage
B) the infant uses basic techniques for getting the B) but red blood cells are delivered from the bone
caregiver to come closer marrow into the circulatory system
C) babies learn very early to control their social C) so in sickle cell anaemia, the cells contain an
environment abnormal type of haemoglobin called
D) they try various techniques for soothing them haemoglobin S
E) the crying of healthy newborn infants is fairly D) which can be caused by either too rapid a loss or
characteristic in both pitch and rhythm too slow a production of red blood cells
E) yet a person cannot absorb enough iron from the
intestines in chronic blood loss
27. Even though there is widespread publicity
about the genetic causes of cancer, ----.
31. Cholesterol is notorious as a possible factor in
A) genetics actually plays a fairly minor role in the heart disease, ----.
development of cancer
B) scientists estimated that lung cancer was nine A) so that the body regulates lipoprotein levels in
times higher among smokers than non-smokers several ways
C) epidemiologists generally agree that sufficient B) because each type of lipoprotein serves a
research evidence exists for infection different purpose
D) the leading cause of cancer deaths for both C) but about one in 500 babies inherits a disease
women and men continues to be lung cancer called hypercholesterolemia
E) these genes protect against cancer by providing D) although it circulates in the blood, mainly in
the code for a protective protein particles called low-density lipoproteins
E) yet it is essential for the functioning of all our cells

28. When people do not have insurance to obtain


health care, ----. 32. People of all sizes eagerly try the best diet
ever on the market, ----.
A) this reluctance has consequences for the
management of their diseases A) in case they cannot afford to consult a physician
B) they are less likely to have regular medical B) hoping that this one will really work
attention and more likely to have a chronic health C) as some diets do not offer a safe and effective
problem plan for weight loss
C) they need to be regularly checked for any serious D) in which they often offer distorted bits of
disorder legitimate research
D) they have a wide experience of receiving health E) so that they do not have to conduct credible
care, from the time they are children to the time research on the benefits or dangers of their diet
they are adults
E) medical dominance began to decline and other
types of health care providers became more 33. A team of medical researchers were granted
prominent financial support to explore ----.

A) whether there was a connection between autism


and a newly discovered class of nerve cells in the
brain called “mirror neurons”
B) although these observations must be taken into
account in any final explanation of autism
C) that the most conspicuous feature of this disorder
has been a withdrawal from social interaction
D) when further investigations also supported the
existence of mirror neurons
E) if they have continued to investigate mirror
neurons in different species
142
34. ----, the jaw should be held in place with the 38. Since salts are lost from the body daily
teeth together and immobile. through sweat, urine and faeces, they must be
replaced by dietary intake.
A) As many jaw fractures can be repaired surgically
B) Just as antibiotics are usually given to a person A) Ter, idrar ve dışkı ile her gün vücuttan kaybedilen
with a jaw fracture tuzların telafi edilmesi ancak besin alımı yoluyla
C) Even though fractures can cause internal olur.
bleeding B) Her gün alınan besinler, vücudun ter, idrar ve
D) If a jaw fracture is suspected dışkı yoluyla kaybettiği tuzların telafi edilmesini
E) While a blow powerful enough to fracture the jaw sağlar.
may cause bleeding within the skull C) Tuzlar, her gün vücuttan ter, idrar ve dışkı yoluyla
kaybedildiği için, besin alımı ile telafi edilmelidir.
D) Her ne kadar ter, idrar ve dışkı yoluyla vücuttan
35. Cancer cells are immortal; they can go on her gün tuz kaybedilse de besin alımıyla bu kayıp
dividing indefinitely ----. telafi edilir.
E) Vücuttan ter, idrar ve dışkı ile her gün önemli
A) once the body’s immune system normally miktarda tuz kaybedilmesine rağmen, bu tuzlar
recognizes a transformed cell as abnormal besin alımı yoluyla hemen telafi edilir.
B) but chemotherapy is used to treat metastatic
tumours
C) as long as they have a supply of nutrients 39. – 41. sorularda, verilen Türkçe cümleye
D) because this process has already been controlled anlamca en yakın Đngilizce cümleyi bulunuz.
E) insofar as many tumours, luckily, can be treated
39. Tiroid bezi işlevini yerine getirmediği zaman,
bazal metabolizma hızı yaklaşık % 40 düşer.
36. – 38. sorularda, verilen Đngilizce cümleye
anlamca en yakın Türkçe cümleyi bulunuz. A) Invariably, the basal metabolic rate falls to 40%
as soon as thyroid function is reduced.
36. Even when we are not moving, our muscles B) When the thyroid gland does not carry out its
are in a state of partial contraction known as function, the basal metabolic rate is reduced by
“muscle tone”. about 40%.
C) Since there is no thyroid function, the basal
A) Hareket etmediğimizde bile, kaslarımız “kas metabolic rate has dropped to a level of 40%.
tonüsü” olarak bilinen kısmî bir kasılma D) The reduction of the basal metabolic rate to a
durumunda olur. level of 40% is inevitable only if the thyroid
B) Hareket etmesek bile, bazı kaslarımız kısmen ceases to function.
kasılır ve bu duruma “kas tonüsü” denir. E) A 40% reduction of the basal metabolic rate
C) “Kas tonüsü” olarak bilinen kısmî kasılma occurs when the thyroid’s function is
durumu, kaslarımız hareket etmediği zaman compromised.
oluşur.
D) “Kas tonüsü”, hareket etmediğimiz halde .
kaslarımızda oluşan kısmî kasılma durumuna 40. Bebeklerin büyüme ve gelişmesinin, yedikleri
verilen addır. yiyeceklerden etkilendiğini hepimiz biliyoruz.
E) Eğer hareket etmezsek, kaslarımız “kas tonüsü”
olarak bilinen kısmî kasılma durumunu sürdürür. A) As far as we know, the kind of food babies eat
has much influence on their growth and
development.
37. Brain activity can be studied by measuring B) All of us know that the food babies eat can have
and recording the “brain waves” given off by some adverse effect on their growth and
various parts of the brain when they are active. development.
C) We are all aware of the fact that babies’ growth
A) Beyin faaliyetini tetkik etmek için ölçülen ve and development depend a great deal on the
kaydedilen “beyin dalgaları”, beynin çeşitli kinds of food they consume.
bölümleri tarafından faal oldukları zaman yayılır. D) We all know that the growth and development of
B) Beynin faal olan çeşitli bölümlerinden yayılan babies are influenced by the food they eat.
“beyin dalgaları”nı ölçerek ve kaydederek, beyin E) It is well known to us all that whatever food
faaliyetlerini tetkik etme olanağı vardır. babies consume essentially affects their growth
C) Beyin faaliyetlerinin tetkik edilmesi demek, beynin and development.
o an faal olan bölümlerinin yaydığı “beyin
dalgaları”nın ölçülüp kaydedilmesi demektir.
D) Beyin faaliyetini, beynin çeşitli bölümlerince faal
durumda iken yayılan “beyin dalgaları”nı hem
ölçerek hem de kaydederek tetkik etmek
mümkündür.
E) Beyin faaliyeti, beynin çeşitli bölümleri tarafından
faal oldukları zaman yayılan “beyin dalgaları”nı
ölçerek ve kaydederek tetkik edilebilir.

143
41. Çoğu sindirim enzimi, yalnızca, sindirim 43. Before treatment was available, most people
kanalında yemek bulunduğu zaman üretilir. who had acute leukemia died within 4 months of
diagnosis. Now many people are cured. ----. The
A) While food is passing through the digestive tract, disease returns in many, but 50 per cent of
a number of enzyme secretions take place. children show no signs of the leukemia 5 years
B) As soon as food enters the digestive tract, many after treatment.
digestive enzymes are produced.
C) Most digestive enzymes are produced only when A) These symptoms include weakness and
food is present in the digestive tract. shortness of breath, resulting from too few red
D) Several kinds of enzymes are secreted into the blood cells; infection and fever, resulting from too
digestive tract whenever there is food in it. few normal white blood cells; and bleeding,
E) Soon after food moves into the digestive tract, a resulting from too few platelets
large number of enzymes are secreted. B) Common blood tests, such as the complete blood
cell count, can provide the first evidence that a
person has leukemia
42. – 46. sorularda, boş bırakılan yere, parçada C) However, a bone-marrow biopsy is almost always
anlam bütünlüğünü sağlamak için getirilebilecek performed to confirm the diagnosis and
cümleyi bulunuz. determine the type of leukemia
D) For more than 90 per cent of people who have
42. In the United States, more doctors are using acute lymphocytic leukemia (usually children), the
hypnosis for procedures in which sedation is first course of chemotherapy brings the disease
inappropriate or patients are allergic to under control
anaesthetics. Yet not even the most enthusiastic E) They are released into the bloodstream and
proponents of hypnosedation suggest it can transported to the liver, spleen, lymph nodes,
replace anaesthesia entirely. ----. Some 60% of brain, kidneys and reproductive organs, where
patients are hypnotizable to some extent; an they continue to grow and divide
additional 15%, highly so. The rest seem to be
unresponsive.
44. People adopt health-related behaviours in
A) Millions of Americans have employed hypnosis to order to stay healthy and to combat disease. ----.
stop smoking cigarettes or cut back on their However, all of these models have some
eating limitations, especially in their ability to predict the
B) Meanwhile, studies using advanced scanning health-related behaviours of people who lack the
technology have shed additional light on how financial resources necessary to pursue proper
hypnosis works to block pain medical attention.
C) Since the early 1990s, hypnosis has increasingly
been used in operating rooms as a substitute for A) Several theoretical models have been formulated
or as a complement to anaesthesia in an effort to explain and predict health
D) This application, now widely employed in Europe, behaviours, and most of these theories have
stems in part from studies showing that some value in predicting and explaining health-
hypnosedated patients suffer fewer side effects related behaviour
than fully sedated ones do B) Although the meaning of these models may seem
E) For one thing, not everybody can be hypnotized; obvious, their definitions have been elusive
to oversimplify, one must be open to the process C) The limitation of each of these models is their
first inability to accurately measure a number of
social, ethnic and demographic factors that also
affect people’s health-seeking behaviour
D) Among these stages is the necessity of
overcoming their optimistic bias; that is, their
belief that, although certain behaviours are
dangerous, the danger pertains to other people
and not to them
E) In the United States and other Western countries,
people are not “officially” ill until they are
diagnosed by a physician, making physicians the
gatekeepers to further health care

144
45. In the United States, child abuse is by no 47. – 51. sorularda, karşılıklı konuşmanın boş
means a recent phenomenon, but the bırakılan kısmını tamamlayabilecek ifadeyi
identification of child battering as a social bulunuz.
problem and its definition as a crime did not occur
on a national level until the 1960s. ----. Evidence 47.
began to accumulate that such beatings were Diane: - Everyone knows that physical exercise
common and a major source of injury for infants lowers our risk for diabetes and heart disease, but
and children. do you know why?
Ed: - ----
A) One common result of intentional injuries is that Diane: - No, it’s because our bodies have evolved
abused children may be vigilant and feel in such a way that our muscles need to be used in
threatened in a variety of situations – the world order for the rest of our bodies to remain healthy.
becomes a dangerous place
B) Treating infants with skull and long bone A) Which types of exercise are best for people with
fractures led paediatricians and radiologists to heart disease?
recognize that these injuries were caused by B) I am afraid overexercising makes joints, muscles,
beatings tendons, and bones ache. Don’t you agree?
C) Children who fail to thrive, especially during the C) I think swimming is one of the best forms of
first year of life, may never catch up exercise, isn’t it?
developmentally or socially with their peers D) Is it because exercise lowers the percentage of
D) Violence is a more common cause of injury and fat in our bodies?
death in the United States than in other E) Well, the recommended heart rate for exercise is
industrialized nations, with violent crimes 60 per cent of a person’s estimated maximum
occurring at a rate of about 8 per 1,000 people heart rate.
E) Abused children also tend to be fearful about
rejection, abandonment and additional abuse
48.
Ann: - My doctor told me that I should have
46. Some obese people who lose weight screening for colorectal cancer.
voluntarily benefit from their weight loss. Kathy: - ----
Research on obese people who lost weight Ann: - Yes, I know; she said that I need a
indicates that a 10% loss is sufficient to produce screening only as a preventative measure
significant decreases in lifetime health risks and because of my age.
personal health-related expenditures. ----. Kathy: - That’s right. The disease has a higher
prevalence in people 50 and older.
A) Dieting is a good choice for some people but not
for others A) You’ll need to be screened at regular intervals,
B) The eating disorders that have received the most won’t you?
attention, both in the popular media and in the B) Does she have a reason to believe you have the
scientific literature, are anorexia nervosa and disease?
bulimia C) Have you had a screening before? I had one last
C) Therefore, even modest weight loss can be month.
important for those who are obese D) Don’t you know that diet plays some role in the
D) On the contrary, the factors that influence obesity risk of colon cancer?
in adolescents are the same as those in adults E) But that doesn’t necessarily mean you have
E) In addition, behaviour modification programmes colorectal cancer.
with obese children have greater success in
promoting permanent weight loss than similar
programmes with adults 49.
Christopher: - My aunt went to the doctor because
she thought she had flu, and the doctor said she
was ill from carbon monoxide poisoning.
Patricia: - ----
Christopher: - Yes, and it’s a good thing she found
out in time. She’ll have to have it checked and
repaired.
Patricia: - I think that people should be educated
more about carbon monoxide, so that they can
take precautions against it.

A) I think the doctor must have made a mistake.


B) How strange! There must be a problem with her
heating system.
C) Carbon monoxide? Where did that come from?
D) She’s lucky she only became ill. Many people die
from carbon monoxide poisoning each year.
E) She should have had a carbon monoxide monitor
installed in her home.

145
50. 53. (I) When a person eats too much, metabolism
Doctor: - ---- favours fat formation. (II) The human body can digest
Richard: - Why not? What’s wrong with it? a wide variety of plant and animal tissues, converting
Doctor: - Studies have shown that second-hand these foods into usable proteins, fats, carbohydrates,
smoke has more toxins and cancer-causing vitamins and minerals. (III) The digestive system
chemicals than what smokers inhale from their takes in food, processes it into particles that can be
cigarettes. absorbed, and excretes the undigested wastes. (IV)
Richard: - Well, I guess I’d better be more careful, The particles that are absorbed through the digestive
then. system are transported through the bloodstream so
as to be available to the whole body (V) This provides
A) You really shouldn’t smoke, you know, when the energy for activity as well as the materials for
there are people around you. body growth, maintenance and repair.
B) Do you still smoke? You know it’s the leading
avoidable cause of death! A) I B) II C) III D) IV E) V
C) Your heart and lungs will be affected negatively if
you don’t quit smoking soon.
D) How many packs of cigarettes do you smoke 54. (I) The relationship between genes and proteins
every day? was first proposed in 1909 when English physician A.
E) Many work places and public places have Garrod suggested that genes dictate phenotypes
banned indoor smoking. through enzymes, the proteins that catalyze chemical
processes in the cell. (II) The idea came from his
observations of inherited diseases. (III) He
51. hypothesized that an inherited disease reflects a
Kelly: - My doctor says that I have something body’s inability to manufacture a particular enzyme,
called pre-diabetes. and he referred to such diseases as “inborn errors of
Eric: - ---- metabolism”. (IV) The function of a gene is to dictate
Kelly: - Maybe not, if I exercise and watch what I the production of a specific enzyme. (V) His
eat. hypothesis was ahead of its time but research
conducted decades later by other scientists proved
A) What are the symptoms? him right.
B) Does that mean you are going to develop
diabetes? A) I B) II C) III D) IV E) V
C) Do you think I should get my glucose levels
checked, too?
D) Could you give me your doctor’s name and 55. (I) Almost everyone has experienced heartburn at
telephone number? one time or another, usually after a meal. (II)
E) I wonder whether you’ll have to change your diet. Heartburn is the painful sensation a person feels
when the cardiac sphincter fails to prevent the
stomach contents from refluxing into the esophagus.
52. – 56. sorularda, cümleler sırasıyla (III) As a matter of fact, anyone who has heart trouble
okunduğunda parçanın anlam bütünlüğünü bozan should consult a doctor. (IV) This may happen if a
cümleyi bulunuz. person eats or drinks too much or both. (V) Tight
clothing and even changes of position (lying down,
52. (I) Anything that irritates the eyes can also irritate bending over) can cause it, too, as can some
the eyelids and cause swelling. (II) The most common medications and smoking.
irritant is an allergy, which can make one or both lids
crinkled or swollen. (III) Allergic reactions may be A) I B) II C) III D) IV E) V
caused by medications instilled into the eyes, such as
eyedrops; other drugs or cosmetics; or pollen or other
particles in the air. (IV) Insect stings or bites as well 56. (I) Some people have especially active,
as infections from bacteria, viruses, or fungi can also decaycausing bacteria in their mouth. (II) A parent
cause the eyelids to swell. (V) In fact, tears are salty may pass these bacteria to a child, presumably by
fluid that continuously bathes the surface of the eye to kissing. (III) However, one must understand that not
keep it moist. all tooth pain is caused by cavities. (IV) The bacteria
flourish in the child’s mouth after the first teeth come
A) I B) II C) III D) IV E) V in and can then cause cavities. (V) So a tendency
toward tooth decay that runs in families doesn’t
necessarily reflect poor oral hygiene or bad eating
habits.

A) I B) II C) III D) IV E) V

146
57. – 60. soruları aşağıdaki parçaya göre 60. It is explained in the passage that when a pain
cevaplayınız. occurs in an internal organ, ----.

Pain is a signal that protects us from harmful stimuli. A) it can be located immediately through the work of
An excess of any type of stimulus such as pressure, certain sensory neurons
heat, cold, excessive mechanical stretch, and specific B) it is usually felt in an area under the skin, which
chemical compounds stimulates pain receptors. In the may not be close to the organ itself
human body, pain receptors are the tips of certain C) pain receptors throughout the body begin to work
sensory neurons found in almost every tissue. efficiently
However, most internal organs are poorly supplied D) it is obvious that the organ cannot cope with a
with pain receptors. For this reason, pain from internal wide range of harmful stimuli
structures is often difficult to locate. In fact, pain is E) the connection between the organ and the spinal
often not projected back to the organ that is cord is seriously disrupted
stimulated. Instead, it is referred to an area just under
the skin that may be some distance from the organ
involved. The area to which the pain is referred is
connected to nerve fibres from the same level of the
spinal cord as the organ involved.

57. According to the passage, although most of


our body is thoroughly equipped with pain
receptors, ----.

A) this is not true for all the internal organs


B) it is in the internal organs that they function most
efficiently
C) they are mostly located in the tissues of some
internal organs
D) some of them are far more sensitive than others
E) the pain caused by heat and cold is much
stronger than other kinds of pain

58. It is pointed out in the passage that, through


pain, ----.

A) the relationship between the spinal cord and an


internal organ becomes extremely compromised
B) the function of the organs in the body is seriously
reduced
C) the area in an internal organ which is most
affected by a harmful stimulus can be identified
right away
D) it is extremely easy to determine the excess of a
harmful stimulus in an internal organ
E) we become aware of those stimuli in the body
that can have an adverse effect

59. As can be understood from the passage, the


stimulation of pain receptors ----.

A) is much stronger in an internal organ than in any


other area of the body
B) takes place only if the stimulus endangers the
organs
C) depends a great deal on the kind of chemical
compounds found in the body
D) occurs when any tissue in the body is
overstimulated
E) clearly indicates the somatic importance of the
spinal cord

147
61. – 64. soruları aşağıdaki parçaya göre 64. The author suggests that ----.
cevaplayınız.
A) the spread of kidney disease in the world will
Our kidneys excrete metabolic wastes and help continue to increase
regulate the volume and composition of body fluids. B) the immune response may be the cause of the
Their vital function is compromised in more than 13 impairment of the kidney’s filtration function
million people in the United States who suffer from C) the number of people who suffer from kidney
kidney disease. In fact, kidney disease ranks fourth in disease in the United States will remain stable
prevalence among major human diseases in the D) glomerulonephritis can lead to the growth of
United States. Kidney function can be impaired by tumours in the kidneys
infections, poisoning by substances such as mercury, E) mercury is the only substance that has a
lesions, tumours, kidney stones, shock or circulatory devastating effect on the kidneys
disease. For instance, one of the most common
kidney diseases both in the United States and in the
world is glomerulonephritis, which is related to the
damage of the kidney’s filtering units. The damage is
thought to result from an autoimmune response.

61. As pointed out in the passage,


glomerulonephritis ----.

A) is the failure of the kidney function caused by


various infections and other factors
B) is widespread and occurs when the kidney fails to
filter properly
C) becomes a deadly disease when the kidney’s
autoimmune response fails
D) has a limited extent of prevalence throughout the
United States
E) can cause serious damage to the filtering units of
the kidney

62. It is pointed out in the passage that the


kidneys ----.

A) have an autoimmune system which is very


efficient, especially against poisoning
B) have a very efficient filtering system which is
immune to certain infections
C) are responsible only for the excretion of waste
matter from the body
D) can fail to function properly owing to a wide range
of causes
E) can be easily infected once a tumour begins to
grow in them

63. As one understands from the passage, the


kidneys play a role ----.

A) in the partial filtration of various types of fluids in


the body
B) especially in regard to the concentration of
mercury in the body
C) in the efficient circulation of body fluids
D) in the body’s response to various infections
E) in relation to the fluids in the body

148
65. – 68. soruları aşağıdaki parçaya göre 68. As one learns from the passage, the current
cevaplayınız. genetic studies of schizophrenia ----.

The causes of schizophrenia are unknown, although A) also focus on the families in which the disease
the disease has a strong genetic component. Studies has a high rate of occurrence
of identical twins show that if one twin has B) have already produced positive results for more
schizophrenia, there is a 50% chance that the other efficient therapies
twin will have it, too. Since identical twins share C) are mostly concerned with cases of twins and
identical genes, this indicates that schizophrenia has how they can be treated effectively
an equally strong environmental component, the D) have already radically changed the norms of
nature of which has not been identified. Current treatment for the disease
treatments for schizophrenia focus on brain pathways E) totally leave out the role of the environmental
that use dopamine as a neurotransmitter. Despite component in the incidence of the disease
their ability to alleviate symptoms, many of the drugs
used to treat schizophrenia have such negative side
effects that patients frequently stop taking them. Now
that the human genome has been sequenced, there is
a vigorous effort under way to find the mutant genes
that predispose a person to the disease. This effort
includes sequencing DNA from families with a high
incidence of schizophrenia
.
65. As one understands from the passage,
identification of genetic mutations that contribute
to schizophrenia ----.

A) can only be useful in the treatment of the disease


if the use of drugs is also continued
B) will take a very long time before an effective
treatment for the disease can be developed
C) is being actively pursued
D) is not a solution since there are different forms of
the disease
E) has led to a wide range of new therapies that are
particularly effective for the disease

66. It is suggested in the passage that an


environmental influence ----.

A) is not of primary importance in understanding the


type of schizophrenia a patient has
B) must be completely ruled out in the diagnosis of
schizophrenia
C) need not be taken into account in a genetic study
of schizophrenia
D) is also considered to be as significant as the
genetic factor in schizophrenia
E) has a very minimal share in a patient’s
predisposition to schizophrenia

67. It is clear from the passage that the treatment


of schizophrenia through the use of drugs ----.

A) is most suitable especially in the case of twins


with identical genes
B) is the most effective therapy which is commonly
practised in the medical world
C) has serious drawbacks and, hence, is often
disrupted
D) also has to take into account the patient’s social
environment
E) is least concerned with the role of dopamine in
the brain

149
69. – 72. soruları aşağıdaki parçaya göre 72. As pointed out in the passage, the
cevaplayınız. communication between the brain and all the
other parts of the body ----.
Protected inside the bony vertebrae of the spine is an
inch-thick gelatinous bundle of nervous tissue called A) is extremely complicated due to the structure of
“the spinal cord”, which acts as the central the spinal column
communication conduit between the brain and the B) is maintained by nerve fibres through the spinal
rest of the body. Millions of nerve fibres carry motor cord
information from the brain to the muscles, while other C) is not affected even when the vertebrae are
fibres bring sensory information from the body to the injured
brain. In its structure and functions, the spinal cord D) can still be maintained even though the spinal
may be compared to a transcontinental telephone cord is broken
cable jam-packed with wires, each of which carries E) depends a great deal on the proper functioning of
messages both ways. But what happens if that cable the brain
is cut? Signals cannot get through, communication is
lost, and the cable must be repaired or replaced. In
humans, though, this is not a simple process due to
the sensitive nature of the spinal cord. The spinal cord
is rarely severed because the vertebrae provide rigid
protection. However, a traumatic blow to the spinal
column and subsequent bleeding, swelling and
scarring can crush the delicate nerve bundles and
prevent signals from passing. The result may be a
debilitating injury.

69. According to the passage, the spinal cord in


the human body ----.

A) is very sensitive to external blows and can be


easily injured
B) only carries the brain’s messages to all the parts
of the body
C) is far more intricate and vulnerable than a mere
telephone cable
D) is not affected by swelling and scarring caused
by an injury
E) can almost never be completely cut as it is
wellplaced within the vertebrae

70. As it is pointed out in the passage, there are


so many nerve fibres in the spinal cord that ----.

A) the appearance reminds one of a telephone


cable consisting of innumerable wires
B) each has a different function to perform for the
brain
C) it takes no time for the messages to reach the
brain
D) any injury to them can be absolutely fatal
E) their delicate nature makes them vulnerable to
injuries

71. As one understands from the passage, while a


severed telephone cable can be repaired or
replaced, ----.

A) the vertebrae make it impossible for us to treat


the spinal cord
B) the nerve bundles inside the spinal cord need to
be carefully treated
C) this is not so easy with the spinal cord due to its
very delicate structure
D) this is not the case with the spinal cord due to
bleeding and swelling
E) in the case of the spinal column the procedure
takes a long time

150
73. – 76. soruları aşağıdaki parçaya göre 76. It is clearly pointed out in the passage that,
cevaplayınız. occasionally, there may be cases when ----.

Immune deficiency, which in fact means an A) chemotherapy drugs have no effect whatsoever
inadequate immune response, may occur for several on the occurrence of immune deficiency
reasons. For example, it is a side effect of most B) there occurs a failure in the development of a
chemotherapy drugs used to treat cancer. Immune child’s immune system
deficiency also occurs naturally. Although the immune C) even antibodies do not provide protection for the
system is not functional at birth, infants are protected infant in the placenta
by antibodies they have received from their mother D) child mortality may not be related to children’s
through the placenta, and infants who breastfeed also exposure to viruses or bacteria
receive antibodies from their mother’s milk. These E) the mother’s milk does not contain antibodies to
antibodies offer protection until the infant’s own protect the infant in the first months of life
immune system develops during the first months of
life. In rare cases, the immune system fails to
develop, leaving the child without immune protection.
Exposure to any virus or bacterium can be fatal to
these children. Therefore, they are sealed into sterile
quarters to isolate them from the microorganisms that
are part of the normal world.

73. According to the passage, the development of


a child’s immune system ----.

A) is not of vital importance since the child is always


protected by various antibodies
B) begins in the placenta before birth and is
completed at birth
C) depends completely on the protective nature of
antibodies in the mother’s milk
D) is only completed a few months after birth
E) is closely related to the child’s physical
development and, hence, takes a very long time

74. As one can understand from the passage, the


causes of immune deficiency ----.

A) in an infant can be understood easily, as the


development of the immune system is yet in its
early stage
B) can sometimes be traced to a microorganism
such as a virus
C) are fundamentally linked with chemotheraphy,
which is used in cancer treatment
D) in children can be traced back to the first months
after birth
E) may vary

75. It is clear from the passage that, at birth,


infants ----.

A) have a non-functioning immune system, and so


rely on antibodies from their mother’s milk for
protection against microorganisms
B) have a relatively well-developed immune system
and are therefore well-protected against viruses
or bacteria
C) are so exposed to viruses or bacteria that the
rate of mortality is usually very high
D) hardly need antibodies gained from breastfeeding
to protect them against the fatal effects of
disease-causing bacteria and viruses
E) have a very strong immune system which
enables them to cope with viruses

151
77. – 80. soruları aşağıdaki parçaya göre 80. It is suggested in the passage that when
cevaplayınız. people discover that they are HIV-positive, they
----.
People who believe they may be infected with HIV, as
well as those who know they are, can benefit from A) adopt an indifferent attitude and continue to live
various psychological interventions. People with high- their usual life
risk behaviours may have difficulty deciding whether B) begin to act responsibly by refraining from
to be tested for HIV, and psychologists can provide contaminating others
both information and support for these people. A C) usually experience various negative feelings
significant minority of homosexual and bisexual men, D) seek help and support from psychotherapist and
intravenous drug users, and a larger proportion of apply for treatment
heterosexual men and women with multiple partners E) volunteer to undergo a series of tests in order to
and inconsistent users of condoms have never been obtain the most effective treatment
tested for HIV. Indeed, an estimated 70% of people
who are HIV-positive have not been tested and thus TEST BĐTTĐ.
do not know their HIV status. Because HIV infection CEVAPLARINIZI KONTROL EDĐNĐZ.
has a long incubation period, at-risk heterosexual
men and women may contaminate others for years
before they learn they have HIV. However, people
learning of an HIVpositive test result typically react
with increased anxiety, depression, anger and
distress. Therefore, trained psychotherapists are
needed to help such people cope with their diagnosis.

77. It is clearly emphasized in the passage that


psychological help ----.

A) and the clinical treatment of HIV are two different


procedures which are irrelevant to each other
B) plays a minimal role in the treatment of HIV
patients
C) has not proven to be useful in the treatment of
HIV patients because they often do not want to
cooperate with psychotherapists
D) is often ignored in HIV treatment not only by
patients but also by physicians
E) can be beneficial to patients with HIV infection

78. As pointed out in the passage, many people


already infected with HIV ----.

A) have agreed to be tested and undergo long


periods of treatment
B) have been receiving help from psychotherapists
to overcome their psychological problems
C) are not aware of their own situation because they
have not been tested
D) mostly include intravenous drug users and
heterosexual men with multiple partners
E) have been identified through a number of tests
they have taken

79. One learns from the passage that because it


takes a long time before HIV infection reveals
itself, ----.

A) heterosexual men and women with HIV may


already have infected many other people
B) homosexuals deliberately avoid taking HIV tests
and, thus, endanger themselves
C) intravenous drug users think that they are free
from any infection
D) this does not mean that bisexual men cannot be
infected
E) homosexuals and heterosexuals must be tested
regularly to make sure that they are free from HIV

152
1. – 18. sorularda, cümlede boş bırakılan yerlere 9. In April 1953, Watson and Crick ---- the
uygun düşen sözcük ya da ifadeyi bulunuz. scientific world with a succinct paper ---- their
model for DNA.
1. Throughout history, many ---- in engineering
and science have come about as the result of the A) were shaking / to explain
development of weapons. B) had been shaking / to have explained
C) have shaken / to be explaining
A) applications B) resolutions C) representations D) shook / explaining
D) innovations E) amplifications E) had shaken / having explained

2. Because of the time needed to develop 10. Common fungicides ---- for the puzzlingly high
expertise, scientists tend to continue working in a levels of DDT still found in some soils, even in
single area for a ---- length of time, perhaps even regions where this potent insecticide ---- decades
throughout their lives. ago.

A) substantial B) thorough C) moderate A) are blamed / would have been banned


D) qualitative E) comprehensive B) could be to blame / was banned
C) were blamed / must be banned
D) might be blamed / would be banned
3. Although the red flames of lithium and E) are to blame / has been banned
strontium appear similar, the light from each can
be separated by means of a prism into ----
different colours. 11. Rockets ---- to have originated with the
Chinese before the thirteenth century, which is
A) excessively B) distinctly C) conventionally when they ---- to appear in Europe.
D) properly E) familiarly
A) may be believed / were beginning
B) could be believed / have begun
4. The continents ---- their existence to Earth’s C) were believed / had begun
long history of plate-tectonic activity. D) have been believed / could begin
E) are believed / began
A) endanger B) result C) proceed
D) compile E) owe
12. Gregor Mendel probably chose to study
garden peas because he was familiar with them
5. For decades, scientists have theorized that ---- his rural upbringing; they were easy to grow,
much of the universe is ---- nearly undetectable and they came ---- many readily distinguishable
dark matter and dark energy. varieties.

A) made up of B) taken over by C) lost in A) from / in B) at / for C) with / on


D) held on E) broken away from D) in / by E) on / over

6. Exploring the human genome for clues to 13. Sunspots, a barometer ---- solar activity in
human evolution and migration is something of a general, seem to have been unusually numerous
battlefield, and the ground rules of this new ---- the last century.
science are still being ----.
A) for / at B) of / over C) within / by
A) worked at B) worked out C) worked through D) for / about E) above / from
D) worked back E) worked for
14. Fish often spend much of their time in the
7. Geology and biology ---- since life ----. deep, cool waters of a lake ---- oxygen levels there
become depleted by decomposers.
A) are intertwined / has begun
B) were intertwined / had begun A) as if B) just as C) now that
C) have been intertwined / began D) unless E) so as
D) would be intertwined / begins
E) could be intertwined / will begin
15. In 2002, ---- Australia’s Great Barrier Reef was
hit hard by unseasonable warming, 95 per cent of
8. ---- missing heat-shield tiles or a failed its coral was adversely affected.
undercarriage door have allowed the airframe ----?
A) so that B) if C) when
A) Could / to melt D) so long as E) in case
B) Would / melt
C) Might / to be melting
D) Can / melting
E) Will / be melting

153
16. ---- a violent storm is over, it leaves a cooler 24. – 35. sorularda, verilen cümleyi uygun şekilde
ocean behind, lowering the likelihood that more tamamlayan ifadeyi bulunuz.
storms will flare up, at least not immediately.
24. Despite the fact that no one has ever seen it
A) Once B) Even so C) Even if happen, ----.
D) Since E) While
A) there is evidence to suggest that rocks of up to
320 kilograms are moved by the wind across the
17. Humans, like all warm-blooded animals, can floor of Death Valley in California
keep their core body temperatures pretty much B) the Grandstand is a 20-metre-high island of rock
constant ---- differences in the temperature in the that looks like the top of a mountain buried in a
world around them. sea of sediment
C) most of southeastern California is a region torn
A) as of B) regardless of C) instead of by earthquakes and eroded by wind and rain
D) in terms of E) because of D) Death Valley lies 86 metres below sea level and
is surrounded by peaks of more than 3,000
metres
18. The part of an animal ---- gases are exchanged E) Death Valley was formed as the Amargosa and
with the environment is called the respiratory Panamint mountain ranges were pulled apart
surface. from each other
A) how B) which C) whatever
D) what E) where 25. As the Hubble Space Telescope continues its
mission, ----.

19. – 23. sorularda, aşağıdaki parçada A) Edwin Hubble encouraged this idea in connection
numaralanmış yerlere uygun düşen sözcük ya da with his own research
ifadeyi bulunuz. B) Eta Carinae is one of the most massive stars
known in the Milky Way, and is thought to be on
Small planes should be safe enough for normal, the verge of becoming a gigantic supernova
nonrisk-taking people to trust their lives to them. C) millions of people have already learned a great
NASA wants (19) ---- the accident rate by 90 per cent deal about the solar system
within twenty-five years. The planes should become D) we tend to think of science in terms of great
fast enough for their effective speed to be at least minds coming up with great ideas
three times (20) ---- great as that of cars on the E) it sends home new revelations about the life and
highway. The existing small-plane fleet averages 150 death of stars and the nature of our expanding
knots; that should be raised to 300 knots within a universe
decade, and eventually to 450 knots, (21) ---- small
planes could compete with the jetliners’ speed. The
planes should be more efficient and environmentally 26. When the first transistor amplifiers came
safer, using less fuel, creating less pollution, and along, ----.
generating less noise. They should be more (22) ----
in their operations and far simpler to fly, much like A) there were a number of design deficiencies
cars that vary little from one rental site to another. B) the engineers identified these problems and fixed
And they should be radically more reliable and them
cheaper to maintain – following the example of C) people would have been astonished by the magic
automobiles, with their quality revolution (23) ---- the of it all
1980s and 1990s. D) Shockley, Bardeen and Brattain developed the
transistor in 1947 and 1948
19. E) the extent of their achievements will never be
A) to have reduced B) reducing appreciated
C) having reduced D) to reduce
E) to have been reducing
27. As soon as scientists realized the power of
20. DNA technology, ----.
A) as B) such C) much
D) so E) more A) early concerns focused on the possibility that
21. they might create new pathogens
A) if only B) in that C) so that B) the Human Genome Project has yielded many
D) by which E) as if other unexpected results
C) they claim that these proteins could be tested for
22. their ability to cause allergic reactions
A) tentative B) consistent C) deliberate D) they began to worry about its potential dangers
D) reluctant E) recurrent E) one safety measure is a set of strict laboratory
procedures designed to protect researchers from
23. infection
A) at B) for C) about
D) of E) with

154
28. Although geologists tended to dismiss the 32. Geologists note that coal is similar to tar ----.
attempt of the physicist Kelvin to estimate the age
of Earth as being too simplistic, ----. A) as we might expect coal to last another 200 years
B) unless environmental issues may limit how much
A) the theory of continental drift might have been of this resource is exploited
accepted decades earlier C) in that both are relatively difficult to mine without
B) early nineteenth-century geologists largely dangerous environmental consequences
accepted that Earth was of unlimited age D) because the world is using these energy sources
C) many people believe that his calculation failed so inefficiently at the moment
through his ignorance of radioactivity E) while tar is also formed by tobacco burning
D) Kelvin began writing on this subject when he was
16
E) the model he used has in fact proved very useful 33. Practically all the problems associated with
in geology the musculo-skeletal and body-fluid systems
could be alleviated or avoided in space ships ----.

29. Since albatross have the longest wings in A) if artificial gravity similar to that on Earth could be
nature, ----. provided
B) as current countermeasures are limited to the use
A) their populations had already begun to decline of exercise equipment
B) they came ashore far more frequently C) since technological progress might have solved
C) they can glide for hundreds of kilometres without this problem
flapping their wings D) before further complications had developed
D) from time to time they went in search of new E) although the spaceship could be linearly
breeding grounds accelerated in the desired direction
E) most pairs mate for life, producing and raising
one chick every two years
34. The Mariner 10 space probe determined ----.

30. Fullerenes are carbon molecules ----. A) since planetary scientists have speculated about
Mercury’s magnetic field
A) that the simplest fullerene molecule, C60, has a B) that Mercury, unlike Venus and Mars, has a
soccer-ball shape significant magnetic field
B) whose shapes are made up of pentagons and C) while there is no way to judge whether iron on
hexagons that meet three at a time, in such a Mercury is solid or liquid
way that no two pentagons are adjacent D) until the new project uses radar reflections to
C) whereas, mathematically, the combinatorics of determine subtle oscillations in Mercury’s rotation
fullerenes is an application of Euler’s formula rate
D) although other fullerenes, such as C80, have E) which presumably creates a strong magnetic field
been made in the laboratory
E) while every fullerene contains exactly 12
pentagons with no limit to the number of 35. One of the most important aspects of our
hexagons it contains planet’s evolution is the formation of the
atmosphere, ----.

31. Twenty years have passed since the accident A) if continents and oceans, encircled by an oxygen-
at Chernobyl ----. rich atmosphere, support familiar life forms
B) although such constant change has
A) when many of the studies have been showing an characterized Earth since its beginning some 4.5
elevated rate of mutations among the animals in billion years ago
the area C) whether understanding the carbon dioxide
B) so that scientists studying the effects find content of the early atmosphere is crucial for
themselves in unpopular positions understanding climatic control
C) wherever policy makers want concrete D) because it is this assemblage of gases that
conclusions and results, not probability estimates allowed life to come out of the oceans and to be
on the dangers of radiation exposure sustained
D) because many public servants do not share the E) as continental shift has been altering the face of
scientists’ enthusiasm for the scientific process Earth for nearly a billion years
E) but the extent to which people and the
environment have been harmed is still being hotly
debated

155
36. – 38. sorularda, verilen Đngilizce cümleye 39. – 41. sorularda, verilen Türkçe cümleye
anlamca en yakın Türkçe cümleyi bulunuz. anlamca en yakın Đngilizce cümleyi bulunuz.

36. In North America, the electrical grid has 39. Darwinizme göre, mümkün olduğunca sık
evolved in piecemeal fashion over the past 100 üremek, her canlının temel amacıdır.
years.
A) Darwinism upholds the view that every organism
A) Kuzey Amerika’daki mevcut elektrik şebekesi, always strives to reproduce so long as it is
geçen 100 yıl boyunca aşama aşama possible.
oluşturulmuştur. B) According to Darwinism, frequent reproduction is
B) Kuzey Amerika’daki elektrik şebekesi, geride what every organism has as a major aim.
kalan 100 yıl boyunca parça parça ancak C) As one infers from Darwinism, for every
kurulabilmiştir. organism, the essential goal is to reproduce so
C) Kuzey Amerika’da, elektrik şebekesi, geçen 100 far as possible.
yıl içinde düzensiz bir şekilde gelişmiştir. D) According to Darwinism, it is the basic goal of
D) Kuzey Amerika’daki elektrik şebekesinin bir every organism to reproduce as often as
bölümü, geçen 100 yıl içinde geliştirilmiştir. possible.
E) Kuzey Amerika elektrik şebekesinin adım adım E) With reference to Darwinism, it is to be stated
gelişmesi, geçen 100 yıl içinde gerçekleşmiştir. that reproduction is what every organism has as
its ultimate aim.

37. Global competition regarding limited


petroleum and natural gas resources is intense, 40. Kozmik ışınlar, aslında, atmosferin tepesine
and even a mild production shortage can send hemen hemen ışık hızına yakın bir hızda çarpan ve
prices skyrocketing, as we have been seeing for çoğunlukla güneş sisteminin ötesinden gelen
some time. iyonlardır.

A) Sınırlı petrol ve doğal gaz kaynakları konusunda A) Cosmic rays, usually called ions, come from
küresel rekabet yoğundur ve, bir süredir across the solar system, hitting the outer layers of
gördüğümüz gibi, hafif bir üretim açığı bile the atmosphere at a speed close to that of light.
fiyatları birden yükseltebilir. B) The fact is that cosmic rays, also called ions,
B) Petrol ve doğal gaz kaynakları sınırlı olduğu için come from the other end of the solar system and
küresel rekabet oldukça yoğundur ve üretimde en constantly hit the top of the atmosphere at the full
ufak bir azalma, son zamanlarda görüldüğü gibi, speed of light.
fiyatları fırlatmaktadır. C) Cosmic rays are in fact ions that strike the top of
C) Sınırlı olan petrol ve doğal gaz kaynaklarına the atmosphere at nearly the speed of light and
yönelik küresel rekabet o denli yoğundur ki, yakın mostly come from beyond the solar system.
zamandan beri gözlemlediğimiz gibi, en küçük bir D) It is true that cosmic rays are ions which cut
üretim açığı bile fiyatları birden yükseltmektedir. across the solar system and strike the upper
D) Küresel rekabetin yoğun olduğu petrol ve doğal layer of the atmosphere at about the speed of
gaz kaynakları oldukça sınırlıdır ve, bir süredir light.
görüldüğü gibi, üretimde oluşan en küçük bir E) What we call ions are in fact cosmic rays that,
kısıtlama bile fiyatları oldukça yükseğe coming from the depths of the solar system,
çekmektedir. strike the upper parts of the atmosphere at
E) Sınırlı petrol ve doğal gaz kaynakları exactly the speed of light.
konusundaki yoğun küresel rekabet nedeniyle, bir
süredir gördüğümüz gibi, üretimin hafif de olsa
düşmesi sonucu fiyatlar alabildiğine 41. Leibniz, bilgisayar programının icadından 250
yükselmektedir. yıl önce yaşamış olmasına rağmen, modern
algoritmik bilgi düşüncesine çok yaklaşmıştı.

38. Until recently, there was no reliable method to A) Although Leibniz lived 250 years before the
measure the age of dinosaurs, and thus, to figure invention of the computer programme, he came
out the conditions in which they grew. very close to the modern idea of algorithmic
information.
A) Son yıllara kadar kullandığımız hiçbir yöntem B) Leibniz, who lived some 250 years before the
dinozorların yaşını ölçmek ve büyüdükleri introduction of the computer programme, was in
koşulları ortaya koymak için güvenilir değildi. fact fully familiar with the idea of modern
B) Dinozorların yaşını ölçmek ve dolayısıyla algorithmic information.
büyüdükleri koşulları ortaya koymak için bugüne C) It was just 250 years before the development of
kadar hiçbir güvenilir yöntem bulamadık. the computer programme that Leibniz lived and
C) Yıllardan beri, dinozorların yaşını ölçmeye ve put forward the modern idea of algorithmic
böylelikle nasıl büyüdüklerini belirlemeye yönelik information.
herhangi bir yöntem bulamadık. D) Living 250 years before the launching of the
D) Uzun zamandan beri, dinozorların yaşını ölçerek computer programme, Leibniz had a notion of
büyüdükleri koşulları kesin olarak belirlemede modern algorithmic information.
kullanılabilecek herhangi bir yöntemimiz yoktu. E) Even though the computer programme was
E) Son zamanlara kadar, dinozorların yaşını ölçmek invented 250 years after Leibniz, he was actually
ve böylece büyüdükleri koşulları anlamak için aware of the idea underlying modern algorithmic
güvenilir bir yöntem yoktu. information.

156
42. – 46. sorularda, boş bırakılan yere, parçada 45. Although a soccer ball can be put together in
anlam bütünlüğünü sağlamak için getirilebilecek many ways, there is one design so ubiquitous that
cümleyi bulunuz. it has become iconic. This standard soccer ball is
glued together from 32 polygons, 12 of them five-
42. Carbon nanotubes have been hailed as a sided and 20 six-sided, arranged in such a way
semiconducting wonder ingredient that will make that every pentagon (five-sided) is surrounded by
materials stronger. ----. Moreover, their ability to hexagons (six-sided). ----. This colour scheme was
act as filters might one day be exploited to build introduced for the World Cup in 1970 to enhance
artificial livers. the visibility of the ball on television, although the
design itself is older.
A) In the molecules of a polar liquid, some atoms
are slightly positively charged while others carry a A) 12 pentagons and 20 hexagons form a figure
balancing negative charge known to mathematicians as a “truncated
B) Some experts in nanotube chemistry have icosahedron”
published extensively B) To a mathematician, the iconic black and white
C) Accordingly, it is possible to make nanotubes soccer ball is an intriguing puzzle
generate electricity C) A number of questions can be tackled about the
D) Thus, their importance has been greatly arrangement of pentagons and hexagons using
overrated the language of mathematics
E) In addition, they will help miniaturize electronics D) The usual way to colour such a ball is to paint the
systems pentagons black and the hexagons white
E) Every soccer ball contains at least 12 pentagons,
but may well contain more
43. Locomotion can be considered to be a flow of
mass from one location to another. ----. They seek
and find paths and rhythms that allow them to 46. This year researchers from some 60 nations
move their mass the greatest distance per are participating in the International Polar Year, an
expenditure of useful energy while minimizing intensive burst of interdisciplinary research
thermodynamic imperfections such as friction. focusing on the polar regions. ----. For instance,
water from the melting ice sheet is flowing into
A) All of these designs allow for the maximum the North Atlantic much faster than scientists had
transfer of material with the least amount of previously thought possible.
resistance
B) A flow is an equilibrium of areas with high and A) Greenland, especially, has become a kind of
low resistivities barometer for the rest of the world because of its
C) Animals move on the surface of Earth in the sensitivity to climate changes
same way as rivers, winds and oceanic currents B) Climatologists have found that the best places to
D) A river basin configures and reconfigures itself so study global warming are the coldest regions on
that the water is discharged with the least Earth
resistance through the mouth of the river C) Thus far, the data the researchers have seen has
E) One of the basic goals of any design – whether been alarming
it’s an animal or a machine – is to get maximum D) A glacier that accelerates with a warming
output for minimum energy atmosphere is within the realm of scientific
expectation
E) Arctic climatologist Konrad Steffen has spent 18
44. Only a few large meteorites have struck the consecutive springs on the Greenland ice cap,
earth. The largest we know about fell in Arizona personally building and installing the weather
and made what is now called Meteor Crater, a hole stations
about a mile across and 600 feet deep. ----. Other
big meteorites fell in ancient times, in Texas, in
Argentina, in northern Siberia and in Greenland.

A) When a meteor reaches the earth, it is called a


meteorite
B) This big meteorite may have fallen as much as
twenty-five thousand years ago
C) The amazing thing about these meteor showers
is that they come year after year
D) Most meteors are small, probably a few inches in
diameter
E) The most remarkable meteor shower was seen in
Connecticut on the night of November 12, 1833

157
47. – 51. sorularda, karşılıklı konuşmanın boş 50.
bırakılan kısmını tamamlayabilecek ifadeyi Ann: - Did you know that the use of graph paper
bulunuz. for plotting functions and data was first made
common by Professor John Perry, when he was
47. still an assistant of the famous physicist Lord
Angela: - How was your visit to Crater Lake Kelvin?
National Park last summer? Jane: - No, I didn’t. How did he make it available to
Sharon: - It was wonderful. The lake is very the public?
beautiful, with a clear, deep-blue colour. And I Ann: - ----
learned something new about it: it’s a closed Jane: - Well, that’s really something.
basin lake.
Angela: - ---- A) He was a tireless educator in engineering and
Sharon: - Well, there are no permanent streams mathematics.
that enter or exit the lake. B) He challenged Lord Kelvin’s hypothesis about the
temperature of the Earth.
A) I plan to visit the lake this summer. C) Perry came up with the idea that heat moved
B) How did you learn that? more easily deep inside the Earth than it does
C) You’re very informed, aren’t you? close to the surface.
D) What does that mean? D) Perry insisted that mathematics was basic to all
E) How many visitors are allowed into the park each the sciences.
year? E) Simply, it was because of him that the price of
graph paper became affordable for everybody.

48.
Ken: - Do you know? I’m really getting very 51.
interested in the movement of glaciers. Peter: - It seems that higher sea-surface
Sherrie: - What have you learned now? temperatures could give rise to ever larger and
Ken: - ---- more frequent hurricanes.
Sherrie: - That’s right; the ice moves out to the Frank: - ----
sides because of the greater weight and pressure Peter: - True. What do you think is going to
at the centre. happen?
Frank: - Let’s just wait and see!
A) That when glaciers move, they don’t only move
straight downhill. A) At present, it is all pure speculation. Let’s change
B) Well, some glaciers flow into the sea, but others the subject.
end on land. B) Why have you become so interested in global
C) Glaciers store about 75% of the world’s warming?
freshwater. C) But which parts of the globe would be affected?
D) I found out that where an ice sheet flows into the D) Yes; I’m familiar with that theory. But there are
ocean and floats, it forms an ice shelf. opposing theories too.
E) Ice sheets flowing over land usually form piles of E) Hurricanes will certainly increase in number and
rocks and dirt at their ending points. severity.

49. 52. – 56. sorularda, cümleler sırasıyla


Michelle: - It says in this article that Jupiter’s moon okunduğunda parçanın anlam bütünlüğünü bozan
Europa has relatively few craters on it – only one cümleyi bulunuz.
or two significant ones.
Kathy: - I wonder why it has so few, when some of 52. (I) In science fiction, the worst threats to space
Jupiter’s other moons and our own moon have so travellers are large ones: asteroids, ravenous
many. creatures, and imperial battle cruisers. (II) The journey
Michelle: - ---- time from Earth to Mars could be reduced from six
Kathy: - Oh, yes, I’ve read about that. It has to do months to less than six weeks. (III) In reality, though,
with tides changing the surface features, doesn’t the scariest menaces for humans in space are the
it? tiniest: fast-moving elementary particles known as
cosmic rays. (IV) On a long journey, these would give
A) The article says that it would be surprising if the astronauts a dose of radiation serious enough to
tides weren’t still active. cause cancer. (V) Unlike most of the other challenges
B) Scientists think that the surface has been of venturing into deep space, which engineers should
completely re-made in the cosmically recent past. be able to solve, cosmic rays pose irreducible risks.
C) The continously changing surface would create
organisms, if there are any, that could adapt A) I B) II C) III D) IV E) V
easily to the changes.
D) Since there are many tiny bodies in the outer
solar system, they would normally have hit
Europa, forming craters.
E) The weak ice on Europa’s surface cannot support
high mountains.

158
53. (I) A dramatic cut in the cost of a super-efficient
new breed of solar cell could put domestic solar
power on a more economic footing. (II) The cells,
which helped take NASA’s electric-powered aircraft
Helios to record altitudes, have until now been too
expensive. (III) But their manufacturer has found a
way to make them as much as 20 times cheaper. (IV)
The cells convert light energy into electricity with an
efficiency of 20 per cent – which means they generate
one-third more electrical power than conventional
silicon solar cells. (V) NASA’s electrically powered
plane Helios soared to altitudes above 96,000 feet (29
kilometres) – a world record for a winged plane not
powered by a rocket engine.

A) I B) II C) III D) IV E) V

54. (I) Our knowledge of cell structure took a giant


leap forward as biologists began using the electron
microscope in the 1950s. (II) Instead of light, the
electron microscope (EM) uses a beam of electrons.
(III) Actually, specimens should have been cut into
extremely thin sections and stained with atoms of
heavy metals such as gold. (IV) The EM has a much
greater resolution than the light microscope. (V)
Under special conditions, the most powerful Ems can
detect individual atoms.

A) I B) II C) III D) IV E) V

55. (I) Early in the 20th century, oranges and


grapefruits were ripened for market in sheds equipped
with kerosene stoves. (II) Before leaves fall, many of
their essential elements are stored in the stem. (III)
Fruit growers thought it was the heat that ripened the
fruit, but when they tried newer, cleaner-burning
stoves, the fruit did not ripen fast enough. (IV) Plant
biologists learned later that ripening in the sheds was
actually due to ethylene, a gaseous by-product of
kerosene combustion. (V) We now know that plants
produce their own ethylene, which functions as a
hormone that triggers a variety of aging responses,
including fruit ripening and programmed cell death.

A) I B) II C) III D) IV E) V

56. (I) Since the terrorist attacks on 11 September


2001, anti-aircraft missile batteries have been
installed to protect buildings in US cities. (II) However,
less drastic solutions have also been suggested. (III)
No software simulation is going to be sufficient to
convince any pilot about the new avionics systems.
(IV) An aerospace company, for instance, has
proposed installing the electronics from its pilotless
plane in passenger aircraft to allow ground control to
take over a hijacked plane and land it remotely. (V)
Others say automatic landing systems could ster
planes to safety without human intervention.

A) I B) II C) III D) IV E) V

159
57. – 60. soruları aşağıdaki parçaya göre 59. According to the passage, Pasteur discovered
cevaplayınız. that ----.

There were many heated debates in the nineteenth A) most French winemakers were far more skilled at
century about the relationship between chemical producing vinegar than wine
reactions and living organisms. Some scientists felt B) the problem French winemakers faced needed to
that fermentation was an activity of living things and, be dealt with immediately
therefore, could not take place outside of living cells. C) most scientists of his time knew little about the
This was proved by the work Louis Pasteur undertook variety of chemical reactions taking place in living
for the French wine industry. Indeed, in the 1850s, the organisms but they were prepared to debate
French wine industry was having serious trouble with about them
wine that had spoiled. The French emperor, Napoleon D) what really spoiled wine was not fermentation
III, called in Pasteur to help. Pasteur knew that the brought about by living yeast cells but that
fermentation which produced wine was caused by caused by bacterial activity
living yeast cells. But now he found that certain E) not only the French wine industry but also its milk
bacteria could also carry out fermentation. He industry could be greatly improved through the
discovered that fermentation by bacteria spoils wine use of pasteurization
because it produces vinegar (acetic acid) instead of
the alcohol produced by yeast. Pasteur suggested
that the winemakers heat the wine for a short time to 60. As pointed out in the passage, the idea that
destroy the bacteria. They were horrified, but it ----.
worked. The process, pasteurization, is still used
today, especially for milk. A) bacteria spoiled wine was accepted among the
French scientists of the nineteenth century, but it
57. It is clear from the passage that the was Pasteur whom Napoleon III appointed to
fundamental principle of pasteurization, especially improve wine-making in the country
as practised in the milk industry today, ----. B) bacteria in wine could best be destroyed through
a prolonged process of heating was
A) aroused a fierce controversy among the French commonplace among nineteenth-century French
scientists of the nineteenth century winemakers
B) was already known in France before the time of C) pasteurization prevented milk from spoiling
Pasteur quickly spread outside France in the nineteenth
C) was given a full scientific explanation by century and has never since been challenged by
scientists working for Napoleon III scientists
D) was discovered only after a long period of D) fermentation by bacteria spoils wine is no longer
experimentation by French winemakers current, since various new methods have been
E) is the destruction, by heating, of the bacteria developed for better wine-making
which cause fermentation E) fermentation was caused by a chemical reaction
in living cells wasn’t accepted by many scientists
until well into the nineteenth century
58. One understands from the passage that, in the
nineteenth century, ----.

A) a major controversy among scientists concerned


whether living organisms played any role in
chemical reactions
B) Napoleon III was seriously interested in scientific
matters and favoured Pasteur more than any
other scientist
C) French winemakers had so much trust in
Pasteur’s work on bacteria that they fully followed
his instructions for the process of pasteurization
D) French scientists especially focused on
fermentation, since the wine industry was of vital
importance for France
E) the French wine industry was remarkably
advanced since it made use of various
innovations and scientific discoveries

160
61. – 64. soruları aşağıdaki parçaya göre 64. It is pointed out in the passage that ----.
cevaplayınız.
A) Aristotle was particularly interested in the fossils
Fossils are the remains of organisms which have found in rocks
endured for fantastic periods of time. Fossils can be B) there are many different kinds of fossils
bones or teeth or even plant or animal imprints C) the earliest kinds of fossils were those of sea
preserved in rock since prehistoric times. The plants and seashells
appearance of fossils in rock has been a source of D) Herodotus and Aristotle were the earliest
wonder and fascination to man for centuries. The pioneers of fossil studies
fossil of an ancient sea animal was even found E) the very first discovery of fossils was in the
among the possessions of a prehistoric man. Many Libyan desert
people have tried to explain fossils. Aristotle believed
they were the remains of living creatures, but thought
the creatures grew in the rocks. Some people
believed that fossils were placed in rocks by evil
spirits. Other explanations were remarkably modern.
For example, Herodotus, an ancient Greek historian,
observed fossil seashells in the Libyan desert in 450
B.C. and guessed that the Mediterranean Sea had
once reached much farther south than it does today.

61. It is clear from the passage that ----.

A) throughout history, there have been many


different views and explanations as regards the
nature and cause of fossils
B) it is only in modern times that there has been any
serious interest in fossils
C) human interest in fossils has only been aroused
through the fascinating explanations and
discoveries made by modern science
D) Herodotus travelled extensively in the
Mediterranean world and was particularly
attracted by the geography of Libya
E) the study of fossils by modern scientists has
mostly focused on the preserved imprints of
plants and animals in rocks

62. It is clear from the passage that fossils ----.

A) greatly fascinated prehistoric peoples, who


revered them as sacred
B) are always found in rocks as bones or teeth
C) date back to very early prehistoric times
D) were not as serious a concern for Aristotle as
they were for others
E) were regarded by prehistoric man as evil spirits
preserved in rocks

63. According to the passage, Herodotus


speculated that the Libyan desert, ----.

A) which was rich in the remains of various


organisms, had been the original home of
prehistoric man
B) which was vast and dry, had been flooded on
several occasions in the past by the
Mediterranean Sea
C) through which he often travelled, had in the past
made up a major part of the Mediterranean Sea
D) where he saw fossil seashells, had once been
covered by the Mediterranean Sea
E) where there were plant and animal imprints in
rocks, had no connection whatsoever with the
Mediterranean Sea

161
65. – 68. soruları aşağıdaki parçaya göre 68. As it is indicated in the passage, if the addition
cevaplayınız. of new members to a population exceeds loss, ----.

A population is a group of individual organisms of the A) this can have a restrictive effect on emigrations
same kind that are limited to some particular space. from the population
The most familiar example is the human population, B) this has no effect whatsoever on the environment
but there are also populations of animals and plants in which the population lives
everywhere on Earth. In fact, scientists regard a C) the survival of the population can be maintained
population as a biological unit that has both structure in a balanced way
and function. The parts of a population are its D) the size of the population shows a growing
individual members. The functions of a population are pattern
similar to those of other biological units: growth, E) new measures must be introduced to prevent
development, and self-maintenance in a changing environmental changes
environment. Individuals enter a population by birth
and by moving in, that is, by immigration. Individuals
leave a population by death and by moving out, that
is, by emigration. If the environment of a population
remains the same, loss and replacement of members
are in balance. The population will be able to survive
in that particular environment. If the environment
changes, however, loss or addition of members
increases or decreases the size of the population.

65. It is pointed out in the passage that the


changes that occur in the environment of a
population ----.

A) have an impact, negative or positive, on the


members of that population
B) speed up the process of replacement of the
members of the population
C) always contribute greatly to the survival of all the
members of that population
D) are mostly caused by the uncontrollable size of
that population
E) can be reduced through an increase in the size of
the population

66. According to the passage, what is called a


“population” in biology ----.

A) can be defined as any group of organisms that is


not subject to loss and replacement
B) is a biological unit that has only the function of
growth
C) is a group of animals and plants that can survive
all kinds of environmental changes
D) solely refers to any human group that lives in a
specific region on Earth
E) is a unit that consists of the same kind of
individual organisms living in a particular area

67. It is clear from the passage that, so long as a


population lives in a constant environment, ----.

A) the growth, development, and self-maintenance


of its members can be fully controlled
B) its size remains more or less stable
C) it usually undergoes a rapid structural change,
which considerably affects its size
D) the replacement of its members is relatively slow,
compared with other populations in different
environments
E) its survival becomes difficult owing to the
uncontrollable increase in its size

162
69. – 72. soruları aşağıdaki parçaya göre 72. As is stated in the passage, from the data
cevaplayınız. provided by the Hubble Space Telescope about
Pluto ----.
Pluto, which was until recently regarded as the
outermost and smallest planet in the solar system, A) some scientists have suggested that its
has never been visited by an exploring spacecraft. So exploration ought to be started soon
little is known about it that it is difficult to classify. Its B) one can conclude that it has a climate which is
distance from Earth is so great that the Hubble Space stable and temperate
Telescope cannot reveal its surface features. C) it has a dull surface with absolutely no variety
Appropriately named for the Roman god of the D) it is understood that there is an ice cap on its
underworld, it must be frozen, dark, and dead. Its north pole
mean distance from the Sun is 5,900 million E) one becomes aware of the fact that every planet
kilometres. In fact, it has the most eccentric orbit in in the solar system has a similar cycle of seasons
the solar system, bringing it at times closer to the Sun
than Neptune. Furthermore, there is evidence that
Pluto has an atmosphere, containing methane, and a
polar ice cap that increases and decreases in size
with Pluto’s seasons. It is not known to have water.
The Hubble Space Telescope’s faint-object camera
revealed light and dark regions on Pluto, indicating an
ice cap at the north pole. It is not known if there is an
ice cap at Pluto’s south pole.

69. According to the passage, Pluto’s orbit around


the Sun ----.

A) takes so long that each of its seasons has a long


period
B) has not yet been described accurately
C) brings it, on occasion, closer than Neptune to the
Sun
D) follows a pattern which is uniform and stable
E) has been studied again and again through the
Hubble Space Telescope

70. As is pointed out in the passage, Pluto ----.

A) is on the outer edge of the solar system


B) has extensive ice caps at both its poles
C) was a major god in antiquity, worshipped by the
Romans as well as by other peoples
D) and Neptune seem to have similar orbits that
bring them closer to the Sun
E) looks so dark that nothing whatsoever can be
observed on it

71. It is stated in the passage that, since Pluto is


so far away from Earth, ----.

A) its regions and poles can best be studied through


a powerful telescope
B) almost nothing is known about even its exterior
C) the density of the methane in its atmosphere
cannot be measured
D) the Hubble Space Telescope clearly shows how
completely frozen its surface is
E) only some minor explorations have so far been
made by means of a spacecraft

163
73. – 76. soruları aşağıdaki parçaya göre 75. As can be understood from the passage, the
cevaplayınız. fact that even some great rivers have from time to
time run dry due to the overuse of their capacity
Today the world faces a growing crisis over the ----.
management of its great rivers. In recent years, most
of the great rivers in the world, such as the Yellow A) demonstrates how the growth of the populations
River in China, the Indus, the Colorado, and the Nile, in some countries has had an adverse effect on
have all periodically run empty because mankind has the water resources
used their every last drop. Indeed, there is a huge B) shows how irresponsible the water engineers of
unmet demand in the world for water. More than a most countries have been
billion people have no access to clean drinking water, C) signifies that there must be a national water
and while it is hoped that this figure will be halved by authority in each country for the preservation of
2015, nobody is sure where the water will come from. the water resources
With today’s trends, one-third of the world population D) makes it urgent for water engineers to discover
will be seriously short of water by 2025. Politicians in new water resources in the southwestern US
China, India, Pakistan, Egypt and other waterstressed E) is a clear indication of how urgent the demand for
countries want their water engineers to find solutions water is in the world today
– and fast.

73. In the passage, there is a clear warning that, 76. One concludes from the passage that efficient
----. management of the water resources of the world
is essential ----.
A) sooner or later, water shortages could lead to
serious political crises in China and other A) if the growing worldwide demand for water is to
countries be met adequately
B) despite the solutions proposed by water B) and the waters of the Nile and the Indus, in
engineers, the people of China, India, Pakistan, particular, must not be used so wastefully
and Egypt will soon face a serious shortage of C) since China and India, with their large
water populations, are heading for a serious shortage
C) unless more precautions are taken, more than a of water well before 2015
billion people will have almost no access to water D) as one-third of the population in China is unable
in the near future to get clean drinking water
D) so long as politicians remain indifferent to the E) in order to maintain political stability in the
growing water crisis in the world, most countries countries most affected by an acute shortage of
will be unable to provide clean drinking water for drinking water
their people
E) by the end of the first quarter of this century,
there will be a severe water shortage affecting
one-third of the world population

74. According to the passage, the water resources


of the world ----.

A) are largely confined to the Indus and the Nile


B) have been increased through the solutions
proposed by water engineers, and so the need
for clean drinking water will be met well before
2015
C) are so limited that it is doubtful whether the
number of people with no access to clean
drinking water can be halved, as hoped, by 2015
D) have become a major concern among politicians
in many countries and, therefore, new policies
have been proposed for an efficient management
of the great rivers
E) need to be upgraded by 2025 in order to catch up
with the growth rate of the world population

164
77. – 80. soruları aşağıdaki parçaya göre 80. As pointed out in the passage, Greenland, with
cevaplayınız. its 5,000-foot-thick ice sheet, ----.

The huge ice sheet covering Greenland, which is the A) is so affected by the atmospheric concentrations
world’s largest island, provides a habitat for many of heat-trapping gases that the amount of the
arctic species and holds nearly 8 per cent of the meltwater on the island has risen to a dangerous
world’s freshwater. It is, on average, 5,000 feet thick level
and is constantly being replaced as snow falls each B) has lost much of its freshwater capacity due to
winter. Over the course of centuries, the snow the process of extensive melting which has been
compacts into ice, which slides towards the ocean. In going on for centuries
recent years, higher atmospheric concentrations of C) will soon lose its ice mass, since the amount of
heat-trapping gases have accelerated that process. snowfall on the island each winter has dropped
As temperatures rise, the top layers melt, giving way dramatically over the course of the last few
to darker, heat-absorbing ice and liquid water. The centuries
meltwater seeps down to the rock below, lubricating D) not only accommodates different kinds of arctic
the ice mass and speeding its slide into the sea. species but also preserves a significant amount
of the world’s freshwater
77. As one can see, the passage ----. E) can no longer provide a habitat for some arctic
species that have lived on the island over the
A) focuses on the importance of Greenland as a course of many centuries
major source of the world’s freshwater
B) is mainly concerned with the geographical
features of Greenland’s surface and highlights its TEST BĐTTĐ.
natural beauty CEVAPLARINIZI KONTROL EDĐNĐZ.
C) deals in detail with the causes of global warming
and its effects on the arctic species in Greenland
D) explains how global warming is having an
environmental impact on Greenland’s ice mass
E) extensively describes the process whereby the
ice mass of Greenland has formed over the
course of centuries

78. It is pointed out in the passage that the slide


into the ocean of the ice mass in Greenland ----.

A) has caused much damage to a wide range of


arctic species and their habitat
B) can be prevented completely so long as
temperatures are stable
C) is of vital importance because, through this
process, the world’s freshwater capacity is
increased
D) has only been observed in recent years, but
environmentally, this phenomenon is of no
significance
E) has been faster than usual in recent years as a
result of global warming

79. According to the passage, when the top layers


of the ice sheet melt, ----.

A) the ice mass ceases to slide towards the ocean


B) water seeps down to the rocks below aiding the
ice mass to slide into the sea
C) it has an adverse effect on various arctic species
D) the rock under the ice mass is fully exposed
E) there is a noticeable increase in the volume of
liquid water

165
1. – 18. sorularda, cümlede boş bırakılan yerlere 8. The Pyramid of the Sun ---- in stages in the
uygun düşen sözcük ya da ifadeyi bulunuz. second and third centuries A.D. and ---- about 200
feet high and 700 feet in length.
1. The process of economic and social reforms
that began in the 1990s in some developing A) was built / measures
countries has had a profound ---- on the B) had been built / may measure
functioning and adjustment of their labour C) could have been built / was to measure
markets. D) has been built / would measure
E) could be built / had measured
A) dismissal B) recovery C) impact
D) recognition E) distrust
9. Underwater archaeology is generally
considered to ---- its first major encouragement
2. Ever since the ---- remains of ancient states and during the winter of 1853-54, when a particularly
cities were first discovered, the collapse of their low water level in a Swiss lake ---- bare enormous
civilizations has been a focus of debate and quantities of wooden posts, pottery and other
inquiry. artifacts.

A) consistent B) implicit C) vulnerable A) be receiving / has laid


D) competent E) impressive B) be received / would lay
C) have been received / had laid
D) receive / could have laid
3. The Arawak Indians were the first to inhabit E) have received / laid
Grenada, but they were all ---- massacred by the
belligerent Carib Indians.
10. Sub-Saharan Africa ---- at a relatively stable
A) fairly B) previously C) eventually rate since the mid-1990s, and its growth ---- in the
D) principally E) rarely following years.

A) was growing / continues


4. In the advertising industry, it is a maxim that a B) has been growing / should continue
message needs to be ---- often in order to be both C) has grown / should have continued
understood and appreciated. D) grew / has continued
E) is growing / will have continued
A) repeated B) engaged C) negotiated
D) involved E) settled
11. In a clever experiment carried out during the
1980s, a team of psychologists at Cornell
5. In narrative poems, characters often ---- certain University ---- that being in a happy mood ----
ideas or heroic qualities which the poet wishes to people generate more creative solutions to
celebrate. problems.

A) head towards B) stand for C) show up A) have found / will help


D) carry out E) try on B) had found / helped
C) are to find / would help
D) found / helps
6. Children can ---- different roles, work through E) would find / has helped
conflicts, and attempt various methods of
communication, all under the pretence of play.
12. Psychological studies show that Belgian and
A) go along with B) give up C) get in French workers place greater importance ----
D) try out E) fall back on personal independence than do workers ---- many
other countries.

7. In the US, the percentage of obese people ---- A) in / of B) on / in C) about / by


over the past two decades, and at present, 35% of D) to / for E) of / within
the population ---- overweight.

A) could have doubled / would be 13. Ancient Pompeii, destroyed in A.D. 79 ---- an
B) had doubled / was eruption of Vesuvius, lay buried ---- rock and ash
C) has doubled / is until the 18th century.
D) would double / will be
E) doubled / has been A) by / under B) at / for C) with / below
D) in / on E) through / over

166
14. During the second half of the 20th century, the 19. – 23. sorularda, aşağıdaki parçada
fracture rates among high-risk European numaralanmış yerlere uygun düşen sözcük ya da
populations grew higher, ---- this increase was ifadeyi bulunuz.
modest compared with that of the urbanized
populations in Southern Asia. The Spanish people take particular pride in their
cultural heritage. (19) ---- the traditional art form of
A) in case B) because C) unless flamenco dance and the three-act drama of the
D) but E) now that bullfight (corrida), the theatres and opera houses of
Spanish cities provide one of the best ways of sharing
the cultural experience (20) ---- Spain. Many activities
15. Citizens of countries that are members of the (21) ---- well after midnight, and so taking full
EU, as well as citizens from the US, Canada, advantage of the afternoon siesta is a good way to
Australia and New Zealand, do not need a visa to (22) ---- for the evening ahead. Further, Spain’s
visit Germany ---- their stay does not exceed three mountain ranges, woodlands and extensive coast
months. offer great potential for scenic tours and sports
vacations (23) ---- alternatives to sunbathing on the
A) whereas B) as if C) so long as beach.
D) whether E) so that
19.
A) In opposition to B) As a result of
16. Every Turkish citizen over the age of 25 is C) In addition to D) Dependent on
eligible to be a deputy in Parliament ---- he or she E) In spite of
has completed primary education and has not
been convicted of a serious crime. 20.
A) at B) for C) by
A) with the idea that D) of E) from
B) from the point of view that
C) despite the fact that 21.
D) by the fact that A) begin B) began C) are to begin
E) on condition that D) have begun E) was to begin

22.
17. The number of frauds in the US ---- the A) maintain B) prepare C) complete
criminal uses someone else’s credit card number D) identify E) participate
doubled to 162,000 cases in 2002.
23.
A) in which B) by whom C) of which A) just B) like C) such
D) which E) with whom D) even E) as

18. Unlike many other European nations, Finland 24. – 35. sorularda, verilen cümleyi uygun şekilde
has not been the destination of large groups of
tamamlayan ifadeyi bulunuz.
foreign workers, ---- has it ever been a colonial
ower.
24. When a rise in civil-servant numbers resulted
in economic problems in Tanzania, ----.
A) and B) nor C) or
D) also E) not,
A) newly emerged leaders often offered civil service
jobs to their supporters in Africa
B) the payment of civil servants is still problematic
C) a painful cut in pay would be inevitable because
of staff numbers
D) the IMF forced the government to cut the total
wage bill
E) holding civil servants to account is another
problem

25. Since older people perform certain tasks beter


than the young do, ----.

A) it is wrong to categorize them as generally less


capable
B) older people often are more socially competent
C) an older employee might prove initially slower at
a particular mental function
D) it is exactly what employers worry about in
quality-control jobs
E) this insight has important implications for the
workplace

167
26. Although it is not known exactly when 31. Small states in Africa like Senegal would
potterymaking began in Cappadocia, ----. probably integrate with the global production
chains of multinationals ----.
A) early pottery was generally shaped by the
wrapping-rolling method A) although Mali and Cameroon could be
B) it is still agreed that the art originated in industrialized by processing foodstuffs
Mesopotamia B) while countries with large populations like Nigeria
C) it dates back at least to Hittite times might develop their own national industries
D) most archaeologists are keenly interested in the C) if ever South African firms expand aggressively
excavations that are going on across the region
E) pottery belonging to other cultures has been D) whereas such obstacles explain why sub-Saharan
found in the townships of Hacıbektaş and countries have only a marginal share in manufactured
Güzelyurt goods
E) in that most African industries are classified as
small-to-medium sized enterprises
27. As long as the inflation rate differs from what
is expected, ----.
32. Some researchers have recently questioned
A) there will always be winners and losers ----.
B) the same is true for unanticipated inflation
C) it is called hyperinflation when rates exceed 50% A) whether drug protocols used in American lethal
per month injections produce death without pain
D) the purchasing power of your wage would be less B) that some people used to believe capital
than you anticipated punishment was wrong
E) many people devoted their time to speculation in C) which lethal injections replaced death by hanging
real estate D) because it is obliged to execute as humanely as
it can
E) when the time has come for renewed
28. On the issue of why men earn more than consideration of the death penalty
women, ----.

A) various studies suggest the opposite is true 33. Dutch was widely known in Europe in the 17th
B) some people are of the opinion that sex century, ----.
discrimination plays an important role
C) American economists observed a gap of about A) since English has had much influence on Dutch
20% among workers aged 26-34 since the Second World War
D) a recent study suggests that Afro-American B) as it has had an influence in Sri Lanka
females earn 86% as much as white females C) when Dutch-derived artistic terms such as
E) others believe that the gender gap is smaller in “landscape” and “sketch” were adopted into
Europe but much larger in Japan English
D) but there was a Low German influence on
English starting with the later Middle Ages
29. Even before the Walt Disney Company signed E) until scholars use the word “Netherlandic” as a
an agreement with the French government to historical term for the various dialects spoken in
open Euro Disney, ----. the Netherlands

A) the park would employ thousands of people and


attract a large number of tourists 34. It was not until the middle of the 19th century
B) Disneyland Paris has more than twice as many ----.
visitors as the Louvre
C) certain attractions are introduced to cater to A) while some geologists were studying the
French tastes stratification of rocks
D) critics began to refer to it as a cultural disaster B) when there were significant achievements in the
E) the French are the largest European consumers newly developing science of geology
of Disney products such as comic boks C) in which these guidelines were to be the basis of
archaeological excavations
D) that the discipline of archaeology became truly
30. The most prominent ancient monument in established
Đzmir is the ruined fortress of Kadifekale, ----. E) before the term “prehistory” itself came into
general use
A) so successive reconstructions were made by the
Romans, Byzantines and Ottoman Turks
B) and all that survives are the fortifications on
Kadifekale
C) but the foundations and a few of the lower
courses were built around 295 B.C.
D) whereas the ruins that one sees there today are
the remains of the citadel
E) which was originally the acropolis of the
Hellenistic city

168
35. Most stutterers can recite poems or sing with 38. Whereas the freedom of the press in America
relative ease, ----. is guaranteed by the First Amendment to the
Constitution, the British press has never enjoyed
A) since they exhibit structural weaknesses in the such a right.
brain’s speech motor centres and auditory areas
B) whether speculation about the causes of the A) Her ne kadar Birinci Anayasa Değişikliği ile
speech problem has been widespread since Amerika’da basın özgürlüğü güvenceye
ancient times kavuşturulmuş ise de, Đngiliz basınının böyle bir
C) so as late as the mid-19th century, physicans haktan yararlanması söz konusu değildir.
were using surgery to correct supposed defects B) Amerika’da Birinci Anayasa Değişikliği sonucu
in the tongue basın özgürlüğünün tam bir güvence altına
D) although stress may also influence the onset and alınmış olmasına karşın Đngiliz basınının böyle bir
durability of stammering hakka sahip olması hiçbir zaman
E) but normal conversation can be a distressing öngörülmemiştir.
exercise in frustration C) Basın özgürlüğü, Birinci Anayasa Değişikliği ile
Amerika’da güvence altına alınırken, böyle bir
hak Đngiliz basını için hiçbir zaman gündeme
36. – 38. sorularda, verilen Đngilizce cümleye gelmemiştir.
anlamca en yakın Türkçe cümleyi bulunuz. D) Amerika’da basın özgürlüğü, Birinci Anayasa
Değişikliği ile güvence altına alındığı halde, Đngiliz
36. The Romans adopted from the Greeks not only basını hiçbir zaman böyle bir hakka sahip
epic and lyric poetry, but also rhetoric, olmamıştır.
philosophy, and the writing of history. E) Đngiliz basınının hiçbir zaman sahip olmadığı
basın özgürlüğü hakkı, Amerika’da Birinci
A) Romalılar, Yunanlılardan sadece destanı ve lirik Anayasa Değişikliği ile kayıtsız şartsız güvence
şiiri değil, aynı zamanda söylevi, felsefeyi ve tarih altına alınmıştır.
yazımını almışlardır.
B) Romalıların Yunanlılardan aldıkları arasında
sadece destan ve lirik şiir değil, aynı zamanda 39. – 41. sorularda, verilen Türkçe cümleye
söylev, felsefe ve tarih yazımı bulunmaktadır. anlamca en yakın Đngilizce cümleyi bulunuz.
C) Romalılar, destan, lirik şiir, söylev, felsefe ve tarih
yazımının hepsini Yunanlılardan almışlardır. 39. Son derece saygın bir yazar ve muhalif olan
D) Destan ve lirik şiirden başka, söylev, felsefe ve Vaclav Havel, 1989’da Çekoslovakya
tarih yazımı da, Romalıların Yunanlılardan cumhurbaşkanı seçilince, ülkedeki aşağı yukarı 42
aldıkları arasındadır. yıllık komünist yönetim sona erdi.
E) Romalıların Yunanlılardan aldıklarını, hem destan
ve lirik şiir, hem de söylev, felsefe ve tarih yazımı A) The election in 1989 of Vaclav Havel, a much
oluşturmaktadır. admired writer and rebel, as the president of
Czechoslovakia ended the 42-year Communist
regime in the country.
37. Until the 1950s, Nepal was a closed society B) In 1989 when Vaclav Havel, a highly respected
ruled by hereditary prime ministers, but since writer and dissident, was elected president of
then it has had an open-door policy for integration Czechoslovakia, nearly 42 years of Communist
with the outside world. rule in the country came to an end.
C) Communist rule in Czechoslovakia had lasted for
A) 1950’lere gelinceye kadar aynı soydan gelen about 42 years before Vaclav Havel, who was
başbakanlarca kapalı bir toplum olarak yönetilen admired greatly as a writer and political rebel,
Nepal, o zamandan beri dış dünya ile was elected president of the country.
bütünleşerek açık kapı politikası benimsemiştir. D) Vaclav Havel was so respected in
B) Aynı soydan gelen başbakanlar, 1950’lere kadar Czechoslovakia as a writer and political figure
Nepal’i kapalı bir toplum olarak yönetmişlerse de that, when he was elected president in 1989, the
daha sonra dış dünya ile bütünleşmek amacıyla, Communist regime in the country, which had
açık kapı politikası benimsenmiştir. lasted for over 42 years, came to an end.
C) Nepal’in, aynı soydan gelen başbakanlarca E) As a dissident and an extremely admired writer,
kapalı bir toplum olarak yönetilmesi 1950’lere Vaclav Havel was elected president of
kadar sürmüştür, fakat daha sonra, dış dünya ile Czechoslovakia in 1989, and this brought to an
bütünleşme hedeflenerek, açık kapı politikası end the 42-year Communist rule in the country.
benimsenmiştir.
D) Her ne kadar 1950’lere gelinceye değin Nepal,
aynı soydan gelen başbakanlarca kapalı bir
toplum olarak yönetilmişse de daha sonra, açık
kapı politikası izlenmiş ve dış dünya ile
bütünleşme sağlanmıştır.
E) 1950’lere kadar Nepal, aynı soydan gelen
başbakanlarca yönetilen kapalı bir toplumdu;
ancak, dış dünya ile bütünleşmek için, o
zamandan beri açık kapı siyaseti izlemiştir.

169
40. I. Dünya Savaşı’nın başında Romanya 42. – 46. sorularda, boş bırakılan yere, parçada
tarafsızlığını ilân etti; ancak, daha sonra Almanya anlam bütünlüğünü sağlamak için getirilebilecek
ve müttefiklerine karşı savaşa katıldı. cümleyi bulunuz.

A) Although Romania was neutral at the time World 42. Although difficulties with names are nothing
War I broke out, it soon decided to declare war new, the nature of name-giving changed with the
on Germany and her allies. introduction of computer technology. ----. In the
B) When World War I started, Romania was old days, a letter from overseas addressed to the
determined to remain neutral but, soon after, it “U.S.” or the “U.S.A.” or even the “E.U.” would
declared war on Germany and her allies. stand a chance of being delivered, but an e-mail
C) At the start of World War I, Romania proclaimed for the corresponding geographic domain must
its neutrality, but later joined the war against have the exact designation “US”; no variation is
Germany and her allies. tolerated.
D) Romania’s neutrality was declared at the start of
World War I, but this did not last long since it later A) For example, the ticker symbols that identify
declared war on both Germany and her allies. securities on the New York Stock Exchange can
E) Romania’s declaration of war on Germany as well be no more than three characters long
as her allies came later, since at the start of B) Broadcast radio stations in the US have call signs
World War I it had decided to remain neutral. of either three or four letters, and the first letter is
always either K or W
C) The names can get so long and intricate that only
41. Yunan alfabesinden kendi alfabelerini a computer can identify them
geliştiren Akdeniz ulusları arasında, M.Ö. 9. D) It is not just the names that are scarce; we are
yüzyılda Đtalya’ya yerleşmiş olan Etrüskler de even running out of numbers
bulunuyordu. E) Place names – and abbreviations for them – offer
a good example of how names have changed
A) The Etruscans, who had invaded Italy during the
9th century B.C., were one of the Mediterranean
peoples who adopted their alphabets from the 43. Capturing your attention and holding it is the
Grek alphabet. prime motive of most television programming,
B) Among the Mediterranean peoples who and this enhances its role as a profitable
developed their own alphabets from the Grek advertising vehicle. ----. So the surest way to get
alphabet were also the Etruscans, who had audiences focused on a programme is to provide
settled in Italy in the 9th century B.C. them with constant stimulation through variety,
C) It was from the Greek alphabet that, like other novelty, action and movement.
Mediterranean peoples, the Etruscans, who had
conquered Italy in the 9th century B.C., developed A) It is difficult to escape the influence of television
their own alphabet. B) The advertisements are often more entertaining
D) Like the other Mediterranean peoples, the than the actual programmes
Etruscans, who invaded and settled in Italy in the C) The only things Americans do more than watch
9th century B.C., adapted the Greek alphabet and television are work and sleep
developed their own. D) Programmers live in constant fear of losing the
E) The Etruscans were among the Mediterranean attention of their audiences
peoples who settled in Italy during the 9th century E) By the age of 20 an average person will have
B.C. and created their own alphabet from the been exposed to at least 20,000 hours of
Greek alphabet. television

44. The arts in Italy have enjoyed a long and


glorious history, and Italians are very proud of
this. ----. Still, efforts are being made throughout
the country to put as many art collections on
show as possible. The performing arts such as
opera and cinema are also underfunded, yet there
are spectacular cultural festivals.

A) A tradition of literary Italian was established back


in the 13th and 14th centuries by Dante and
Petrarch, who wrote in a cultured Florentine
dialect
B) In the land of Verdi and Rossini, opera is
naturally well supported
C) However, it does cost a lot to look after them
D) It was the artists and scholars of 15th-century
Florence who inspired the Renaissance in
Europe
E) On the other hand, Rome, Florence and Venice
are naturally the main tourist destinations

170
45. In its first case concerned with global 48.
warming, the US Supreme Court ruled in April that Maria: - Natives of the areas near the Arctic Circle
greenhouse gases such as carbon dioxide are air say that people who come from Europe or
pollutants which the Environmental Protection America are essentially lonely.
Agency (EPA) can regulate. As a consequence, Bob: - ----
experts agree that greenhouse emissions from Maria: - Well, I think they’re trying to say that
automobiles and possibly power plants will face Europeans and Americans are not so closely
regulation. The debate will now focus on how involved with their neighbours or relatives.
strict or flexible those rules will be. ----. Moreover, Bob: - That idea deserves some thought, I
the agency also stated that even if it did have the suppose.
power to regulate these gases, it would not do so.
A) Do you ever feel lonely? I never do.
A) If the agency were to refuse, there would be a B) In fact, natives of the Arctic used to live together
lawsuit against it with their extended families.
B) However, the EPA had long claimed to have no C) I completely disagree.
authority in regulating these gases D) What do they mean by that?
C) Siding with the EPA were several industry groups E) Which natives do you mean?
and ten states, a number of which rely heavily on
coal, electricity or motor vehicle production
D) The Supreme Court ruling may represent a 49.
milestone in the legal battle over climate change Oscar: - This article tells how a poor woman in
E) Notably, 12 states had been seeking waivers Bangladesh borrowed money from a bank and set
from the Agency permitting them to pass laws up a small profitable business.
requiring reduced car emissions of greenhouse Ernie: - ----
gases from the 2009 model year onward Oscar: - I had never heard of it until I read this
article. Very interesting.
Ernie: - Imagine. When she has finished paying off
46. Whether or not the number of pilots increases the loan, she’ll make an even greater profit!
in the long run, for the foreseeable future small
planes could make a difference mainly if they A) So you are interested in economic matters, are
constitute the operating fleet for a new national you?
system of air taxis. ----. And, in the race to create B) Oh, I know. It is an example of the micro-credit
this fleet, two companies are deeply involved. system practised in Bangladesh.
C) How did she find a bank that would lend her the
A) They will be looking for jet planes priced at well money?
under a million dollars apiece D) As you may recall, the per capita income in
B) The people racing to create new systems of air Bangladesh has always been very very low.
transportation are nearly all men who learned to E) Bangladesh is not an industralized country, and
fly as teenagers the rate of unemployment there is very very high.
C) Both these companies have already begun large-
scale production of a genuinely new small
airplane 50.
D) In fact, most airlines have introduced Helena: - I’m reading an article on the US
considerable reductions in their air fares government’s breakup of the Standard Oil Trust,
E) A supply of inexpensive, safe, comfortable small which occurred in 1911.
planes could bring freedom and convenience to a Ron: - ----
broader share of the travelling public Helena: - Well, the trust was controlling over 90%
of the market for refined petroleum products, and
using unfair means to do so. The government
47. – 51. sorularda, karşılıklı konuşmanın boş broke it up into several new companies to
bırakılan kısmını tamamlayabilecek ifadeyi encourage competition and reduce prices.
bulunuz. Ron: - It’s a good thing they did, too.

47. A) Why don’t you try reading something a little more


Woody: - Hi, Amy. How are you? current?
Amy: - Oh, I’m fine, but I’m worried about my B) Yes; there are many similar stories in US
daughter. Her notes in maths have fallen this year, economic history.
and I think it’s mostly due to her teacher. C) Oh? What happened?
Woody: - ---- D) I’ve always found American industrial history
Amy: - Oh, the teacher is qualified enough, but I fascinating.
don’t think she’s used to teaching at my E) I don’t think it should have been broken up.
daughter’s age level.

A) Why? Isn’t he or she a good enough teacher?


B) I had the same worries with my son last year, but
everything turned out alright.
C) Why don’t you talk to the school principal?
D) Are you sure your daughter is working hard
enough?
E) What about your son? How’s his maths this year?

171
51. 55. (I) To answer the question “why?” is the most
Fred: - Have you read this article about Lady Bird difficult task in archaeology. (II) The past 30 years
Johnson, the wife of former US president Lyndon have seen the re-emergence of the use of
Baines Johnson? archaeological theory. (III) Indeed, it is the most
Bob: - ---- challenging and interesting task in any science or field
Fred: - That’s right. The article is largely about her of knowledge. (IV) For with this question, we go
commitment to cleaning up America and making it beyond the mere appearance of things and on to a
more beautiful by planting trees and flowers in the level of analysis that seeks in some way to
cities. understand the pattern of events. (V) Indeed, this is
Bob: - Well, that’s a worthy cause, and I think she the goal motivating many researchers who take up
succeeded during her lifetime. the study of the human past, whether through
archaeology or history.
A) No, I haven’t. How do you find the time to read so
many articles? A) I B) II C) III D) IV E) V
B) No; I was never a big fan of either President or
Mrs Johnson.
C) No. What does it say about her? 56. (I) An engineer may choose either to work or not
D) No, but I’d like to have a look at it when you’re to work in defense-related industries and be ethically
through. justified in either position. (II) Many reasonable
E) No, I haven’t. Didn’t she die last year? engineering professionals feel that, ethically, they
cannot work on designs that will ultimately be used to
kill other humans. (III) Their remoteness from the
52. – 56. sorularda, cümleler sırasıyla actual killing doesn’t change this feeling. (IV)
okunduğunda parçanın anlam bütünlüğünü bozan Everyone in his working life is likely to face problems
cümleyi bulunuz. that cannot be solved easily. (V) Even though they
won’t push the button or may never actually see the
52. (I) It has been estimated that at least 100,000 victims of the weapon, they still find it morally
children testify in court cases in the US every year. (II) unacceptable to work on such systems.
This figure does not include the much larger number
of instances in which children provide evidence A) I B) II C) III D) IV E) V
outside court. (III) In recent years, a number of
researchers have performed studies that speak
directly to this important question. (IV) The cases in
which children testify have a range of topics, but the
most frequent category among criminal trials,
accounting for about 13,000 cases each year, is child
sexual abuse. (V) In most instances of alleged abuse,
the child witness is also the target of the abuse.

A) I B) II C) III D) IV E) V

53. (I) In 1958, following the signing of the Treaty of


Rome in the previous year, the European Economic
Community (EEC), now the European Union (EU),
was born, and Brussels became its headquarters. (II)
Today, the city remains home to most of the EU’s
institutions. (III) The European Commission, the EU
body that formulates the policies, is currently based in
various buildings in the city. (IV) The city is also one
of the seats of the European Parliament. (V) Each
nation has a certain number of votes according to
their size in the Council of Ministers.

A) I B) II C) III D) IV E) V

54. (I) The World Cup football tournament has grown


much more competitive due to globalization. (II) More
and more players from teams with no history of
success are getting the chance to play for more
experienced teams in other countries. (III) Indeed,
football is the most popular sport in South America.
(IV) Further, the trend now in world football is for
games in the competition to be decided by much
closer scores. (V) This means more suspense for the
spectators, a fact that makes watching the matches
more enjoyable.

A) I B) II C) III D) IV E) V

172
57. – 60. soruları aşağıdaki parçaya göre 59. According to the passage, following in the
cevaplayınız. footsteps of Adam Smith, a number of capitalist
economists in Britain ----.
The most important idea of the nineteenth century in
Britain was that everyone had the right to personal A) were much upset about the practices of factory
freedom, and this became the basis of capitalism. owners and, therefore, made proposals to the
This idea, which had originated with Adam Smith in government to stop child labour
the eighteenth century, spread widely due to the B) believed that more laws were needed to
popularity of his book The Wealth of Nations. After encourage commercial and industrial activities
Adam Smith, several capitalist economists argued throughout the country
that the government should not interfere in trade and C) were so concerned with the happiness of the
industry at all. Fewer laws, they claimed, meant more majority in society that they provided employment
freedom, and freedom for individuals would lead to even for women and children
happiness for the greatest number of people. These D) did their best to improve the economy of the
ideas were eagerly accepted by the growing middle country and fully supported the government’s
class. However, it soon became very clear that the economic policies
freedom of factory owners to do as they pleased had E) maintained that commercial and industrial
led to slavery and misery for the poor, not to activities were to be completely free from any
happiness or freedom. By 1820, more and more government interference
people had begun to accept the idea that the
government must interfere to protect the poor and the
weak. The result was a number of laws to improve 60. Though it was still widely believed in
working conditions. For instance, one of the laws, nineteenthcentury Britain that laws limit freedom,
which went into effect in 1833, limited the number of ----.
hours that women and children were allowed to work.
A) the government in Britain did not hold with this
57. As pointed out in the passage, in the early belief
decades of the nineteenth century in Britain ----. B) factory owners felt that they needed the
protection of laws
A) a growing number of people shared the view that C) laws to improve working conditions in the
the poor had to be protected against capitalist factories were eventually introduced
abuse D) Adam Smith did not share this view
B) the English middle class strongly supported the E) the book, The Wealth of Nations, discredited this
government’s proposals for the improvement of theory
life for the poor
C) most factory owners introduced a wide range of
measures to improve the working conditions of
women and children
D) the government adopted a new policy which
ultimately led to the abolition of slavery in the
country
E) the government strongly held the view that new
legislative steps had to be taken to increase the
effectiveness of capitalism in the country

58. It is stated in the passage that the idea of


individual freedom, ----.

A) widely popular among factory owners, led to a


serious decline in trade and industry in
nineteenth-century Britain
B) first put forward by Adam Smith in the eighteenth
century, led to the rise of a capitalist economy in
Britain
C) supported by capitalist economists, in fact had
nothing to do with the development of the
capitalist economy in Britain
D) which formed the essence of Britain’s
government policies, had been originally attacked
by Adam Smith
E) which was confined to economic activities, had
much influence on the government’s economic
policies in Britain in the eighteenth century

173
61. – 64. soruları aşağıdaki parçaya göre 64. It is clear from the passage that those
cevaplayınız. emigrating from England in the seventeenth
century, ----.
The seventeenth century is probably the first in
English history in which more people emigrated than A) were largely criminals hoping to make a fresh
immigrated. In the course of the century, something start in life
over one-third of a million people, mainly young adult B) were mostly Catholics headed for Europe and
males, emigrated across the Atlantic. The largest Maryland
single group made for the West Indies; a second C) were usually reluctant to do so
substantial group made for America, in particular D) were not usually serious about finding
Virginia and Catholic Maryland, and even Puritan New employment
England. The pattern of emigration was a fluctuating E) included a small number who were simply looking
one, but it probably reached its peak in the 1650s and for good pay and adventure
1660s. For most of those who emigrated, the search
for employment and a better life was almost certainly
the principal cause of their departure For a clear
minority, however, freedom from religious persecution
took precedence. Moreover, an increasing number
were forcibly transported as a punishment for criminal
acts. In addition to these transatlantic emigrants, an
unknown number emigrated to Europe and settled
there. The largest group were probably the sons of
Catholic families making for religious houses in
France and elsewhere. There were also some
adventurers who were willing to fight in any cause if
the pay were good.

61. As clearly pointed out in the passage, for a


very large majority of people who left England in
the seventeenth century for America and the West
Indies, ----.

A) the main motive was to find work and improve


their way of life
B) New England seemed to offer far beter economic
opportunities than any other place
C) religious freedom was of vital importance and
became the main reason for emigration
D) the real attraction was a life filled with excitement
E) Virginia and Maryland provided better conditions
for employment than the West Indies

62. According to the passage, seventeenth-entury


emigration from England ----.

A) was mostly in the direction of Europe


B) included as many women as men
C) surpassed immigration to England
D) did not include men wishing to fight in foreign
wars
E) to the West Indies exceeded emigration there
from many other European countries

63. It is stated in the passage that the emigrants


from England to America in the seventeenth
century ----.

A) had to choose between Virginia and New


England
B) were almost all in search of religious freedom
C) found what they were looking for there
D) included criminals who, as a punishment, were
being deported from the country
E) followed a very stable pattern since most of the
emigrants shared the same aims

174
65. – 68. soruları aşağıdaki parçaya göre 67. According to the passage, every recognized
cevaplayınız. Athenian male citizen ----.

Ancient Greece consisted of a number of city-states, A) had the right of direct involvement in collective
of which Athens was one of the greatest. In the fifth decision-making
century B.C., all citizens native to Athens could both B) attending a public assembly was expected to
vote and speak in a government assembly; but this, of speak on laws and foreign affairs
course did not apply to women and slaves. This C) was dissatisfied with the system of direct
system of “direct democracy” was feasible because democracy as practised by a number of citystates
Athens was a small community. Each individual could in Greece
be involved, gathering collectively in the public square D) had the right to hold whatever religious beliefs he
where decisions on government matters, such as chose to
laws and foreign affairs, were made. City E) believed that representative democracy was
administrators were expected to account for their more feasible than direct democracy
decisions. What counted in ancient Athens was the
authority of the community as a whole. This took
precedent over the liberty of the individual. The 68. In the passage, attention is drawn to the fact
freedom of the individual to make private decisions, that administrators in ancient Athens ----.
such as choosing a religion, was restricted on the
grounds that the interests of society were paramount. A) were so powerful that no citizen dared to speak
However, this simple form of democracy had its against them in the assembly
drawbacks. While subsequent political thinkers B) were elected by the members of the public
praised the concept of direct political involvement, it assembly
was recognized that this would be impractical in larger C) were normally held responsible for their decisions
communities. Indeed, societies with populations of of government
thousands or millions would never be able to manage D) did their best to reconcile the liberties of the
the logistical problem of direct participation. It was, individual and the interests of the community
therefore, natural that in modern times there emerged E) were biased against direct democracy and
the idea of representative democracy. advocated representative democracy

65. It is clearly stated in the passage that, in


ancient Athens, ----.

A) women enjoyed the same democratic rights as


men and took part in the process of collective
decision-making
B) what was to the benefit of the community counted
more than the personal interests of the individual
C) representative democracy was fully practised
although it had certain drawbacks with regard to
the process of decision-making
D) everybody living in the city was required to take
part in public assemblies and vote for the election
of city administrators
E) the authority of city administrators was so great
that their decisions on government matters were
final

66. As pointed out in the passage, the system of


direct democracy ----.

A) fully safeguarded individual liberties while it


restricted the authority of the community as a
whole
B) was so efficient that, with the exception of
Athens, every Greek city-state adopted it
C) would obviously be unworkable in large
communities
D) was short-lived as it resulted in so many
disagreements
E) enabled a number of Greek city-states to resist
the interference of Athens in their foreign affairs

175
69. – 72. soruları aşağıdaki parçaya göre 71. It is pointed out in the passage that, due to the
cevaplayınız. economic depression in the West in the 1930s,
----.
During the economic depression that affected the
whole Western world in the 1930s, with its mass A) most governments curbed public expenditure and
unemployment, poverty and other social ills, changed their system of taxation
governments, for the most part, did nothing. The B) there was unemployment on a very large scale
accepted wisdom was that, given time, the free C) the increasing demand for goods had to be
market would solve its own problems and that prevented through harsh economic policies
government interference would only make things D) many governments introduced a series of
worse. John Maynard Keynes, the British economist measures to solve social problems
who challenged this belief, argued that it was the E) it was almost impossible for people to borrow
proper responsibility of governments to prevent both Money
booms and recessions in order to maintain gradual
economic growth and permanent full employment. He
maintained that this could be done by manipulating 72. As it is stated in the passage, in the 1930s, ----.
taxation, credit and public expenditure. If the economy
was growing too fast, then money and, therefore, A) all governments in the West carefully followed the
demand could be taken out of the economy by higher economic policies proposed by Keynes
taxes, lower government spending and by making it B) permanent full employment was achieved
harder to borrow money. If there was recession and through an efficient implementation of free
growing unemployment, then the government could market policies
put money into the economy through lower taxes, C) governments generally felt that the free market
higher public expenditure and easier credit. Thus, was the only way of solving the problems of the
demand could be encouraged. If, as a result, there depression
was money in people’s pockets, then more would be D) despite high unemployment, people had so much
spent on goods and more people would be needed to money that the demand for goods could not be
make the goods to fulfil the extra demand, and this controlled
would reduce unemployment. E) the British government gave Keynes full
responsibility to improve the economy
69. According to the Keynesian argument
summarized in the passage, in order to bring
down unemployment, ----.

A) new economic policies would be formulated by


the government, so that demand could be curbed
B) governments would follow a policy of
nonintervention in the economy and allow the
problem to be solved through the free market
C) the government had to make new loans available
for businesses at very high interest rates
D) the government would prefer to increase taxation,
so that people would spend less
E) one of several measures to be introduced by the
government would be to encourage an increase
in public spending

70. As is clear from the passage, Keynes ----.

A) differed little from his contemporaries in his


economic theories
B) had much trust in the free market, which he
believed had a positive impact on the economy of
the 1930s
C) did not think that the economic depression of the
1930s was serious enough to justify government
interference
D) firmly believed that government intervention in
the management of the economy could be
necessary
E) argued that economic prosperity should not be
the prime aim of any government

176
73. – 76. soruları aşağıdaki parçaya göre 75. It is asserted in the passage that, when the
cevaplayınız. Suez Canal was opened in 1869, ----.

Since the dawn of civilization, the Middle East, a A) European powers had already begun to colonize
region at the crossroads of Africa, Asia and Europe, the Middle East on a large scale
has been important to large and small powers alike, B) an increasing number of Europeans began to
from the empires of the East to the imperial powers of make long journeys through Asia
the West. The opening of the Suez Canal in 1869, C) the flow of Middle Eastern oil to the West
which transformed maritime travel between Europe increased steadily
and Asia, added to European interest. The region’s D) European powers made a joint effort to get
other riches also encouraged European intervention control of the region
and rivalries. This resulted in a series of E) travel by sea between East and West underwent
confrontations between the Ottoman Empire and its great changes
European adversaries, and finally in the collapse of
the former and the direct or indirect European
colonization of large parts of the region in the course 76. It is pointed out in the passage that, from the
of the nineteenth and early twentieth centuries. But nineteenth century onwards, European
what added to the Middle East’s importance in the involvement in the Middle East ----.
twentieth century was oil, which was found in
abundance in the Persian Gulf and in parts of North A) was solely concerned with the security of the
Africa. Moreover, in the strategic context of the Cold Suez Canal
War, the region’s geopolitical importance provided an B) was strongly resisted by the Ottoman Empire,
additional reason for the superpowers to increase which thus prevented the colonization of the
their role and presence. whole region
C) has largely been motivated by the region’s
73. According to the passage, during the Cold geopolitical importance
War, ----. D) led to various conflicts with the Ottoman Empire
and finally ended its presence in the region
A) the Middle East was geopolitically so important E) centred only on the Suez Canal and the opening
that the superpowers came to believe that of the sea-route to Asia
involvement in the area was desirable
B) the geopolitical importance of the Middle East
was recognized for the first time
C) the European powers competed with each other
in the Middle East in order to have full control of
the oil reserves in the Persian Gulf
D) the superpowers not only increased their
activities in the Middle East but also formed
alliances with the countries of the region
E) the superpowers were attracted by the oil of the
Middle East more than by its geopolitical position

74. It is stressed in the passage that, from very


early times, every Eastern as well as every
Western power ----.

A) has ignored the geopolitical position of the Middle


East
B) has tried to colonize the Middle East and exploit
its natural resources
C) has maintained its presence in the Middle East
on account of its oil
D) has regarded the Middle East as having crucial
importance
E) has followed a policy of peace and cooperation
with the Middle East

177
77. – 80. soruları aşağıdaki parçaya göre 79. The writer claims in the passage that
cevaplayınız. Aphrodisias, with its wonderful ruins, ----.

Until the early 1960s, the picturesque ruins of A) has been a place of attraction for archaeologists
Aphrodisias were scattered in and around the very for many centuries
pretty village of Geyre, where the houses had been B) is among the most attractive and loveliest of the
built largely from remnants of the ancient city. But the archaeological sites in Turkey
present excavations, which began in 1961, have now C) has now been completely excavated and
reached such a scale that the village and its declared an archaeological zone
inhabitants have been moved to another site nearby. D) still lies hidden under the modern village of Geyre
Some of the superb sculptures unearthed are now and, hence, needs to be unearthed
exhibited in a new museum, which is located in what E) is a Graeco-Roman archaeological site, settled
was once Geyre’s village square, while others can be for the first time in the sixth century B.C.
seen around the archaeological zone, one of the most
interesting and beautiful sites in all of Turkey.
Surprisingly, the excavations at Aphrodisias have 80. It is stated in the passage that the temple of
unearthed remains of a settlement dating back to Aphrodite in Aphrodisias ----.
about 5,800 B.C.. The site seems to have been a very
ancient shrine of Ishtar, the fertility goddess of A) was the very first site to be excavated in the early
Nineveh and Babylon, who was one of the 1960s and is today one of the most captivating
predecessors of Aphrodite, the Greek goddess of sights at Geyre
love. In fact, the earliest Greek sanctuary of Aphrodite B) was designed and built in full imitation of the
on this site dates from the sixth century B.C., and it temple of the Babylonian fertility goddess Ishtar
was from this sanctuary during the next four centuries on the same site
that the cult of Aphrodite spread throughout the C) lost its importance completely once the cult of the
Graeco-Roman world. goddess had spread in the Graeco-Roman world
D) is still regarded by most archaeologists as an
77. As one learns from the passage, the architectural wonder, which, with its superb
excavations at Aphrodisias ----. sculptures, surprises everybody
E) gave rise to the popularity among the Greeks and
A) originally had a single aim: to find out whether the Romans of the worship of the goddess
there was any connection between this city and
the Mesopotamian cities of Nineveh and Babylon TEST BĐTTĐ.
B) confirm that Ishtar was reborn as Aphrodite CEVAPLARINIZI KONTROL EDĐNĐZ.
C) have unearthed only a small part of the ancient
settlements, which are very extensive and need
to be carefully identified
D) have focused on unearthing Aphrodite’s
sanctuary as well as other Graeco-Roman
remnants
E) have been going on for over four decades and
reveal that the city dates back to very early times

78. It is pointed out in the passage that the people


of modern Geyre ----.

A) have always played an active part in the


excavations, since they are very keen to learn
about the past of their village
B) were forced to re-settle at a new site quite far
from that of Aphrodisias itself
C) used the remains of ancient Aphrodisias as
building material for their homes
D) had been moved to a new site for re-settlement
before the excavations began in the early 1960s
E) have insisted that the sculptures unearthed
during the excavations be exhibited in the village
square

178
1. – 18. sorularda, cümlede boş bırakılan yerlere 8. Psychologists, psychiatrists and
uygun düşen sözcük ya da ifadeyi bulunuz. neuroscientists ---- for years over how much of
our behaviour ---- driven by our genes versus the
1. When taken in doses of more than 10 times the environment in which we grow up and live.
recommended daily ----, vitamins A and D are
toxic. A) had argued / had been
B) used to argue / would be
A) maintenance B) exposure C) application C) have argued / is
D) advice E) allowance D) argue / would have been
E) argued / will be

2. Many people with a regular exercise programme


accept minor injuries and soreness as an almost 9. For a long time, scientists believed that
---- component of their programme. functional deficits in certain brain regions ----
autism – the result of complications in brain
A) irrelevant B) available C) inevitable structure that no change in wiring among neural
D) expansive E) irreversible networks ----.

A) might cause / had fixed


3. Despite a preoccupation with body image and B) caused / could fix
weight loss, the prevalence of obesity in the US C) have caused / fixes
continues to rise ----. D) cause / could have fixed
E) would have caused / fixed
A) collectively B) adequately C) randomly
D) respectively E) dramatically
10. Epidemiology, which ---- as a science until the
19th century, is a branch of medicine that
4. The goal of the Neuroscience Research investigates factors ---- to improved health, or the
Programme is to ---- our understanding of the occurrence of a disease in a particular population.
neuronal systems involved in a variety of
neurological disorders. A) could not have evolved / having contributed
B) had not evolved / to contribute
A) rank B) relate C) implicate C) has not evolved / to have contributed
D) increase E) interact D) did not evolve / contributing
E) could not evolve / to be contributing

5. The body resists starvation by ---- its own


tissues and using them as a source of calories – 11. Although 25 to 30 per cent of all people ----
much like burning the furniture to keep the house some form of excessive mood disturbance during
warm. their lifetime, only about 10 per cent ---- a disorder
severe enough to require medical attention.
A) getting off B) breaking down
C) carrying out D) following up A) would experience / have had
E) making for B) experienced / will have had
C) had experienced / had had
D) will have experienced / would have
6. Billions of useful bacteria colonize our guts, but E) experience / have
because antibiotics are lethal to a whole range of
microbes, drugs taken for a chest infection, for
example, ---- friendly bacteria too. 12. In biofeedback, biological responses are
measured ---- electronic instruments, and the
A) wipe out B) sort out C) build up status of those responses is immediately
D) take back E) move in available ---- the person being tested.

A) with / at B) by / to C) in / about
7. Prevention is the ideal way to approach pain, D) from / on E) through / with
and several educational programmes that ----
workers to avoid lower back injuries ---- some
effectiveness. 13. Perhaps the most crucial factor ---- patient
noncompliance is poor verbal communication ----
A) are training / would show the practitioner and the patient.
B) would train / had shown
C) have trained / showed A) for / by B) of / among C) about / of
D) train / have shown D) in / between E) to / for
E) trained / will show

179
14. ---- physical activity can enhance physical 19. – 23. sorularda, aşağıdaki parçada
functioning, reduce anxiety, stress, and numaralanmış yerlere uygun düşen sözcük ya da
depression, it also poses some hazards to one’s ifadeyi bulunuz.
physical and psychological health.
People may bruise easily because of fragile
A) Although B) Since C) Whether capillaries in the skin. Each time these small blood
D) In case E) Unless vessels break, a little blood (19) ----, leaving tiny red
dots in the skin and bluish-purple bruises. Women
seem more prone than men to bruising from a minor
15. ---- surgeons in France had performed the first injury, especially (20) ---- the thighs, buttocks, and
partial face transplant late in November, upper arms. Older people are especially susceptible
psychologists began to question whether the to bruising after bumps and falls (21) ---- they have
patient was mentally stable enough to handle the fragile blood vessels and a thinner layer of fat under
stressful, high-risk procedure. the skin, which normally serves as a cushion to help
protect against injury. For most people, the condition
A) Suppose that B) So long as C) As soon as isn’t serious, but bruising easily (22) ---- a sign that
D) Because E) If something is wrong with the blood clotting elements,
most likely the platelets. Blood tests can determine if
(23) ---- problems exist.
16. ---- people have been diagnosed with
cardiovascular disease (CVD), they typically enter 19.
a cardiac rehabilitation programme to change A) passes by B) comes up C) runs away
their lifestyle and thereby avoid subsequent CVD. D) breaks off E) leaks out
A) Whereas B) After C) While
D) Even though E) As if 20.
A) on B) with C) throughout
D) through E) over
17. In veterinary medicine, federal governments
and professional associations keep data on
animal euthanasia and, ---- the research results 21.
obtained so far, have developed guidelines and A) unless B) even so C) because
procedures. D) whereas E) so that
A) in the hope of
B) as of 22.
C) in case of A) has to be B) should be C) would be
D) in accordance with D) may be E) will be
E) on behalf of

23.
18. The anaemia of folate deficiency, ---- DNA A) such B) no C) much
synthesis slows and the body’s cells lose their D) so E) many
ability to divide, is characterized by large,
immature blood cells.
24. – 35. sorularda, verilen cümleyi uygun şekilde
A) in which B) for whom C) how tamamlayan ifadeyi bulunuz.
D) in that E) why
24. Even after a drug has been approved by the
Food and Drug Administration (FDA), ----.

A) such drugs are occasionally studied first in a


small number of healthy volunteers
B) comprehensive premarketing studies detected
adverse reactions about once in every 1,000
doses
C) many drugs are rejected at this stage, too
D) the manufacturer must conduct postmarketing
surveillance and report any previously
undetected adverse drug reactions
E) it was 3 years before the drug came onto the
market

180
25. Because symptoms of sleep apnea ocur 30. According to recent studies, bananas are the
during sleep, ----. most promising food for an edible vaccine against
the hepatitis B virus, ----.
A) severe apnea can result in headaches, excessive
daytime sleepiness and slow mental activity A) but this virus is very common in many African
B) they must be described by someone who countries
observes the person sleeping B) so that potatoes, tomatoes and other vegetables
C) snoring is associated with episodes of gasping might serve to immunize people some day
and choking C) just as symptoms of acute viral hepatitis usually
D) most people have been treated successfully and begin suddenly
can now sleep comfortably D) which lives in about 5 per cent of the world’s
E) people who snore are advised to sleep on their population
side or face-down E) whereas scientists have discovered a way to
boost the level of immune-triggering hepatitis
protein in the fruit
26. Even when a cell becomes cancerous, ----.

A) antigens released into the bloodstream by some 31. Cancer cells are often perceived as all having
cancers are sometimes called tumour markers the same potential to proliferate and spread within
B) it is a cell whose biological function has been the body, ----.
altered
C) a fully functioning immune system can’t always A) because cancer treatments must target cancer
destroy all these cancer cells stem cells to eradicate the disease
D) the immune system may regard these cells as B) but in many types of cancer, only a small subset
foreign of tumour cells has that power
E) the immune system can often destroy it before it C) where the study of stem cells is shedding light on
replicates cancer treatments today
D) so one among thousands of tumour cells may be
a cancer stem cell responsible for driving the
27. While some drugs, once absorbed, tend to disease
stay within the watery tissues of the blood and E) since current treatments focus on killing the
muscle, ----. greatest number of cancer cells

A) others concentrate in specific tissues such as the


liver and kidneys 32. Audiometry measures hearing loss precisely
B) drugs penetrate different tissues at different with an electronic device ----.
speeds
C) all drugs are either metabolized or excreted intact A) that produces sounds at specific pitches and
D) these drugs accumulate in the fatty tissues specific volumes
E) they rapidly circulate through the body B) if a person listens to a series of two-syllable,
equally accented words presented at specific
volumes
28. Armed with gene sequences and the latest C) although it measures how loudly words have to
gene chip technologies to measure the activity of be spoken to be understood
thousands of genes simultaneously, ----. D) because loud tones presented to one ear may
also be heard by the other ear
A) high-risk groups can be targeted with more E) even if it detects how much sound passes
aggressive screening programmes through the middle ear
B) antibody-based therapies have an inherent
limitation
C) the tissue that surrounds a tumour plays a crucial 33. For any pain syndrome, surgery is the most
role in its development extreme approach and is usually recommended
D) anti-cancer drugs have been largely a matter of ----.
dosing people and waiting to see what happens
E) researchers are now several years into the task A) because it has its own potential dangers and
of mapping which ones grow abnormally possibilities for complications
B) in case the pain starts suddenly and stays in a
particular area of the back
29. Although the cause of Reye’s syndrome is C) only when all the other means of treatment have
unknown, ----. failed to produce a positive result
D) unless back surgery is only partially successful in
A) most doctors are unaware of its effects repairing tissue damage
B) people who take aspirin have an increased risk of E) as long as drugs, skin stimulation and surgery
bleeding can all be effective for controlling pain
C) certain viruses, such as influenza A or B or
varicella virus, may be involved
D) the doctor would perform a liver biopsy and
spinal tap to diagnose it
E) no specific treatment can be proposed to prevent
it

181
34. Bulimia nervosa occurs with equal prevalence 37. The severity of a burn depends on the amount
in various social classes and ethnic groups, ----. of tissue affected and the depth of the injury,
which is described as first, second, or third
A) while people who have this disease are degree.
distressed by it
B) but both eating disorders occur far more often in A) Bir yanığın şiddeti, etkilenen doku miktarına ve
women than men hasarın birinci, ikinci veya üçüncü derece olarak
C) even though anoxeria nervosa has a mortality tanımlanan derinliğine bağlıdır.
rate of 5% to 10% B) Etkilenen dokuların durumu ve oluşan hasarın
D) since an antidepressant drug can often help derinliği, yanığın birinci, ikinci veya üçüncü
control the disease derece olarak belirtilen şiddetini ortaya koyar.
E) whereas anorexia nervosa appears primarily C) Bir yanığın birinci, ikinci veya üçüncü derece
among the upper classes olarak sınıflandırılan hasar derinliği ile zarar
görmüş doku miktarına dayanılarak o yanığın
şiddeti belirlenir.
35. People with extensive burns have an D) Bir yanığın şiddetinin birinci, ikinci veya üçüncü
increased risk of infection ----. derece olarak tanımlanması, zarar gören doku
miktarıyla ve ortaya çıkan hasarın derinliğiyle
A) so that a new layer of epidermis grows slowly ilişkilidir.
from the edges of the burned area E) Birinci, ikinci veya üçüncü derece olarak
B) whereas many people who have been burned dlandırılan hasar derinliği ve zarar gören
need to take an analgesic, often a narcotic, for at dokuların çokluğu göz önüne alınarak bir yanığın
least a few days şiddeti saptanabilir.
C) although damaged red blood cells in the injured
area may make the burn bright red
D) because the damaged skin opens up the body to 38. Because much is still unknown about breast
invasion by harmful organisms cancer and no single treatment works all the time,
E) but life-threatening burns require immediate care, doctors may have different opinions about the
preferably at a hospital equipped to treat burns most appropriate treatment.

A) Göğüs kanseri için her zaman sonuç veren tek bir


36. – 38. sorularda, verilen Đngilizce cümleye tedavi yönteminin bulunmamasının yanı sıra
anlamca en yakın Türkçe cümleyi bulunuz. hastalıkla ilgili yeterli bilginin olmaması nedeniyle
hekimler arasında en etkili tedaviye ilişkin farklı
36. Most infectious diseases are caused by görüşler mevcuttur.
microorganisms that invade the body and multiply B) Göğüs kanserine ilişkin çok az şey ortaya
there. konduğu ve her zaman sonuç veren belirli bir
tedavi yöntemi bulunamadığı için, hekimler en
A) Pek çok bulaşıcı hastalığın nedeni vücuda geçerli tedavi hakkında farklı düşünceler ileri
yayılan ve vücutta üreyen mikroorganizmalardır. sürmektedir.
B) Mikroorganizmalar, vücuda girerek ve vücutta C) Hekimler göğüs kanseri için en uygun tedavi
çoğalarak, çeşitli bulaşıcı hastalıkların hakkında çeşitli görüşler ileri sürmektedir; çünkü,
oluşmasına yol açar. bu hastalık hakkında yeterli bilginin olmaması,
C) Çoğu bulaşıcı hastalık, vücudu işgal eden ve tek bir tedavinin her zaman geçerli kılınmasını
vücutta çoğalan mikroorganizmalar nedeniyle engellemektedir.
oluşur. D) Göğüs kanseri hakkında yeterli bilgi olmadığı için
D) Bulaşıcı hastalıkların nedenlerinin çoğu, vücuda her zaman sonuç veren tek bir tedavi yöntemi
girerek çoğalan mikroorganizmalarla ilgilidir. yoktur ve bu nedenle hekimler en uygun tedaviye
E) Üreyip vücudu ele geçiren mikroorganizmalar ilişkin çelişkili düşünceler ortaya atmaktadır.
nedeniyle birçok bulaşıcı hastalık ortaya E) Göğüs kanseri hakkında çok şey hâlâ
çıkmaktadır. bilinmediğinden ve hiçbir tedavi tek başına her
zaman sonuç vermediğinden, hekimler, en uygun
tedaviye ilişkin farklı görüşlere sahip olabilirler.

182
39. – 41. sorularda, verilen Türkçe cümleye 41. Tümör büyümesine neden olan kanser kök
anlamca en yakın Đngilizce cümleyi bulunuz. hücrelerinin varlığı, çeşitli kan kanseri ve tümör
türlerinde tespit edilmiştir; ancak, bu habis kök
39. Yoksul ülkelerdeki çoğu insan, sıtma, AIDS, hücrelerinin nasıl oluştuğu hâlâ belirsizdir.
verem ile zengin ülkelerde daha az bilinen birçok
hastalık çekmektedir; çünkü, hükümetleri, ilâç A) It has been shown that a wide range of blood
şirketlerinin istediği fiyatları karşılayamamaktadır. cancer and tumour types result from malignant
stem cells whose properties are not yet fully
A) Malaria, AIDS, tuberculosis and a number of known.
other diseases not known in rich countries affect B) Cancer stem cells that cause tumour growth have
many people in poor countries, where the been discovered in a number of blood cancer and
governments are reluctant to pay the prices tumour types, but it is not yet clear why they
companies demand for drugs. become malignant.
B) Most people in poor countries suffer from malaria, C) Different blood cancer and tumour types are
AIDS, tuberculosis and many diseases lesser caused by cancer stem cells, although there is no
known in rich countries, since their governments evidence to show how these malignant stem cells
cannot afford the prices drug companies want. come into being and drive tumour growth.
C) Since malaria, AIDS, tuberculosis and various D) Even though it is not yet certain how malignant
other diseases little-known in rich countries are stem cells form, they are often found in various
very common among the people of poor blood cancer and tumour types, causing them to
countries, the governments find it hard to pay the grow.
prices demanded by companies for drugs. E) The existence of cancer stem cells that drive
D) While a growing number of people in poor tumour growth, has been established in several
countries suffer from malaria, AIDS, tuberculosis blood cancer and tumour types, but how these
and several other diseases unknown in rich malignant stem cells arise is still uncertain.
countries, their governments refuse to pay the
prices companies ask for their drugs.
E) The governments in poor countries cannot afford 42. – 46. sorularda, boş bırakılan yere, parçada
to pay the prices companies want for their drugs, anlam bütünlüğünü sağlamak için getirilebilecek
but more and more people are being affected by cümleyi bulunuz.
malaria, AIDS, tuberculosis and other diseases
that do not occur in rich countries. 42. As anyone who has sat through a lecture on a
warm day knows, attentiveness and mental
alertness can be hard to sustain. ----. Its
40. Đnsan vücudu, her biri, yaşamı sürdürmek için counterpart is sleep, a state in which we continue
gerekli olan bir işlevi yerine getiren ayrı ayrı organ to receive stimuli but are not conscious of them.
ve dokulardan oluşmuş son derece karmaşık bir
sistemdir. A) It is also important to regulate sleep
B) Researchers can study the electrical activity in
A) The human body is a highly complex system the brain during arousal and sleep
made up of separate organs and tissues, each C) It filters out some familiar and repetitive
performing a function essential to maintaining life. information that constantly enters the nervous
B) Since the human body consists of many different system
organs and tissues, each of which has a function D) Arousal is a state of awareness of the outside
essential for the maintenance of life, it is to be world
compared to an extremely complex system. E) In general, the less mental activity that takes
C) The human body, which is an extremely complex place, the more regular are the brain waves
system, is made up of innumerable organs and recorded on the electroencephalogram (EEG)
tissues which have separate functions essential
for the maintenance of life.
D) The presence of many different organs and
tissues, each of which performs a function
necessary for maintaining life, makes the human
body a very complicated system.
E) The human body is so complicated a system that
each of the organs and tissues it consists of
performs a function vital for maintaining life.

183
43. Most people are familiar with one important 46. Ulcerative colitis is a disease in which
function of the kidneys – to rid the body of waste extensive areas of the walls of the large intestine
materials that are either ingested or produced by become inflamed and ulcerated. Some clinicians
metabolism. A second function that is especially believe that it results from an allergic or immune
critical is to control the volume and composition destructive effect, but it could also result from a
of the body fluids. For water and virtually all chronic bacterial infection. ----.
electrolytes in the body, the balance between
intake and output is maintained in large part by A) The person may also have a fever and a poor
the kidneys. ----. appetite
B) Even then, the ulcers sometimes fail to heal, and
A) Each kidney in the human body is made up of the only solution is removal of the entire colon
about 1 million nephrons, each capable of C) Moreover, about 10 per cent of people who have
forming urine the disease have only simple attacks
B) This regulatory function of the kidneys maintains D) Whatever the cause, there is a strong hereditary
the stable environment of the cells necessary for tendency for susceptibility to this disease
them to perform their various activities E) Unlike Crohn’s disease, ulcerative colitis never
C) Blood flow to the two kidneys is normally 21 per affects the small intestine
cent of the cardiac output, or about 1,200ml/min
D) The two kidneys lie on the posterior wall of the
abdomen, outside the peritoneal cavity 47. – 51. sorularda, karşılıklı konuşmanın boş
E) These include urea, uric acid, creatinine and bırakılan kısmını tamamlayabilecek ifadeyi
metabolites of various hormones bulunuz.

47.
44. In 1862, the English doctor John Langdon Patient: - Dr Jameson, what are the safest
Down, who was the director of a home for exercises for me, as a middle-aged man, to
mentally handicapped children, described the improve my fitness?
case of one of the children, who was short and Dr Jameson: - There are several. Let me think
had stubby fingers and unusual eyelids. The boy’s which would be suitable for you.
condition was later labelled by this doctor’s Patient: - ----
surname. ----. In 1959, the French paediatrician Dr Jameson: - Those would be fine. Cycling can
Jérome Lejeune discovered that these children also be useful.
have three copies of chromosome 21, instead of
two. A) It will take time to make a decision, won’t it? I’ll
call you to learn your recommendations.
A) Physical limitations continue to challenge these B) I would prefer walking and swimming. Forget the
individuals rest.
B) Today, more than 350,000 Americans have C) At my age, would aerobics really be suitable?
Down’s syndrome D) Let it be something natural: no exercising
C) But the cause of Down’s syndrome was not machines for me!
uncovered for another century E) I have to climb three flights of stairs to get to my
D) Scientists confirmed that within this chromosome home. Isn’t that enough exercise?
are the genes that cause both Down’s syndrome
and Alzheimer’s disease
E) In 2004, American scientists tracked the effects 48.
of other genes on chromosome 21 Eleanor: - This article is about patients in a
persistent vegetative state. Some of them wake up
many years later, and some of them never do.
45. In the 1990s, most psychiatrists in the US Bob: - How do doctors decide whether or not
started to treat children and teens with these patients should be kept on life-support?
antidepressants designed for adults, since they Eleanor: - ----
wanted something more for them than talk Bob: - I suppose so. Obviously there are legal and
therapy. ----. Now, however, studies have found medical concerns, but I think the family of the
that some antidepressants might interfere with patient should have some rights in the matter, too.
normal patterns of growth in children’s still
developing brains. A) Sometimes a patient’s brain works around the
damaged part, allowing the patient to begin to
A) Although these drugs had not been tested in the function normally again.
young brains of children, they hoped the benefits B) One patient in a minimally conscious state started
would outweigh the risks to speak and move about 19 years after he’d
B) Several brain areas are commonly associated been injured.
with depression such as those involved in mood, C) I don’t know how; but they do have to come to a
sleep, appetite, desire and memory decision.
C) Finding these effects in mice is a long way from D) It is difficult for doctors to make accurate
proving that the same thing happens in humans diagnoses of unconscious patients.
D) Adults with depression usually recognize that E) A patient shows sensory responses to pain, even
they have a problem and they want to fix it, but when the nervous system isn’t connected to the
children rarely ask to see a psychiatrist centres of the brain that cause it to experience it.
E) Concerns over antidepressant use led the FDA to
issue a warning label for these medications

184
49. 52. – 56. sorularda, cümleler sırasıyla
Senior Doctor: - I hear you’ve been doing research okunduğunda parçanın anlam bütünlüğünü bozan
on alcoholism in Turkey. cümleyi bulunuz.
Junior Doctor: - Yes. I feel the findings are not all
that alarming. 52. (I) Mycobacterium tuberculosis has plagued
Senior Doctor: - ---- humans throughout history. (II) Improved public health
Junior Doctor: - In a way, yes. For instance, nearly brought the epidemic under control, and the advent of
8 per cent of adults in the US have a serious antibiotics in the 1940s seemed to ensure successful
problem with alcohol use. treatment. (III) But the bacteria have adapted to
nearly every drug used against them. (IV) As a result,
A) Really? You say that in comparison with other there has been a recent resurgence, and the disease
developed countries? now kills a person every 15 seconds, worldwide. (V)
B) Aren’t they? It’s a fact that alcohol produces both The germ in question is the tuberculosis-causing
psychological and physical dependence. Mycobacterium tuberculosis, once the leading cause
C) In your report, you claim that men are four times of death in the US.
more likely than women to become alcoholics.
D) In general, alcoholics often can’t manage their A) I B) II C) III D) IV E) V
behaviour and tend to drive while drunk.
E) Well, alcohol is rapidly absorbed from the small
intestine into the blood stream. 53. (I) The most commonly used illegal drug in the US
is marijuana. (II) Its potential for serious health
consequences is still debated, but few authorities
50. regard it as a major health risk. (III) In fact, a recent
Dr Clark: - Let me stress that the most study found no increased death rates for marijuana
troublesome problem of peritoneal dialysis is the users, except for men who died of AIDS. (IV)
risk of infection. Moreover, marijuana has been used medically to treat
Student: - We can use antibiotics to clear up the glaucoma and to prevent the vomiting and nausea
infection. associated with chemotherapy. (V) Although this
Dr Clark: - ---- study does not determine cause and effect, it seems
Student: - So it’s inevitable that in peritoneal safe to assume that marijuana did not cause death
dialysis, complications can occur any time. from AIDS, but rather that men receiving a diagnosis
of AIDS may have subsequently increased their use
A) Generally, peritoneal dialysis is not performed in of marijuana.
people who have abdominal wall infections.
B) Yet that’s not all. Other problems are also A) I B) II C) III D) IV E) V
associated with this type of dialysis.
C) In peritoneal dialysis, a catheter is inserted
through a small incision in the abdominal wall into 54. (I) The causes of depression aren’t fully
the peritoneal space. understood. (II) A number of factors may make a
D) Moreover, inflammation of the kidneys can also person more likely to experience depression, such as
be caused by an infection. a family tendency, side effects of certain medications,
E) In fact, when the kidneys fail, waste products and an introverted personality, and emotionally upsetting
excess water can be removed from the blood by events. (III) For some reason, women are twice as
haemodialysis. likely as men to experience depression. (IV)
Psychological studies show that changes in hormone
levels before menstruation and after childbirth may be
51. factors. (V) Hormones also control the volume of fluid
Jerome: - Did you see the documentary on TV and the levels of salt and sugar in the blood.
about the US healthcare system?
Mark: - ---- A) I B) II C) III D) IV E) V
Jerome: - That’s right, many families have gone
bankrupt paying for treatment for a family
member. 55. (I) Like all medicines, sleep medicines have side
Mark: - Yes, and that’s not all. Some patients have effects. (II) On the other hand, daytime drowsiness
even been refused treatment by hospitals. can be best avoided by taking the lowest dose
possible. (III) The most common of these effects are
A) Yes; to tell the truth, I was hoping that it would be dizziness, lightheadedness, and difficulty with
more thorough. coordination. (IV) Sleep medicines can also make one
B) Yes, but I didn’t have time to watch all of it. sleepy during the day. (V) How drowsy one feels
C) No, but I wanted to watch it. Did you? depends upon how one’s body reacts to the medicine.
D) Yes. It’s getting worse and worse.
E) No; I watched a film on another channel instead. A) I B) II C) III D) IV E) V
Was it very informative?

185
56. (I) Doctors have noted a dramatic increase in 59. The point is made in the passage that what an
bone fractures in children and teenagers in recent autistic person normally does ----.
years. (II) Bones are rigid but not inflexible; they will
bend in response to external forces. (III) For example, A) can best be assessed through intensive sessions
if you fall forward with your hands outstretched, the of psychotherapy
bones of your hands, wrists, and arms will flex to B) is always tolerated, although it often draws other
absorb the shock and then return to their original people’s attention
shape. (IV) However, as many of us know from C) is always ignored by other people, since it does
personal experience, the skeletal system has its not interest them at all
limits. (V) If a force is applied that exceeds a bone’s D) in time develops into extreme fascinations or
resiliency, the result is a broken bone, or fracture. fixations
E) is never taken into consideration in deciding the
A) I B) II C) III D) IV E) V kind of treatment to be given

57. – 60. soruları aşağıdaki parçaya göre 60. It is stated in the passage that autism ----.
cevaplayınız.
A) can always be successfully treated by means of
Autism, from the Greek word for “self,” was first psychotherapy
identified as a disorder in 1943. Initially, it was thought B) is an abnormality which has not yet been
to be a psychological disorder brought on by cold or researched adequately
unemotional mothers, and curable by intensive C) is far more widespread among male than female
sessions of psychotherapy. During the 1960s, children
specialists realized that autistics frequently had D) usually has only one symptom, which reveals
epilepsy and abnormal brain scans, which led to the itself as epilepsy
condition being recognized as a brain disorder by the E) is a single kind of neurological abnormality, which
1970s. Autism is now known to be a hereditary can be treated most effectively through
neurological condition, about three times more antidepressants
common in boys than girls. Usually, autistics lack the
ability to relate normally to other people and have an
anxious desire to maintain a routine, which evolves
with age into intense interests or obsessions. Many
autistic people deliver monologues on topics while
unaware of other people’s comments or possible
discomfort. There are several related, but different,
forms of autism. Depending on the severity,
symptoms can sometimes be alleviated with carefully
controlled antidepressants, although sufferers
typically find it difficult to function normally in society.

57. As is clear from the explanation given in the


passage, ----.

A) antidepressant drugs should not be used in order


to reduce the severity of autism symptoms
B) autism should not be regarded as a neurological
disorder inherited from the family
C) autism is not a single type of disorder with only
one level of severity
D) autism was a condition recognized by the ancient
Greeks
E) autistics can usually have a normal relationship
with other people

58. It is pointed out in the passage that it has


taken the medical world several decades to ----.

A) decide to what extent psychotherapy can be used


in the treatment of autism
B) decide on which antidepressants are the best for
the treatment of autism
C) discover how the relationship between mothers
and their children has an effect on autism
D) establish clearly which methods are most efficient
to alleviate the symptoms of autism
E) understand clearly what autism is and the causes
that lead to it

186
61. – 64. soruları aşağıdaki parçaya göre 64. It is explained in the passage that, when the
cevaplayınız. level of oxygen in the tissue fluid decreases, ----.

Because oxygen is one of the major substances A) a long time is required to get it back to normal
required for chemical reactions in the cells, it is B) the activity of the red blood cells increases
fortunate that the body has a special control immediately to carry more oxygen to the cells
mechanism to maintain an almost exact and constant C) haemoglobin lets out enough oxygen to bring it
oxygen concentration in the extracellular fluid. This back to normal
mechanism depends principally on the chemical D) the amount of oxygen provided through the lungs
characteristic of haemoglobin, which is present in all increases in a remarkable way
red blood cells. Haemoglobin combines with oxygen E) this has an adverse effect on the chemical
as blood passes through the lungs. Then, as the efficiency of haemoglobin
blood passes through the tissue capillaries,
haemoglobin, because of its own strong chemical
affinity for oxygen, does not release oxygen into the
tissue fluid if too much oxygen is already there. If the
oxygen concentration is too low, however, sufficient
amounts are released to re-establish adequate tissue
oxygen concentration. Thus, the regulation of oxygen
concentration in the tissues depends principally on
the chemical characteristics of haemoglobin itself.

61. The passage describes ----.

A) the structural and chemical characteristics of the


cells in the human body
B) the process of the maintenance of an adequate
level of oxygen in body tissues by haemoglobin
C) the chemical reactions which take place in all the
cells throughout the body
D) the vital importance of the lungs in providing
oxygen for the red blood cells
E) why oxygen plays a basic role in the chemical
reactions of the cells

62. It is stressed in the passage that haemoglobin


----.

A) is responsible for an exact and constant


maintenance of oxygen in the tissues
B) is a special mechanism whereby the level of the
extracellular fluid in the body is regulated
C) has a chemical characteristic which helps the
blood pass through the tissue capillaries
D) continually releases small amounts of oxygen
into the tissue fluid in the body
E) circulates through the blood only when the
oxygen concentration in the tissue fluid becomes
too low

63. One understands from the passage that


oxygen ----.

A) in the extracellular fluid always increases when


there is a chemical reaction
B) in the tissues has to be reduced in order to
prevent over-concentration
C) and haemoglobin in the body do not always have
a constructive relationship
D) is a substance that sometimes plays an adverse
role in the body’s chemical reactions
E) is necessary for the chemical reactions that take
place in the cells

187
65. – 68. soruları aşağıdaki parçaya göre 67. As one learns from the passage, some
cevaplayınız. suggest that, in assessing the risk of breast
cancer, one should ----.
The evidence linking dietary fat with cancer is less
conclusive than for heart disease, but it does suggest A) also pay attention to the question of whether
an association between total fat intake and some kcalorie intake affects the risk of heart disease
types of cancer. Dietary fat seems not to initiate B) focus primarily on the type of cancer and ignore
cancer development but to promote cancer once it the role of saturated fat
has arisen. Some studies report a relationship C) take into consideration total energy intake rather
between specific cancers and saturated fat or dietary than the levels of kcalories from fat
fat from animal sources, which is mostly saturated. D) closely study the variations observed in the
Thus, health advice to reduce the risk of cancer percentages of kcalories from saturated fat
parallels that given to reduce the risk of heart disease: E) be concerned not only with total energy intake
reduce total fat intake, especially saturated fat. The but also with the percentages of kcalories from
relationship between dietary fat and the risk of cancer fat
differs for various types of cancers. In the case of
breast cancer, some studies indicate little or no
association between dietary fat and cancer. Others 68. It is suggested in the passage that every
find that total energy intake is a beter predictor than cancer ----.
the percentage of kcalories from fat. In the case of
prostate cancer, there does appear to be a strong A) seems to develop as a result of the harmful
association with fat. This association appears to be effects of dietary fat
due primarily to saturated fat from meats; fat from milk B) is of a different kind and cannot necessarily be
or fish has not been implicated in cancer risk. associated with dietary fat
C) can be predicted through kcalorie percentages
65. It is clear from the passage that, according to related to dietary fat
some studies, saturated fat ----. D) is in some way associated with heart disease
E) is just as fatal as breast or prostate cancer
A) and its effect on a number of diseases, including
heart disease, has yet to be further researched
B) plays a primary role in the development of all
kinds of cancer rather than in heart disease
C) should only be consumed moderately so as to
prevent all kinds of cancers
D) does seem to have a link with certain types of
cancer, of which prostate cancer is an example
E) has only recently come to be regarded, in the
light of extensive research, as a major cause of
cancer

66. A point is made in the passage that the


medical advice given for the reduction of the risk
of cancer ----.

A) is the same as the medical advice given for the


reduction of the risk of heart disease
B) does not depend on any kind of conclusive
evidence and, hence, can be ignored
C) is only partially sound, since the cancer studies
undertaken so far have not reached a convincing
conclusion
D) not only concerns the intake of saturated fat but
is also related to the state of the individual’s
health
E) has been challenged by some specialists on the
grounds that the causes of cancer have not been
established fully

188
69. – 72. soruları aşağıdaki parçaya göre 72. As one can understand from the passage,
cevaplayınız. people in East Asia began to contract beriberi ----.

People who fail to eat enough food to meet energy A) due to excessive alcohol consumption
needs risk nutrient deficiencies, including thiamin B) when the hulls were removed from their rice
deficiency. Inadequate thiamin intakes have been C) due to damaged nerve processes caused by
reported among malnourished and homeless people. excess thiamin
Similarly, people who derive most of their energy from D) only because rice was excluded from their regular
empty-kcalorie items, like alcohol, risk thiamin daily diet
deficiency. Alcohol contributes energy, but provides E) even though the food they took in contained all
few, if any, nutrients and often displaces food. In the nutrients needed
addition, alcohol enhances thiamin excretion in the
urine, doubling the risk of deficiency. Prolonged
thiamin deficiency can result in the disease “beriberi,”
which was first observed in East Asia when the
custom of polishing rice became widespread. Rice
provided 80 per cent of the energy intake of the
people of that area, and rice hulls (the outer skin of
rice) were their principal source of thiamin. When the
hulls were removed, beriberi spread like wildfire.
Because thiamin participates in nerve processes,
paralysis sets in when it is lacking. The symptoms of
beriberi include damage to the nervous system as
well as to the heart and other muscles.

69. According to the passage, thiamin deficiency


----.

A) has been attributed mainly to excessive


consumption of alcohol
B) has always been very common among East
Asian people
C) and its effects were understood, to some extent,
after the case of the East Asian people
D) affects the nervous system and can lead to
paralysis
E) has been observed mostly among the homeless
people of East Asia

70. As is clear from the passage, undernourished


people ----.

A) usually suffer from a number of diseases which


are linked to prolonged thiamin deficiency
B) are likely to contract various diseases, in
particular, beriberi
C) are particularly vulnerable to thiamin deficiency
and frequently suffer from heart disease
D) must avoid alcohol and keep to a diet which is
rich in thiamin
E) may face nutrient deficiencies since they do not
have enough energy intake

71. In the passage, it is pointed out that alcohol


----.

A) has a positive effect on nutrients in food and,


thus, increases one’s intake of energy
B) is a major source of all kinds of nutrients,
especially of thiamin
C) is an empty-kcalorie beverage and increases the
urinary excretion of thiamin
D) plays a positive role in overcoming one’s nutrient
deficiencies
E) provides a great deal of energy whereby
inadequate thiamin intake is tolerated

189
73. – 76. soruları aşağıdaki parçaya göre 76. According to the passage, if a person with
cevaplayınız. hypertension loses weight, ----.

The single most effective step people can take A) other nondrug treatments should be disregarded
against hypertension is to find out whether they have B) regular blood pressure readings may be
it. At check-up time, a health-care professional can discontinued
provide an accurate resting blood pressure reading. C) every attempt should be made to read the blood
Under normal conditions, blood pressure fluctuates pressure accurately
continously in response to a variety of factors D) it may be possible to decrease the dosage of
including such things as talking or shifting position. hypertension medication
Some people react emotionally to the procedure, E) effective drugs should be used to reduce
which raises the blood pressure reading. For these abdominal fat in the first place
reasons, if the resting blood pressure is above
normal, the reading should be repeated before
confirming the diagnosis of hypertension. Thereafter,
the blood pressure should be checked regularly. In
general, efforts to reduce high blood pressure focus
on weight control, because excess body fat,
especially abdominal fat, can precipitate
hypertension. Indeed, weight loss alone is one of the
most effective nondrug treatments for hypertension.
Those who are using drugs to control their blood
pressure can often reduce or discontinue the drugs if
they lose weight. Even a modest loss of 5 kilograms
may significantly lower blood pressure.

73. According to the passage, weight loss ----.

A) is certainly one way of overcoming hypertension


effectively
B) can be a very exhausting and boring process for
some people
C) can most effectively be achieved through the use
of certain drugs
D) is by itself not effective enough to reduce high
blood pressure
E) should be taken into consideration only when
drug treatment for hypertension has failed

74. It is clear from the passage that, before


diagnosing hypertension, the health-care
professional ----.

A) must make sure that the patient has no emotional


problems
B) should advise the patient on the most effective
methods of losing weight
C) ought to recommend to the patient certain drugs
that are effective for weight loss
D) should repeat the blood pressure test if the first
reading is above normal
E) must first deal with excess body fat and bring the
blood pressure under control

75. It is pointed out in the passage that, normally,


blood pressure ----.

A) should not be regarded as a means to find out


about hypertension
B) has an irregular pattern in those people who have
too much abdominal fat
C) fluctuates widely among people who cannot
control their emotions
D) can be controlled and regulated through repeated
readings
E) is not stable, but constantly changes due to
various reasons

190
77. – 80. soruları aşağıdaki parçaya göre 80. In this passage, the writer ----.
cevaplayınız.
A) is concerned with oral cancers in regard to their
Oral cancers develop in 30,000 Americans and cause nature, early detection, and effects as well as
8,000 deaths each year, mostly in people over age their incidence rate in America
40. This represents about 2.5 per cent of cancer B) explains in detail the importance of oral hygiene
cases and 1.5 per cent of all cancer-related deaths. and how cancerous growths in the mouth can be
Clearly this is a high rate considering the small size of prevented
the mouth in relation to the rest of the body. Along C) gives a wide range of reasons for widespread
with cancers of the lung and skin, cancers of the cases of lung cancer in America
mouth are more preventable than most other cancers. D) gives a full description of how oral cancers
Non-cancerous and cancerous growths can originate spread to many parts of the body, including the
in any type of tissue in and around the mouth, lymph nodes of the jaw and neck
including bone, muscle, and nerve. Rarely, cancers E) suggests that cancerous and non-cancerous
found in the mouth region have spread there from growths in the mouth can only be differentiated
other parts of the body – most commonly the lung, through a full medical and dental examination
breast and prostate. Screening for oral cancer should
be an integral part of medical and dental
examinations because early detection is critical. TEST BĐTTĐ.
Cancers less than a half inch across can usually be CEVAPLARINIZI KONTROL EDĐNĐZ.
cured easily. Unfortunately, most oral cancers aren’t
diagnosed until they’ve spread to the lymph nodes of
the jaw and neck. Because of delayed detection, 25
per cent of oral cancers are fatal.

77. It is made clear in the passage that oral


cancers ----.

A) account for 1.5 per cent of deaths from cancer in


Americans
B) are always cured effectively, since they are
detected right away
C) are more fatal than other cancers, such as breast
and prostate cancers
D) have only in the past few years become a major
concern among dentists in the US
E) are invariably caused by cancers in the rest of
the body

78. One understands from the passage that


medical examinations ----.

A) should be carried out once dental examinations


confirm a non-cancerous growth in the mouth
B) must always precede dental examination to
detect cancerous growths in the mouth
C) should include a check for the early detection of
oral cancers
D) must be made of all the body before the
treatment of an oral cancer can be initiated
E) should focus on cancerous rather than
noncancerous growths in various parts of the
body

79. It is emphasized in the passage that the delay


in the diagnosis of most cancers in the mouth ----.

A) can have a harmful effect on other parts of the


body, especially on the lungs
B) can lead to the spread of these cancers to the
jaw and neck areas
C) should not be tolerated, since these cancers can
spread to the breast and prostate
D) is a primary cause of death among middle-aged
people
E) is not as serious as a delay in detecting skin
cancers

191
1. – 18. sorularda, cümlede boş bırakılan yerlere 8. Cosmologists believe that equal amounts of
uygun düşen sözcük ya da ifadeyi bulunuz. matter and antimatter ---- in the early universe, but
since matter and antimatter annihilate each other,
1. Europe’s deep-ocean margin, stretching from something ---- to create an excess of matter,
the Arctic to the Mediterranean and to the Black leading to the universe we see today.
Sea, contains an ---- of biological energy and
mineral resources. A) are created / happened
B) have been created / had happened
A) extension B) influence C) abundance C) had been created / has happened
D) element E) assumption D) may have been created / must have happened
E) would be created / has been happening

2. Cloud seeding, a technique which attempts to


make precipitation by dispersing silver iodide 9. Since 1997, when the spacecraft ‘‘The Mars
particles into clouds, remains ---- because it is Global Surveyor’’ ---- over the surface of Mars for
quite difficult to prove whether it actually works. the first time, scientists ---- by the considerable
magnetic anomalies identified on the planet.
A) valuable B) confidential C) essential
D) fascinating E) controversial A) has flown / are intrigued
B) flies / were intrigued
C) had flown / had been intrigued
3. Some frozen areas of Greenland have always D) was flying / may have been intrigued
melted each summer, but recent research has E) flew / have been intrigued
shown that the extent of snowmelt in Greenland
increased ---- between 1992 and 2005.
10. In 1998, 16 per cent of the world’s coral reefs
A) vaguely B) roughly C) scarcely ---- by bleaching caused by El Nino, but half of
D) drastically E) marginally those reefs ---- signs of recovery, especially in
protected areas where it is illegal to harvest coral.

4. Twenty-five per cent of excess nitrogen from A) have been killed / showed
overfertilization of fields ---- into the oceans via B) were killed / are showing
rivers. C) had been killed / would have shown
D) have been killed / show
A) brings B) carries C) swims E) could be killed / had shown
D) expels E) flows
11. If all of the Arctic ice ----, global sea levels ----
5. The SOS (Space Observatories in School) by 23 feet, submerging most coastal areas.
programme was ---- to make young people more
aware of opportunities to study the fundamental A) melted / will rise
sciences, especially those related to the universe. B) is to melt / are rising
C) were to melt / could rise
A) thought over B) built in C) made up D) will melt / rise
D) set up E) found out E) had melted / rose

6. Ever since the sixteenth century, when Central 12. We need to worry ---- the effects of fossil-fuel
America first appeared on European maps, carbon dioxide ---- the atmosphere.
schemes have been ---- to build canals there.
A) onto / below B) on / by C) for / at
A) set off B) put forward C) run down D) about / in E) towards / on
D) taken up E) sent off
13. There’s a broad range of opinions ---- the
7. Scientists predict that should the current rate of biological consequences of being exposed ---- the
deforestation in the rainforests ----, a great many contaminated environment near Chernobyl.
of the species they support ---- completely by the
turn of the 22nd century. A) for / of B) in / with C) on / to
D) about / at E) through / from
A) continue / will have disappeared
B) is continued / will disappear
C) was continued / would disappear
D) had continued / would have disappeared
E) will continue / would have disappeared

192
14. Recently, researchers have found that ---- 19. – 23. sorularda, aşağıdaki parçada
certain genes remain activated long enough, they numaralanmış yerlere uygun düşen sözcük ya da
can dramatically enhance an organism’s health ifadeyi bulunuz.
and extend its life span.
Although radon causes many deaths, it is clear that a
A) even if B) while C) in case lot of them are preventable. Radon tests are cheap,
D) if E) although and when the gas (19) ----, diverting it from buildings
is (20) ---- a simple matter of fitting vents, fans or
membranes. (21) ---- in people’s homes, rates of
15. Tourists gathered to admire the mushroom testing and remediation have been slow-moving. And
clouds during nuclear tests in Nevada between (22) ---- it comes to workplaces, the dangers (23) ----
1951 and 1963 ---- at the time there was complete radon are barely recognized.
ignorance of the dangers of radioactive fallout.
19.
A) since B) so as to C) when A) was found B) is found C) will find
D) so that E) unless D) finds E) found

16. Genetic engineering is primarily considered a 20.


field of applied microbiology, ----, the exploitation A) hardly B) ever C) never
of microorganisms for a specific product or use. D) usually E) finally
A) but rather B) as such C) and then
D) for example E) that is 21.
A) Though B) Also C) Just
D) If E) Yet
17. A laptop maker recently released a model that
lets users change the processor, graphics card
and other parts by just removing one panel, ---- 22.
spending hours disassembling the computer. A) while B) when C) until
D) since E) after
A) due to B) on behalf of C) instead of
D) with regard to E) in order to
23.
A) within B) at C) in
18. Wetlands filter out excess nutrients and D) on E) of
pollutants by trapping them in roots and soil, ----
plants and bacteria break them down into less
harmful substances. 24. – 35. sorularda, verilen cümleyi uygun şekilde
tamamlayan ifadeyi bulunuz.
A) where B) which C) wherever
D) how E) whether
24. Although the evolutionary origin of animals is
unclear, ----.

A) scientists are no longer working on new theories


B) evolutionary biologists have abandoned
traditional views about it
C) the cells that make up the animal body are
specialized to perform specific functions
D) biologists have gathered enough evidence to put
forward a new theory about it
E) much of the vegetation on land is similar to that
found in the water

25. ---- if Gregor Mendel had never lived?

A) Is it possible to compare the science of genetics


to mathematics in that it consists of some basic
principles
B) Can the basic rules of genetic inheritance in
human beings be fully discovered
C) Are geneticists interested in the study of the
transmission of genes
D) Will our understanding of the relationship
between an organism’s genes and its
characteristics be more advanced
E) Would the development of the science of
genetics in the 20th century have been any
different

193
26. ---- that large amounts of Greenland’s melted 30. Scientists worry ----.
freshwater could dilute the salinated water of the
Gulf Stream. A) so that NASA may develop ways to shield
astronauts from radiation on Mars
A) With regard to the new data, it has been B) just as exposure to radiation is a serious danger
questioned for manned spaceflight
B) A team of ecologists from Norway have C) unless serious measures are taken to prevent fire
wondered on a space shuttle
C) An increasing number of scientists from around D) that high-energy cosmic rays could cause much
the world believe damage to space shuttles
D) In view of new evidence, one is puzzled E) provided that monitoring systems are placed
E) Despite a great deal of reliable evidence, most throughout a spacecraft
environmentalists will not be certain

31. ----, it rode aboard the massive Saturn V


27. ----, but let’s not forget where we live now – rocket.
that is, our own planet.
A) Even if the scientists at NASA were concerned
A) We have almost completely focused on space about the quality of the spacecraft Apollo 11
exploration B) Since the spacecraft Apollo 11 had been
B) As scientists, we are able to monitor and predict designed by a special team of engineers
environmental threats C) Whether the spacecraft Apollo 11 was to be used
C) Satellite images help researchers track for lunar missions
hurricanes and glacial melting D) Although a lot of money was spent for the
D) Scientists pose intricate questions about the land, development of the spacecraft Apollo 11
sky and oceans E) When the spacecraft Apollo 11 went to the Moon
E) Scientists make precise predictions about climatic in 1969
changes

32. Sulphur dioxide emissions in Europe, ----, were


28. As investigators learn more about what is reduced by 67% between 1980 and 2000.
happening in the brain during the exact moment
of insight, ----. A) which contribute to Arctic haze
B) as environmental pollution has adverse effects on
A) physicians are placing their hopes on stem cells, our life
which have reversed some symptoms in C) because there have been serious attempts so far
monkeys D) if the melting of the Arctic ice can be prevented
B) one aspect of mental imagery has been more or E) since pollution from industry and forest fires has
less neglected become a major concern
C) the primary visual cortex seemed to be activated
during the process of imagination
D) people are able to attribute new meanings to 33. By the end of the year, the NASA scientists
objects in their inner eye should know for certain ----.
E) all of us want to know what we can do to
maximize the conditions that allow us to have A) where the colourful images of Mars’ south pole
brilliant thoughts give indications of water
B) whether there is water on Mars in the form of ice
C) even though water once flowed over the surface
29. ---- because its ice sheet exerts a tremendous of Mars
influence on many ecological cycles. D) as life, similar to our own on Earth, could have
existed on Mars
A) We probably have more information on nearby E) because the surface of Mars could only have
planets than we do on Greenland been formed by flowing water
B) The climatic change on Greenland is an indicator
of things to come in the rest of the world
C) What happens on Greenland over the next 34. Future computers will be able to tell us ----.
decade does not worry scientists
D) The average temperature on Greenland has A) even if the universe consists mostly of dark
been steady in recent years matter and dark energy
E) Less than one per cent of all the water on Earth B) unless we produce more work for less effort
is drinkable and not locked up in ice C) so far as human beings can find new sources of
food
D) while landmines in war zones will be detected
E) how we can improve our performance at work

194
35. ----, one cannot notice right away that it is 38. By stimulating dead brain tissue,
moving. neuroscientists have concluded that a specific
receptor found in the outer layer of neurons
A) While an iceberg is simply a chunk of ice functions differently in schizophrenic brains.
B) When glacier ice reaches the sea
C) Since a glacier creeps downhill at a few metres a A) Ölü beyin dokularını uyaran sinirbilimciler,
year şizofren hastalarda sinirlerin dış katmanındaki bir
D) If glacier ice is not static tür reseptörün daha farklı işlev gösterdiğini ortaya
E) Before a glacier forms from accumulated snowfall koymuşlardır.
B) Sinirbilimcilerin ölü beyin dokularını uyarmaları,
şizofrenlilerin beyinlerinde bulunan özel bir
36. – 38. sorularda, verilen Đngilizce cümleye reseptörün sinirlerin dış katmanında farklı işlevler
anlamca en yakın Türkçe cümleyi bulunuz. gösterdiğini kanıtlamıştır.
C) Sinirbilimciler, ölü beyin dokularının uyarılması
36. The change in temperature distribution in the durumunda, sinirlerin dış katmanında bulunan
Arctic would also affect ocean currents in the özel bir reseptörün şizofren hastaların
Atlantic, which would, in turn, influence beyinlerinde farklı tepkilere neden olduğunu
atmospheric circulation in the region. anlamışlardır.
D) Ölü beyin dokularını uyararak, sinirbilimciler
A) Kuzey kutup dairesindeki sıcaklık değişimi sinirlerin dış katmanında bulunan belirli bir
Atlantik okyanusunda akıntıları da etkileyeceği reseptörün şizofrenik beyinlerde farklı biçimde
için bölgede atmosfer dönüşümü de değişecektir. işlev gösterdiği sonucuna varmışlardır.
B) Kuzey kutbunda sıcaklık dağılımının değişmesi E) Sinirbilimcilerin elde ettiği sonuçlara göre, ölü
sonucu, Atlantik’teki okyanus akıntıları da buna beyin dokularının uyarılması yüzünden, sinirlerin
karşılık bölgedeki hava dolaşımını etkileyecektir. dış katmanında yer alan belirli bir reseptör
C) Kutuplarda sıcaklık değişiminin dağılımı şizofren hasta beyinlerinde daha farklı
Atlantik’te de okyanus akıntıları üzerinde etki çalışmaktadır.
gösterecek, buna bağlı olarak, bölgedeki
atmosfer basıncı da etkilenecektir.
D) Sıcaklık dağılımının kutuplardaki değişimi 39. – 41. sorularda, verilen Türkçe cümleye
Atlantik’te okyanus akıntılarına da yansıyacak, bu anlamca en yakın Đngilizce cümleyi bulunuz.
da bölgenin hava dolaşımı üzerinde etkili
olacaktır. 39. Tek bir türün yok olması çok etkili değildir ama
E) Kuzey kutup bölgesinde sıcaklık dağılımındaki bir ikinci, üçüncü veya daha fazla sayıda tür yok
değişme Atlantik’teki okyanus akıntılarını da olduğunda ekosistemin tutarlılığı tehdit altına
etkileyecek, dolayısıyla, bu durum bölgedeki girer.
hava dolaşımını etkileyecektir.
A) The disappearance of a single species is not
dramatic but when a second, third, or greater
37. Breaking the hypersonic barrier of 6,000 km/h number of species becomes extinct, the stability
for commercial air transport is a very serious of the ecosystem is threatened.
research field for today’s European aircraft B) The stability of an ecosystem is not threatened by
manufacturers. the disappearance of a single species but with
the extinction of a second or third species, the
A) Günümüzde, Avrupalı hava taşıtı üreticilerinin çok risk becomes dramatic.
ciddi bir araştırma alanı da, ticari hava C) Not only is the disappearance of a single species
taşımacılığında 6.000 km/s lik hiper ses hızı dramatic but the extinction of a second, third, or
sınırını aşabilmektir. greater number of species also threatens the
B) Ticari hava taşımacılığı için, günümüzde Avrupalı stability of the ecosystem.
hava taşıtı üreticilerinin çok ciddi bir araştırma D) While the disappearance of a single species may
alanı 6.000 km/s lik hiper ses hızı sınırının not be dramatic, when a second, third, or greater
aşılması konusudur. number of species becomes extinct it may
C) Ticari hava taşımacılığındaki 6.000 km/s lik hiper threaten the stability of the ecosystem.
ses hızı sınırını aşmak, günümüz Avrupalı hava E) When a single species disappears, there is no
taşıtı üreticileri için çok ciddi bir araştırma cause for alarm; however, with the extinction of a
alanıdır. second, third, or greater number of species, the
D) 6.000 km/s lik hiper ses hızı sınırını aşmak, very stability of the ecosystem is threatened.
günümüzde Avrupalı hava taşıtı üreticilerinin
ticari taşımacılıkta çok ciddi bir araştırma alanı
haline gelmiştir.
E) Ticari hava taşımacılığı için 6.000 km/s lik hiper
ses hızı sınırını aşmak, günümüzde Avrupalı
hava taşıtı üreticileri için çok ciddi bir araştırma
alanı olarak görülmektedir.

195
40. 2006 yazında Mars’ın atmosferinde oluşan 42. – 46. sorularda, boş bırakılan yere, parçada
bulutlar, ilk defa olarak beklenmeyen bir anlam bütünlüğünü sağlamak için getirilebilecek
yükseklikte sıcaklığın -193°C olduğu 80-100 km cümleyi bulunuz.
arasında gözlemlendi.
42. Is there such a thing as a “gay brain”? ----. Gay
A) At an unexpected height of 80 to 100 km and a men tended to have brains that were more like
temperature of -193°C, cloud formations were those of straight women than of straight men –
observed in the atmosphere of Mars for the first the right and left sides were about the same size,
time in the summer of 2006. the researchers found. Gay women's brains
B) Scientists observed clouds in the atmosphere of tended to be more like those of straight men than
Mars for the first time in the summer of 2006, at of straight women – the right side tended to be
the unusual height of between 80 and 100 km, slightly larger than the left.
where the temperature is -193°C.
C) For the first time in the summer of 2006, clouds A) Such research is full of uncertainty, and it could
were observed forming in the atmosphere of not rule out the possibility that the findings were
Mars at an extreme height between 80 and 100 the result of changes that occurred in response to
km with a temperature of -193°C. experiences and behaviours, rather than being
D) In the summer of 2006, clouds forming in the inborn
atmosphere of Mars were observed for the first B) Some scientists say the new findings are part of
time at an unexpected height between 80 and an increasingly convincing body of evidence that
100 km, where the temperature was -193°C. suggests sexual orientation results from
E) Clouds were observed forming in the atmosphere fundamental developmental differences that are
of Mars for the first time since the summer of probably caused by hormonal exposures in the
2006 at an unusual height between 80 and 100 womb
km, where the temperature is -193°C. C) Some scientists remain sceptical, saying that
there has been a history of jumping to
conclusions and over-interpreting findings in the
41. Dil teknolojilerinin en büyük ilerleme field of human sexual orientation
gösterdiği uygulama alanlarından biri ses D) In 1991, brain scientists reported that the
komutuyla çalışan ev aletleri alanıdır. hypothalamus, which is involved in sexual
behaviour, tended to be smaller in gay men than
A) Voice command operated domestic appliances in straight men
constitute the field of application in which E) To find out, scientists used magnetic resonance
language technology is progressing the fastest. imaging, or MRI, to compare the symmetry of the
B) One of the fields of application in which language brains of 25 “straight” men and 25 “straight”
technology is progressing the most is that of women with those of 20 gay men and 20 gay
domestic appliances operated by voice women
command.
C) One of the newest and most exciting fields of
application for language technology is that of 43. In 1815, on the Indonesian island of Sumbawa,
domestic appliances operated by voice Mount Tambora blew up in the largest volcanic
command. eruption in recorded history, ejecting 20 times the
D) Domestic appliances which are operated by voice amount of rock that flew from Vesuvius in 79 A.D.
command are an interesting, if not new, Superhot ash and rock burned or buried all in its
application of language technology. path, including the tiny kingdom of Tambora. The
E) Language technology’s most profitable and death toll was 92,000. Wind-blown clouds from the
promising field of application is surely domestic 27-mile-high plume of ash dimmed the Sun’s rays.
appliances operated by voice command. ----. After the eruption, Sumbawa was largely
uninhabited for decades.

A) Scientists have uncovered three houses under


ten feet of ash near Tambora
B) As a result of the darkness, crops in the
surrounding regions failed, and many people
went hungry
C) Volcanologists are searching the area with
ground-penetrating radar
D) Archaeologists will continue excavating the area
E) Some scientists say that Tambora could be the
Pompeii of the East

196
44. The honeybees that pollinate agricultural 46. ----. Forests are dying, most impressively by
crops in the US were bred from stock originally burning. The damage done by wildfires in the US,
brought over from Europe in the 1600s. ----. the vast majority of them in the western states,
Whether caused by a new insecticide, disease, or has increased greatly since the late 1980s. In
a mix of stressors, the losses have resulted in a 2006, nearly ten million acres were destroyed.
great deal of research and a query: Could native With temperatures in the region up by 2°C over
bees take their place? Of the thousands of US the past 30 years, spring is coming sooner to the
species, some efficiently tend crops like apples western mountains. The snowpack – already
and alfalfa. A few, like the orchard mason bee, are diminished by drought – melts earlier in the year,
already in commercial use. Advocates are fighting drying the land and creating perfect conditions for
to preserve wild and weedy lands that support wildfires. As hotter summers extend into autumn,
these natives. the fires are ending later as well.

A) The US honey yield for 2006 was 155 million A) People in the western part of the US are not yet
pounds, and 33% of the US diet was tied to suffering from water shortage, but trees are
honeybee services B) It is thought that precipitation in the southwestern
B) The big, social colonies of honeybees are ideal US will decline steadily over the next few
for commercial pollination, and entire colonies decades
are often transported to different farms C) The fires are not only more frequent; they are
C) However, some once common native bee also hotter and more damaging
species are in decline and data on others are D) Most forests in the southwest of the US have
incomplete always burned frequently, but at low intensity
D) Honeybees remain important workers in the US, E) The typical tree bears the marks of many such
where the value of their pollination work is $14.6 fires, with black scars where the flames
billion a year consumed the bark
E) Since 2006, however, hundreds of thousands of
these honeybees have died out in what is being
called colony collapse disorder 47. – 51. sorularda, karşılıklı konuşmanın boş
bırakılan kısmını tamamlayabilecek ifadeyi
bulunuz.
45. The biofuels currently manufactured in the US
are doing great things for some farmers and some 47.
large agricultural companies, but little for the Fred: - I am sure you are familiar with Darwin’s
environment. The corn used to manufacture theory of evolution, aren’t you?
ethanol requires large doses of herbicide and Student: - Well, I can’t say I am. Can you
nitrogen fertilizer and can cause more soil erosion summarize it for me?
than any other crop. ----. Biodiesel from soybeans Fred: - ----
is only slightly better. Environmentalists also fear Student: - Now I can understand clearly what he
that rising prices for both crops will push farmers meant by his theory.
to plow up some 35 million acres of marginal
farmland now set aside for soil and wildlife A) I would suggest that Darwin’s theory
conservation, potentially releasing even more revolutionized our perception of life science.
carbon from the soil in those fields. B) Simple. For Darwin, new species arise naturally
by a process of evolution.
A) Unlike the ancient carbon unlocked by the C) It would be useful first to do some research on
burning of fossil fuels, the carbon in biofuels Darwin’s theory.
comes from the atmosphere and is returned there D) Let me first urge you to do some reading in life
when the fuels are burned science.
B) Such renewable fuels could also improve the US E) Before we discuss the theory, let’s remember
economy and help it to become less dependent how Darwin formed his theory.
on other countries
C) The boom in corn production has pushed corn
prices to levels not seen in years, causing US 48.
growers to plant the largest crop since World War Karen: - Did you know that nicotine actually
II changes the structure of the brain in a way that
D) Additionally, producing corn ethanol consumes may cause addiction?
just about as much fossil fuel as the ethanol itself Scott: - ----
replaces Karen: - And that’s not all; between 1998 and 2004,
E) The key to intelligent biofuel production is to learn tobacco companies increased the amount of
how to make it from plant material other than nicotine in cigarettes.
food, such as plant stalks, grasses, fastgrowing
trees, or even algae A) Yes, and ten per cent of new smokers become
addicted within only two days!
B) But aren’t there new anti-smoking medications?
C) That’s right; after one smoke-free year, the risk of
coronary artery disease is reduced by half.
D) I read that each year more than 19 million people
try to quit smoking.
E) Most people who want to stop smoking try many
times before they succeed.

197
49. 52. – 56. sorularda, cümleler sırasıyla
Brian: - Have you ever heard of “exoplanets”? okunduğunda parçanın anlam bütünlüğünü bozan
Duncan: - Exoplanets? No. What are they? cümleyi bulunuz.
Brian: - ----
Duncan: - So you mean they are extrasolar planets 52. (I) More people than ever before live in areas
orbiting their own suns. vulnerable to natural disasters. (II) For average
citizens, this research translates to beter
A) The vast majority of exoplanets use what is called preparedness when faced with the unexpected. (III)
“the radial-velocity method”. Over a billion people live under the shadow of the
B) Well, most of these planets are of a type known world’s 1,900 active volcanoes, yet only a few of
as hot gas giants. these volcanoes are adequately monitored. (IV)
C) So far, some 300 such bodies have been Accurate weather forecasting is the best defense
detected. against hurricanes, but landfall predictions remain
D) Put simply, they are planets outside our own inaccurate by an average of 70 miles, and many
solar system. hurricane warnings go unheeded. (V) Tsunamis can
E) They are planets with their own life forms. form too quickly for an official warning, but
recognizing the immediate signs, such as a rapidly
receding ocean, can give people precious minutes to
50. reach safety.
Stephen: - Currently, the plastics industry is
undergoing a major change. A) I B) II C) III D) IV E) V
Timothy: - How do you mean?
Stephen: - ----
Timothy: - That is good news because, as you 53. (I) Moving slowly helps the pygmy three-toed sloth
know, petrochemicals have adverse escape the notice of predators. (II) The sloth can
environmental effects. move quickly when threatened, but generally expends
no more energy than is necessary to hang around all
A) Plastic shopping bags, which are produced from day eating leaves. (III) Confined to one remote island
petrochemical materials, are widely used that separated from Panama long ago, this smaller
throughout the world. cousin of mainland sloths has managed to survive
B) As you know, the plastics industry has always quietly so far, but any disruption to its habitat could
depended on petrochemicals as raw material. have huge consequences. (IV) Algae that sometimes
C) It is a fact that the production and burning of coats its fur adds another level of camouflage. (V)
petrochemical plastics increases CO2 levels in the Nearby development could well be the end of it.
atmosphere.
D) Evidently, recycling bioplastics into fuel could A) I B) II C) III D) IV E) V
reduce concerns about the use of food crops in
biofuels production.
E) There is a radical shift in the industry from 54. (I) The magnificent Philippine eagle is one of the
petrochemicals to bio-based renewable planet’s most endangered birds of prey. (II) There is
polymers. no competition for prey from tigers, leopards, bears,
or wolves in the Philippine islands, the eagle’s only
home, so it became the top predator in the rain forest.
51. (III) However, with deforestation rates in the
Anthony: - Our research shows that the continents Philippines among the highest in the world, the eagle
contain rocks up to 4 billion years old. has been reduced to a population estimated at
Simon: - That is amazing, isn’t it? several hundred breeding pairs. (IV) The forest that
Anthony: - ---- allowed them to prosper is almost gone, and if the
forest disappears, the eagle will become extinct. (V) A
A) To the contrary. We have found evidence that series of devastating floods and mud slides in the
supports the theory of plate tectonics, which past decade has convinced Filipinos that the loss of
explains how plates converge and disperse. forest affects not just wildlife, but people too.
B) According to the theory of plate tectonics, the
Earth has a rigid outer layer known as the A) I B) II C) III D) IV E) V
lithosphere.
C) Indeed. It’s really dazzling to think of how the
landmasses we see around us today were 55. (I) The global climate is changing as it always has;
formed billions of years ago. species will go extinct as they always have; other
D) As most scientists suggest, over millions of years, species will prosper. (II) Humans may be influencing
mountains rise where plates collide, and oceans the change, but they didn’t create the change. (III)
form where plates diverge. Global warming is as dynamic as many other earthly
E) In fact, it is generally accepted that almost all of processes. (IV) Of course it is important to monitor
the oceanic floor is less than 180 million years which changes are being influenced by humans, but it
old. must be remembered that change is natural, and
Earth will continue to change no matter what humans
may or may not do. (V) Humans as a species must
accept this, and figure out how best to adapt to the
change, not how to control it.

A) I B) II C) III D) IV E) V

198
56. (I) The fate of the polar-region ice sheets will 59. It is clear from the passage that the mass and
determine how much the sea level rises in the coming size of dark matter ----.
century. (II) Under the frozen surfaces of Himalayan
glaciers on the flanks of Mount Everest and its fellow A) have been measured through the use of a
giant peaks, caves wind through the ice. (III) They threedimensional map of space
follow twisting paths carved out by flowing meltwater, B) have ceased to be one of the greatest mysteries
with unusual underground formations and narrow ever known in science
passages that open into huge galleries. (IV) The way C) distort the fabric of space and, therefore, cannot
meltwater moves inside glaciers is poorly understood, be explored properly
so scientists are going beneath the surface to track D) are so immense that it is out of question to study
how water eats away at glaciers from the inside. (V) them in detail
What we see on the surface is just part of the story. E) can only be understood through the curves made
by the light from galaxies
A) I B) II C) III D) IV E) V

60. One can maintain that the passage ----.


57. – 60. soruları aşağıdaki parçaya göre
cevaplayınız. A) focuses solely on the process whereby Einstein
was able to locate dark matter
Dark matter is the invisible and mysterious material B) deals with the nature of dark matter and how its
that makes up 22 per cent of the stuff in the universe. presence has been revealed
It is one of the greatest scientific unknowns. It does C) clearly explains where in space dark matter can
not emit light; nor does it reflect light or absorb it. be located and observed
While we are unable to see dark matter itself, we are D) sheds a great deal of light on the amount of
able to create maps of it. We can clearly pinpoint its research astronomers have done
location by observing the effects of its mass on light E) largely dwells on the question of how light from
from distant galaxies. This can be explained with galaxies becomes curved in space
reference to Einstein, who points out that a massive
object will curve the fabric of space and that light will
follow this deformed path. So we can look at how light
from galaxies has been bent and, consequently, infer
the quantity and location of the matter that did the
bending. In fact, by using this method, a team of
astronomers have recently managed to create the first
three-dimensional map of the immense structure of
dark matter.

57. One understands from the passage that dark


matter ----.

A) has a very complex structure that has caused


much controversy among astronomers
B) has been thoroughly explored and studied by a
number of astronomers
C) accounts for more than half of the material that
makes up the universe
D) has a bending effect on the light that comes from
distant galaxies
E) with its great mass was already known by
Einstein and a team of astronomers

58. It is pointed out in the passage that the


mapping of dark matter ----.

A) has only been possible on the basis of a theory


formulated by Einstein
B) has been an easy task for astronomers, since
they know its exact location
C) was originally suggested by Einstein, but it is only
now that this has been achieved
D) has revealed a much closer and more extensive
interaction among galaxies
E) was first attempted by Einstein, who had already
studied the light emitted by galaxies

199
61. – 64. soruları aşağıdaki parçaya göre 64. In the passage, the writer ----.
cevaplayınız.
A) clearly explains the adverse effects that coalfired
Using coal to make electricity accounts for about a power plants have in America
third of America’s carbon emissions. As a result, B) criticizes the United States government for not
tackling emissions from coal-fired power plants following a clear energy policy
represents our best opportunity to make sharp C) is fully in favour of the use of coal, as it is a
reductions in greenhouse gases. Fortunately, the readily-available and cheap energy source
United States already has the technology to do that. D) calls for the development of new technologies for
Unfortunately, right now the country is addicted to the reduction of carbon emissions
coal, a cheap, abundant power source. Burning coal E) is worried about how gigantic volumes of
produces more than half the country’s electricity, powerplant waste can be efficiently treated
despite its immense human and environmental costs.
Air pollutants from coal-fired power plants cause
somewhere between 20,000 and 30,000 premature
deaths in the United States each year. Besides, fifty
tons of mercury are pumped into the atmosphere
annually from coal plants. In addition, the extraction of
coal, from West Virginia to Wyoming, devastates the
physical environment, and its processing and burning
produce gigantic volumes of waste.

61. It is stated in the passage that coal-fired power


plants in the United States ----.

A) are noted for lower carbon emissions than other


kinds of power plants
B) are concentrated in West Virginia and Wyoming
more than in any other area
C) produce a great amount of the country’s
electricity
D) are blamed more for mercury emissions than for
carbon emissions
E) have caused widespread environmental
destruction in West Virginia and Wyoming

62. Emphasis is put on the fact that the United


States ----.

A) has the ability to drastically decrease greenhouse


gases in the country
B) has developed efficient technologies for the
prevention of environmental pollution
C) is the only country in the world that largely
depends on coal for its energy needs
D) has made great technological advances in
processing large amounts of waste
E) is seriously concerned about the human and
environmental effects of its energy policy

63. As one learns from the passage, coal ----.

A) has always been used as a primary source of


energy, but new technologies are needed to
extract it more cheaply
B) is so abundant in America that more and more
coal-fired power plants are being constructed
throughout the country
C) is extracted in gigantic amounts in West Virginia
and Wyoming, since these two states have the
richest reserves in the country
D) is so indispensable for the production of
electricity that nobody is concerned about its
adverse effects on the environment
E) is a major power source in America, although it
has various human and environmental
disadvantages

200
65. – 68. soruları aşağıdaki parçaya göre 68. In the passage, attention is drawn to the fact
cevaplayınız. that the theory of the Higgs field ----.

According to the most accurate scientific theory ever A) has been used as the standard model for an
created and generally known as the standard model, explanation of magnetic fields
all of space is filled with a mysterious stuff called “the B) is absolutely reliable and sheds light on how the
Higgs field”. Unlike magnetic or gravitational fields, universe was formed
which vary from place to place (as, for instance, the C) helps us understand how to avoid the dangerous
fact that things weigh more on Earth than on the effects of solar rays
surface of the Moon), the Higgs field is exactly the D) constitutes the basis of nuclear physics, since it is
same everywhere. What varies is how the different concerned with nuclear elements
fundamental particles interact with it. That interaction, E) is indispensable for an understanding of the
the theory goes, is what gives particles mass. In other Moon’s gravity and its effects
words, the Higgs field is what makes some particles,
such as protons and neutrons, relatively heavy, others
(like electrons) subatomic lightweights, and still others
(like photons) utterly massless. If photons weren’t so
light, a person would be shredded by a photon
hailstorm every time he or she was exposed to a
sunbeam. Then again, if protons and neutrons weren’t
so heavy, one wouldn’t dare to go outside to sunbathe
anyway. So without mass and its affinity for gravity,
there would be no galaxies, no stars, and no us.

65. One learns from the passage that, in magnetic


or gravitational fields, ----.

A) photons have an equal mass to that of protons


and neutrons
B) things do not interact at all and are therefore
massless
C) the weight of things is never the same, but
changes according to location
D) there are still many mysteries that need to be
explained accurately
E) it is not clear how different fundamental particles
interact with each other

66. It is emphasized in the passage that mass ----.

A) is the weight of a thing and is wholly dependent


on gravity
B) is fundamentally different from weight and the
two terms should not be confused
C) can only be observed in magnetic and
gravitational fields
D) shows no difference on Earth and on the surface
of the Moon
E) is essentially a function of how particles interact
with the Higgs field

67. As is pointed out in the passage, the Higgs


field ----.

A) covers space completely and is of a


homogeneous nature
B) has the same characteristics as a magnetic or
gravitational field
C) has been known for centuries and led to the
theory of gravity
D) has had no impact on the formation of galaxies
and stars
E) is only related to the interactions of photons,
protons, and neutrons

201
69. – 72. soruları aşağıdaki parçaya göre 71. It is explained in the passage that a hurricane
cevaplayınız. ----.

Hurricanes, which are circular storms spinning around A) can only be destructive so long as the surface
a region of low atmospheric pressure, are powered by temperatures of tropical oceans continue to rise
energy released by spiralling surface winds that draw steadily
heat from the ocean. Warmer seas provide more B) releases its energy when the temperature of the
energy and make hurricanes stronger. This is what sea surface increases markedly and causes
happened during Hurricane Katrina in August 2005, spiralling winds
which submerged New Orleans and the vicinity. In C) develops from spiralling surface winds that,
fact, according to climate scientists, both the intensity according to climate scientists, mostly happen in
and destructiveness of hurricanes have increased tropical regions
markedly since the 1970s. In other words, the energy D) is a storm that has a circular pattern and moves
released by an average hurricane appears to have quickly around an area of low atmospheric
increased by about 70 per cent within the past 30 pressure
years. This increase correlates very closely with rises E) derives its energy from the oceans, whose
in sea surface temperatures. Furthermore, tropical surface temperature has changed very little over
oceans have warmed about one degree Fahrenheit in the past 50 years
the past 50 years, a rise that is believed to be chiefly
the result of global warming.
72. One of the points emphasized in the passage
69. One understands from the passage that is that, over the last 30 years, ----.
Hurricane Katrina was obviously extremely
destructive because ----. A) the average hurricane’s energy has risen by
almost three-quarters
A) no measures had been taken over the last 30 B) Katrina has been the only destructive hurricane in
years to protect New Orleans and its the New Orleans area
surroundings C) climate scientists have made no progress in
B) New Orleans, situated so close to the ocean, has understanding hurricanes
always had adverse effects on global warming D) there has been a marked decrease in the
C) at the time, global warming affected the New strength of spiralling surface winds
Orleans area more than anywhere else E) a great deal of research has been done regarding
D) the increasingly warm ocean must have provided the causes of global warming
it with an unusual amount of energy
E) climate scientists did not believe that such a
hurricane could happen in the New Orleans area

70. It is suggested in the passage that global


warming ----.

A) has always been considered to be the single


most important cause of hurricanes throughout
the world
B) has been a major topic of research among
climate scientists over the past 50 years or so
C) is most intense in regions where atmospheric
pressure is very low and sea surface
temperatures very high
D) was first recognized by climate scientists in the
1970s and has always been correlated with
hurricanes
E) may have played a major role over the years in
the rise of temperature in the oceans in tropical
regions

202
73. – 76. soruları aşağıdaki parçaya göre 76. The passage gives us ----.
cevaplayınız.
A) a biological and ecological description of rivers
Rivers and streams generally support communities of and streams
organisms quite different from those of lakes and B) a warning about the dangers of pollution in rivers
ponds. A river or stream changes greatly between its and streams
source and the point at which it empties into a lake or C) a full picture of the kinds of organisms found in
the sea. Near the source, a stream’s water is usually rivers and lakes
cold, low in nutrients, and clear. The channel is often D) an insight into adverse environmental effects on
narrow, with a swift current that does not allow much rivers and streams
silt to accumulate on the bottom. Most of the E) an account of how water animals, including
organisms found here are supported by the worms and insects, feed
photosynthesis of algae attached to rocks or by
organic material, such as leaves, carried into the
stream from the surrounding land. Downstream, a
river or stream generally widens and slows. The water
is usually warmer and may be cloudier because of
sediments and other particles suspended in it. Worms
and insects that burrow into the mud are abundant, as
are waterfowl, frogs, fish, and other water animals.

73. It is maintained in the passage that, near the


end of its course, a river or stream ----.

A) flows so fast that no sediment or silt accumulates


on the bottom
B) becomes so polluted that no use can be made of
its water
C) shows a significant ecological and physical
difference from its source
D) is usually rich in algae indispensable for the
survival of various water animals
E) loses much of its nutrient capacity and turns into
a muddy waterway

74. A point made in the passage is that the


organisms found in rivers and streams ----.

A) live in the silt that is formed by the accumulation


of various kinds of organic material
B) are completely consumed by waterfowl, frogs,
and fish
C) are more abundant nearer to the source
D) depend on worms and insects for their food
rather than on algae and other nutrients
E) are quite unlike the ones found in lakes and
ponds

75. As is clear from the passage, there is a sharp


contrast between ----.

A) the variety of waterfowl upstream and


downstream
B) rivers and lakes as regards the kind of water
plants found in them
C) the kinds of organic material found upstream and
downstream
D) the quality of the water at a river’s source and
downstream
E) the types of organisms found in different rivers
and streams

203
77. – 80. soruları aşağıdaki parçaya göre 80. It is stressed in the passage that, over the last
cevaplayınız. two centuries, ----.

People have been pushing into forestlands for A) the demand in the world for lumber and paper
thousands of years, but during the last century, has reached alarming levels
scientists say, the rate of global forest reduction has B) many lost species of trees have been recovered
reached alarming levels. About 50 million acres of in Europe’s forestlands
forest are cleared every year. Much of Europe’s C) no pesticides and chemicals have been allowed
original forests are gone. The forests of North in North America’s forests
America, which once dominated the landscape, have D) nearly half of North America’s forestlands have
shrunk by almost 40% in the last two centuries to been cleared for various reasons
make room for people and meet the demand for E) the area of natural forests has been sustained
lumber and paper. Not only have many of the animals throughout the world
that depend on these ecosystems disappeared, but
various species of trees have also been depleted.
Timber farms on land that once sustained natural TEST BĐTTĐ.
forests have little of the biodiversity of the original CEVAPLARINIZI KONTROL EDĐNĐZ.
forests, with pesticides and other chemicals allowing
the land to support only a few kinds of life.

77. The passage as a whole deals with ----.

A) the process of deforestation in Europe taking


place over thousands of years
B) the steps taken for the restoration of lost
forestlands in North America
C) the biodiversity that the lost forests of Europe
once had
D) the alarming question of deforestation and its
harmful impact on ecosystems
E) the uses of pesticides and chemicals for the
protection of forestlands

78. It is claimed in the passage that forest farms


planted for timber ----.

A) can be most useful in recovering the lost


forestlands in Europe
B) lack the extensive biodiversity found in natural
forests
C) have a beneficial impact on the environment
because of the variety of trees there
D) have become widespread in North America and
are preferable to natural forests
E) have increased throughout the world over the last
two centuries

79. According to the passage, it is over the last


century that ----.

A) deforestation in the world has become


dangerously extensive
B) scientists have become aware of the variety of
ecosystems in North America
C) the paper industry in America has become
dependent on forest farms
D) solutions for the conservation of natural
forestlands have gone into effect
E) the reduction of natural forests has been brought
under control

204
1. – 18. sorularda, cümlede boş bırakılan yerlere 9. When Germany ---- Yugoslavia 1941, Bosnia
uygun düşen sözcük ya da ifadeyi bulunuz. and Herzegovina ----- part of the Nazi-controlled
Croatia.
1. In the Philippines, with its numerous scandals
and continuing power struggle, the public is A) has invaded / had been made
frustrated, and economic ---- is in jeopardy. B) had invaded / would have been made
C) invaded / were made
A) progress B) decline C) recession D) invades / have been made
D) depression E) failure E) was to invade / could have been made

2. Both Saudi Arabia and China know that it is in 10. If she ---- more pessimistic, most probably she
their ---- interest to cooperate economically and ---- up, but instead, she went on trying.
increase trade between them.
A) were / had given
A) notable B) precious C) mutual B) has been / has given
D) responsible E) implicit C) had been / may have given
D) is / will give
E) were / would have given
3. Whereas in the past, some Asian countries
were driven ---- by ideology, today, economics has
become their driving force. 11. There ---- no doubt that the oil sector ---- the
basis of the Iraqi economy for many years to
A) respectively B) unequally C) likely come.
D) entirely E) favourably
A) has been / may have formed
B) is / will form
4. As the automotive sector continues to ---- its C) was / has formed
operations, it creates even more jobs and D) had been / formed
opportunities in the communities where it does E) will be / would form
business.

A) settle B) extract C) expand 12. ---- an average of five credit cards per person,
D) contract E) reduce the people in the United States buy more ---- credit
than the people in any other country in the world.

5. According to the World Bank’s latest figures A) By / from B) For / of C) To / with


from 2005, of 196 countries around the world, 131 D) With / on E) At / by
---- food imports to feed their citizens.

A) keep up B) rely on C) drop out 13. The Bush Administration’s restoration of


D) bring about E) run out diplomatic relations with Libya ---- more than a
quarter of a century of often violent confrontation
is largely the result of several years ----
6. Few people know that many research projects painstaking and mostly secret diplomacy.
are ---- solely by the EU.
A) for / by B) in / over
A) settled down B) put up with C) written up C) after / of D) through / through
D) set out E) carried out E) at / with

7. Since 1985, Bolivia ---- economic changes that 14. Perfectionism constricts people just when the
---- phenomenally successful. fast-moving world requires more flexibility, and
----, it turns them into success slaves.
A) implemented / had been
B) has implemented / have been A) consequently B) nevertheless C) still
C) was implementing / were D) however E) contrarily
D) implements / are
E) had implemented / will be
15. ---- Americans are generally marrying less,
they still marry, divorce and especially remarry at
8. For a long time, psychologists ---- that major rates higher than in most other countries.
personality makeovers ---- impossible.
A) As B) When C) Since
A) have believed / are D) Just as E) Even though
B) believe / are going to be
C) believed / will be
D) would have believed / have been
E) had believed / had been

205
16. ---- English contains a vast lexical inheritance 24. – 35. sorularda, verilen cümleyi uygun şekilde
from Latin, Persian has absorbed a great number tamamlayan ifadeyi bulunuz.
of Arabic loanwords.
24. ----, which are cheap but durable.
A) As long as B) Because C) Even if
D) Just as E) Whether A) In Africa, China has found a ready market for its
manufactured goods
B) Most Chinese products are ideal for African
17. Some 30 women are running in the Kuwaiti conditions
general election, ---- few, if any, are expected to C) Some African states are still trying to recover
win. from internal conflicts
D) Africa can learn far more from China than it can
A) so that B) in that C) whether from the developed nations
D) because E) although E) The US has openly criticized China for selling
arms to Africa’s undemocratic regimes
18. Studying the origin of language was once
thought to be ---- an endeavour ---- scientific 25. ---- that the US is trying hard to increase the
societies actually forbade it. share of its oil imports from sub-Saharan Africa.
A) rather / than B) such / that C) not / but A) There is a great deal of controversy
D) either / or E) both / and B) It will be interesting
C) It has been revealed in a recent report
D) There has been much debate
19. – 23. sorularda, aşağıdaki parçada E) Most African states have been approached
numaralanmış yerlere uygun düşen sözcük ya da
ifadeyi bulunuz.
26. ----, but much is being done to improve
The first half of this interesting book provides an facilities.
introduction to the theory and practice (19) ---- logic.
The authors provide a historical (20) ---- of models of A) The Moroccan government has invested in the
logic from Aristotle to choice theory and “fuzzy logic.” expansion of some airports
They show (21) ---- the use the principles to analyze B) Morocco needs to reduce its reliance on foreign
and construct logical arguments. (22) ---- they give aid
some medical examples, this section is generic and C) The main obstacle in Morocco at present to
thus, this book (23) ---- well as a text for an further growth in mass tourism is the lack of
introductory course in a philosophy curriculum. Good infrastructure
illustrations and boxed definitions help to guide the D) Agadir is Morocco’s most developed coastal
reader, too. resort
E) The World Bank has agreed to provide Morocco
19. new funds
A) of B) in C) at
D) under E) within
27. ----, who were later joined by Germanic
peoples.
20.
A) quotation B) controversy C) omission A) Prague, one of Europe’s most beautiful capital
D) overview E) result cities, has a population of just over one million
B) Today we all know that the Czech Republic is
one of Europe’s youngest nations
21.ĐPTAL C) In 1993 Czechoslovakia was peacefully divided
A) what B) whether C) whom into two independent states as the Czech
D) which E) how Republic and Slovakia
D) The Czech Republic has produced writers,
artists, and musicians of world renown
22. E) Around 500 B.C. the area now known as the
A) Just as B) Although C) Since Czech Republic was settled by Celtic tribes
D) After E) Because

23.
A) will have served B) has served
C) had served D) would serve
E) may have served

206
28. ----, although the sculptural decorations were 33. A report issued by the International Energy
not completed until 1795. Agency warned that oil prices could be volatile in
2009, ----.
A) The Brandenburg Gate has witnessed many
historical events A) even though Venezuela had suspended trade
B) Berlin’s Brandenburg Gate was designed by Carl agreements with several countries
Gotthard Langhans B) as several of America’s largest banks had
C) The Brandenburg Gate is crowned by the famous announced high profits
sculpture of a chariot drawn by four horses C) just as the inflation rate in the EU economic zone
D) The Brandenburg Gate in Berlin was erected has risen well under 1%
between 1788 and 1791 D) so that Europe’s finance ministers came closer to
E) The Brandenburg Gate is the most striking agreeing radical reforms in banking
symbol of the city of Berlin E) because production and refining capacity are not
enough to meet world energy needs

29. ---- when it was conquered by Scipio


Africanus. 34. The more fundamental question is ----.

A) Spain, originally inhabited by Celts, became a A) because the government has put into effect new
part of the Roman Empire in 206 B.C. policies in agriculture
B) From the 12th to the 15th century, Aragon and B) that most EU countries are in favour of radical
Castile were the only Spanish states reforms in the economy
C) Off Spain’s east coast in the Mediterranean are C) whether such a biased foreign policy can serve
the Balearic Islands the interests of the country
D) In 711, the Muslims under Tariq entered Spain D) in case there may be a call from the government
from Africa for an early election
E) Today Spain occupies 85% of the Iberian E) while there have been introduced new plans to
Peninsula, which it shares with Portugal change working conditions

30. After World War I, the League of Nations gave 35. One of the many criticisms made of the British
France a mandate over Syria, ----. media is ----.

A) as it formed in 1958, together with Egypt, the A) why Britain has been the final destination for
United Arab Republic illegal immigrants from the Middle East
B) since France recognized it in 1930 as an B) how it can best engage its own audience in public
independent republic affairs and international problems
C) even though nationalist demonstrations in 1945 C) whether the politicians opposed to the
broke into actual fighting government’s monetary policy are right
D) but the French were soon opposed by several D) that it is not interested in the everyday interests
nationalist uprisings of its own listeners and readers
E) whereby it became independent on 29 E) because it has mostly been indifferent to the
September 1961, following a revolution problem of rising unemployment

31. Finding less expensive accommodation in big


cities can be difficult, ----.

A) while most cities have several international chain


hotels
B) unless one stays outside the city centre
C) whereas high prices generally apply to hotels in
historic buildings
D) if a room in a pension or private home is usually
more reasonable
E) since most hotels prefer to create a family
atmosphere

32. Present-day Austria emerged in 1919, ----.

A) when the lands of its former Habsburg Empire


were granted independence
B) since it was bordered by Switzerland and
Germany to the west and north
C) because it has existed as a country for less than
100 years
D) while it has thrived thanks to its position at the
heart of Europe
E) even though visitors are attracted by the glories
of its imperial past
207
36. – 38. sorularda, verilen Đngilizce cümleye 39. – 41. sorularda, verilen Türkçe cümleye
anlamca en yakın Türkçe cümleyi bulunuz. anlamca en yakın Đngilizce cümleyi bulunuz.

36. An effective work group should be designed 39. Geçmişin ne anlama geldiğini
well from the start, bringing together people who sorguladığımızda, şüphesiz farklı insanlara farklı
can contribute to the right mix of knowledge, şeyler ifade eder.
skills, tools and other resources necessary to
succeed. A) If we ask what the past may mean, it is clear that,
for different people, it means different things.
A) Đşin başında tasarlanmış etkin bir çalışma grubu, B) When we question the meaning of the past,
bilgi, beceri, gereç ve diğer kaynaklar konusunda undoubtedly different people interpret it
katkıda bulunabilen insanları bir araya differently.
getirmektedir. C) Since the past means different things to different
B) Etkin bir çalışma grubunun bir araya getirilmesi people, we must question what the meaning is.
için, bilgi, beceri, gereç ve diğer kaynakların D) When we ask what the past means, it certainly
doğru biçimde kaynaştırıldığı süreçlere katkıda means different things to different people.
bulunabilecek kişilerin daha baştan planlanması E) As the meaning of the past varies from people to
gerekir. people, clearly we must ask how these meanings
C) Bilgiye, beceriye, gereçlere ve başarı için gerekli are made.
diğer kaynaklara katkıda bulunarak etkin bir
çalışma grubu oluşturacak kişiler, henüz işin
başındayken doğru biçimde bir araya 40. Suudi Arabistan kralı Abdullah’ın Çin’e
getirilmelidir. seyahati, iki ülke arasında diplomatik ilişkilerin
D) Etkili bir çalışma grubunun, başarılı olmak için 1990’da kurulmasından beri, bir Suudi kral
bilgiye, becerilere, gereçlere ve diğer kaynaklara tarafından yapılan ilk seyahatti.
katkıda bulunabilecek insanları bir araya
getirerek tasarlanması gerekir. A) King Abdullah of Saudi Arabia’s trip to China was
E) Etkin bir çalışma grubu, başarmak için gerekli the first by a Saudi king since the diplomatic
bilginin, becerilerin, gereçlerin ve diğer relations were established in 1990 between the
kaynakların doğru bileşimine katkıda bulunabilen two countries.
kişileri bir araya getirerek baştan iyi bir biçimde B) The trip which King Abdullah of Saudi Arabia
tasarlanmalıdır. made to China in 1990 was the first made by a
Saudi king since the establishment of diplomatic
relations between the two countries.
37. Bad things will come and find you, but for the C) When King Abdullah of Saudi Arabia travelled to
good things you have to keep the door open, go China in 1990, it was the first trip ever made by a
hunt for them and find them. Saudi king since the establishment of diplomatic
relations between the two countries.
A) Kötülükler gelip seni bulacaktır, ancak iyilikler için D) Following the establishment in 1990 of diplomatic
kapıyı açık tutman, onları araman ve bulman relations, King Abdullah of Saudi Arabia’s trip to
gerekir. China was the first by a Saudi king.
B) Gelip seni bulanlar kötü şeylerdir ama senin E) After the diplomatic relations between China and
kapıyı açıp izlediğin ve bulduğun şeyler iyi Saudi Arabia were established in 1990, King
şeylerdir. Abdullah made his first trip to this country.
C) Kötülükler gelir seni bulur, ama iyi şeyler için
kapıyı açık tut, onları ara ve bul.
D) Kötü olaylar gelir seni bulur, ama iyi olaylarla
karşılaşmak istiyorsan, kapıyı açık tutmalı, onları
arayıp bulmalısın.
E) Kötü şeyler gelip seni bulsa da iyi şeyleri arayıp
bulmalısın ve kapını her zaman açık tutmalısın.

38. Couples, who repeatedly break up and then


make up, often find themselves caught between
their desire for freedom and their fear of regret.

A) Devamlı ayrılıp barışan çiftler, bağımsız olma


isteği ile pişman olma korkusu arasında kalırlar.
B) Her zaman ayrılıp barışan çiftler, genellikle
kendilerini bağımsız olma istekleri ile pişman
olma korkuları arasında bulurlar.
C) Tekrar tekrar ayrılıp barışan çiftler, çoğu kez
kendilerini bağımsız olma istekleri ile pişman
olma korkuları arasında kalmış bulurlar.
D) Bağımsız olma isteği ile pişman olma korkusu
arasında kalan çiftler, tekrar tekrar ayrılıp
barışırlar.
E) Bağımsız olma isteği ile ayrılan çiftler, genelde
pişman olma korkusu ile barışırlar.

208
41. Đsviçre yatırım bankası UBS, çoğunlukla Asya 43. Until recently, Sudan’s Nubian history
ve Avrupa üzerinden kanalize edilen petrol remained largely unknown. ----. They have
dolarlarının, ABD carî açığının beşte ikisini recognized that these pharaohs came from a
finanse ettiğini tahmin etmektedir. strong African civilization that had flourished on
the southern banks of the Nile for 2,500 years,
A) UBS, the Swiss investment bank, claims that going back at least as far as the first Egyptian
generally Asia and Europe channel petrodollars dynasty. The most important remains of this
in order to pay for two fifths of the US current civilization are the pyramids in the Nubian Desert,
deficit. which are greater in number than those in Egypt.
B) According to the estimation of UBS, the Swiss
investment bank, two fifths of the US current A) Human figures in Egyptian, Greek, and Roman
deficit are mostly funded by the petrodollars art indicate a clear awareness of racial features
channelled through Asian and European banks. B) Only in the past four decades have
C) The Swiss investment bank UBS has announced archaeologists uncovered the story of the
that Asian and European banks are chanelling socalled ‘‘black’’ pharaohs
most of petrodollars in order to fund two fifths of C) Sudan’s pyramids are extremely interesting
the US current deficit. structures and have not been properly studied
D) As UBS, the Swiss investment bank, has stated, D) Therefore, The Sudanese government has
the petrodollars, generally channelled through decided to build a dam on the Nile, 970 kms
Asia and Europe, are being used to pay for two upstream from Egypt’s Aswan High Dam
fifths of the US current deficit. E) The Nubian Desert, which is vast and extremely
E) UBS, the Swiss investment bank, estimates that uninhabitable, needs to be fully explored
petrodollars, mostly channelled through Asia and
Europe, are funding two fifths of the US current
deficit. 44. The United Arab Emirates’ (UAE’s) poultry
sector has lost $5.5 million because of a 50% cut
in consumption due to bird flu fears. Many small
42. – 46. sorularda, boş bırakılan yere, parçada farms have been forced to close, and production
anlam bütünlüğünü sağlamak için getirilebilecek has been cut by 30%, with surplus meat being
cümleyi bulunuz. frozen and sold off cheaply. The consumption of
eggs has not been affected. ----.
42. For many Central American immigrants
heading north, the first dangerous border A) Nevertheless, the country claims to be free of the
crossing is not the one into the US. It’s southern disease
Mexico where the danger begins. Every year, B) Moreover, Kuwait and Saudi Arabia have
hundreds of thousands of Central Americans announced cases of bird flu on their territories
cross illegally into Mexico along the country’s C) In fact, the UAE’s oil income has doubled over
southern border. ----. The pace of illegal migration the last few years due to a sharp rise in its
north has increased dramatically over the past exports
decade, propelled in part by the lingering effects D) Among the Gulf states, it is the UAE that most
of the 1970s and 80s civil wars in Guatemala, El aspires to become a major global investor
Salvador, and Nicaragua. E) However, the UAE is facing big increases in the
price of consumer goods and transport
A) The US wants Mexico’s southern border to more equipment imported from Europe
secure, even as American employers demand
cheap labour and American drug users demand
smuggled cocaine 45. Jordan’s 1994 peace deal with Israel has
B) Guatemalan border towns are growing quickly as paved the way for growing economic links
they cater to the growing number of migrants between the two countries. Jordan is now taking
north to Mexico the lead in encouraging other Arab states to
C) Nobody knows exactly how many of those improve relations with Israel. -----. He has argued
migrants are headed to the United States, but that future peace initiatives should come out of
most experts estimate the figure to be 150,000 or the region itself, rather than being designed and
more a year imposed by foreign authorities.
D) The simplicity with which people and goods pass
illegally across Mexico’s southern border is A) The US permits all Jordanian textiles to be
obvious to anyone who spends time there exported to the US as long as they have a
E) The US government has already put much minimum 8% Israeli content
pressure on Mexico so that illegal migration can B) According to some diplomats, economic
be stopped completely cooperation between Jordan and Israel is rather
more advanced than political cooperation
C) King Abdullah of Jordan has also raised hopes
that the Saudi Middle East peace plan of 2002
might be revived by persuading the Arab League
to make the plan more attractive to Israel
D) Actually, the two countries signed an economic
agreement in 2002 in order to establish joints
ventures in all the sectors
E) Jordan has made relatively rapid economic
progress during the five years since King
Abdullah ascended to the throne
209
46. This year, the map of international adoptions 48.
will be redrawn. The US, which adopts the most Sarah: - The book you are reading is about Brazil,
children from abroad, will become a full isn’t it? I know a little bit about this country’s
participant in the Hague Convention on economy.
Intercountry Adoption. The treaty regulates Henry: - It is, indeed. It says, since 1930,
adoption among the 74 members and helps successive governments have persistently
ensure that adoption agencies comply with pursued policies for economic growth and
convention rules, which call for counselling for Amazon area development.
adoptive parents and ban child buying. ----. Sarah: - ----
Although it is a Hague signatory, the country has Henry: - That explains partly why by the 1990s
been accused of child trafficking. Brazil has become one of the world’s largest
economies.
A) Numbers of adoptions to the US from other
Hague participants are expected to rise A) Unfortunately, development has destroyed much
B) When the Cold War ended, Russia and former of the ecosystem that has characterized the
Soviet-bloc countries opened to adoption Amazon basin.
C) With a dropping birthrate, Russia now favours B) This is not surprising, since Brazil has vast
local families, as does China natural resources and a huge labour force.
D) Adoptive parents often embrace the culture of C) Geographically, the north eastern region of Brazil
their new child is semiarid scrubland, heavily settled and poor.
E) That could be a problem for Guatemala, a D) However, it is true that today more than one out
mainstay of US adoption for a decade of four Brazilians survive on less than $1 a day.
E) Well, Brazil’s narrow coastal belt includes most of
the major cities, of which Rio de Janeiro is the
47. – 51. sorularda, karşılıklı konuşmanın boş most famous one.
bırakılan kısmını tamamlayabilecek ifadeyi
bulunuz.
49.
47. Frank: - Have you ever heard about what is called
Robert: - I hear that, with regard to the current oil “landscape archaeology”?
crisis, the US Senate is planning to set up a Jane: - No, I haven’t. Can you tell me what it is?
subcommittee. Frank: - ----
James: - What do you think the subcommittee will Jane: - So, in other words, the view from above
be authorized to do? can yield archaeological insights on the ground.
Robert: - ----
James: - Clearly, this is a matter that the A) In fact, there are unstable areas in the Middle
subcommittee must look into as soon as possible. East, where landscape archaeology cannot be
practised.
A) As far as I am concerned, the role of the B) Among archaeologists, it is regarded as
multinational oil companies in the current crisis extremely useful for a comprehensive description
urgently needs to be investigated. of the excavations at an archaeological site.
B) It has been reported that new oil discoveries can C) Look, according to this article, it is the kind of
keep up pace with predicted growth in demand. archaeology that, through the use of satellite
C) In my opinion, no country can claim that oil imagery, reveals a landscape’s hidden
companies have not behaved in a responsible archaelogical details, such as buried roads and
manner. canal systems.
D) You know that the Middle East with its rich D) In the end, archaeologists have accepted the fact
reserves supplies most of the oil consumption in that landscape archaeology contributes a lot to
the world. our understanding of a site.
E) In fact, during the first half of this year, there was E) Historically, not until the invention of the airplane
a rising demand for all petroleum products. did the idea of aerial photography become
practical.

210
50. 53. (I) The mountain kingdom of Bhutan has decided
John: - What do you know about Bertolt Brecht? to build its national policies on improving its gross
Friend: - As I remember, he was a modern German national product. (II) The “Easterlin Paradox”, which
dramatist. states that a nation's happiness actually declines in
John: - ---- relation to its economic growth, was once thought to
Friend: - As far as I know, he also encouraged be limited to rich Western countries. (III) However,
audiences to develop an active and critical researchers have recently shown that it also applies
attitude to a play’s subject. to developing countries. (IV) For example, even as
China experienced extraordinary economic growth
A) In my opinion, as a genre, comedy has between 1990 and 2000, the percentage of Chinese
undergone many changes since classical who described themselves as very happy fell from 28
antiquity. per cent in 1990 to 12 per cent in 2000. (V) Experts
B) Moreover, drama is distinct from literature in that attribute this phenomenon to the fact that people’s
it is a performing art. desires and expectations change along with their
C) The drama that contains elements of tragedy and material fortunes.
comedy is called tragicomedy.
D) Right. More than that, he was one of the most A) I B) II C) III D) IV E) V
influential figures in the 20th-century theatre.
E) On the other hand, theatre history can be traced
back to the ancient Greek festivals for Dionysus. 54. (I) Britain’s character has been shaped by its
geographical position as an island. (II) Never
successfully invaded since 1066, its people have
51. developed their own distinctive traditions. (III)
George: - This summer, I spent my holiday in Founded by the Romans in the first century A.D. as
Spain. I was particularly impressed by Madrid. an administrative centre and trading port, London is
Mark: - I am sure you were. While there, you must the capital of the country. (IV) This is also reflected in
have found time enough to visit the Prado the country’s historical heritage of castles, cathedrals
Museum. and stately homes. (V) Indeed, although today a
George: - ---- member of the European Union, Britain continues to
Mark: - Originally, all these paintings, especially maintain its nonconformity.
the major works of Velásquez and Goya, were in
the royal collection. A) I B) II C) III D) IV E) V

A) Of course, certainly, because it houses the


world’s greatest collection of Spanish paintings 55. (I) “Each day is a journey, and the journey itself is
from the 12th to the 19th centuries. home”, the poet Matsuo Basho wrote more than 300
B) Actually, the origins of the city date back to A.D. years ago in the first entry of his masterpiece Narrow
852, when the Moors built a fortress near the Road to a Far Province. (II) Today, Basho is the first
Manzanares river. writer that many Japanese read seriously, and many
C) In fact, it was under the Habsburgs that the city people admire his poetry. (III) Thousands of people
acquired some of its most notable landmarks, make pilgrimages to his birthplace and burial shrine,
including its splendid palaces. and travel parts of Basho’s Trail. (IV) After three
D) Let me tell you that the museum was designed in centuries, his Narrow Road still speaks to readers
1719 by the royal architect Juan de Villanueva. around the world. (V) After his master died, Basho
E) In Spanish culture, the 18th century was an era of spent most of his life in Kyoto.
French influence, following the Bourbon
accession to the Spanish throne. A) I B) II C) III D) IV E) V

52. – 56. sorularda, cümleler sırasıyla 56. (I) France, Russia and the United States are the
okunduğunda parçanın anlam bütünlüğünü bozan world’s biggest exporters of small arms. (II) Last year
cümleyi bulunuz. they produced 75% of all small arms sold in the world.
(III) But several other countries, such as Egypt and
52. (I) Christine Lagarde, France’s first female Israel, are also emerging as major producers and
minister for finance and the economy, says it is time traders. (IV) Especially in the Middle East, the
for French people to “roll up their sleeves” and stop problem of arms smuggling is widespread. (V) For
thinking about holidays. (II) The former international instance, in 2006 Israel exported $175 million worth of
lawyer, impressed by the work ethic she witnessed small arms.
during her time in the US, wants to instill the same
spirit in her countrymen and women. (III) The French A) I B) II C) III D) IV E) V
government’s plans to reorganize the retail industry
have brought fears that greater competition will
threaten livelihoods. (IV) Her approach is calm and
rational, bearing little resemblance to the harsher
style of the French president, Nicolas Sarkozy. (V) Yet
Christine Lagarde shares the president’s convictions
when it comes to the French people and the world of
work.

A) I B) II C) III D) IV E) V

211
57. – 60. soruları aşağıdaki parçaya göre 60. According to the passage, the beginnings of
cevaplayınız. historical linguistics ----.

In 1786 Sir William Jones, a British judge serving in A) were marked by the discovery in the eighteenth
India, made a discovery that transformed knowledge century that the languages of the South Asian
about prehistory and began the formal study of subcontinent had evolved from Sanskrit
historical linguistics. Turning his spare time towards B) can be dated back to the early nineteenth century
the study of Sanskrit, the ancient language from when the term “Indo-European” was introduced
which the predominant languages of the South Asian C) are traced back to the work of Sir William Jones
subcontinent derive, Jones discovered that Sanskrit in the eighteenth century
shares features of grammar and vocabulary with Latin D) have always been controversial, since Sir William
and ancient Greek to an extent inexplicable by sheer Jones’s theory about the Indo-European
coincidence. His interest further aroused, he then languages was based on a false assumption
examined the early Germanic language called E) were confined only to Jones’s study of Sanskrit
“Gothic”, the ancient Celtic languages of Europe, and and did not include his study of the other Indo-
Old Persian, and found that they, too, exhibited European languages
marked similarities to Sanskrit. He concluded that all
these languages must have evolved from a common
but now-extinct linguistic source. In the early
nineteenth century, both this ancient language and
the later languages that derived from it, were labeled
“Indo-European,” reflecting their wide distribution from
India and Ireland.

57. One can conclude from the passage that


Gothic ----.

A) originally derived from Latin and had the same


grammatical features
B) had no relationship whatsoever with Sanskrit and
other languages
C) was an old language which was spoken
throughout Europe, including Ireland
D) was the only language in which Sir William Jones
took a special interest
E) was the language spoken by early Germanic
peoples

58. It is pointed out in the passage that Sanskrit


----.

A) is the only language which is widely spoken


among the peoples of South Asia
B) and other Indo-European languages are closely
related with each other
C) was first studied by Sir William Jones in the
eighteenth century
D) was the most ancient language from which Latin
and Greek have evolved
E) has a grammatical structure which is entirely
different from that of Old Persian

59. As one understands from the passage, Sir


William Jones ----.

A) made fundamental changes in the judicial system


of eighteenth-century India
B) was a professional linguist and spent his time in
India by the study of Sanskrit
C) rejected the view that the European Celtic
languages derived from a common source
D) studied Sanskrit only when he was free from his
judicial responsibilities
E) was very interested in the prehistory of India and
made several discoveries

212
61. – 64. soruları aşağıdaki parçaya göre 63. It is suggested in the passage that humans ----.
cevaplayınız.
A) in the Stone Age were very skilled and far
The Stone Age is, in fact, divided into various stages. advanced in making tools for their farming needs
Dominating the period is the Palaeolithic Age, which B) seem to have first developed their religious ideas
most anthropologists would extend down to roughly during the Upper Palaeolithic Era
11,000 B.C. Within the Stone Age in general, C) in the Upper Palaeolithic Era knew how to defend
however, scholars also speak of an Upper themselves when they were attacked by hunter-
Palaeolithic Era, beginning around 40,000 B.C. They gatherers
draw attention to some significant changes in human D) in the Palaeolithic Age were skilled not only in
behaviour around this date, including the appearance hunting but also in different kinds of cloth-making
of sophisticated cave paintings, and evidence of E) in the Stone Age had no notion of religion and,
religious ideas. Humans also began producing the therefore, did not know how to worship
most effective, finely crafted tools such as fishhooks,
arrowheads, and sewing needles made from organic
materials, such as wood or animal bone. Yet, despite 64. It is pointed out in the passage that, with the
these important developments, the basic patterns of Upper Palaeolithic Era beginning around 40,000
human life changed little during this era. Virtually all B.C., ----.
human societies before 11,000 B.C. consisted of
small bands of hunter-gatherers that moved A) most human societies worshipped various gods
incessantly in search of food. Because they could not B) cave painting became very popular among
stay in any one location for long, these groups left no hunter-gatherers
continuous archaeological record whereby we might C) fully organized human societies began to appear
trace the development of their culture. Our knowledge D) human beings ceased to be hunter-gatherers
of them is, therefore, very limited. E) man entered a new period of important
developments
61. It is stressed in the passage that, during the
Upper Palaeolithic Era, ----.

A) there were no settled human communities but


groups of wandering hunter-gatherers
B) humans practised different forms of art but
preferred to specialize in cave painting
C) hunter-gatherers had plenty of food whereby they
were able to feed themselves easily
D) bands of hunter-gatherers produced different
kinds of tools whereby the development of culture
can be traced
E) there appeared new and most developed
cultures, which have been brought to light
through archaeological excavations

62. According to the passage, the Stone Age ----.

A) was, according to archaeologists, the most


sophisticated and developed period of early
human history
B) and the cultures of the period have been fully
revealed by archaeologists
C) is generally known as the Upper Palaeolithic Era,
during which man learned how to produce food
D) is not one continuous period but consists of
different eras
E) is generally dated to 11,000 B.C. and has always
been a serious academic concern among
anthropologists

213
65. – 68. soruları aşağıdaki parçaya göre 68. It is stated in the passage that Hong Kong ----.
cevaplayınız.
A) was the major exporter of cheap products to the
By the end of the twentieth century, East Asia had United States and Europe
become a centre of industrial and manufacturing B) contributed enormously to the emergence of
production. Especially China began to establish China as an economic power in East Asia
commercial ties with the West in the 1970s and C) was not affected at all by the decline of prosperity
became the world’s leading heavy industrial producer and economic growth in the rest of China
by the year 2000. Its state-owned companies D) took immediate measures to prevent the looming
acquired contracts from Western firms to produce energy crisis in the area
products cheaply and in bulk, for sale back to home E) was a major centre of industrial and
markets in the United States and Europe. Moreover, manufacturing production in China in the 1970s
the Chinese government established semicapitalist
commercial zones around major port cities like
Shanghai. These commercial zones were intended to
encourage massive foreign investment on terms that
left China a favourable balance of trade for its huge
volume of cheap exports. Yet, in practice, they
enjoyed only mixed success. Problems in farming and
a looming energy crisis hampered prosperity and
economic growth, but Hong Kong only managed to
maintain its traditional economic and cultural ties with
the rest of the world. However, in recent years, China
has overcome most of these problems and radically
upgraded its economic performance.

65. The passage is mainly concerned with ----.

A) the extent and complexity of problems in Chinese


agriculture
B) the variety and volume of Western investments in
China in the 1970s
C) Hong Kong’s economic prosperity and its impact
on social life in the city
D) China’s economic development and performance
since the last century
E) the improvement of China’s balance of trade
since the 1970s

66. It is claimed in the passage that China’s


commercial zones ----.

A) absolutely provided China with a huge volume of


foreign investment
B) were not as successful as had been originally
intended
C) were solely located around Shanghai and Hong
Kong
D) have been the most efficient ones throughout
East Asia
E) were originally intended for state-owned
companies

67. It is pointed out in the passage that Western


firms ----.

A) made contracts with Chinese companies


because of favourable production conditions in
China
B) made huge investments in commercial zones, but
they failed to make any profit
C) established closer commercial ties with China to
benefit from cheap exports
D) were mainly concentrated in Hong Kong, since it
was a major port city
E) cooperated with China in order to upgrade its
economic performance

214
69. – 72. soruları aşağıdaki parçaya göre 72. According to the passage, although Europe
cevaplayınız. had achieved a balance of power by 1914, ----.

In 1914, Europe had built a seemingly stable peace. A) Germany, Austria and Italy continued their efforts
Through the complex negotiations of great power to build very large armies and began to challenge
geopolitics, Europe had settled into two systems of the Allied Powers
alliance: the Allied Powers which consisted of Britain, B) European leaders encouraged the development
France and Russia, and the Central Powers that of advanced technologies for a comprehensive
included Germany, Austria, and Italy. Within this victory
balance of power, the nations of Europe challenged C) this did not prevent the European nations from
one another for economic, military, and imperial challenging each other for various advantages
advantage. The rivalry for colonies abroad D) military leaders strongly argued that larger armies
accompanied a fierce arms race at home, where were needed in order to protect the colonies
military leaders assumed that superior technology and overseas
larger armies would result in a quick victory in a E) political leaders were extremely suspicious of
European war. Indeed, in the prevailing atmosphere military leaders and, therefore, did their best to
of international suspicion, such a war seemed likely to prevent a war
many of Europe’s political and military leaders. Yet
none of them predicted that the war would break out
so soon. Nor did many expect that the assassination
in June 1914 of the Austrian archduke and his wife
would spark off that war, which engulfed all of Europe
in just over a month’s time.

69. It is suggested in the passage that, when the


Austrian archduke and his wife were assassinated
in June 1914, ----.

A) the European balance of power was not affected


by this incident, and the European peace was
maintained
B) the British government immediately began
negotiations for an alliance with France and
Russia
C) Austria turned to Germany and Italy for an
alliance and declared war on Britain and France
D) most European leaders did not think that this
incident would soon lead to a war
E) many European leaders believed that this was a
plot of the Allied Powers against the Central
Powers

70. It is pointed out in the passage that, while the


European powers were competing with each other
for colonization overseas, ----.

A) their main concern was how to benefit from the


geopolitical importance of their colonies
B) they were also involved in an escalating process
of rearmament
C) they held the view that technological superiority
was essential for the exploitation of their colonies
D) they held various negotiations for the prevention
of a possible war that would destroy Europe
E) it seemed to many politicians that international
problems could be solved through negotiations

71. The writer asserts in the passage that the


peace in Europe in 1914 ----.

A) was essentially the achievement of political and


military leaders
B) was maintained efficiently by the Allied Powers
C) mainly depended on superior technology and
very strong armies
D) seemed a lasting one, but it was, in fact, fragile
E) was not affected at all by the assassination of the
Austrian archduke

215
73. – 76. soruları aşağıdaki parçaya göre 76. It is clear from the passage that Iran’s nuclear
cevaplayınız. technology ----.

Since 1993, China has invested in more than fifty oil A) has been aided by China through its logistical
and gas projects in some thirty nations. In particular, support
China has focused on acquisitions and partnerships in B) is not so advanced and efficient as that of China
Sudan and Iran. In Sudan alone, China has reportedly C) has been strongly criticized by the United States
spent $15 billion developing oil fields. In the D) has been financed through its oil exports to a
meantime, China has also begun to use its military to number of countries
protect its oil investments abroad. Reportedly, troops E) can develop fully even though China is not willing
disguised as oil workers patrol Chinese oil to cooperate
infrastructure in Sudan. Moreover, in recent years,
China has strengthened its military presence in the
oil-and gas-rich parts of the South China Sea, over
which sovereignty is still disputed. Perhaps most
significant in the short term is China’s relationship with
Iran. With Saudi Arabia and Iraq clearly within the
American sphere of influence, China has been
steadily courting Tehran and aims to become the
biggest buyer of Iranian oil. In return for oil, China has
supplied Iran not only with conventional weapons but
also with technology and materials that can be used
for the manufacturing of nuclear weapons.

73. It is claimed in the passage that the Chinese


oil workers in Sudan ----.

A) have increased steadily because China has


acquired many oil and gas fields in this country
B) are, in fact, military personnel in disguise,
employed to protect the Chinese oil investments
in this country
C) have been employed in over fifty oil and gas
projects, for which China has already spent
billions of dollars
D) have been extremely efficient and built the
extensive oil infrastructure that this country has
E) mostly prefer to work for the companies that
China has set up in this country for partnership in
oil and gas projects

74. One learns from the passage that there are ----.

A) many Chinese workers already employed in


nearly thirty nations
B) several Chinese companies involved in oil
projects in the South China Sea
C) a number of economic issues that China faces in
the South China Sea
D) many oil fields in Iran that have been developed
by China
E) areas in the South China Sea which are rich in oil
and gas

75. One understands from the passage that, just


as the United States maintains its dominant
position in Saudi Arabia and Iraq, so China ----.

A) has made efforts to develop its partnership with


Iranian oil companies
B) is resolved to increase its military presence in
some thirty countries
C) has sought to establish closer economic relations
with Iran
D) has decided to invest heavily in Iran’s various oil
and gas projects
E) tries hard to persuade Iran to become an ally in
the region

216
77. – 80. soruları aşağıdaki parçaya göre 79. As one understands from the passage, in
cevaplayınız. informal imperialism, ----.

The term “imperialism” means the process of A) the colonizing nations sign treaties with native
extending one nation’s control over another; it is a governments to help them solve economic
process that takes many forms. Historians distinguish problems
between “formal imperialism” and “informal B) the independence of the weaker nation is
imperialism.” Formal imperialism is colonialism, and it respected by the colonizing nation
was exercised by the Europeans in the past mainly by C) the sovereignty of the colonized nation is totally
direct rule: the colonizing nations annexed territories disregarded by the colonizing power
outright and established their own governments to D) the management of the ports in a country is
subjugate and administer the peoples of these undertaken by economically stronger nations
territories. Sometimes formal imperialism was E) native governments are granted certain privileges
exercised through indirect rule: the conquering and political powers by stronger nations
nations reached agreements with native leaders and
governed them. There was no single practice of
colonial management, and resistance from the natives 80. It is clear that the passage ----.
forced colonial powers to shift strategies frequently.
As for “informal imperialism,” it refers to a more subtle A) gives an account of the historical reasons why
and less visible exercise of power, in which the formal imperialism in the past was more widely
stronger nation allows the weaker one to maintain its practised than informal imperialism
independence while reducing its sovereignty. For the B) is a detailed account of how the Europeans
Europeans in the past, informal imperialism took the colonized other peoples in the 18th and 19th
form of carving out zones of European sovereignty centuries
and privilege, such as treaty ports, within other C) is mainly concerned with the process of
countries. Essentially it meant using European resistance that colonial peoples put up against
economic, political, and cultural power to get the European nations
advantageous treaties or terms of trade. Informal D) is a full description of the economic and cultural
imperialism was not only common, it played an even privileges which, in the past, the Europeans got
more fundamental role in shaping global power from native governments
relations in the 18th and 19th centuries. E) theoretically explains imperialism and refers to
the European practice of it in the past
77. It is asserted in the passage that, in the past,
informal imperialism ----.
TEST BĐTTĐ.
A) enabled the Europeans to conquer the lands of CEVAPLARINIZI KONTROL EDĐNĐZ.
other peoples and face no resistance
B) was preferred by the natives who were involved
in free trade with the European nations
C) was far more effective than formal imperialism in
the development of power relations in the world
D) was commonly practised by the Europeans
because it allowed them to make use of various
strategies
E) provided the European nations with an economic
power which enabled them to rule the rest of the
world

78. According to the passage, although the


practice of imperialism may be varied, ----.

A) native peoples in the European colonies were


always in favour of formal imperialism
B) it was formal imperialism in the past which most
suited the Europeans for their trade overseas
C) the European nations followed a common
strategy in the past in order to set up colonies in
other parts of the world
D) for historians, it is mainly divided into “formal” and
“informal” imperialism
E) in the 18th and 19th centuries the European
nations gave up formal imperialism because of
native resistance

217
1. – 18. sorularda, cümlede boş bırakılan yerlere 8. French authorities ---- trials of gene therapy
uygun düşen sözcük ya da ifadeyi bulunuz. after a boy ---- a disease similar to leukaemia.

1. Because saliva offers some natural protection A) had suspended / has contracted
against tooth ----, less saliva can lead to more B) suspend / may have contracted
cavities. C) suspended / contracts
D) will have suspended / had contracted
A) treatment B) surgery C) care E) have suspended / contracted
D) growth E) decay
9. Long ---- as a rat poison, arsenic ---- to halt a
2. The ---- thin walls of the alveoli allow oxygen to deadly blood cancer called acute promyelocytic
move from the alveoli into the blood in the leukaemia.
capillaries.
A) being used / can be shown
A) consciously B) adversely C) unexpectedly B) using / was shown
D) extremely E) alternatively C) used / has been shown
D) to be used / is shown
E) having used / had been shown
3. When healthy, the lining of the mouth (oral
mucosa) is reddish pink, and the gums, which fit
---- around the teeth, are paler pink. 10. Heatstroke is a life-threatening condition
which ---- from extreme exposure to heat, in which
A) protectively B) attractively C) rarely a person ---- enough to lower body temperature.
D) abruptly E) randomly
A) had resulted / does not sweat
B) resulted / won’t sweat
4. Roasting some green vegetables makes them C) has resulted / would not sweat
as acidic as carbonated drinks, and ---- to dental D) results / cannot sweat
erosion. E) will result / did not sweat

A) maintains B) enables C) prevents


D) contributes E) converts 11. I can still remember my excitement when I ----
for the first time how one’s understanding of the
concepts of probability and risk ---- to and
5. Eye doctors once ---- high fluid pressure in the enhance diagnostic and therapeutic problems in
eye to diagnose glaucoma, but now a six-year clinical care.
study has indicated that glaucoma can be
diagnosed earlier by also measuring damage to A) saw / could be applied
the optic nerve. B) had seen / could have been applied
C) could see / will be applied
A) gave up B) relied on C) thought up D) have seen / was applied
D) put forward E) looked through E) see / had been applied

6. Laboratories can ---- a variety of tests that help 12. Lung cancer is the most common cause of
doctors assess disorders of the liver and other death ---- cancer ---- both men and women.
organs.
A) about / with B) from / in C) with / for
A) find out B) put down C) keep on D) in / over E) through / about
D) look into E) carry out
13. While dialysis is an option ---- people suffering
7. If the doctor ---- for additional tests to be from kidney disease, no similar treatment is
performed, then this illustrates that he ---- other available for people ---- severe liver disease.
problems.
A) in / without B) with / at C) for / with
A) had asked / suspects D) of / within E) over / on
B) would ask / may suspect
C) asks / suspects
D) has asked / had suspected 14. ---- oral decongestants reduce cold symptoms
E) asked / will suspect by 20%, nasal sprays can reduce symptoms by as
much as 70%.

A) While B) Because C) Unless


D) As long as E) Supposing

218
15. ---- a new anti-viral drug is developed, 24. – 35. sorularda, verilen cümleyi uygun şekilde
scientists first test it on the hepatitis C virus. tamamlayan ifadeyi bulunuz.

A) Even if B) Before C) Since 24. Whereas many insomniacs believe that they
D) Because E) Whenever lack sufficient sleep, ----.

A) the chronic inability to either fall asleep or keep


16. A brain abscess may be fatal ---- it is treated sleeping are symptoms of insomnia
with antibiotics. B) they should arrange their retiring and rising times
so that they spend eight hours in bed
A) but B) whereas C) just as C) evidence shows that they are in fact getting at
D) unless E) in case least as much as they require
D) many people are usually drowsy early in the
morning
17. Lycopene is an important part of the body’s E) studies have shown that sleeping seven hours a
cell protection system which neutralizes free night is associated with the lowest mortality risk
radicals and ---- decreases the risk of
cardiovascular disease.
25. ----, but one sees an upright image.
A) on the contrary B) on the way
C) as well D) yet A) The analysis of the image by the brain proceeds
E) in this way piecemeal
B) Refractive errors can be corrected with glasses or
lenses
18. Toxic amblyopic, usually involving only one C) Disorders of the nervous system can damage the
eye, is a condition ---- retrobulbar neuritis, which optic nerve, which carries impulses to the brain
usually affects both eyes. D) The lens in your eye casts an upside-down image
on your retina
A) the same as B) similar to C) as regards E) The perception of a face depends largely on the
D) according to E) as well as relative positions of the features

19. – 23. sorularda, aşağıdaki parçada 26. When the mother contracts German measles
numaralanmış yerlere uygun düşen sözcük ya da during the first trimester of pregnancy, ----.
ifadeyi bulunuz.
A) it is impossible to repair intracardiac defects while
Fifteen million Europeans are infected by the hepatitis the heart is still pumping
B and C viruses, and chronic hepatitis is responsible B) the same defect has been known to occur in
for two-thirds of all cirrhoses and cancers of the liver. identical twins and in succeeding generations
Thanks to experience with HIV, we (19) ---- multi- C) congenital defects of the heart are also
therapy strategies in our fight (20) ---- hepatitis, associated with other congenital defects of the
combining several drugs to reduce the development body
of any resistance. These strategies now allow us to D) these complications may affect only the pregnant
completely (21) ---- the virus from the patient’s body in woman or both the woman and the foetus
50% of hepatitis C cases. (22) ----, multi-therapy E) this often causes the development of heart
treatment can only halt hepatitis B, not cure it. This is defects in the foetus
why vaccination is (23) ---- important.

19. 27. Because lymphocytes make up a relatively


A) have developed B) had developed small proportion of white blood cells, ----.
C) may have developed D) will have developed
E) must have developed A) lymphocytopenia is an abnormally low
lymphocyte count – below 1,500 cells per
20. microlitre of blood in adults
A) to B) against C) about B) the two main types of lymphocytes are
D) towards E) of Blymphocytes, also called B-cells, and
Tlymphocytes, also called T-cells
21. C) a reduction in their number may not cause a
A) eradicate B) cancel C) delay significant decrease in white blood cell count
D) clean E) disappear D) lymphocytopenia can be caused by a variety of
diseases and conditions
22. E) people who have low T-lymphocyte counts
A) Accordingly B) Besides usually have more severe lymphocytopenia than
C) As a result D) On the other hand people who have low B-lymphocyte counts
E) As well as

23.
A) such B) just C) as well
D) both E) so

219
28. ---- whether the prostate gland is enlarged. 33. You may increase your risk of contact
dermatitis, especially on sensitive skin, ----.
A) Routine urine analysis includes chemical analysis
to detect protein, sugar and ketones A) on condition that you have satisfied hydration and
B) Proteinuria is usually a sign of kidney disease radiance concerns
C) Blood in the urine is detectable by examination B) just because your skin feels smoother
under a microscope C) if you keep changing your skin-care products
D) The doctor performs a rectal examination in a D) as you have a problem limited only to the skin
man to feel E) before you become worried about wrinkles
E) Pain in the bladder is most often caused by a
bacterial infection
34. Infections are more likely and usually more
severe in the elderly than in younger adults, ----.
29. ----, though they may tire easily and may not
be able to keep up with a demanding schedule. A) while for serious infections, combinations of
antibiotics are often used
A) Doctors consider the possibility of multiple B) even though many long-term disorders that are
sclerosis in younger people who suddenly common in the elderly also increase the risk of
develop blurred vision infection
B) People with multiple sclerosis may have a few C) in that both physical barriers and the immune
more white blood cells than normal system defend the body against organisms that
C) People who become weak and unable to move can cause infection
easily may develop bedsores D) although the elderly are more likely to be in a
D) Leber’s hereditary optic atrophy is more common hospital or nursing home, where the risk of
in men acquiring a serious infection is greater
E) People with multiple sclerosis can often maintain E) probably because aging reduces the
an active lifestyle effectiveness of the body’s immune system

30. ----, the body quickly pulls water from tissues 35. ----, some infections, such as typhoid fever,
outside the bloodstream in order to keep the cause a slower rate than would be expected for
blood vessels filled. the severity of the fever.

A) Just as blood pressure falls A) As long as infection and fever generally make a
B) When blood is lost person breathe faster
C) In case anaemia is caused by excessive bleeding B) Although most infections increase the pulse rate
D) Since the body’s oxygen supply decreases C) As abnormalities of brain function may occur in
E) Before simple blood tests can identify anaemia severe infection
D) While a serious infection may cause stress ulcers
in the upper intestine
31. A peptic ulcer is a well-defined round or oval E) Unless an organism gets through the body’s
sore ----. natural barriers

A) even though the treatment is the same for


children as for adults
B) whether the pain occurs at a specific time of day
C) unless the x-ray study is normal, and the doctor
still suspects an ulcer
D) where the lining of the stomach has been
damaged or eroded by stomach acid and
digestive juices
E) since young children have difficulty in describing
their symptoms precisely

32. Most people can, to some extent, protect


themselves from HIV, ----.

A) as psychologists have played an important role in


combating the spread of infection
B) but infants born to HIV-infected mothers are
always susceptible
C) because the disease is easily transmitted during
medical or surgical procedures
D) even though HIV progresses over a decade or
more
E) so long as they have found meaning and positive
experiences in their lives

220
36. – 38. sorularda, verilen Đngilizce cümleye 38. Pain is not an inevitable consequence of
anlamca en yakın Türkçe cümleyi bulunuz. cancer, but most people with cancer experience
pain either as a result of the progression of the
36. Despite the chance that antibiotics may disease or due to the various treatments for
increase the risk of infection by suppressing cancer.
some bacteria and allowing others to grow faster,
these drugs can greatly reduce the risk of A) Ağrı kanserin kaçınılmaz bir sonucu değildir;
infection when used properly. fakat, kanserli pek çok kişi hastalığın
ilerlemesinin bir sonucu olarak veya çeşitli kanser
A) Antibiyotiklerin enfeksiyon riskini artırma tedavileri yüzünden ağrı çeker.
olasılığına karşı, bazı bakterileri bastırıp, B) Kanserin kaçınılmaz sonucu ağrı değildir, ancak,
diğerlerinin hızla büyümesi sağlanarak, bu kanserli hastalar hastalığın ilerlemesi sonucu,
ilaçların enfeksiyon riski uygun kullanımla önemli uygulanan tedavilere bağlı olarak ağrı çekerler.
ölçüde azaltılabilir. C) Hastalığın ilerlemesi ya da çeşitli tedaviler
B) Antibiyotiklerin bazı bakterileri bastırıp, sonucu kanserli hastaların ağrı çekmesi, ağrının
diğerlerinin daha hızlı büyümesine neden olması kanserin kaçınılmaz sonucu olduğunu göstermez.
yüzünden enfeksiyon riski artmaktadır; D) Kanserin sonu, kaçınılmaz olan ağrılar değildir;
enfeksiyon riskini azaltmak için bu ilaçların ancak kanser hastalarının çoğu ağrıyı ya
dikkatle kullanılması gerekmektedir. hastalığın ilerlemesi ya da çeşitli kanser tedavileri
C) Her ne kadar antibiyotikler enfeksiyon riskini nedeniyle çekerler.
artırıyorlarsa da, bazı bakterilerin bastırılmasına, E) Kanserde kaçınılmaz olan sonuç ağrı değildir;
diğerlerinin de hızlı büyümesine imkân tanınarak fakat pek çok kanserli insan hastalığın ilerlemesi
bu ilaçların uygun kullanımıyla enfeksiyon riski sonucu çeşitli kanser tedavileri gördükleri için ağrı
önemli oranda azaltılabilir. çekerler.
D) Antibiyotiklerin bazı bakterileri bastırıp diğerlerinin
daha hızlı büyümesine imkân sağlayarak
enfeksiyon riskini artırabileceği olasılığına 39. – 41. sorularda, verilen Türkçe cümleye
rağmen, bu ilaçlar, uygun bir şekilde anlamca en yakın Đngilizce cümleyi bulunuz.
kullanıldıklarında enfeksiyon riskini önemli ölçüde
azaltırlar. 39. Panik atak belirtileri o kadar ciddi ve
E) Bazı bakterileri bastırıp, diğerlerinin hızla korkutucudur ki insanlar bunları sıklıkla kalp krizi
büyümesini sağlamak antibiyotiklerin enfeksiyon belirtileriyle karıştırıp, hasteneye koşarlar.
riskini artırsa da, bu ilaçların doğru oranlarda
kullanımı enfeksiyon riskini büyük oranda A) Rushing to the hospital for a panic attack is not
azaltabilir. necessary, even though its symptoms are so
severe and frightening that they are often
mistaken for those of a heart attack.
37. Among the most exciting recent developments B) The symptoms of a panic attack are severe and
in medical technology are techniques that allow frightening, and people often mistake them for
physicians to see the organs and organ systems heart attack symptoms and rush to the hospital.
without resorting to surgery. C) Because panic attack symptoms are so severe
and frightening, people often rush to the hospital,
A) Medikal teknolojideki en heyecan verici mistaking them for signs of a heart attack.
gelişmeler arasında yer alan son teknikler, D) People often rush to the hospital when
doktorların organları ve organ sistemlerini experiencing a panic attack, as the symptoms are
ameliyata başvurmadan görmelerine imkân as severe and frightening as those of a heart
tanıyanlardır. attack.
B) Doktorların organları ve organ sistemlerini E) Panic attack symptoms are so severe and
ameliyata başvurmadan görmelerine imkân frightening that people often mistake them for
tanıyan teknikler, medikal teknolojideki heyecan signs of a heart attack and rush to the hospital.
verici en son gelişmelerden biridir.
C) Medikal teknolojideki en heyecan verici son
gelişmeler arasında doktorların organları ve
organ sistemlerini ameliyata başvurmadan
görmelerine imkân tanıyan teknikler gelmektedir.
D) Doktorların organları ve organ sistemlerini
ameliyata başvurmadan görmelerine imkân
tanıyan teknikler, medikal teknolojideki en
heyecan verici gelişmeler arasında sayılmaktadır.
E) Medikal teknolojide doktorların organları ve organ
sistemlerini görmesini sağlayan tekniklerin
arasında en heyecan verici olanı, ameliyata
gerek bırakmayan en son tekniktir.

221
40. Genetikçiler, çeşitli çevresel faktörlerin yanı 42. – 46. sorularda, boş bırakılan yere, parçada
sıra, en az on beş genin ankziyete ve strese karşı anlam bütünlüğünü sağlamak için getirilebilecek
hassasiyete neden olabileceğini tahmin ediyorlar. cümleyi bulunuz.

A) At least fifteen genes, together with some 42. The liver receives blood from both the
environmental factors, are estimated by intestine and the heart. Tiny capillaries in the
geneticists to be likely to contribute to anxiety intestinal wall drain into the portal vein, which
and susceptibility to stress. enters the liver. ----. The hepatic artery brings
B) According to geneticists, anxiety and blood to the liver from the heart. This blood
susceptibility to stress are likely caused by at carries oxygen for the liver tissue itself as well as
least fifteen genes, as well as a few cholesterol and other substances for processing.
environmental factors.
C) Geneticists believe that at least fifteen genes and A) The liver converts substances in digested food
different environmental factors contribute to into proteins
anxiety and susceptibility to stress. B) The liver manufactures about half of the body’s
D) Geneticists estimate that, along with various cholesterol
environmental factors, at least fifteen genes may C) Abnormalities of liver function can be divided
contribute to anxiety and susceptibility to stress. broadly into two groups
E) Geneticists are of the opinion that, besides D) Sugars are stored in the liver as glycogen
serious environmental factors, at least fifteen E) The blood then flows through tiny channels inside
genes do contribute to anxiety and susceptibility the liver
to stress.

43. Normally, the pancreas secretes pancreatic


41. Doğumdan sonra meydana gelen ciddi juice through the pancreatic duct to the
hormonal dalgalanmalar, hassas kadınlarda duodenum. ----. Blockage of the pancreatic duct
doğum sonrası depresyonuna neden olabilir, stops the flow of the pancreatic juice. Usually, the
ancak, bu rahatsızlığın nedenleri tam olarak blockage is temporary and causes limited
anlaşılmamıştır. damage, which is soon repaired.

A) Dramatic hormonal fluctuations that occur after A) The pancreas is a leaf-shaped gland about five
delivery may cause postpartum depression in inches long
susceptible women, but the causes of the B) Acute pancreatitis is a sudden inflammation of
disorder have not been fully understood. the pancreas
B) It is thought that postpartum depression may be C) This pancreatic juice contains digestive enzymes
caused by the dramatic hormonal fluctuations D) Almost everyone with acute pancreatitis suffers
that occur after delivery, although the causes of severe abdominal pain in the upper midabdomen
the disorder have not been adequately E) No single blood test proves a diagnosis of acute
understood. pancreatitis
C) While postpartum depression in susceptible
women is linked to the dramatic hormonal
fluctuations occurring after delivery, the causes of 44. ----. Studies on rats show that every drop of
the disorder have not been properly understood. sugar syrup they swallow causes a surge in their
D) Some women may be susceptible to postpartum dopamine levels – a sign of desire and a
depression as a result of dramatic hormonal biochemical marker of substance abuse. Boosting
fluctuations that occur after delivery, but doctors dopamine time after time is what drugs of abuse
have not yet fully understood the causes of this do. That makes scientists wonder whether food
condition. also has addictive properties.
E) Even though the causes of postpartum
depression have not been understood yet, A) Some researchers have begun to suspect that
doctors suspect that, in susceptible women, it obesity, eating disorders, and even the ordinary
may be caused by dramatic hormonal urges of appetite might resemble drug addiction
fluctuations that occur after delivery. B) Evolution has furnished us with several
neurochemicals and neural circuits that make
eating a deeply pleasurable activity
C) Scientists now realize that appetite and hunger
are not regulated only by energy balance in the
body, but also by human psychology
D) The appetite-regulation network in the human
body has evolved over millions of years to err on
the side of over-eating
E) The same brain receptors that bind the opioids
released by eating high-calorie food also respond
to morphine and heroin, with more pronounced
results

222
45. The McGill Pain Questionnaire (MPQ) is the 47. – 51. sorularda, karşılıklı konuşmanın boş
most frequently used pain questionnaire. It has bırakılan kısmını tamamlayabilecek ifadeyi
been used to assess pain relief in a variety of bulunuz.
treatment programmes and has demonstrated
some validity in assessing multiple pain 47.
syndromes. ----. Sensory qualities of pain are its Polly: - This is terrible! Have you read this article
temporal, spatial, pressure and thermal on dirty needles being used for vaccinations in
properties; affective qualities are its fear, tension poor countries?
and autonomic properties that are part of the pain Karin: - Yes, and it is terrible. Apparently, anti-
experience; and evaluative qualities are the words tetanus shots given with used needles are
that describe the subjective overall intensity of spreading HIV.
the pain experience. Polly: - ----
Karin: - Well, you know, they may not have enough
A) Pain has physical and psychological elements, needles, so they may have to use them more than
both of which can be quantified and measured once.
B) The measurement of pain is important because it
allows clinicians to quantify their patients’ pain A) I think wealthy countries should donate
C) Self-reports of pain include simple rating scales, disposable needles to poorer countries.
standardized pain inventories, and standardized B) How can the health workers in those countries do
personality tests such a thing?
D) The questionnaire provides a subjective report of C) They should test everyone’s blood before giving
pain and categorizes it in three dimensions a vaccine.
E) Pain rating scales make no distinction, for D) Tetanus and HIV are both dangerous infectious
example, among pains that are pounding, diseases.
shooting, stabbing or hot E) The health workers are probably selling the clean
needles illegally.
46. Although skin cancer is associated with a
behavioural risk (voluntary exposure to the sun 48.
over a long period of time), it also has a strong Doctor: - Have you gained any weight since you
genetic component. Light-skinned, fair-haired, started taking this high blood pressure
blue-eyed individuals, compared with darkskinned medication?
people, are more likely to develop skin cancer, Patient: - ----
and much of their damage occurs with sun Doctor: - No, you shouldn’t. This particular drug is
exposure during childhood. ----. They must take known to cause weight gain, so it’s normal. You
protective measures, including using sunscreen should just take care to eat properly and get
and wearing protective clothing while exposed to enough exercise.
the sun. Patient: - Alright, then.
A) Not all skin cancers are innocuous; one form, A) No, I don’t think so, and I’m glad I haven’t.
malignant melanoma, can be deadly B) How do you know? Well, I’m going on a crash
B) Twenty-five per cent of adults in the US sunbathe diet now to try and lose it.
frequently, and one-fourth of those do not use C) I can see you are worried.
sunscreens at the recommended levels D) As a matter of fact, I have. Should I be worried
C) Skin cancer is the most common form of cancer, about it?
but most types of skin cancer are curable E) I’ve heard that many drugs are related to obesity
D) During the past 50 years, the relationship in patients.
between skin cancer mortality rates and
geographic latitude has gradually decreased
E) These people should avoid prolonged and
frequent exposure to the sun

223
49. 52. – 56. sorularda, cümleler sırasıyla
Doctor: - How have you been feeling since we okunduğunda parçanın anlam bütünlüğünü bozan
started you on the cholesterol-lowering diet and cümleyi bulunuz.
drug regimen?
Harry: - Well, it’s been really hard for me. I’m not 52. (I) Cigarette smoking is most likely to cause oral
allowed to eat what I want, and I’ve been feeling cancer. (II) When nicotine is delivered to the brain,
depressed and aggressive for some reason. catecholamines, that is, neurotransmitters that include
Doctor: - ---- epinephrine and norepinephrine, are released. (III)
Harry: - In other words, they are only temporary These substances act as stimulants, increasing
then. cortical arousal, which can be measured by an
electroencephalograph (EEG). (IV) In addition,
A) Well, you’ll just have to continue with the smoking releases beta-endorphins, and the
regimen. You have your cardiovascular health to pleasurable effects of smoking may be due to the
consider, you know! release of these opiates produced by the body. (V)
B) Would you like to try psychological counselling to Nicotine also increases the metabolic level, which
help to confront these feelings? explains the tendency for smokers to be thinner than
C) Unfortunately, those are common side-effects of nonsmokers.
the drug you are taking.
D) Maybe we should consider alternate medication A) I B) II C) III D) IV E) V
for your condition.
E) Why don’t you try stopping the diet and
continuing with the drug, then? 53. (I) The sugars lactose, sucrose, and maltose are
broken down by the enzymes lactase, sucrase, and
maltase, which are located in the lining of the small
50. intestine. (II) Normally, the enzymes break these
Peter: - Here’s an interesting job advertisement. It sugars into simple sugars, such as glucose, which are
seems they’re looking for a person with a PhD in then absorbed into the blood through the intestinal
physical sciences to assist in a nanomedicine wall. (III) If the necessary enzyme is lacking, the
study. sugars are digested, and they can’t be absorbed. (IV)
George: - If the research topic is medicine, why do Thus, they remain in the small intestine, and the
they need a physicist? resulting high concentration of sugar draws fluid into
Peter: - ---- the small intestine, causing diarrhea. (V) The
George: - I guess that makes sense. Most doctors nutritional deficiencies resulting from malabsorption
don’t have that kind of training. can cause additional symptoms.
A) I would assume that they need someone with A) I B) II C) III D) IV E) V
experience in handling nanoparticles.
B) They must need help with something outside
their realm. What is nanomedicine, anyway? 54. (I) Identifying bacteria is a critical business for
C) I can’t guess, but I think nanomedicine has doctors and food safety experts, but involves either
promise for the future. culturing the bacteria until there are enough to look at
D) The reason is unclear. Maybe there’s a mistake under a microscope or amplifying the bacteria’s DNA.
in the advertisement. (II) Both these processes take hours, sometimes
E) I don’t know. Why don’t we look at the website to days. (III) The sensor is 25 times smaller than
find out? previously tested sensors, and is made of silicon, so it
requires the same fabrication technique as computer
chips and costs just eight cents. (IV) Now
51. bacteriophages, the viruses that prey on bacteria and
Martha: - It has been stated that schizophrenia is a are notoriously choosy about which species they
major public health problem throughout the world. attack, are being put to work in an electrical sensor
Colleague: - Yes, I’ve read somewhere that that detects bacteria within minutes. (V) The sensors
schizophrenia is more prevalent than Alzheimer’s take up less than a square millimetre each, so to
disease, diabetes, or multiple sclerosis. identify unknown bacteria, hundreds could be
Martha: - ---- integrated onto a single microchip with a different
Colleague: - Well, what makes a person vulnerable bacteriophage in each sensor.
to schizophrenia isn’t known, but may include
genetic predisposition. A) I B) II C) III D) IV E) V
A) In fact, as a serious mental disorder, it is
characterized by loss of contact with reality. 55. (I) Drugs cause rashes in several ways. (II) As
B) However, over longer periods, the prognosis of with severe burns, the skin loss is life threatening. (III)
schizophrenia varies. Most drug rashes are allergic reactions to
C) I strongly believe that it is to do with a person’s medications. (IV) After taking the first dose of a
genes. particular drug, a person may become sensitized to
D) Clearly, the severity and types of symptoms can the drug. (V) Later exposure to the drug may trigger
vary significantly. an allergic reaction.
E) It is true that schizophrenia is associated with
about 10 per cent risk of suicide. A) I B) II C) III D) IV E) V

224
56. (I) The first step in considering the possibility of a 59. It is pointed out in the passage that viral
genetic abnormality is obtaining a family history. (II) A respiratory infections ----.
doctor or genetic counsellor constructs a family tree
by asking about medical problems affecting family A) and pneumonia have no connection whatsoever
members. (III) For an accurate appraisal of genetic B) can be almost completely prevented throughout
risks, information about three generations in the the world
family is usually needed. (IV) The state of health or C) are mild illnesses that are most common among
cause of death of all first-degree and second-degree elderly people
relatives is noted. (V) One of the most common tests D) pose a serious threat, especially to children in
used to diagnose genetic abnormalities in a foetus is their first year
ultrasound scanning. E) cause no harm at all to the heart and lungs

A) I B) II C) III D) IV E) V
60. In the passage, attention is particularly drawn
to the fact that people with chronic cardiac
57. – 60. soruları aşağıdaki parçaya göre disease or chronic bronchitis ----.
cevaplayınız.
A) cannot be treated effectively against viral
Viral infections of the respiratory tract are certainly the infections of the respiratory tract
most common cause of infectious illness in most B) may experience severe complications because of
countries. People in all age groups are susceptible, a viral respiratory infection
but for those at the extremes of life these illnesses are C) develop pneumonia far more rarely than those at
particularly hazardous. Viral respiratory infections are the extremes of life
more common in children under the age of five years, D) should avoid any hazardous contact with other
but in the first year of life, they are more severe. In the patients
elderly, degenerative processes of the heart or lungs E) are less vulnerable to viral respiratory infections
make pneumonia a more frequent and serious than any other age group
complication, and the same is true for patients of all
ages suffering from chronic cardiac disease or chronic
bronchitis. On the whole, the vast majority of viral
respiratory infections are mild though often
uncomfortable conditions, but sometimes the illness is
severe and constitutes a threat to life. Unfortunately,
there is no method whereby the spread of infection
can be prevented. Adequate ventilation and
avoidance of crowds are clearly advisable.

57. It is clear from the passage that, especially in


the case of elderly people, ----.

A) having their rooms well-ventilated will prevent


viral respiratory infections
B) viral respiratory infections can lead to serious
consequences
C) the treatment of pneumonia may involve a very
complicated process
D) chronic cardiac disease can rarely be prevented
E) all kinds of viral infections must be avoided under
all circumstances

58. According to the passage, among the age


groups, it is the very young and very old ----.

A) that are more frequently threatened by


pneumonia
B) that suffer most from chronic bronchitis as well as
pneumonia
C) whose health is far more easily affected by
crowds
D) who always have a mild infection of the
respiratory tract
E) who are more susceptible to viral infections of the
respiratory tract

225
61. – 64. soruları aşağıdaki parçaya göre 64. According to the passage, some parents think
cevaplayınız. that ----.

As adolescents begin to assert their individuality, A) current norms of behaviour in society can never
family tensions increase, and battles are fought over be tolerated
clothes and hair styles, late nights and so on. The B) children must never be allowed to act
doctor may find himself consulted; some parents feel independently or behave differently
that, if their children get into trouble or disagree with C) it is not always helpful to consult a doctor about
them, they must be ill. Others seek a referee or an the behaviour of their children
accomplice in the battle of the generations. Often the D) any disagreement with their children is an
younger doctor in a partnership has an advantage in indication of their abnormality
dealing with such problems, being able to bridge the E) tensions in the family will always lead to an
generation gap and communicate well with both unending generation battle
parents and teenagers. Both generations need
education about the other and particularly about
current norms of behaviour. Children may have to be
reminded that their parents also have rights, and
parents, especially those with unrealistic ambitions for
their offspring, or those determined to live their lives
again through their children, must be taught to give
their children more independence.

61. It is suggested in the passage that the


generation gap between parents and their teenage
children ----.

A) can best be bridged through mutual


understanding and tolerance
B) has ruined many families and caused many
problems in education
C) has always been a primary concern among
younger doctors
D) is unnecessarily exaggerated by young doctors
E) could easily turn into a major problem that cannot
be solved

62. One understands from the passage that, in the


case of adolescents, ----.

A) parents rather than doctors know how to get over


problems of behaviour
B) staying out late at night must never be allowed by
parents
C) it is seldom that there arises a conflict with their
parents
D) changes in behaviour do, in fact, indicate a
statement of individuality
E) any disagreement with their parents mostly
results from family tensions

63. In the passage, reference is made to some


parents ----.

A) who have not been educated properly and,


hence, fail to understand each other
B) who have unattainable aspirations for their
children
C) whose only aim in life is to provide a good
education for their children
D) that deal with the problems of their children far
better than doctors
E) that are completely indifferent to the development
of their children’s individuality

226
65. – 68. soruları aşağıdaki parçaya göre 68. It is stressed in the passage that sensory
cevaplayınız. nerves ----.

Most of the functions of the brain are still unknown, A) have nothing to do with the body’s reflexive
and the ones we know about are very poorly actions
understood. The brain is assumed to be the organ of B) perform a limited number of voluntary functions
higher mental function, of the mind and intellect, but C) are so complicated that very little is known about
there is surprisingly little evidence for this, and no one them
has any idea what physical structures or mechanisms D) perform most of the functions controlled by the
perform these functions. The brain is known to control brain
all bodily functions by means of motor and other E) carry impulses from the body to the brain
nerves which carry impulses from the brain outwards
to all parts of the body. Sometimes these are under
our voluntary control; mostly, they are involuntary,
reflexive or automatic. Reflexive actions are the result
of impulses passed inwards from the body towards
the brain by means of sensory nerves. Information
arriving in the brain about various sensations like
heat, pain, touch, position, the need for saliva or
gastric juice or even the thought or smell of food are
acted on in the various “centres” in the brain.

65. It is clear from the passage that every part of


the human body ----.

A) is not sensitive to all the impulses sent by the


brain
B) does not show the same kind of reaction against
sensations
C) is connected to the brain through all kinds of
nerves
D) has a different kind of sensation not immediately
acted on by the brain
E) is controlled and acted on by a specific centre in
the brain

66. According to the passage, the question of how


the brain functions ----.

A) arises from problems related to the “centres” in


the brain
B) is essentially related to the question of how
reflexive actions take place
C) has been thoroughly investigated and,
consequently, resolved
D) has only become a major concern in medicine in
recent decades
E) does not yet have a full answer and still needs to
be studied

67. As one can see from the passage, one thing is


certain about the brain, and that is the fact that
----.

A) it only functions through motor nerves


B) all the impulses coming from it are involuntary
C) some intellectual functions take place in it
D) every bodily function is controlled by it
E) its main function consists of reflexive actions

227
69. – 72. soruları aşağıdaki parçaya göre 72. As one understands from the passage, it is
cevaplayınız. usually due to poor brushing and flossing that ----.

Gingivitis is the inflammation of the gums. Under this A) orthodontic appliances can be dangerous
condition, the gums become red and swollen and B) the gumline of the teeth has plaque, which
bleed easily. An extremely common condition, causes gingivitis
gingivitis can develop any time after a person’s teeth C) birth control drugs can be harmful to the teeth
come in, and it is almost always the result of D) the gums get rapidly swollen
inadequate brushing and flossing, which allows E) dentures and bridges are soon deformed
plaque to remain along the gumline of the teeth.
Plaque, which is a soft, sticky film made up primarily
of bacteria, accumulates especially in faulty fillings
and around the teeth next to poorly cleaned partial
dentures, bridges, and orthodontic appliances. When
plaque stays on the teeth for more than 72 hours, it
hardens into tartar, which can’t be completely
removed by brushing and flossing. Although plaque is
the main cause of gingivitis, other factors can make
the inflammation worse, especially pregnancy,
puberty, and birth control drugs.

69. It is pointed out in the passage that gingivitis


----.

A) in women usually develops only when birth


control drugs are taken
B) is rare, even when the teeth are not brushed
properly
C) can increase in severity due to various factors
D) is generally seen in grown-ups rather than
children and young people
E) can be treated most effectively through efficient
flossing

70. As explained in the passage, tartar ----.

A) is a major factor that causes bleeding along the


gumline of the teeth
B) forms around the teeth if plaque is not removed
within three days
C) generally results from faulty fillings and unflossed
partial dentures
D) is best removed from the teeth through frequent
flossing
E) can be effectively prevented by the use of
suitable orthodontic appliances

71. It is stressed in the passage that, even as early


as childhood, ----.

A) a great majority of people understand the bad


effects of plaque
B) people should be taught how to brush their teeth
C) many people become aware of the importance of
flossing
D) some people know how to get rid of tartar
E) people may suffer from gingivitis

228
73. – 76. soruları aşağıdaki parçaya göre 76. According to the passage, a strong heart
cevaplayınız. muscle ----.

Narrowly defined, fitness refers to the characteristics A) is indispensable to get rid of physical stress
that enable the body to perform physical activity. B) can only be developed through hard physical
These characteristics include flexibility of the joints, activities
strength and endurance of the muscles, including the C) is one of the indications of being fit
heart muscle, and a healthy body composition. A D) need not be related to physical fitness
broader definition of fitness is the ability to meet E) is the only indication of a healthy body
routine physical demands with enough energy reserve composition
to rise to a sudden challenge. This definition shows
how fitness relates to everyday life. Ordinary tasks
such as carrying heavy suitcases, opening a stuck
window, or climbing four flights of stairs, which might
strain an unfit person, are easy for a fit person. Still
another definition is the body’s ability to withstand
stress, meaning both physical and psychological
stress. These definitions do not contradict each other;
all three describe the same wonderful condition of the
body.

73. According to the passage, fitness, among


other things, enables ----.

A) an unfit person to perform various tasks in


everyday life
B) the heart to perform its task regularly
C) people to understand their body composition fully
D) the body to overcome psychological stress
E) a person to conserve his or her energy efficiently

74. Each definition of fitness given in the passage


----.

A) is not complete and has already aroused much


controversy among specialists
B) refers to various things and is therefore widely
different from the others
C) is perfectly compatible with the others and draws
attention to the same thing
D) offers guidelines about the development of a
healthy body composition
E) underlines the ways whereby the muscles can
best be made stronger

75. It is implied in the passage that, if one is not


fit, one ----.

A) can still find it easy to climb the stairs or open a


stuck window
B) should still try to do all kinds of tasks that cause
physical and psychological stress
C) must do his or her best to increase the energy
reserve of the body
D) should only carry out ordinary tasks in everyday
life
E) may find it hard to do the routine tasks of
everyday life

229
77. – 80. soruları aşağıdaki parçaya göre 80. It is stressed in the passage that, if a diet
cevaplayınız. contains too many calories, ----.

The immediate cause of obesity is the prolonged A) the body becomes far more active and healthy
consumption of a diet containing more calories than B) in the long run, it will most likely lead to obesity
are needed to provide for the body’s tissue repair, C) it usually has a good effect on the body’s vital
vital functions and physical activities. In modern functions
society, food has become very plentiful and attractive, D) its consumption should be stopped immediately
and the physical effort demanded by many E) people in most civilized communities know how to
occupations has diminished. Most people in civilized avoid obesity
communities eat more than they require, and it is
surprising that obesity is not more common than it is.
It is difficult to escape the conclusion that there exists TEST BĐTTĐ.
some unknown mechanism by which the body is CEVAPLARINIZI KONTROL EDĐNĐZ.
enabled to get rid of the surplus calories which would
otherwise be stored as fat. If there were not such a
mechanism, obesity would be much more common.

77. According to the passage, it is not clear yet by


what means ----.

A) fat is stored in the body and causes the


development of obesity
B) obesity can be prevented most effectively or
treated properly
C) the vital functions of the body can be maintained
most efficiently
D) the consumption of calorie-rich foods can be
increased in civilized communities
E) the body is able to eliminate its extra calories

78. One understands from the passage that


calories ----.

A) must be sufficient for the body to meet its


physical demands
B) are mainly derived from foods which are rich in
fat
C) are for the body to carry out its tasks and
activities
D) must be reduced so as to enable the body to
repair tissues
E) taken through the consumption of various foods
must always be limited

79. It is pointed out in the passage that, in most


professions, ----.

A) foods with a lot of fat should never be consumed


B) the amount of calories needed has increased
dramatically
C) people pay a lot of attention to the kind of food
they consume
D) obesity has never been a serious concern
E) the amount of the physical effort previously
required has declined

230
1. – 18. sorularda, cümlede boş bırakılan yerlere 8. NASA ---- experiments on the surface of the
uygun düşen sözcük ya da ifadeyi bulunuz. moon for eight years when they ---- them down in
1977 for financial reasons.
1. The cells of all living organisms have the ---- to
harvest energy from the breakdown of organic A) conducted / would have to shut
fuel molecules. B) has been conducting / have to shut
C) had been conducting / had to shut
A) amount B) clarity C) performance D) was conducting / have had to shut
D) reliance E) ability E) would have conducted / would have had to shut

2. Bacteria are truly remarkable in terms of their 9. It is predicted that the world’s fossil fuel
adaptations to extreme environments and their sources ---- completely by the turn of the century
abilities to survive in parts of Earth that are ---- to unless strict policies to use them wisely ---- at
other forms of life. present, before it is too late.

A) distasteful B) cordial C) inhospitable A. will have been exhausted / are implemented


D) persuasive E) discreet B. will be exhausted / would be implemented
C. would be exhausted / would have been
implemented
3. In December 1831, at the age of 22, Darwin D. would have been exhausted / were implemented
began a round-the-world sea voyage that ---- E. were exhausted / were to be implemented
influenced his thinking.

A) obscurely B) legitimately C) hopefully 10. If microchips ---- back in the late 1950s,
D) respectfully E) profoundly computer technology ---- as advanced as it is
today.

4. All organisms have some capacity to adapt to A. weren’t developed / isn’t


environmental changes, but the extent of this B. hadn’t been developed / wouldn’t be
adaptive capacity ---- greatly. C. wouldn’t have been developed / weren’t
D. wouldn’t be developed / wouldn’t be
A) executes B) varies C) discards E. weren’t being developed / won’t be
D) abolishes E) merges
11. It is hoped that alternative forms of energy,
5. The geological history of the Grand Canyon such as wind energy and geothermal energy, ----
region ---- to be a lot more complex than more widely used in the near future, once
previously thought when scientists examined the technical problems ---- completely resolved.
area in more detail.
A. become / had been
A) put off B) gave up C) brought about B. are becoming / will be
D) turned out E) set out C. will become / are
D. have become / were
E. would have become / have been
6. Most space rockets plunge into the oceans or
become space litter when they are ---- completely.
12. Insects that live in colonies, such as ants,
A) taken up B) fallen out C) given out bees, wasps, and termites, have long fascinated
D) made up E) used up everyone, ---- naturalists ---- artists.

A) among / with B) about / between


7. Scientists ---- that by the year 2050, robot brains C) in / of D) between / above
based on computers that have the ability to E) from / to
execute 100 trillion instructions per second ----
rivalling human intelligence. 13. A gene giving humans a preference ---- sweet
foods was recently identified ---- researchers.
A) are predicting / are starting
B) have predicted / would start A) over / among B) on / with C) to / at
C) predicted / would have started D) for / by E) into / within
D) predict / will start
E) would predict / will have started
14. The existence of galaxies was not recognized
until the early 20th century, ---- since then galaxies
have become one of the focal points of
astronomical investigation.

A) even B) because C) but


D) if E) whether

231
15. A gemstone used in the making of jewellery 19. – 23. sorularda, aşağıdaki parçada
must be durable, ---- it cannot withstand the numaralanmış yerlere uygun düşen sözcük ya da
process of being handled or shaped. ifadeyi bulunuz.

A) thus B) even if C) although Air pollution is one of the major challenges that most
D) or else E) while major cities face. The task of cleaning up air pollution,
(19) ---- difficult, is not believed to be insurmountable.
Use of fuels that are low in pollutants, such as low-
16. ---- it has a great potential for creating new sulphur forms of petroleum; more complete burning of
organisms, experimental recombination of genes fossil fuels, often in (20) ---- with a recycling of the
is viewed by some scientists as dangerous and pollutants; and the shift to less polluting forms of
unethical. power generation, such as solar energy (21) ---- place
of fossil fuels – all are methods that can be used for
A) As long as B) Unless C) Just as controlling pollution. Over the past few decades, the
D) Before E) Because example of London as well as of some other cities
(22) ---- that 10 years or less is enough to control this
problem to a certain extent. In fact, this period is
17. ---- does geology provide a better (23) ---- enough to achieve major improvements in air
understanding of the Earth’s evolution and its quality.
present features, but it also serves society in a
variety of practical ways. 19.

A) Either B) So C) Not only A) though B) as if C) unless


D) So long as E) Not once D) whether E) in case

18. Geometry, ---- name is derived from Greek 20.


words meaning “earth measurement,” is one of
the oldest branches of mathematics. A) discrimination B) combination
C) purification D) authorization
A) as B) which C) that E) utilization
D) whose E) its

21.

A) over B) at C) for
D) by E) in

22.

A) had shown B) would have shown


C) will have shown D) has shown
E) was showing

23.
A) nor B) either C) as well as
D) neither E) also

24. – 35. sorularda, verilen cümleyi uygun şekilde


tamamlayan ifadeyi bulunuz.

24. Whereas ordinary solids have a definite size


and shape, ----.

A. most liquids do, too


B. gases have none
C. their volumes are definite, as well
D. liquids don’t take the form of the container
E. gases consist of a vast number of molecules

232
25. Although rain forests occupy a very small 29. Physicists have established that, before the
percentage of the land surface on Earth, ----. universe existed in its present form, ----.

A. it is true that, in addition to high rainfall and A. the Milky Way is one of billions of galaxies in the
humidity, mild winters are good for them universe
B. most governments have introduced severe B. astronomy attracted many high school students
measures to stop and eliminate environmental to study it in depth
pollution C. all of its matter was concentrated in one mass
C. they contain a very large variety of plant and D. they knew what to look for
animal species that are mostly rare E. their existence would not have been questionable
D. many species living in those areas are faced with
the threat of extinction
E. the Brazilian government has long-term plans to 30. Experts differ greatly in their estimates of the
open up the Amazon basin for mining number of plant and animal species, ----.

A. since the exact number of these species has only


26. ----, but a significant part of this land is recently been identified
unsuitable for either agriculture or habitation. B. but they all agree that the distribution of these
species is highly uneven
A. China’s first nature reserve was established in C. in case many of the currently endangered
the 1950s species go extinct in the near future
B. China is located in Far East Asia D. unless they focus their attention on those species
C. The population of China has been increasing that have increased in number
dramatically E. despite the fact that the populations of certain
D. One of China’s major problems is environmental species have soared dramatically
pollution
E. China covers a vast area in Asia
31. Atoms have several properties ----.

27. Experts fear that, unless significantly higher A. in case they have different numbers of protons
yields are achieved with genetically modified (GM) and electrons
crops, ----. B. in which they are the smallest particles of all
matter
A. genetic engineers are determined to continue to C. that help distinguish one type of atom from
work on crops that can grow in a shorter period of another
time D. because physicists have split the atom into many
B. these trends led to economic development and a subatomic particles
significant reduction in global hunger and poverty E. while the electrons in an atom are very much
C. certain circles are concerned about the possible smaller than the protons and neutrons
negative effects of GM crops on human health
D. massive environmental degradation will be the
inevitable result of trying to feed the world’s 32. Quantum theory specifies new rules for
increasing population describing the universe, ----.
E. the impacts of climate change on agriculture will
not be fully understood in the years ahead A. although efforts to explore it are continuing
B. as it is considered an ancient science
C. unless the universe cannot be described with the
28. ----, which, in turn, makes animal husbandry help of its principles
more profitable than it used to be in the past. D. just as it has introduced new ways of thinking
about matter and energy
A. People with rising incomes consume more meat E. for it has not been proven or validated
and livestock products scientifically
B. The world’s poor have been getting poorer and
less able to produce food
C. Much of the world’s starving population lives in 33. Climatologists are not certain about the
the sub-Saharan region, where farming is poor number of hurricanes that occurred worldwide
D. Debate continues over whether climate change before 1970, ----.
will further reduce the world’s ability to feed itself
E. Most countries in Africa need to invest more in A. although the historical data are too inconsistent
their agriculture sectors to allow firm conclusions
B. given that sea-surface temperatures are a key
driver of hurricane formation
C. since an unprecedented four hurricanes hit
Florida in the summer of 2004
D. which global warming has led to more intense
storms
E. when satellite observations became routine

233
34. Some of the world’s natural resources, such 37. The discovery in Japan of a 130-million-year-
as natural vegetation and solar energy, are old lizard fossil may re-determine the date when
renewable, ----. flowering plants are believed to have emerged on
Earth.
A. which cannot be renewed easily at a low cost
B. while others like fossil fuels are non-renewable A. Çiçekli bitkilerin yeryüzünde ortaya çıktığı tarihin
C. since they are not reliable sources of energy yeniden belirlenmesi, Japonya’da 130 milyon
D. if conserving them is not possible yaşında bir kertenkele fosilinin keşfiyle
E. some of which are non-renewable gerçekleşmiştir.
B. Japonya’da 130 milyon yaşında bir kertenkele
fosili keşfedildikten sonra, çiçekli bitkilerin
35. New bioplastics are a lot less hazardous to our yeryüzünde ortaya çıktığına inanılan tarih
planet and our natural resources, ----. yeniden belirlenmiştir.
C. Japonya’da 130 milyon yaşında bir kertenkele
A. even though their reserves are gradually fosilinin keşfi, çiçekli bitkilerin yeryüzünde ortaya
shrinking due to increasing use by industries çıktığına inanılan tarihi yeniden belirleyebilir.
B. as plastics cannot be degraded biologically like D. Japonya’da 130 milyon yaşında bir kertenkele
organic materials fosilinin keşfedilmesinden önce, çiçekli bitkilerin
C. since they require less oil and energy to produce yeryüzünde daha yakın bir zamanda ortaya
than traditional plastics çıktığına inanılıyordu.
D. so traditional plastics are used in a wide range of E. Japonya’da 130 milyon yaşında bir kertenkele
industries, including textiles and packaging fosilinin keşfedilmesiyle, çiçekli bitkilerin
E. for the damaging effects of plastic waste on the yeryüzünde ortaya çıktığına inanılan tarih
environment are undeniable yeniden belirlenmiştir.

36. – 38. sorularda, verilen Đngilizce cümleye 38. With an estimated 200,000 species of plants
anlamca en yakın Türkçe cümleyi bulunuz. and animals, Madagascar is among the top five
most biologically diverse countries in the world.
36. According to some physicists, the universe is
just a little lighter than a black hole of the same A. Aşağı yukarı 200.000 bitki ve hayvan türüne
size. sahip olan Madagaskar, biyolojik çeşitliliği en
fazla olan dünyadaki ilk beş ülkeden biridir.
A. Bazı fizikçilere göre, bir kara delik, aynı boyuttaki B. Biyolojik çeşitlilik bakımından dünyanın en önemli
evrenden sadece biraz daha ağırdır. beş ülkesinden biri olan Madagaskar, tahminî
B. Bazı fizikçiler, evrenin, aynı büyüklükteki bir kara olarak 200.000 bitki ve hayvan türüne sahiptir.
delikten, sadece biraz daha hafif olduğunu öne C. Madagaskar’ın sahip olduğu bitki ve hayvan türü
sürmektedirler. 200.000 civarındadır ve böylece dünyada
C. Bazı fizikçiler, evrenin, aynı boyuttaki bir kara biyolojik çeşitliliği en zengin olan ilk beş ülke
delik kadar hafif olduğunu varsaymaktadırlar. arasındadır.
D. Bazı fizikçilere göre, evrenin ağırlığı, aynı D. Madagaskar, gerek zengin biyolojik çeşitliliği ve
boyuttaki bir kara deliğin ağırlığını gerek sahip olduğu 200.000 civarındaki bitki ve
geçmemektedir. hayvan türüyle, dünyanın ilk beş ülkesi arasında
E. Bazı fizikçilere göre, evren, aynı boyuttaki bir yer almaktadır.
kara delikten yalnızca biraz daha hafiftir. E. Tahminî 200.000 bitki ve hayvan türüyle,
Madagaskar, dünyada, biyolojik olarak en çok
çeşitliliğe sahip, ilk beş ülke arasındadır.

234
39. – 41. sorularda, verilen Türkçe cümleye 41. Çin’deki sanayileşme, öyle büyük çevresel
anlamca en yakın Đngilizce cümleyi bulunuz. değişikliklere yol açmıştır ki fabrikaların yarattığı
kirlilik dağlık alanlardaki yağışı, yüzde 50’ye varan
39. X-ışını görüntüleme yöntemleri, kırıkların miktarda düşürmüştür.
tanılanmasını bir yüzyıldan fazla zamandır daha
kolay kılmıştır, ancak X-ışınları, her zaman daha A. Industrialization in China has brought about such
ince ayrıntıların saptanmasında yardımcı olamaz. massive environmental changes that pollution
created by factories has reduced the rainfall in
A. Although the diagnosis of fractures has been mountainous areas by as much as 50 per cent.
easier for over a century, thanks to X-ray imaging B. Massive environmental changes in China have
methods, X-rays do not always help to detect the resulted from industrialization, and, consequently,
finer details. the rainfall in mountainous areas has dropped as
B. X-ray imaging methods have made diagnosing much as 50 per cent due to the pollution caused
fractures easier for more than a century, but X- by factories.
rays cannot always help to detect the finer C. The amount of the rainfall in the mountainous
details. areas in China has fallen by as much as 50 per
C. For over a century, it has been much easier to cent because of the pollution caused by factories,
diagnose fractures thanks to X-ray imaging as massive industrialization led to environmental
methods, even though X-rays fail to detect the changes.
intricate parts. D. Industrialization in China has caused so much
D. X-rays have helped to detect the inner parts of environmental change that the amount of the
fractures, and, therefore, the diagnosis of rainfall in the country’s mountainous areas has
fractures has been much easier for over a fallen 50 per cent due to the pollution brought
century. about by factories.
E. Thanks to X-ray imaging methods, the diagnosis E. China’s industrialization has led to extensive
of fractures has been possible for more than a environmental changes – so much so that,
century, but the details of fractures cannot always because of the pollution caused by factories, the
be detected fully. rainfall in mountainous areas has dropped to 50
per cent.

40. Her element, diğer bir elementin atomlarından


farklı, tek tip atomdan oluşur. 42. – 46. sorularda, boş bırakılan yere, parçada
anlam bütünlüğünü sağlamak için getirilebilecek
A. An element contains only one type of atom, cümleyi bulunuz.
entirely different from other atoms.
B. The kind of atom that makes up an element is 42. Photosynthesis, the process by which green
usually different from the atoms of other plants and certain other organisms use the energy
elements. of light to convert carbon dioxide and water into
C. Each element consists of one kind of atom, which simple sugar glucose, occurs in green plants,
is different from the atoms of any other element. seaweeds, algae, and certain bacteria. These
D. The atoms that each element has are completely organisms are veritable sugar factories,
different from those of other elements. producing millions of new glucose molecules per
E. Each element has one set of atoms which are second. ---- Some plants produce more glucose
very different from the atoms of all the other than they use, however, and they store it in the
elements. form of starch and other carbohydrates in their
roots, stems, and leaves.

A. The sugar we use in our everyday lives is derived


from plants like sugar cane and is processed in
factories.
B. An extremely important by-product of
photosynthesis is oxygen, on which most
organisms depend.
C. Being unable to produce it, humans and other
animals rely on the glucose produced by these
organisms.
D. Plants use much of this glucose as an energy
source to build leaves, flowers, fruits, and seeds.
E. In plants, photosynthesis occurs in leaves and
green stems within specialized cell structures
called “chloroplasts”.

235
43. Insects are often regarded as pests because 45. Ecology is the study of the relationship of
some bite, sting, spread diseases, or compete plants and animals to their physical and biological
with humans over crop plants. ---- Without insects environment. The physical environment includes
to pollinate flowers, the human race would soon light and heat or solar radiation, moisture, wind,
run out of food because many of the crop plants nutrients in soil, water, and atmosphere. The
that we rely on would not be able to reproduce. In biological environment includes organisms of the
addition, insects themselves are valued as food in same kind as well as other plants and animals. ----
many parts of the world. Also, they help to recycle For this reason, it draws upon such fields as
organic matter by feeding on wastes and on dead climatology, hydrology, oceanography, physics,
plants and animals. chemistry, geology, and soil analysis as well as
animal behaviour, taxonomy, physiology, and
A. Therefore, humans use chemicals to get rid of mathematics.
harmful plants growing among agricultural crops.
B. Scientists have so far been able to identify about A. Different kinds of organisms often share the
one million species of insects. same environment, which makes the relationship
C. Nevertheless, they are of undeniably great value between them significant.
to nature and to humans. B. However, due to the climate and other natural
D. Pollination is the process by which most plants factors, physical and biological environments
reproduce, but there are exceptions. show great diversity in the world.
E. While some insects protect themselves with their C. An increased public awareness of environmental
poison, others use camouflage. problems has made ecology a common but often
misused word.
D. Our understanding of environmental problems
44. ---- As a residential fuel, it is burned in has increased by a very large extent over the last
furnaces, water heaters, cooking stoves, and few decades.
clothes dryers. As an industrial fuel, it is burned E. Although the field is a distinct scientific discipline,
in special furnaces to bake bricks and ceramic it has a close relationship with other disciplines.
tiles and to produce cement. In addition to its use
as a fuel, natural gas serves as a raw material for
creating petrochemicals, which are used as a 46. The principle of all the sciences, including
base product for fertilizers, detergents, physics, is generally considered to be the
pharmaceuticals, plastics, and numerous other ordering of the complex appearances detected by
goods. our senses. In other words, this is the ordering of
what we often refer to as the “world around us”. In
A. Historical records show that natural gas was fact, many people think of science as a
burned as a fuel as early as 250 A.D. in China. mechanical process of collecting facts. ----
B. There is a growing concern among developed Essentially, science is a creative activity that in
nations about the future availability of energy many respects resembles other creative activities
sources. of the human mind.
C. Among the most common residential fuels are
natural gas, coal, and bottled butane and
propane. A. This is not the case at all.
D. Natural gas is used both as a fuel and as a raw B. Aristotle believed that the natural state of an
material in the manufacture of chemicals. object is to be at rest.
E. The use of fossil fuels is generally regarded as C. It was Galileo who founded our modern view of
harmful to the environment, since it is a major motion.
cause of global warming. D. As a result of Einstein’s theory of relativity, our
concepts of space and time have been
completely altered.
E. The list of ways in which physics, for example,
relates to other fields is extensive.

236
47. – 51. sorularda, karşılıklı konuşmanın boş 50.
bırakılan kısmını tamamlayabilecek ifadeyi Jeffrey: - The swine flu outbreak seems to have
bulunuz. emerged without warning, don’t you think?
Nathan: - Yes, you have a point. But I think the
47. virus existed for some time before it was
Joseph: - I read that Taiwanese researchers have discovered.
developed a motorcycle that runs on compressed Jeffrey: - ----
air. Nathan: - The answer is likely “yes”, if more
Maria: - So, what’s so special about it? attention had been paid to the human-animal
Joseph: - ---- interactions that enable new viruses to emerge.
Maria: - Oh, then it really is a significant invention.
A. Do you think governments have taken the
A. The prototype has not aroused much interest necessary measures against it?
among the public yet. B. Could there be a more effective way of
B. It requires electricity to compress the air. monitoring the course of the disease?
C. Well, it could help to solve the problem of C. Do you think the world faces more pandemics like
pollution. this one?
D. Actually, they have been working on it for quite D. Couldn’t it have been detected and the spread of
some time. it prevented?
E. Well, they might soon start mass producing them. E. Do you think vaccination against such pandemics
would solve the problem?

48.
Customer: - I’d like to buy a good camera for a 51.
reasonable price. What would you recommend? Peter: - Weren’t you thinking of getting a new job
Shop assistant: - Well, we have this professional as a laboratory assistant? Well, here is an
series, with which you can get extra high- advertisement.
resolution images and at least three types of Joanna: - Oh, I have already seen that one, but I’m
lenses. Their prices start from $3,000. not the one they are looking for.
Customer: - ---- Peter: - ----
Shop assistant: - Then, maybe you’d consider one Joanna: - Yes, all but a very important one. I’m not
from this series, which comes with only the a British citizen.
standard lens, but can be upgraded when
necessary. Plus, they provide equally high- A. Don’t you want to work in Britain?
resolution images. B. But why? You meet all the requirements.
C. Why? Don’t you have a licence to work as a lab
A. Oh, I didn’t expect that they would be so simple. assistant?
B. Actually, photography has been a major interest D. Are they looking for better qualified people?
of mine for a long time. E. You’re wrong; I’m sure they don’t care about
C. I can see you are very informed about cameras. citizenship.
D. Oh, I don’t think I want something so
professional.
E. Well, that really is a good value. 52. – 56. sorularda, cümleler sırasıyla
okunduğunda parçanın anlam bütünlüğünü bozan
cümleyi bulunuz.
49.
Thomas: - Did you know that e. coli, the bacteria 52. (I) The Anacostia River, which flows through the
that upsets stomachs and kills people, can heart of Washington, DC, is just one of dozens of
actually be useful? severely polluted rivers in the US. (II) It is
Julie: - How so? contaminated with the molecular remnants of dyes,
Thomas: - ---- plastics, asphalt, and pesticides. (III) Recent tests
Julie: - Wow! That means it is going to serve as an have shown that up to 68% of the river’s brown
alternative energy source. bullhead catfish suffer from liver cancer. (IV) Water
pollution comes from many sources, such as
A. You know, it is actually present in the intestines. wastewater from textile and pulp mills, agricultural
B. Well, scientists have genetically engineered it to waste, and residential sewage. (V) Wildlife officials
excrete biodiesel. recommend that anyone who catches the river’s fish
C. It obtains the energy it needs from the intestinal toss them back uneaten, and swimming has been
flora. banned.
D. The new method involves mixing the bacteria
with sugar cane. A) I B) II C) III D) IV E) V
E. The newly-developed biodiesel is not as efficient
as a fossil fuel.

237
53. (I) Weight, lift, thrust, and drag are the four
primary sources acting upon an airplane. (II) Since
the 1950s, aircraft technology has developed at an
amazing speed, making especially great advances in
thrust. (III) Weight, which is the force caused by
gravity, is also the force that offsets lift, which is the
upward force on the plane, because it acts in the
opposite direction. (IV) Thrust, which is provided by a
propeller or a jet engine, or a combination of the two,
is the force that pushes the airplane forward in the air.
(V) The fourth force, drag, is created by the friction
that the airplane produces when moving through the
air.

A) I B) II C) III D) IV E) V

54. (I) Volcanic eruptions in populated regions are a


significant threat to people, property, and agriculture.
(II) The danger is mostly from fast-moving hot flows of
explosively erupted materials, falling ash, and highly
destructive flows of lava and volcanic debris. (III) On
the contrary, they can also be rich in natural
resources, especially with the materials that they
erupt. (IV) In addition, explosive eruptions, even from
volcanoes in unpopulated regions, can eject ash high
into the atmosphere, posing a danger to distant
populations. (V) Such explosions also create drifting
volcanic ash clouds that represent a serious hazard to
airplanes.

A) I B) II C) III D) IV E) V

55. (I) Water vapour is the most common greenhouse


gas in the atmosphere, accounting for about 60 to 70
per cent of the natural greenhouse effect. (II) Humans
do not have a direct effect on water vapour levels in
the atmosphere. (III) But human activities do increase
the concentration of other greenhouse gases in the
atmosphere, producing warmer temperatures. (IV)
Indeed, global warming has now become one of the
most passionately argued issues of our time. (V) This
results in an increase in the evaporation of oceans,
lakes, and rivers, raising the amount of water vapour
in the atmosphere.

A) I B) II C) III D) IV E) V

56. (I) For a long time, science was more or less a


united whole known as “natural philosophy,” and
wasn’t that far removed from art. (II) Then, in the 18th
century, the distinctions between the scientific fields
began to appear. (III) Later, in the 19th century, to take
two examples, physics and chemistry went their
separate ways. (IV) Actually, several physicists in the
1930s and 1940s tried to apply their knowledge to
microbiology. (V) So, the sharp border that we now
see between the arts and sciences is but only a few
centuries old.

A) I B) II C) III D) IV E) V

238
57. – 60. soruları aşağıdaki parçaya göre 59. According to the passage, stratigraphy ----.
cevaplayınız.
A. helped scientists to understand the relationship
Palaeontology was once limited to digging up fossils among only fossil records
and trying to deduce their age with inaccurate B. deals mainly with vegetative life in a given region
methods. However, fossil analysis improved C. started to be used in the study of fossils long
dramatically in the 1960s, with the advent and after its emergence as a science
refinement of two techniques: radiometric dating and D. later established the basis for DNA dating
stratigraphy. The first radiometric method was also E. allowed scientists to refine the methods of
known as carbon-14 dating, and it was usable for biostratigraphy
specimens younger than 50,000 years. Later,
potassium-argon dating revolutionized the field by
enabling scientists to detect the radioactive decay of 60. The passage is mainly concerned with ----.
elements found naturally in rocks and soil surrounding
much older fossils. Stratigraphy, which is the study of A. the advanced methods palaeontologists use to
rock layering, actually was developed well before the date fossils
1960s, but that was the decade scientists began to B. how palaeontology came to be recognized as a
better understand how geological conditions, earthen field of science
layers, and fossil records all relate. The resulting C. recent advances made in the study of genetic
refinement of biostratigraphy, i.e., the study of the mutations
complete life of a stratum of earth, allowed scientists D. the development of DNA testing and its use in
to determine the environment and lifestyle of human deciphering the genetic code
ancestors based on fossilized flora and fauna found E. the relationship between humans and the
within the same layer as the hominine fossils. Since environment they live in
the 1960s, DNA testing has come to be used widely.
As all living organisms have the same genetic code,
scientists can use DNA variations as a molecular
clock. After splitting with a common ancestor, each
generation develops a constant rate of genetic
mutations. The molecular clock allows scientists to
calculate how long ago the split occurred based on
the number of differences between species. The
method is now helping scientists map the routes that
humans took out of Africa.

57. It can be understood from the passage that the


carbon-14 method ----.

A. involves the use of radioactivity to make fossils


decay
B. can be used to date fossils of any age
C. revolutionized potassium-argon dating
D. was no longer practiced after the 1960s
E. was the earliest form of radiometric dating

58. It is clearly stressed in the passage that,


before the development of different dating
methods, ----.

A. the molecular clock was used by scientists to


determine the age of fossils
B. the age of fossils could not be determined reliably
C. palaeontology was regarded as a field of study
that was only concerned with the excavation of
fossils
D. it was impossible to know whether Africa was the
home of the first human beings
E. only the age of fossils which were older than
50,000 years could be determined accurately

239
61. – 64. soruları aşağıdaki parçaya göre 64. The passage ----.
cevaplayınız.
A. doesn’t explain clearly how toothed whales
Baleen whales and toothed whales each have a vocalize
unique way of vocalizing. Only baleen whales B. gives a full account of the study researchers have
produce long sequences of deep sounds known as made of baleen whales
whale songs. They have a larynx, an organ at the top C. focuses more on toothed whales than on baleen
of the trachea, which may be involved in sound whales
production. Researchers are unclear about the D. points out the similarities of sound production in
organ’s role in the songs as whale larynxes are unlike whales and human beings
those of humans, which have vocal chords. Toothed E. describes in detail how toothed whales hunt in
whales, on the other hand, rely on sequences of high- total darkness
pitched clicks and whistles for both echo location and
communication with their mates. Their phonic lips, a
structure analogous to human nasal passages, press
together when air is forced through them, vibrating the
surrounding tissue. The sound waves then penetrate
an oily organ in the whale’s head, called the “melon”,
where they are focused into a beam of sound. When
this beam strikes a fish, the seabed, or another
object, the sound is reflected back to the whale as an
echo. Toothed whales can thus locate prey and
navigate in total darkness. However, during their long,
deep dives, toothed whales cannot inhale air every
time they want to produce a sound. So they collect it
in a sac at the back of their head and reuse it.

61. It is pointed out in the passage that there is


some uncertainty as to ----.

A. why both baleen and toothed whales use sounds


in order to find their ways
B. whether toothed whales can travel long distances
in the sea without inhaling fresh air
C. how a baleen whale’s phonic lips function in the
production of sound
D. how baleen whales produce sound
E. how far whale songs travel in the sea when
whales communicate with their mates

62. According to the passage, baleen whales are


different from toothed whales because they ----.

A. use their trachea to produce sounds


B. communicate with other whales through whistles
C. can dive to immeasurable depths and find their
ways in full darkness
D. are much more efficient in locating feeding areas
in the sea
E. produce what is called the “whale song”

63. It is clear from the passage that toothed


whales ----.

A. can hardly find where their prey is located


B. navigate through the echoes of the sounds they
make
C. often come up to the sea surface to inhale
enough air
D. generally swim close to the seabed while they
are hunting
E. are better hunters than baleen whales, especially
in deep waters

240
65. – 68. soruları aşağıdaki parçaya göre 68. As claimed in the passage, by 2050, the
cevaplayınız. world’s population will ----.

The human population continues to grow by more A. increase by more than half
than 75 million people annually. Since the first Earth B. definitely reach a stable level
Day in 1970, emission rates have remained steady at C. cease to have any effect on gas emissions
about 1.2 metric tons of carbon per person per year. D. have a much lower growth rate
Unfortunately, the 1997 Kyoto Protocol has had little E. decrease by 75 million people per year
measurable effect on these per-capita emissions,
even in the countries that have agreed to national
targets. More than any other factor, population growth
drives rising carbon emissions, and the US Census
Bureau and United Nations both project that the
global population, currently 6.6 billion, will surpass 9
billion before 2050. The implication is that one of the
best strategies for reducing future greenhouse gas
emissions is population stabilization, as quickly as
can be achieved by non-coercive means.

65. It is suggested in the passage that one


feasible way of reducing greenhouse gas
emissions in the future ----.

A. may be through the control of population growth


B. is through the enlargement of the measures
included in the 1997 Kyoto Protocol
C. will be through the assignment of more
responsibility to the United Nations
D. can best be found through cooperation among all
the countries in the world
E. depends to a large extent on the revision of the
1997 Kyoto Protocol

66. One understands from the passage that the


1997 Kyoto Protocol ----.

A. was originally inspired by the celebration of Earth


Day in 1970
B. can be considered a major achievement by the
United Nations and the United States
C. does contain a set of extremely harsh sanctions
on gas emissions
D. has been most effective in the reduction of
greenhouse gas emissions
E. has been largely disregarded by the countries
that have signed it

67. It is emphasized in the passage that annual


population growth in the world ----.

A. can be reduced effectively through the


implementation of the Kyoto Protocol
B. has the effect of raising the amount of carbon
emissions
C. plays no role in the increase in greenhouse gas
emissions
D. is constantly watched by the US Census Bureau
so that it can be kept stable
E. has alarmed many countries and forced them to
take measures to prevent it

241
69. – 72. soruları aşağıdaki parçaya göre 71. It is clear from the passage that natural spider
cevaplayınız. silk is produced ----.

Scientists are exploring ways of producing spider silk A. in large quantities for use in the production of
artificially, a process difficult to repeat effectively. A various materials
spider’s silk gland is a very efficient chemical factory. B. only when the spider’s silk gland has stored
Inside its gland, the spider stores a mixture of liquid enough liquid proteins
proteins, which it is able to transform into light, strong C. shortly before the spider’s silk gland undergoes a
fibres. Artificial spider silk could have many chemical reaction
applications, from lightweight and durable packing D. through the transformation in the spider’s silk
materials to parachutes, surgical sutures, and even gland of liquid proteins into fibres
bullet-proof vests. Producing it synthetically is a two E. even though the spider’s silk gland fails to have
part process: scientists must first manufacture the an adequate amount of liquid protein mixture
proteins and then find a way to form them into
superfine threads. They have had success with the
first part, by producing proteins through genetic 72. It is pointed out in the passage that, in making
modification. Binding proteins into fibres as thin and synthetic silk, scientists have so far been
strong as spider silk, however, has proved to be a successful in ----.
challenge. Recently, however, a group of German
scientists have attempted to solve that problem by A. imitating the entire natural process of silk
using a device modelled on a spider’s glands. Like the production
arachnid method, the proteins are mixed with B. producing fibres suitable for parachutes and
potassium phosphate, and then the pH is lowered bullet-proof vests
before pressure is applied as the mixture flows C. understanding the importance of proteins
through tiny channels, hardening and binding the D. producing artificial fibres about an inch in
proteins. So far, researchers have been able to make thickness
fibres of only a fraction of an inch long, but they hope E. obtaining the necessary proteins through genetic
to be able to produce longer, stronger fibres in the modification
future.

69. As stressed in the passage, with regard to the


production of artificial spider silk, the main
problem is ----.

A. the difficulty in mixing proteins with potassium


phosphate
B. that scientists are not seriously interested in it
C. how to make fibres as fine and long as natural
spider silk
D. whether the material can be used for various
purposes
E. whether it can be commercially and
technologically viable

70. According to the passage, artificial spider silk


----.

A. could have various uses in industry


B. is produced only in chemical factories
C. is commonly known as a very profitable product
D. is a material that has already had a wide range of
applications
E. is currently being produced in threads longer than
an inch

242
73. – 76. soruları aşağıdaki parçaya göre 76. One understands from the passage that
cevaplayınız. astrobiologists ----.

An organism discovered deep in the ground has taken A. have focused all their efforts on the search for life
astrobiologists by surprise. The organism’s unique on other planets
ability to live in complete isolation from other species, B. regard organisms as indispensable for the
or even light or oxygen, suggests it could be the key solution of environmental problems
to life on other planets. It was discovered in fluid-filled C. were astonished by the discovery of so unusual
cracks in a South African gold mine, nearly three an organism in the depths of the Earth
kilometres beneath the Earth’s surface. When US D. have now shifted their attention to the study of
scientists analyzed the fluid, they expected to find organisms deep in the Earth
genes from a mix of species. Instead, they found that E. have been particularly interested in the DNA
99.9 per cent of the DNA belonged to just one structure of a large variety of organisms
bacterium, a previously unknown species. Such a
self-sufficient organism is virtually unheard of. It
means that this organism extracts everything it needs
from an otherwise dead environment. Almost all other
known organisms on the Earth that do not use
sunlight directly do use some product of
photosynthesis. However, this newly-found organism
gets its energy from the radioactive decay of uranium
in the surrounding rocks. It also has genes to extract
carbon and nitrogen from its environment, both of
which are essential for making proteins. Scientists
believe that this organism is just the type that could
survive on a planet other than the Earth.

73. As suggested in the passage, for scientists,


the newly-discovered organism ----.

A. can be useful in preventing environmental


deformation
B. is unique only to Africa’s ecosystem
C. can provide clues about life on other planets
D. seems to reveal the beginnings of life on the
Earth
E. can survive only in an environment where there is
plenty of protein

74. According to the passage, upon the analysis


of the fluid found in a South African gold mine,
scientists have learned that ----.

A. proteins discovered in it were made through


carbon and nitrogen
B. it contained only one type of organism, unlike any
of those already known
C. the Earth and other planets have similar
environmental conditions
D. the radioactive effects of uranium can be seen
very deep down in the Earth
E. all kinds of organisms can be found not only on
the Earth but also on other planets

75. It is clear from the passage that what makes


the newly-found organism unique is that it ----.

A. lives in an environment that lacks any kind of


organic life
B. can survive only through photosynthesis
C. feeds on the remains of dead organisms
D. does not have to produce proteins
E. exists only in the depths of the Earth

243
77. – 80. soruları aşağıdaki parçaya göre 80. According to the passage, the validity of a
cevaplayınız. theory ----.

A scientific view of something is always an intimate A. can seldom be rejected once all the theoreticians
mixture of theories and observed facts. The theories share a common view
are broad, general ideas together with arguments B. is always hard to maintain, since scientists
based on them. The arguments are designed to show usually come up with contrary views
that, if the general ideas are accepted, then this or the C. can only be established after a series of
other thing ought to be observed. If this, that, or the arguments put forward by various theoreticians
other actually is observed, then the theory is a good D. is determined through the test of its comparison
one; if not, then the theoreticians have to think again. with observed facts
Thus, theoretical ideas and arguments are continually E. always makes scientists proud of their work for
subjected to the severe test of comparison with the the progress of science
facts, and scientists are proud of the strictness with
which this is done. On the other hand, theories often
suggest new things to look for; in other words, they TEST BĐTTĐ.
lead to predictions. These predictions are frequently CEVAPLARINIZI KONTROL EDĐNĐZ.
successful, and scientists are entitled to be proud of
that, too. But it follows that no theory is immutable;
any scientific view of any subject may, in principle, be
invalidated at any time by the discovery of new facts.

77. The point made in the passage is that theories


are subject to revision or may lose their validity
----.

A. if and when new facts are brought to light


B. so long as they are not supported through
arguments
C. because the arguments about them are not
based on facts
D. even though they have been tested and
compared with facts
E. due to the fact that most of the observations they
are based on cannot be tested

78. It is asserted in the passage that, when


something is understood scientifically, ----.

A. it shows how essentially the theory rather than


the observation is important
B. this involves not only the relevant theory but also
an observation of facts themselves
C. this means the rejection of the arguments put
forward against it
D. the predictions made so far lose their significance
and are not taken into consideration
E. this is because new facts have not been
discovered yet

79. It is clear from the passage that theories ----.

A. usually provide guidelines for new discoveries


B. always arouse a great deal of controversy among
scientists
C. sometimes exist separately from observed facts
D. can be maintained unless they are validated by
new facts
E. are specific formulations that are taken for
granted by many scientists

244
1. – 18. sorularda, cümlede boş bırakılan yerlere 8. Upon the completion next month of its
uygun düşen sözcük ya da ifadeyi bulunuz. renovation and expansion, the museum ---- its
exhibition space and added an auditorium ---- for
1. Organized youth camps provide young people performances and lectures.
with not only different forms of outdoor ---- but
also training in co-operation, initiative, and A. will have doubled / to be used
resourcefulness. B. doubled / to have used
C. has doubled / to have been used
A) sustenance B) irresponsibility D. doubles / being used
C) permanence D) relevance E. had doubled / to use
E) recreation
9. Without television cameras, the famine now
2. During the seventeenth and eighteenth ravaging Ethiopia ---- the attention of the well-fed
centuries, there was a common view that elegant world that ---- with offers of money and other
speech was a mark of social prestige in that it was forms of aid.
---- of having come from a good family.
A. won’t have caught / will respond
A) possessive B) perceptive C) indicative B. did not catch / responded
D) inclusive E) extensive C. will not catch / had responded
D. would not have caught / has responded
E. is not catching / is responding
3. Since many people are unaware of the miseries
of poverty and hunger, it is ---- easy for them to
feel contented. 10. The finest achievement of Europe’s post-1945
leaders was their recognition that, unless
A) painfully B) barely C) violently Germany ---- into the evolving Western system,
D) offensively E) usually insecurity ---- across the continent.

A. has been integrated / had reigned


4. The pattern of world affairs is very complicated, B. had been integrated / has reigned
and by no means everything that happens can be C. is integrated / reigned
---- to the influence of the super-powers. D. integrates / will reign
E. was integrated / would reign
A) attributed B) entitled C) declared
D) sustained E) resolved
11. Our sense of self ---- by the roles and qualities
that our peers and teachers ---- to us.
5. Historically, the development of capitalism has
---- several phases, following the period of feudal A. was formed / have assigned
organization of society. B. is formed / assign
C. is being formed / will be assigning
A) taken after B) called in D. had been formed / assigned
C) brought forward D) gone through E. will have been formed / would assign
E) turned down
12. The Houston Caribbean Festival brings a feast
6. Rice is by far China’s most important ---- music and colour ---- the streets of downtown
agricultural product and ---- over half the total Houston each year.
cereal production.
A) on / by B) at / over C) of / to
A) lets in B) accounts for C) sets forth D) above / off E) to / within
D) falls through E) looks after
13. Mount Kilimanjaro in Tanzania, the highest
7. Janet Malcolm, who is an admired mountain ---- Africa, rises 5,895 metres ---- sea
photographer and ---- pictures since the early level.
1960s, generally ---- her summers photographing
people and places in Africa. A) over / at B) across / into C) at / through
D) below / to E) in / above
A. took / has been spending
B. had been taking / will spend
C. has been taking / spends 14. It is assumed ---- an educational programme
D. takes / has spent should emphasize the valid aspects of the cultural
E. had taken / would spend and historical past.

A) whereby B) since C) as
D) that E) in case

245
15. The most rewarding aspect of taking 19. – 23. sorularda, aşağıdaki parçada
photographs is to be able to immortalize on your numaralanmış yerlere uygun düşen sözcük ya da
film people’s hearts, smiles, and soul ---- you can ifadeyi bulunuz.
always feel like you are a part of their world.
Despite the American people’s growing (19) ---- to this
A) while B) as though C) so that policy, more than 100,000 US soldiers remain in Iraq,
D) whereas E) whenever where they are (20) ---- involved in training Iraqi
forces to replace them. However, it is understood that
they stand ready to intervene directly should the
16. Painters ---- diverse ---- Goya, Manet, and security situation again (21) ----. Politically, Iraq is still
Picasso were inspired by Titian and other a troubled country, with major rows among political
Renaissance painters. parties on the one hand and tribal leaders on the
other (all of whom are solely interested (22) ----
A) as / as B) both / and expanding their own power base) being a regular
C) not only / but also D) so / that occurrence. In Anbar province, for example, the
E) such / as arguments between the sheikhs and other influential
figures (23) ---- are competing with one another for
power and popularity often turn violent.
17. It is generally thought that animals love us ----
who or what we are. 19.
A) approval B) discrepancy C) opposition
A) so long as B) no matter D) subversion E) failure
C) provided that D) because of
E) for the sake of
20.
A) relatively B) hardly C) rarely
18. If there is one thing that is more astonishing D) mainly E) scarcely
than the ability of the adult human to talk, it is the
process ---- he learns to do it.
21.
A) whichever B) where C) that A) increase B) commence C) extend
D) however E) by which D) emerge E) deteriorate

22.
A) at B) about C) for
D) in E) with

23.
A) which B) who C) where
D) when E) whom

24. – 35. sorularda, verilen cümleyi uygun şekilde


tamamlayan ifadeyi bulunuz.

24. As families become more fragmented and


dispersed, ----.

A. children tend to turn more to other people for


affection
B. ties between mother and father get stronger
C. traditionally, singleness is seen as a mark of
immaturity
D. working conditions for young people have
improved a great deal
E. the increasing rate of crime in big cities has
always been a serious concern

246
25. Because its habitat is threatened by climate 30. Sceptics often claim that reports of
change, ----. unidentified flying objects (UFOs) are examples of
shared delusions, ----.
A. the elephants in Africa have dwindled
considerably A. since many people have seen them in recent
B. the survival chances of many species in the years
Amazon area are getting worse B. despite the fact that we never believed it
C. the polar bear has been the first to be granted C. though that doesn’t discourage those who insist
protection under the Endangered Species Act on having seen them
D. the Asian tiger’s black-striped yellow fur is very D. now that what UFO enthusiasts say they have
attractive seen is taken seriously
E. there is a very profitable market for elephant E. until a team of scientists examined some
tusks wreckage found in 1947

26. While children in their early years are learning 31. The traditional Middle Eastern diet, which
about the world around them, ----. relies heavily on lean meat, salads, vegetables,
and fruit, is a healthy one, ----.
A. they have always been warned by their parents
B. he makes use of his previous experiences A. even though it is often served in expensive
C. it is obvious that child care ought to be a priority restaurants
D. there are some diseases that have no effective B. just as one must pay close attention to one’s own
treatment health
E. they often confuse the real with the imaginary C. as long as you intend to share it with other
people
D. if one is careful enough to follow it in moderation
27. After Dali was expelled from art school in and not to excess
Madrid in 1926, ----. E. when more and more Europeans have travelled
to the Gulf Area
A. he had been much influenced by the earlier
Spanish painters
B. a number of paintings depict unusual landscapes 32. ----, others are more resistant to change and
C. especially mentally disturbed people had become dysfunctional and fail.
interested him enormously
D. he joined a group of painters who called A. While most companies quickly adopt new
themselves “surrealists” information technologies and thus survive
E. today his paintings still amaze but also entertain B. Because companies differ in their goals and the
many people strategies designed to reach them
C. That most senior managers do not realize how
fragile the ongoing viability of a company can be
28. ----, although most French people believe that D. Provided that the most flexible companies rapidly
this tradition has a much more recent history. acquire new knowledge and apply it quickly
E. Unless the best companies meet the challenges
A. Even France’s everyday wines are widely that typically arise in the business environment
appreciated worldwide
B. Most of the great wines of France have long been
produced in its southern regions 33. Taiwan’s pink dolphins have been listed as
C. The range, quality, and reputation of the fine “critically endangered”, ----.
wines of Bordeaux have made them world-
famous A. as soon as the extensive fieldwork on these
D. Each of the wine-producing regions in France creatures is started
has its own traditional identity B. as if the work of scientists and conservationists
E. Winemaking in France dates back to pre-Roman marked a victory for them
times C. if their losses could be halted and reversed
D. when the effort to identify them along the western
coast started in 2004
29. ----, until the Europeans began to settle there E. since they face the threat of extinction
in the 18th century.

A. Anthropologists believe that aboriginal people in 34. ----, all mammals are known today to have
Australia initially arrived from Asia some form of emotion.
B. In Australia, various aboriginal tribes had
inhabited the region now known as South Wales A. Despite the fact that there are striking similarities
C. Australia’s leading city, Sydney, has experienced between human beings and animals
alternating periods of growth and decline B. Although some scientists long maintained
D. Australia has always been inhabited by the otherwise
aboriginal people C. Because even birds delight us with their singing
E. Sydney is a modern cosmopolitan city that has a D. Even if studies suggest that animal behaviour is
distinctive cultural identity instinctive
E. If they know how different species communicate

247
35. Lethal injection replaced execution by 37. Logic suggests that having options allows
hanging, the gas chamber, and the electric chair, people to select precisely what makes them
----. happiest, but as studies show, excessive choice
often makes for misery.
A. so some researchers have challenged the
efficacy of the drug protocols underlying the A. Mantıksal olarak, çok sayıda seçeneğe sahip
practice olmak insanları en mutlu eden şeydir; oysa
B. in which three poisonous chemicals are çalışmalar çok fazla seçeneğimiz olduğunda
administered to the condemned genellikle kararsız kaldığımızı göstermektedir.
C. thus making this method far from foolproof B. Mantık, insanların kendilerini tam olarak en mutlu
D. each of which had at some point been judged to eden şeyi seçmelerini önerir; oysa çalışmaların
be inhumane and excessively violent da gösterdiği gibi, çok sayıdaki seçeneğimiz
E. but many US states soon adopted it as their only arasında bizi mutsuz edebilecek şeyler de vardır.
form of capital punishment C. Mantık, seçeneklere sahip olmanın insanlara
kendilerini tam anlamıyla en mutlu kılacak olanı
seçme imkânı tanıdığını belirtir; ancak,
36. – 38. sorularda, verilen Đngilizce cümleye çalışmaların da gösterdiği gibi, gereğinden fazla
anlamca en yakın Türkçe cümleyi bulunuz. seçenek çoğu kez mutsuzluğa neden olmaktadır.
D. Đnsanların sunulan seçenekler arasından
36. This new collection of cosmetics has been kendilerini tam olarak en mutlu edecek olanı
scientifically developed to help correct the visible seçmeleri mantık açısından da önerilir; ancak
signs of aging, such as fine lines and wrinkles, yapılan çalışmalara göre, genellikle gereğinden
enlarged pores, and skin imperfections. fazla seçeneğe sahip olmak insanlarda sıkıntıya
yol açmaktadır.
A. Bu yeni kozmetik serisi, ince çizgiler ve kırışıklar, E. Çok sayıda seçenek genellikle kararsızlık yaratsa
genişlemiş gözenekler ve cilt bozuklukları gibi da, mantık yine de kendilerini en mutlu edecek
gözle görülür yaşlanma belirtilerini düzeltmeye olanı seçebilmeleri için insanların farklı
yardımcı olmak için bilimsel olarak geliştirilmiştir. seçeneklere sahip olmaları gerektiğini belirtir.
B. Bu kozmetik serisi, ince çizgiler ve kırışıklar,
genişlemiş gözenekler ve cilt bozuklukları gibi
gözle görülür yaşlanma belirtilerini düzeltmeye 38. China’s one-child policy, enforced by the
yardımcı olmak için yeni geliştirilmiştir. authorities through heavy fines, came into force in
C. Bu yeni kozmetik serisi, ince çizgiler ve kırışıklar, 1979 after a period of explosive population growth
genişlemiş gözenekler ve cilt bozuklukları dâhil in the country.
her türlü yaşlanma belirtisini yok etmeye yardımcı
olmak için geliştirilmiştir. A. Yetkililerce ağır para cezaları koyularak dayatılan
D. Bu yeni kozmetik serisi, ince çizgiler ve kırışıklar, Çin’in tek çocuk politikası, ülkede aşırı ölçüde
genişlemiş gözenekler ve cilt bozuklukları gibi nüfus artışının yaşandığı bir dönemden sonra
gözle görülür yaşlanma belirtilerini bilimsel 1979’da yürürlüğe girmiştir.
yollarla düzeltmeye yardımcı olmak için B. Çin’de yetkililerin uymayanlara ağır para cezaları
geliştirilmiştir. öngördükleri tek çocuk uygulaması, ülkede
E. Bu yeni kozmetik serisinin bilimsel yollarla patlama denilebilecek bir nüfus artışının
geliştirilmesinin amacı, ince çizgiler ve kırışıklar, yaşandığı 1979 yılından sonra güç kazanmıştır.
genişlemiş gözenekler ve cilt bozuklukları gibi C. Çin’de yetkililerin 1979 yılında güç kullanarak ağır
gözle görülür yaşlanma belirtilerini düzeltmeye para cezaları karşılığında kabul ettirdikleri tek
yardımcı olmaktır. çocuk politikası, aşırı nüfus artışının yaşandığı bir
dönemden sonra uygulamaya girmiştir.
D. Çin’in 1979 yılında yaşanan aşırı nüfus artışından
sonra uygulamaya koyduğu tek çocuk politikası,
yetkililer tarafından ağır para cezaları sayesinde
kabul ettirilmiştir.
E. Ağır para cezaları konularak halka kabul ettirilen
Çin’deki tek çocuk politikası, ülkenin yaşamış
olduğu aşırı nüfus artışı döneminden sonra
yetkililer tarafından 1979’da yürürlüğe
konulmuştur.

248
39. – 41. sorularda, verilen Türkçe cümleye 41. Đlk kez rıhtım bölgesinde çalışan Güney
anlamca en yakın Đngilizce cümleyi bulunuz. Carolina’lı zenciler tarafından yapılan Çarliston
dansı, George White tarafından Birinci Dünya
39. Dünyanın en eski ve en büyük çarşısı olan Savaşı’nın hemen sonrasındaki yıllarda tanıtılmış
Kapalıçarşı, Đstanbul’un fethinden hemen sonra ve 1920’lerin bir özelliği olmuştur.
Fatih Sultan Mehmet’in emriyle, Ayasofya’ya gelir
sağlamak amacıyla inşa edilmiştir. A. Launched by George White during the early
years following the First World War, the
A. The Grand Bazaar, which was built by Sultan Charleston had originally been danced by Negro
Mehmed the Conqueror to provide income for St. dockworkers in South Carolina before it became
Sophia right after the conquest of Istanbul, is one characteristic of the 1920s.
of the oldest and largest bazaars in the world. B. The Charleston, danced by black dockworkers,
B. Being the oldest and the largest bazaar in the originated in South Carolina and became the
world, the Grand Bazaar was built soon after the characteristic of the 1920s once George White
conquest of Istanbul on Sultan Mehmed the introduced it soon after the First World War.
Conqueror’s order to provide income for St. C. The characteristic dance of the 1920s, the
Sophia. Charleston, was first introduced by South
C. Built upon Sultan Mehmed the Conqueror’s order Carolina Negro dockworkers and became well-
to provide income for St. Sophia, the Grand known only when George White taught it after the
Bazaar has been the oldest and the largest First World War.
shopping area in the world since the conquest of D. Although the Charleston was danced first by
Istanbul. black people at the docks in South Carolina, it
D. The Grand Bazaar, which is the oldest and the was not until George White introduced it that it
largest bazaar in the world, was built after the became characteristic of the post-war era in the
conquest of Istanbul when Sultan Mehmed the 1920s.
Conqueror ordered that it should provide income E. The Charleston, first danced by the South
for St. Sophia. Carolina Negro population working in the
E. It was soon after the conquest of Istanbul that dockland area, was launched by George White in
Sultan Mehmed the Conqueror ordered that the the years immediately following the First World
Grand Bazaar, the oldest and the largest bazaar War, and it became characteristic of the 1920s.
in the world, be built so that it could provide
income for St. Sophia.
42. – 46. sorularda, boş bırakılan yere, parçada
anlam bütünlüğünü sağlamak için getirilebilecek
40. Medyadaki şiddet görüntülerini izlemenin daha cümleyi bulunuz.
sonra çocuklarda saldırgan davranışlara yol
açtığını kesin biçimde ortaya koyamasalar da, 42. The carpet is of eastern origin and in general
aradaki bağlantıyı araştıran çalışmalar şiddet can be said to differ from tapestry by the
izlenmesi ile sonradan ortaya çıkan saldırganlık technique of its knotted weave. The refined
arasında dikkate değer bir ilişki kurmaktadırlar. technique and ornate geometrical patterns found
on fragments discovered in eastern Turkestan,
A. However significant the link between observation dating from the 5th and 6th centuries A.D., indicate
of violence and later aggression may be, a long evolution in the history of the carpet. ----
correlational studies cannot claim that viewing The Abbasid caliphs of Baghdad were also well-
media depictions of violence produces known patrons of carpet manufacture: one made
subsequent aggressive behaviours in children. for Caliph Hashin early in the 8th century
B. No matter how clearly correlational studies measured 100 yards by 50 yards. The most
establish a significant link between viewing media famous area of production was, however, north-
depictions of violence and subsequent western Persia.
aggressive behaviours in children, they still
cannot demonstrate this unequivocally. A. The large carpets that decorate the mosque of
C. Although correlational studies are unable to Alaaddin date back to the 13th century.
establish a significant link between observation of B. The earliest fragment of carpet still existing was
violence and later aggression, they can found in Russia and is now in the Hermitage
demonstrate unequivocally that viewing media Museum in St. Petersburg.
depictions of violence produces subsequent C. A change in the composition and decoration of
aggressive behaviours in children. carpets was made in Persia at the end of the 15th
D. Despite their inability to demonstrate century.
unequivocally that viewing media depictions of D. During the Sassanid dynasty in Persia (in the 6th
violence produces subsequent aggressive and 7th centuries A.D.), carpets of wool and silk
behaviours in children, correlational studies pile were already in great demand.
establish a significant link between observation of E. The carpet industry in Egypt must have been of
violence and later aggression. great importance at the end of the Middle Ages.
E. The fact that viewing media depictions of
violence produces subsequent aggressive
behaviours in children has been poorly supported
by the correlational studies which have
established a link between observation of
violence and later aggression.

249
43. Castes are known in several societies, but the 46. During the 1960s, although Tanzania had
best known and most rigid example developed in adopted Swahili as its national and official
India. It may have grown in the first two millennia language, it was the medium of instruction only
B.C., chiefly with the object of preserving the pale- until high school, where it was replaced by
skinned Aryan invaders from admixture with the English. ---- Since then, Swahili has gradually
darker-skinned, indigenous Dravidians. ---- been replacing English as the medium of
However, an increasing subdivision of castes was instruction throughout all levels of the education
firmly established by the 5th century B.C., with system.
strict rules of social conduct.
A. British colonial education policies, like those of
A. Since British society was divided by class, the the Germans before them, were seen to be
British attempted to equate the Indian caste grounded in linguistic imperialism.
system to their own social class system. B. Swahili also serves a symbolic function for the
B. In Hindu religious terms, the four main castes country, namely one of nationalism and
were of divine origin, having been born from unification.
different parts of Brahma. C. This situation lasted until an official change in
C. Caste rules have been considerably relaxed, policy was written into the country’s Third Five-
especially in the last 40 years or so. Year Plan in 1976.
D. The caste system was carried over into Indian D. However, the role of English was central to the
Islam by those who converted to this faith. shaping of Tanzanians’ cultural identities in
E. Distinctions were not particularly rigid in the early colonial times.
stages, and intermarriage was known. E. During the postcolonial 1950s and 1960s, many
Third World societies concentrated on their socio-
economic needs.
44. ---- In the argument, additional claims are
made in support of the claim we are trying to
persuade people to accept. But these additional 47. – 51. sorularda, karşılıklı konuşmanın boş
claims may be challenged as well. Recognizing bırakılan kısmını tamamlayabilecek ifadeyi
this, authors frequently anticipate the need to bulunuz.
supply further support for their arguments.
47.
A. Recognizing that people generally require Mike: - Do you know that they are going to start
reasons to accept a controversial claim, we set rationing water?
forth an argument. Cindy: - Yes. I happened to hear it on the news this
B. Distractions make it hard, even for reasonable morning. They said that the reservoir had dipped
people, to reliably differentiate between below half its capacity, and the volume of water
reasonable and unreasonable courses of action. entering the reservoir is the smallest in the last
C. We think of reason as the capacity to use ten years.
disciplined intelligence to solve problems. Mike: - It’s not surprising, actually, because we
D. The word “argument” is ambiguous, for one of its haven’t had as much rain as usual this year.
senses is the dispute between two people. Cindy: - Well, if it could rain even a drop, my
E. There comes a point for most people where it is flowers would certainly come back. The soil has
no longer possible to suspend judgment. been baked so hard.
Mike: - ----
45. For advertisers, the most important A. From now on, the best thing to do is to water your
classification of advertisements is by the type of garden regularly every day.
consumer, and they spend huge amounts of B. Forget it. It looks as if it’s going to get a lot worse
money and effort in trying to divide up the public before it gets better.
so that it can be more precisely targeted. This C. You know, even before the rationing
process entails endless discussions about announcement, they suddenly turned off the
whether the best divisions are those of lifestyle, water yesterday without a warning.
socioeconomic class, personality type, or of D. I don’t blame you. Why don’t you hire a gardener
something else altogether. Then there is the fact to help you?
that certain products are more often bought by E. Just remember, I told you to pay the water bills
men than by women, by the rich, by a certain age on time.
group, and so on. ----

A. The advertiser must anticipate a prospective


customer’s preferences and persuade him to buy
the product in question.
B. Advertising has become too widespread
geographically and too multifaceted in terms of
media for any study to be comprehensive.
C. Nevertheless, the issue of defining an ad is
closely linked to that of defining its categories.
D. In other words, when an ad appears on a wall or
even a shirt, it is not the substance of this
environment that matters, but its social meaning.
E. So a given advertising approach will have far
more of an effect on one group than on another.
250
48. 50.
Steve: - How are things going with you and your Ruth: - What do you think of global warming and
roommate? humans’ alleged role in it?
Bob: - ---- Larry: - Global warming is a problem that will
Steve: - Then, why don’t you tell him that he largely have to be solved through energy
should pay you for half of what you spend on conservation, but ----
food? Ruth: - What do you really mean?
Bob: - I would find that a bit hard to do, Larry: - I mean it’s environmental hypocrisy!
considering how he treats me to a good meal in a Ruth: - Hypocrisy?
restaurant every weekend. Larry: - Yes. These people preach clean energy,
simple living, and use of mass transportation
A. He’s such a snob! He wasn’t like that before he while doing none of these things themselves.
got that new job.
B. Actually, I dislike the food he prepares, so I A. people who care about children, born or unborn,
generally eat out. should be aware of the lurking environmental
C. We get along well, except for the times he dangers.
practices playing the guitar and singing off-key. B. industrial agriculture has lowered the nutritional
D. We’re supposed to share the grocery shopping, value of staple vegetable crops far below what it
but he ends up eating everything I have put in the used to be.
fridge without buying any of it himself. C. these high-profile environmentalists who talk and
E. I’ve got sick and tired of his nagging me all the talk and do little else are worse than useless.
time. If he refuses to mend his ways, I’ll ask him D. people must take scientific facts into account
to move out. when making important life choices.
E. there may be no solution to the coming nightmare
of world overpopulation.
49.
John: - Each time we switch to Daylight Savings
Time, I always forget to put the clock forward an 51.
hour. Joanne: - Hi, Tom! You look worried. What’s on
Michael: - Me, too. I’ve always had problems with your mind?
Spring forward and Fall-back. Tom: - I really need that job I’ve interviewed for,
John: - I think adjusting ourselves twice a year is but I just found out that there are many younger
harmful to our long-term health, too. applicants. I guess my chances are pretty slim
Michael: - You know, with modern technology, we now.
could slowly ease into Daylight Savings Time by Joanne: - I wouldn’t say that! You are older but
digitally having clocks add a few seconds a day highly qualified and experienced. I am sure you
for three months in the spring and the reverse in will get it!
the autumn. Tom: - But I still think that any younger applicant
John: - ---- has an advantage over me.
Joanne: - ----
A. Then the change would be imperceptible,
provided all clocks moved in the same manner. A. Those young guys may only be trying to get their
B. I really love that first evening after Daylight foot in the door so they can move on to better
Savings Time has taken effect. positions later. Most employers know this.
C. It is indeed busywork, and it has caused me to B. Anyway, face facts: you are eligible for
miss important things a couple of times. retirement, and that field has moved on without
D. My clock makes the changeover automatically, so you.
I have no problem. C. You’re right! One of the most difficult things an
E. What a great idea! But I don’t think it will do any older jobseeker has to deal with is the younger
good with the traffic situation, do you? competition.
D. Having retirement income, you should start out at
an entry-level salary, which should be satisfying
for you.
E. Just use your common sense. Have you ever
seen anyone over 40 doing that job? There’s a
reason for that.

251
52. – 56. sorularda, cümleler sırasıyla 55. (I) The practice of counting population is, in fact, a
okunduğunda parçanın anlam bütünlüğünü bozan very old one. (II) An essential feature of a census is
cümleyi bulunuz. that the enumeration is direct and names each
individual person. (III) The Roman census, which was
52. (I) Today’s young people in their teens and 20s, designed primarily as the basis of the taxation system
who have been dubbed “digital natives”, have never under which citizens were classified into six classes
known a world without computers, the Internet, and according to their property, also ascertained the
cell phones. (II) Many of these natives rarely enter a number of men available to serve in the army.
library, but instead use Google, Yahoo, and other (IV) The census takers declared publicly the number
online search engines for information they need. of persons counted in Rome, and the proportion of
(III) The neural networks in the brains of these digital orphans and widows. (V) In 5 B.C., the census was
natives differ dramatically from those of adults. extended to the whole of the Roman Empire.
(IV) Investigators have reported that most of
technology exposure, such as watching television and A) I B) II C) III D) IV E) V
videos or listening to music, is passive. (V) Basic
neural networks in most adults are said to have been
laid down during a time when direct social action was 56. (I) After the arrival of road and rail links, the
the norm. commercial importance of Amsterdam’s 75 km of
canals declined, although barges still ply the waters of
A) I B) II C) III D) IV E) V the Amstel River. (II) Canals do, however, play a
crucial role in the city’s tourist industry, as well as
providing Amsterdammers with transport routes and a
53. (I) Various studies have indicated that people place to live. (III) In 1994, the city council passed laws
exhibit much behaviour in about the same order, even aimed at reducing pleasure-boat traffic and imposing
though they have experienced very different stricter rules on houseboats. (IV) A rapid increase in
environments. (II) We may speak of learning to read population in the latter half of the century caused a
as a process and of reading abilities as the products demand for housing in the city. (V) In spite of this, the
or end results. (III) Learning as a process is of primary canals can get very clogged in summer, and there are
importance in deciding the level of the materials of still around 2,500 houseboats officially moored within
instruction. (IV) What an individual can learn is the city’s boundaries.
determined not only by his level of physical and
mental maturation but also by what skills, attitudes, A) I B) II C) III D) IV E) V
interests, tastes, and knowledge he has already
developed. (V) If the height of an individual’s
enjoyment in reading is Donald Duck, for example, it
would be a mistake to begin with Lady of the Lake to
advance his literary taste.

A) I B) II C) III D) IV E) V

54. (I) Scientists hoping to show that Chopin died


from cystic fibrosis, not tuberculosis as is widely
believed, and they have been refused permission to
prove their theory with tests on the composer’s heart.
(II) When researchers at Warsaw’s Institute of
Molecular and Cell Biology asked for access to the
heart, which has been preserved in alcohol since his
death in 1849, the Polish government said, “No”.
(III) When Poland emerged from under Soviet
domination, the people were expecting a new era of
accessible government to begin. (IV) Their goal was
to demonstrate that this hereditary disorder need not
be a barrier to achieving greatness. (V) But much to
their disappointment, Chopin remains unavailable for
scientific research, at least for the time being.

A) I B) II C) III D) IV E) V

252
57. – 60. soruları aşağıdaki parçaya göre 60. Hume’s belief was that poverty was mainly
cevaplayınız. caused by ----.

Scottish philosopher and historian David Hume A. the amount of money in circulation
emerged as an economist also with the publication of B. the scarcity of produced goods
his Political Discourses. The famous Adam Smith was C. the abuse of poor nations by rich ones
a friend of his and may have been influenced by D. the lack of a complete economic theory
Hume: they had similar principles, and both were very E. Adam Smith’s poor grasp of economics
good at illustrating and supporting these from history.
Although Hume did not formulate a complete system
of economic theory, as did Smith in his Wealth of
Nations, he introduced several of the new ideas
around which the “classical economics” of the 18th
century was built. His economic philosophy can be
understood from his main arguments: that wealth
consists not of money but of commodities; that the
amount of money in circulation should be kept related
to the amount of goods in the market; and that poor
nations impoverish the rest because they do not
produce enough to be able to take much part in trade.
Beyond this, he urged society to welcome the shift
from an agricultural to an industrial economy, without
which civilization could not be achieved.

57. According to Hume, ----.

A. rich nations should produce more to be able to


feed their citizens
B. rich nations should provide financial help to
poorer nations
C. poor nations have a negative effect on richer
nations
D. poverty can be overcome by increasing the
production capacity of rich nations
E. poor nations can take part in international trade
only when rich nations are impoverished

58. According to the text, Adam Smith ----.

A. was very much under the influence of Hume


B. formulated a complete system of economic
theory
C. had ideas that conflicted with Hume’s
D. was uncertain about Hume’s principles
E. had a great effect on Hume

59. According to the text, Hume ----.

A. was against the ideas on which the “classical


economics” of the 18th century was based
B. misunderstood the principles that his friend Adam
Smith believed in
C. was not the only one who excelled at illustrating
and supporting his principles from the past
D. argued that money in circulation had to be barely
related to the amount of goods in the market
E. stated that civilization required advances in both
agricultural and industrial production

253
61. – 64. soruları aşağıdaki parçaya göre 63. The passage points out that Hong Kongers’
cevaplayınız. identity crisis ----.

The people of Hong Kong have been experiencing an A. arises from their having lived under British
identity crisis ever since the British returned the colonialism in the past
colony to China in 1997 and it became a Special B. is related to their communications with their
Administrative Region with special privileges (for 50 families on the mainland
years). Although they are proud of their Chinese C. is closely linked with their city’s economic
ethnicity, culturally they have always felt downturn
overwhelmingly Western and therefore much different D. has been made worse with the emergence of
from their cousins on the mainland. Now they have a Shanghai as a future competitor
new worry: a growing threat to Hong Kong’s economic E. did not exist before 1997
success. The Chinese government recently
announced its plan to turn the city of Shanghai into a
global financial and shipping centre by 2020, a move 64. We learn from the passage that Hong Kong’s
that is seen as weakening Hong Kong’s traditional long-term planners ----.
and profitable position as international gateway to
mainland China. Even worse, China’s friendlier A. hope to give the city a natural advantage in
relationship with former enemy Taiwan is already education, environmental-related industries, and
reducing transit commerce through Hong Kong. After medical technology
an economic contraction of almost 8% earlier this B. identified six economic areas deserving of
year, Hong Kong is feeling real pain, and the jobless government support
rate could approach record levels. So the normally C. advised the government to carry out two rounds
hands-off local government has sprung into action: it of tax cuts
has announced two rounds of tax cuts and various D. are concerned about how to meet the challenge
handouts to the poor and to businesses. In addition, represented by Taiwan
the city’s long-term planners have recommended that E. are planning to boost the city’s economy by 2020
Hong Kong’s government focus on developing six in order to be equal to Shanghai
fields – including education, environmental-related
industries, and medical technology – in which Hong
Kong already has an edge.

61. According to the passage, the Hong Kong


government ----.

A. has been investing heavily in medical-technology


companies
B. views the government of Taiwan as its enemy
C. supports the plan to make Shanghai a global
financial and shipping centre
D. is planning to bring about a rise in the jobless
rate in the city
E. usually does not interfere in the local economy

62. We can infer from the passage that for many


years ----.

A. Hong Kong has made a lot of money from others’


trading with China
B. Hong Kong has been suffering from severe
economic contraction
C. the Chinese government has pressured the
people of Hong Kong to give up their Western
ways
D. China and Taiwan have enjoyed friendly relations
E. the Hong Kong government has given out money
to the poor people of the city

254
65. – 68. soruları aşağıdaki parçaya göre 68. The passage points out that the proponents of
cevaplayınız. menu labelling ----.

On July 1, 2009, the US state of California began A. don’t really know if it will be successful
enforcing a new menu-labelling law that requires B. often have to defend the lifestyle choices of
chain restaurants to post on their menus the calories obese Americans
contained in their food items. Three other states– C. are themselves regular customers of chain
Oregon, Maine, and Massachusetts – have already restaurants
passed similar regulations, as have 11 city and county D. oppose the tendency of Los Angeles residents to
governments. The trend has gathered strength gain weight every year
quickly, mostly because of concern about the nation’s E. believe much of the new law is irrelevant
expanding waistlines. The next step is to deploy the
practice nationally, and the Congress is about to
debate such a law. Pressure for this type of move is
coming from the obese, who represent more than a
third of American adults, and their defenders.
Overweight people often struggle to estimate the
number of calories they consume when eating out
and make mistakes when calculating how much food
they should order. Proponents of menu labelling hope
that knowing what is in their food may direct people to
healthier items. In Los Angeles, for example, officials
optimistically predict that menu labelling could prevent
nearly 40% of the annual weight gain there. However,
the effect of menu labelling on dietary choices
remains unclear, and the regulations are too new to
produce much evidence. Furthermore, some critics of
the trend believe the public-health benefits of the new
legislation are irrelevant. For them, the new
regulations are welcome as part of a consumer’s
wide-ranging right to know.

65. The passage indicates that the new menu-


labelling law in California has the potential to ----.

A. force chain restaurants there to offer less food to


the public
B. encourage the habit of overeating in the
population at large
C. persuade other states to consider passing similar
laws affecting chain restaurants
D. help people become more health-conscious
E. reduce the population in that state by nearly half

66. We can infer from the passage that obese


people in the US ----.

A. want to control the number of calories they are


ingesting in chain restaurants
B. are under attack by other Americans
C. eat most of their meals at chain restaurants
D. have difficulty making mathematical calculations
E. have their own special representatives in the US
Congress

67. The passage informs us that menu-labelling


laws ----.

A. were passed in the other US states after


California enforced its version of the law
B. will all be cancelled when the US Congress
passes a national requirement
C. are a sign of the reaction to Americans’ getting
fatter and fatter
D. will result in the disappearance of obese
Americans in the future
E. forbid fat people to eat too much in chain
restaurants

255
69. – 72. soruları aşağıdaki parçaya göre 71. It is clear from the passage that the
cevaplayınız. international prohibition of ivory exports ----.

Although an extension of the worldwide ban on ivory A. has been strongly opposed by the government of
exports to discourage the illegal killing of African Zimbabwe
elephants has been greeted enthusiastically in many B. has been totally ignored by the governments of
places, the rhinoceroses (rhinos) of southern and China and Yemen
eastern Africa are still paying with their lives for their C. has been in force throughout Africa since the
horns, which remain prized by the Chinese for their early 1960s
medicinal and aphrodisiac qualities, and by the D. has completely prevented the illegal hunting of
Yemenis for making dagger handles. According to a elephants in Africa and Asia
group, called Traffic, that monitors the wildlife trade E. will be continued, and this has been widely
throughout the world, this illegal business is on the welcomed
rise. Last month, the group called for stronger
international cooperation along smuggling routes and
for more secure management of legal horn stocks. 72. According to the passage, in view of the
For its part, Zimbabwe, where there are a lot of illegal growth of the illegal wildlife trade, the group
killings, has taken a very radical decision: it says it will Traffic ----.
start dehorning its rhinos. Today only five species of
rhino survive in Africa and Asia. In the past, especially A. is concentrating on shutting down the Chinese
in the 19th and 20th centuries, they were slaughtered and Yemeni wildlife markets
on a large scale by white hunters. By the 1960s, B. has put a great deal of pressure on African
fewer than 70,000 black rhinos were left in Africa, countries, especially Zimbabwe, to begin
and, over the next two decades, illegal hunters wiped dehorning their rhino populations
out 96% of them. But since 1995, thanks to vigorous C. has increased its involvement in the campaign to
conservation efforts, the number of black rhinos has save African elephants from extinction
gone up again, to around 3,700. The number of white D. is urging the international community to step up
rhinos has nearly doubled over the same period, to its efforts to stop the smuggling
over 14,500. E. aims to have governments legalize what is now
the illegal business of wildlife trade
69. As one learns from the passage, black rhinos
in Africa ----.

A. are preferred by the Yemenis over white rhinos


for making dagger handles
B. are so much in demand for medical uses that the
government of Zimbabwe is devoting more
resources to conserving them
C. were hunted almost to extinction in the past by
Chinese doctors
D. were almost completely exterminated during the
1970s and 1980s as a result of illegal hunting
E. and white rhinos in Asia are the only surviving
species of rhinoceros in the world

70. As is pointed out in the passage, there is ----.

A. absolute indifference in the world towards


Zimbabwe’s decision to dehorn its rhinos
B. a full international consensus on the ending of
the wildlife trade in Zimbabwe
C. widespread illegal killing of wild animals in
Zimbabwe
D. now wide support for a new policy of limited legal
killing of African elephants
E. today an ongoing fall in the number of African
rhinos, especially in Zimbabwe

256
73. – 76. soruları aşağıdaki parçaya göre 75. According to the passage, the OECD carried
cevaplayınız. out its investigation of child welfare in its
members ----.
Many governments these days feel that the path to
happiness for society as a whole lies through A. to see if their stated goals were being reached
spending on the welfare of its youngest members: B. because many governments believe that
their health, education, and general well-being. A spending on child welfare leads to societal
recent report from a leading international happiness
organization, the OECD, examined these efforts C. in order to learn where government funds
among its 30 member countries in order to learn if the directed at child welfare were being wasted
aim was being achieved. Specifically, the researchers D. to prove that more spending on child welfare
investigated 21 variables that were then grouped into guarantees positive results
six main categories. The results surprisingly showed E. despite the fact that a key correlation was weak
that while some kinds of spending on children do
work, many should be improved or scrapped. Also,
total government spending per child was seen to vary 76. The passage mentions the case of Australia in
considerably, as did outcomes, but the correlation order to demonstrate ----.
between these was not strong. Moreover, the
differences in spending levels among countries were A. a contrast with other members of the OECD
not directly linked to their relative levels of prosperity. B. the resistance of some OECD members to the
For example, rich Sweden is, as expected, kind to its pressures coming from children’s lobbies
children, but poorish Hungary turns out to be C. the similarities between it and America in terms of
generous, too. Up-and-coming South Korea might be their child-welfare policies
expected to be a bit reluctant to part with so much D. the wisdom of the Swiss government’s decisions
money, but the stinginess of Switzerland is totally affecting child welfare in Switzerland
unexpected. Children’s lobbies always want more E. how spending relatively less on child welfare
funds, but the OECD report suggests that more doesn’t necessarily produce harmful results
money does not reliably yield better results. America
has one of the highest levels of spending per child,
and among the worst outcomes. In contrast, Australia
spends less, with better outcomes.

73. The passage points out that an OECD member


country’s level of prosperity in relation to other
members’ ----.

A. will determine if its child-welfare programmes


should be improved or scrapped
B. is associated with the constant efforts of
children’s lobbies to obtain more funds
C. generally convinces it to be more generous in
providing for its children
D. depends on its geographic location
E. was shown to have almost no connection to its
amount of spending on child welfare

74. The passage makes it clear that typical


expectations about a country’s spending on child
welfare ----.

A. often match the reality found by the researchers


B. can easily turn out to be wrong
C. are essentially the same for most of the OECD
D. cannot be separated from its record in the six
main categories
E. do not take into consideration all the 21 variables
studied by the researchers

257
77. – 80. soruları aşağıdaki parçaya göre 79. In the passage, it is clear that Juanes ----.
cevaplayınız.
A. wishes he were Cuban instead of Colombian
He is young, dark, and handsome, with a beautiful B. claims to have no political purpose with his
light tenor voice, and he swept the Latin Grammy planned concert
Awards in 2008. But should Juan Esteban Aristizabal, C. has a tense, hostile relationship with the Cuban-
better known as Juanes, perform his songs at an American community in Miami
upcoming “peace concert” in the Cuban capital, D. is more concerned with his own career than with
Havana? The debate over this Colombian rock star, helping people in need
who is based in the US city of Miami, has been raging E. was invited to perform in Havana because of the
on that city’s Spanish-language radio and television Latin Grammy Awards he had won earlier
shows. Older Cuban-Americans, who left Cuba in the
immediate aftermath of the 1959 revolution there,
vehemently oppose the concert. They argue that it is 80. We can infer from the passage that the overall
just the latest attempt by the Cuban regime to Cuban-American community ----.
manipulate public opinion. Traditionally, these older
exiles have held all the political power in Miami. But A. loves pro-Cuban musical artists like Bonnie Raitt
younger ones are pushing back, especially when it and The Police
comes to the arts. Beyond the overall Cuban- B. believes that Juanes is lying about his reasons
American community, the Juanes concert is seen as a for performing in Havana
potential great turning point in US-Cuban relations. A C. is much more emotional about US-Cuban
successful outcome could smooth the way for a relations than the wider American public is
further softening in American policy towards Cuba. D. does not speak or understand English very well
Artists other than Juanes have attempted this kind of E. has almost no interest in the arts
bridge-building with Cuba before; for example, left-
leaning musicians like Bonnie Raitt and The Police
appeared there in 1999, but they had an overtly TEST BĐTTĐ.
political agenda. However, Juanes himself, who is CEVAPLARINIZI KONTROL EDĐNĐZ.
widely admired for his humanitarian work in his native
Colombia, denies having any thought of politics. “My
only message is one of peace, of humanitarianism,
and of tolerance,” he said recently.

77. The passage states that the relationship


between the US and Cuban governments ----.

A. depends on the approval of the older Cuban-


American exiles
B. has been affected by the debate over Juanes
taking place in Miami’s Spanish-language media
C. will change now that younger Cuban-Americans
are disagreeing with the older generation
D. may become friendlier if the Juanes concert in
Havana goes well
E. was greatly improved by the appearance in
Havana of Bonnie Raitt and The Police

78. We learn from the passage that older Cuban-


Americans oppose Juanes’s concert because ----.

A. they believe the Cuban government is using him


for propaganda purposes
B. Miami’s Spanish-language radio and television
programs have been severely criticizing it
C. they are disturbed by the younger Cuban-
Americans’ artistic choices
D. they left Cuba soon after the 1959 revolution
E. they never want to see a further softening in
American policy towards Cuba

258
1. – 18. sorularda, cümlede boş bırakılan yerlere 7. For a long time, scientists ---- by the fact that
uygun düşen sözcük ya da ifadeyi bulunuz. the French ---- to eat fatty diets rich in red meat
but to live as long as those on lean and vegetarian
1. The ---- of potassium in the human body is diets.
dramatically demonstrated when too much or too
little of it is secreted. A. had been intrigued / are tending
B. were intrigued / would have tended
A) comparison B) attribution C) significance C. would be intrigued / have been tending
D) marginality E) demand D. will have been intrigued / would tend
E. have been intrigued / tend

2. In the past decade, technology has become ----


to freeze human embryos and then successfully 8. Although people ---- with both “good” brown fat
transplant them into host mothers, thereby giving cells and normal white fat cells, usually the former
some young women the option of postponing ---- after infancy.
childbirth.
A. have been born / lose
A) appropriate B) negligible C) potential B. were born / will lose
D) degraded E) available C. are born / are lost
D. had been born / had lost
E. are to be born / had been lost
3. Most multiple-sclerosis patients suffer
weakness and visual problems, and they become
---- more disabled. 9. As part of an effort by the UK government to
track down people at risk of a heart attack, British
A) hardly B) progressively C) absolutely doctors ---- to investigate the family history of
D) solely E) plausibly those of their patients who ---- high cholesterol.

A) had been told / are having


4. Traditionally, medicine has taken a paternalistic B) are telling / have had
stance towards patients, with the all-knowing C) would be told / will have
physician ---- wisdom from on high, but that is D) have been told / have
becoming increasingly unacceptable. E) had told / had had

A) informing B) withdrawing C) requesting


D) providing E) commenting 10. The latest research finding that older adults
are generally happier than younger ones ----
superficially at odds with many studies that ----
5. Short-term memory records the information in older people are at higher risk for depression and
front of us right now, though a portion of this other mental health problems.
seems to evaporate soon afterward; the remainder
---- a process in the brain called consolidation, A. seems / have found
which makes it permanent. B. seemed / will find
C. has seemed / would find
A) goes through B) gives off C) gets away with D. had seemed / have been finding
D) puts off E) comes up with E. could have seemed / would have found

6. More than 350,000 people in the UK (25,000 of 11. In Britain, the highest-level medical jobs
them children) suffer from Type I diabetes, an continue to be dominated by men of 55 or more,
auto-immune condition that is ---- by a variety of but these distinguished consultants and
largely unknown genetic and environmental professors ---- within the next ten years, and this
factors. ---- more women to reach the top tier.

A) taken part in B) brought about A. will have been retiring / is allowing


C) cut down on D) done away with B. have retired / allowed
E) got over C. would have retired / will be allowing
D. had been retiring / has allowed
E. will be retiring / will allow

12. In the late 1950s and early 1960s, 85,000 baby


teeth were collected ---- children in the
Midwestern US as part of a study of a possible
link ---- the testing of atomic bombs and human
health.

A) around / beside B) with / from


C) inside / throughout D) from / between
E) under / among

259
13. Evidence clearly shows there is a preventive 19. – 23. sorularda, aşağıdaki parçada
effect ---- aspirin and other anti-inflammatory numaralanmış yerlere uygun düşen sözcük ya da
drugs ---- colorectal cancer. ifadeyi bulunuz.

A) after / with B) on / to C) of / on Ultraviolet (UV) light from the sun is classified into
D) without / by E) between / beside three types: ultraviolet A (UVA), ultraviolet B (UVB),
and ultraviolet C (UVC), depending on its wavelength.
UV light in small amounts is (19) ----, as it helps the
14. ---- the so-called swine flu turns out to be less body produce Vitamin D. (20) ----, larger amounts of
frightening than first feared, it is only a matter of UV light damage DNA (the body’s genetic material)
time before a deadlier one comes along. and alter the amounts and kinds of chemicals (21) ----
the skin cells make. UV light also (22) ---- folic acid,
A) Unless B) Even if C) Whether sometimes resulting in deficiency of that essential
D) While E) When vitamin in fair-skinned individuals. Although UVA
penetrates deeper (23)---- the skin, UVB is
responsible for at least three quarters of the
15. All efforts by the Chinese government to damaging effects of UV light, including tanning,
eliminate the most widespread parasitic infection burning, premature skin aging, wrinkling, and skin
in the country failed, ---- a new drug arrived in the cancer.
1980s that was said to be highly effective in
controlling the disease. 19.
A) neutral B) beneficial C) indeterminate
A) just as B) so that C) in case D) minimal E) economical
D) even after E) whereby

20.
16. After Hurricane Katrina hit the US city of New A) Additionally B) Similarly C) However
Orleans in 2005 and left it without electricity for D) Indeed E) Otherwise
many weeks, mold and spores easily grew in the
still-habitable houses, ---- respiratory and skin
problems are still widespread. 21.
A) that B) what C) where
A) because B) so C) although D) whatever E) wherever
D) yet E) whenever

22.
17. Many surgeons believe that a patient ---- face A. should break down
bears a calm expression immediately before an B. must break down
operation is likely to require less anaesthesia C. broke down
during the procedure. D. might have broken down
E. may break down
A) what B) which C) whatever
D) that E) whose
23.
A) with B) from C) into
18. In the developing world, the public-sector D) without E) beside
health care infrastructure is typically
overwhelmed, which is not surprising considering
---- national governments spend on health as a 24. – 35. sorularda, verilen cümleyi uygun şekilde
share of national income.
tamamlayan ifadeyi bulunuz.
A) how little B) few C) so much
24. Since the response to a cholera outbreak is
D) just as much E) how many
often led by medical professionals, ----.

A. epidemics can easily escalate into pandemics


when medications are scarce
B. another disease should be chosen by those in
charge
C. doctors and nurses must learn to cooperate in
emergency and non-emergency situations alike
D. other aspects, such as environmental or
communication issues, might tend to be
neglected
E. cholera often results in severe dehydration and
even death

260
25. ---- that is often associated with extreme 29. Ever since scientists learned how to
tiredness. manipulate genes, ----.

A. Medical investigators decided to study the A. doctors and patients have pinned their hopes on
biochemical clock that keeps bodies running this knowledge being transmuted into readily
parallel with the sun available medical procedures
B. Molecular biologists are feverishly at work on a B. advances in gene therapy are expected to be
new research project announced by research teams in Japan and
C. Most disorders requiring surgery during Argentina in the near future
pregnancy centre on the abdominal region C. all gene therapy rests on the idea that the
D. Occupational safety regulations issued by the US patient’s genes can be manipulated in such a
government have resulted in an industry record way that the organism fixes itself
E. Rheumatoid arthritis is one example of a disease D. various other therapies, such as bone-marrow
transplants, have proved inadequate in making
headway against the rare disorder of severe
26. If it is proven to be successful in clinical combined immuno-deficiency
studies, ----. E. no clinical trial expects to get ultimate approval in
the near future by health authorities in the US
A. significant results in scar reduction have already and Europe
been reported by the University of Manchester
B. an experimental drug has been designed to
reduce certain types of scarring 30. Laser light usually passes easily through
C. this new preparation comes after decades of bone, ----.
research into its anti-scarring properties
D. the new treatment could be used in the early A. except in cases where its use has been ruled out
management of wounds from surgery and injury B. unless the diagnosis is incorrect
E. patients are being told not to get their hopes up C. as will many orthopaedic surgeons
of the treatment being available soon D. whereas bone seldom displays resistance to it
E. but sound does not

27. Although it has a shameful history of


sabotaging the many attempts to bring universal 31. ----, he was one of three people awarded the
health care to the American people, ----. 2008 Nobel Prize for Medicine.

A. the medical-insurance industry now accepts the A. Only if Harald zur Hausen’s earlier work with
need for radical changes in this area retroviruses is excluded
B. political leaders are increasingly unhappy about B. Unless scientists other than Harald zur Hausen
the opposition of the insurance industry had not received credit for their discoveries
C. a typical insurance company does not want to C. Unlike the medical breakthroughs achieved by
see such a model introduced in the US other men that had been honoured previously
D. many politicians agree with the position D. Because Harald zur Hausen had discovered that
traditionally taken by these companies human papilloma viruses cause cervical cancer
E. a majority of the population is frightened of the E. Now that human papilloma viruses are known to
government interfering in their relationships with cause cervical cancer
their insurers

32. ----, many women will welcome the opportunity


28. Although it is not an automatic correlation, ----. to take some preventive action against bone
thinning.
A. shock-absorbing shoes with cushioned soles
have long been considered better for runners A. As they enter middle age
B. there are two types of diabetes, Type I and Type B. Despite the fact that their emotional outbursts are
II seldom justified
C. people who have a family history of cancer may C. Knowing that sports injuries are almost inevitable
be at greater risk for the disease in the highly active
D. cholesterol-lowering drugs are covered by the D. Unless they are told to do so by their doctors
more comprehensive medical-insurance plans E. When doctors forget to advise their patients
E. one’s doctor should inspire a feeling of trust, not
fear

261
33. The waiting lists for organ transplants in the 36. – 38. sorularda, verilen Đngilizce cümleye
Western countries are becoming longer and anlamca en yakın Türkçe cümleyi bulunuz.
longer ----.
36. Certain kidney disorders are diagnosed by
A. so that desperate people in developing countries injecting a radioactive chemical into the patient’s
try to sell their own kidneys blood and then measuring the amount of
B. as more of the ageing populations there develop radioactive material passed in the urine.
life-threatening conditions
C. although China and India, for example, are trying A. Bazı böbrek bozuklukları, hastanın kanına
to stop the illegal trade in organs radyoaktif bir kimyasal zerk edilerek ve sonra
D. in case people hoping to receive a new liver or idrara geçen radyoaktif maddenin miktarı
heart don’t die prematurely ölçülerek teşhis edilir.
E. even if a greater number of surgeons could be B. Bazı böbrek bozukluklarını teşhis etmek için,
persuaded to perform such operations hastanın kanına radyoaktif bir kimyasal zerk
edildikten sonra, bu radyoaktif maddenin idrara
ne miktarda geçtiği ölçülür.
34. Common bacterial infections have evolved to C. Hastanın kanına zerk edilen radyoaktif bir
become a nightmare for hospitals worldwide ----. kimyasalın, radyoaktif madde olarak ne miktarda
idrara karıştığı ölçülerek belli bazı böbrek
A. because they have developed resistance to bozukluklarının teşhisi yapılır.
overused antibiotics D. Belli başlı böbrek bozukluklarının teşhisi,
B. despite the fact that hygiene levels are not as hastanın kanına radyoaktif bir kimyasalın zerk
high as they should be edilerek idrara karışan radyoaktif maddenin
C. if the staff within them don’t continue to follow miktarının ölçülmesi ile gerçekleştirilir.
proper sterilization procedures E. Böbrek bozukluklarından bazılarının teşhis
D. that are struggling with ever-increasing surgical edilmesi, radyoaktif bir kimyasalın, hastanın
and administrative costs kanına zerk edildikten sonra, idrara geçen
E. before antibiotics came into widespread use after radyoaktif madde ölçülerek gerçekleştirilir.
the Second World War

37. Milk and other dairy products have long been


35. The British Medical Association is coming recognized as highly nutritious foods, rich in
under pressure to support compulsory childhood protein and minerals necessary for good teeth
vaccinations, ----. and strong bones.
A. when such interventions are insisted on by the A. Protein ve mineraller bakımından zengin olan süt
parents ve çeşitli süt ürünlerinin, sağlam diş ve kuvvetli
B. whereas there are several diseases for which kemikler için önemli derecede yararlı gıdalar
there are no vaccinations olduğu her zaman kabul görmüştür.
C. after a false scare about autism led to a drop in B. Eskiden beri kabul edildiği üzere, sağlam diş ve
vaccinations and a big jump in measles cases kuvvetli kemikler için zengin protein ve mineraller
D. most of which were of no value after a certain içeren süt ve süt ürünleri, oldukça besleyici
age gıdalardır.
E. but this fails to take into account the views of the C. Sağlam diş ve kuvvetli kemikler için zengin
general public protein ve mineraller içeren süt ve süt ürünlerinin,
çok yararlı gıdalar olduğu eskiden de kabul
edilirdi.
D. Süt ve diğer süt ürünleri, eskiden beri, sağlam diş
ve kuvvetli kemikler için gerekli olan protein ve
mineraller bakımından zengin, son derece
besleyici gıdalar olarak kabul edilmektedir.
E. Süt ve çeşitli süt ürünleri, sağlam diş ve kuvvetli
kemikler için zengin protein ve mineraller içerirler
ve bu bakımdan her zaman son derece yararlı
gıdalar olarak kabul edilirler.

262
38. Almost all the molecules a cell makes are 41. Egzersiz, kalp hastalığı riskini yarıya
composed of carbon atoms bonded to one indirebilir, ancak çoğu yetişkin, tavsiye edilen
another and to atoms of other elements. egzersizleri yapmayı ihmal etmektedir.

A. Hem birbirine hem de diğer elementlerin A. Adults in general neglect the kind of exercises
atomlarına bağlı karbon atomlarından oluşan they have been advised to do even though they
moleküllerin tümü, hücrelerce üretilir. are aware that exercise reduces the risk of heart
B. Bir hücrenin ürettiği moleküllerin hemen hemen disease by half.
tümü, birbirine ve diğer elementlerin atomlarına B. The risk of heart disease can be reduced by half
bağlı karbon atomlarından oluşmaktadır. through exercise, although a great majority of
C. Moleküllerin tümü, gerek birbirlerine gerek diğer adults are indifferent to the exercises they have
elementlerin atomlarına bağlı karbon atomları been advised to do.
olup, hücrelerde oluşur. C. Exercise can cut the risk of heart disease in half,
D. Hemen hemen bütün moleküller, hücrelerde but most adults neglect to do the exercises
üretilerek, birbirlerine ve ayrıca diğer elementlerin recommended.
atomlarına bağlı karbon atomlarını içerirler. D. Since many adults fail to do the recommended
E. Birbirlerine olduğu kadar diğer elementlerin exercises, they always face the risk of heart
atomlarına da bağlı karbon atomlarının disease that can be cut down in half through
oluşturduğu moleküllerin neredeyse tümü, exercise.
hücrelerde üretilir. E. Exercise is indispensable in order to reduce the
risk of heart disease by half, but a growing
number of adults refuse to do their recommended
39. – 41. sorularda, verilen Türkçe cümleye exercises.
anlamca en yakın Đngilizce cümleyi bulunuz.
42. – 46. sorularda, boş bırakılan yere, parçada
39. Kemoterapinin yan etkileri, ilaçların, hızla anlam bütünlüğünü sağlamak için getirilebilecek
bölünen normal hücreler üzerindeki etkilerinden cümleyi bulunuz.
ileri gelmektedir.
42. Down’s syndrome, muscular dystrophy, and
A. It is the adverse effects drugs have on rapidly haemophilia may be among the best-known
divided normal cells that lead to the side effects genetic diseases, but they are most certainly not
of chemotherapy. alone. Several thousand human genes are linked,
B. Since drugs have adverse effects on normal cells when they fail to work properly, to more than
that divide very fast, this leads to the side effects 4,000 heritable genetic diseases. Moreover, only a
of chemotherapy. handful of these diseases are treatable. ----
C. The side effects of chemotherapy are closely
related to the effects that drugs have on rapidly A. Deactivating such genes during embryonic
divided normal cells. development does not kill the embryo, which
D. The side effects of chemotherapy are due to the suggests that disease-related genes are recently
drugs’ effects on normal cells that rapidly divide. evolved.
E. Due to the rapid division of normal cells, brought B. Researchers have found that the majority of
about by the effects of drugs, chemotherapy has disease-causing genes were originally present in
side effects. single-celled organisms and that most of the rest
arose much later.
C. Haemophilia, for example, achieved widespread
40. Bazı kanser türlerine bağlı ölüm oranları son visibility among the European public when it
yıllarda düşmüştür, ancak kanserin yol açtığı afflicted several generations of royal families on
ölümlerin genel oranı, endişe verici bir şekilde the Continent.
artmaktadır. D. The incidence of juvenile diabetes, another
heritable disorder, is on the rise in the US, much
A. In recent years, death rates connected with some to the alarm of public-health authorities.
kinds of cancer have fallen dramatically, while the E. Any way of systematizing knowledge about them
number of deaths caused by other types of would thus be welcome, starting with features
cancer has shown much increase. that the genes that cause diseases have in
B. There has been a noticeable decrease in recent common.
years in certain cancer death rates even though
the average rate of deaths related to cancer in
general has been increasing seriously.
C. In recent years, a remarkable decrease of deaths
related to various types of cancer has been
noticed, although there is a worrying increase in
the number of deaths due to cancer in general.
D. Some types of cancer have a low rate of death,
but in recent years there has been a worrying
increase in the overall rate of deaths due to
cancer in general.
E. Death rates due to some types of cancer have
decreased in recent years, but the overall rate of
deaths caused by cancer is alarmingly on the
rise.

263
43. In 1572, Michel de Montaigne, a French 45. ---- Their findings raise possible health
philosopher, observed that “there are men on concerns for those working in the manufacture of
whom the mere sight of medicine is operative.” the materials. Carbon nanotubes are rolled-up
Over the centuries, all manner of sugar pills and sheets of graphite thousands of times thinner
bitter tonics have been given to patients in the than a human hair. Because they are immensely
belief that they might do some good and probably strong and are good electricity conductors, they
will do no harm. ---- While some consider this a are poised for use in a wide range of fields, from
virtuous lie, others argue it is unethical. engineering to medicine. However, their similarity
in shape to asbestos fibres, which are known to
A. The problem is that doctors have usually damage the lungs, is giving rise to fears of their
prescribed such placebos while telling patients adverse effects on human health.
that they are sure to make them feel better.
B. An added benefit of this approach is that the A. Carbon nanotubes are unlikely to pose risks to
prescribed items are available at much lower cost the general public when incorporated into
than standard pharmaceuticals, thus relieving products.
families financially. B. In studies done on mice, inhaling nanotubes
C. Fortunately, the American Medical Association affected the function of T-cells, a type of white
advised its members in 2006 to use a placebo for blood cell that organizes the immune system.
diagnosis or treatment only if the patient is so C. Scientists are trying to determine if the production
informed and agrees to it. of carbon nanotubes has any biological after-
D. Similarly, so-called faith healing carried out by effect.
unconventional religious leaders has hundreds of D. Suppression by nanotubes of the immune system
thousands of believers worldwide. in mice has been halted by administering one of
E. There are two explanations for the apparent the standard anti-inflammatory drugs.
popularity of placebos: they do, in fact, work; and E. Inhaling carbon nanotubes can suppress the
they are used by overworked doctors to deal with immune system, according to scientists who have
troublesome patients. just completed a study of this new substance.

44. Like many other activities, global health has 46. Infections in the roots of teeth are very difficult
fashions. ---- Recently, though, the focus has to treat. ---- The infected material must then be
shifted to malaria. This tropical disease kills a cleaned out completely and the drilled section
million people a year, most of them children, and filled in. Although the procedure is routine, it is
debilitates hundreds of millions more. That is why common for some of the bacteria to survive and,
researchers are racing against one another to be therefore, for infections to re-emerge shortly after
the first to devise an effective vaccine. treatment.

A. However, not all developing countries are A. Wound infections bear some similarity to this and
struggling with health-related issues. can result in gangrene if not treated successfully.
B. Before Jonas Salk came up with his polio B. The tooth needs to be drilled into, right down to
vaccine, many parents lived in fear of their the bottom of the nerve-carrying canal that runs
children being struck down by the disease. through the root.
C. In Eastern Europe, for example, there has been C. This may help to explain the related mystery of
much concern about illicit drug use since the why dentists have one of the highest suicide
Cold War ended. rates among professions.
D. For the past couple of decades, AIDS has D. Surprisingly, salty foods are just as damaging to
captured both the imagination and the research the teeth as sweet desserts like candy and cake
dollars. are.
E. In contrast, tropical diseases have not always E. Unfortunately, most medical-insurance plans do
received the attention they deserve from either not cover such procedures, even though they are
national governments or international as traumatic to the body as some surgical
organizations. operations.

264
47. – 51. sorularda, karşılıklı konuşmanın boş 49.
bırakılan kısmını tamamlayabilecek ifadeyi Nancy: - Things are getting crazier and crazier in
bulunuz. science. Now the US government has approved a
new drug that is not made in a factory but in
47. genetically-modified goats!
Robert: - Like most people, you probably think that Julie: - ----
the billions of dollars poured by the West into Nancy: - They secrete the drug in their milk, and
fighting diseases in poor countries have produced it’s then used to untangle blood clots in patients.
only good results. Julie: - I suppose that’s okay, as long as the
Tom: - Of course, I do. For one thing, it’s well- animals are not harmed in the process.
known that millions of people are alive today
because of the coordinated distribution of HIV A. Is that a drug that you and I are likely to need in
drugs in developing countries. the future?
Robert: - ---- B. Won’t that lead to unemployment in the drug
Tom: - That’s a serious issue that needs to be industry?
addressed. C. Are the goats even aware of what has been done
to them?
A. The World Health Organization has been D. Why has the government allowed the producers
overseeing disease-elimination programmes that of this drug to go so far beyond what is normal?
are making excellent progress. E. What do the goats do that a factory can’t?
B. You’re absolutely right. In fact, funding for health
support in poor countries has increased
enormously in recent years. 50.
C. That’s true; but there’s a minus side, too: all that Doctor: - Now that you are receiving
money has undermined some countries’ health chemotherapy, I must warn you against
systems by luring doctors and nurses away from opportunistic infections.
hospitals to work for Western organizations. Patient: - What exactly are you talking about?
D. What’s more, in 2000, there was a worldwide Doctor: - ----
reaction of outrage over the situation in Africa, Patient: - So that probably means bacteria that are
where many were dying of AIDS because of the usually harmless can overwhelm my body and
unavailability of HIV drugs there. cause disease.
E. Don’t you know that an investigative report on the
impact of all that money on hospitals and clinics A. Well, you know chemotherapy weakens your
in the poorer countries has just been published? immune system.
B. Perhaps you should bring me your vaccine
report, so that we can check what diseases you
48. are immune to.
Barbara: - Isn’t your mother taking special herbal C. Well, I think it would be best if you avoided close
medicines for her arthritis? contact with people who have even the common
Mark: - Yes, she decided to go the natural route cold.
after reading about possible side effects of D. Once your chemotherapy is over, we’ll have to
conventional medication. check your body functions and infection
Barbara: - ---- resistance again.
Mark: - Then I’d better speak to her and urge her to E. Well, there are infections that affect people of a
see a specialist. certain age group.

A. If we lived in Britain, she wouldn’t have to pay to


see an arthritis specialist under the National
Health Service there. But here, cost is always a
consideration, unfortunately.
B. I’ve always confused rheumatoid arthritis with
osteoarthritis. Which one does your mother
have?
C. Many people believe they can better benefit from
herbs and minerals instead of drugs they
perceive to be risky.
D. Yet, it has now been proved that many herbal
medicines and alternative therapies do nothing to
help people with arthritis.
E. What kind of side effects are you referring to? I
think your mother has done the right thing.

265
51. 54. (I) Sutures have a long and bizarre history, dating
Michael: - I have finally decided to quit smoking back to ancient Egypt, where everything from tree
while I still have my health. bark to hair was used to stitch human flesh back
Dorothy: - Good for you! It’s hard to believe that together again. (II) In Egypt’s wars with its
cigarette smoking, which is banned inside most neighbours, unknown numbers of young men were
places now for health reasons, used to be wounded while fighting. (III) Fifty years ago, a soldier
encouraged by the government. injured on the battlefield would have been sewn up by
Michael: - You must be joking! medics using sheep’s gut. (IV) A hundred years
Dorothy: - ---- earlier, they would have used silk; before that, metal
wire. (V) Today, surgeons often prefer plastics such
A. You know I never joke about social problems and as polypropylene.
the situation of the poor. There’s nothing funny
about people going hungry. A) I B) II C) III D) IV E) V
B. Smoking has been proven to greatly increase the
likelihood of later getting lung cancer, heart
disease, and several other terrible conditions. 55. (I) The government of Botswana, knowing that up
C. No, I’m serious. In wartime, the generals told the to a third of its population had HIV or AIDS,
government they needed “tobacco as much as announced in 2001 that it would offer free
bullets,” and the government happily sent both. antiretroviral treatment to every citizen with AIDS. (II)
D. Yes, actually. Do you want to hear even funnier It was a major medical step forward for sub-Saharan
jokes? I know one about two nurses who worked Africa. (III) By the time the HIV drugs had hit the
in a children’s hospital. shelves, just about everybody in Botswana knew of it.
E. The harmful effects of passive smoking on (IV) And yet, on the last day of 2003, more than two
people who are not themselves smokers are only years after the launch of the programme, only about
now becoming known by the general public. 15,000 people had come forward for treatment. (V)
Due to widespread starvation and disease, the African
continent will always be a risky place to raise children.
52. – 56. sorularda, cümleler sırasıyla
okunduğunda parçanın anlam bütünlüğünü bozan A) I B) II C) III D) IV E) V
cümleyi bulunuz.

52. (I) The coming convergence of biology and 56. (I) Some pathogens are spread from one person
engineering will be led by information technologies, to another by direct contact. (II) They leave the first
which in medicine means the digitization of medical person through body openings, mucous membranes,
records and the establishment of an intelligent and skin wounds, and they enter the second person
network for sharing those records. (II) That essential through similar channels. (III) Many pathogens that
reform will enable many other big technological once meant certain death for people are now dealt
changes to be introduced throughout the health-care with easily by wide-spectrum antibiotics. (IV) Other
sector. (III) Just as important, it can make personal pathogens involve an intermediary carrier, such as an
medical information available to the patients too, insect. (V) The malarial parasite, for example, spends
enabling them to make decisions related to their own part of its life cycle in mosquitoes, and then enters a
health. (IV) Technology in general is advancing so person's bloodstream when the mosquito bites the
quickly that many people believe it will improve the person.
quality of life significantly. (V) However, many doctors,
and some patients, believe that patients lack the A) I B) II C) III D) IV E) V
knowledge to make informed decisions about their
own health.

A) I B) II C) III D) IV E) V

53. (I) One of the drawbacks of drugs in general is


that they have a variety of adverse side effects. (II)
Treatment of vertigo can be divided into three general
categories: specific, symptomatic, and rehabilitive. (III)
In the first category are included antibiotics,
anticoagulants, and surgery. (IV) In fact, many
different classes of drugs have been found to have
antivertiginous properties. (V) However, all of these
drugs can cause metabolic complications, so the
decision on which drug or combination to use is
based on their known properties and on the severity
and duration of the vertigo.

A) I B) II C) III D) IV E) V

266
57. – 60. soruları aşağıdaki parçaya göre 60. It is suggested in the passage that the human
cevaplayınız. brain ----.

The brain’s capacity for finding new information A. needs to be adjusted before deaf people can
processing pathways is thought to explain the hear with the implants
success of artificial cochleas, which have been B. has a history of entering into relationships with
implanted in the ears of approximately 100,000 some machines
hearing-impaired people around the world. They C. consists exclusively of neurons dedicated to the
typically have an array of electrodes, each of which sense of hearing
channels electrical signals toward the auditory nerve. D. is capable of creating new ways of processing
The electrodes can stimulate not just a single neuron information
in the brain but many simultaneously. When cochlear E. always needs to be supported by artificial
implants first appeared in the 1980s, many cochleas
neuroscientists expected them to work poorly, given
their primitive design. But the devices work well
enough for some deaf people to converse over the
telephone, particularly after an adjustment period
during which channel settings are fine-tuned to
provide the best reception. Patients’ brains somehow
figure out how to make the most out of the strange
signals. The surprising effectiveness of artificial
cochleas – together with other evidence of the brain’s
adaptability – has fuelled optimism about the
prospects for brain/machine substitution. A case in
point is an ongoing project at the University of
Southern California that seeks to create implantable
brain chips that can restore or enhance memory.

57. The passage makes clear that cochlear


implants ----.

A. have helped many people with hearing difficulty


to hear better
B. were enthusiastically approved by neuroscientists
when they were introduced
C. can only be used with a specially fine-tuned
telephone
D. will in the future be inserted into the brains of
patients
E. weaken the brain’s ability to remember electrical
signals

58. According to the passage, artificial cochleas


have enabled certain deaf people to ----.

A. understand the purpose of strange signals


B. conduct telephone conversations
C. look forward to future brain/machine cooperation
D. stimulate the neurons in their brains
E. hear just as well as normal people

59. We see from the passage that implantable


brain chips ----.

A. are now in use at the University of Southern


California
B. have already managed to improve people’s
memories
C. have already been developed to improve the
quality of hearing
D. represent the latest generation of
telecommunications technology
E. may be developed in the future to strengthen
memory

267
61. – 64. soruları aşağıdaki parçaya göre 63. According to the passage, six months is the
cevaplayınız. age ----.

By six months of age, the infant’s capacity to digest A) from which all future meals until adulthood are
and absorb a variety of dietary components as well as decided on
to metabolize and excrete the resulting products is B) when the greater importance of infancy becomes
near the capacity of the adult. Consideration of the clear
long-term effects of inadequate or excessive intakes C) in which any kind of replacement food is
during infancy now assumes greater importance. discontinued
These considerations about delivery of adequate D) when the addition of other foods to the baby’s diet
amounts of nutrients are the basis for many of the is recommended
feeding practices advocated during the second six E) when breast feeding should be stopped
months of life. Although it is clear that all nutrient
needs during this period can be met with reasonable
amounts of currently available infant formulas, 64. The passage states that complementary foods
addition of other foods after four to six months of age ----.
is recommended. In contrast, the volume of milk
produced by many women may not be adequate to A. are chosen according to how well they match the
meet all nutrient needs of the breast-fed infant beyond formula being given
about six months of age, especially iron. Thus, for B. should be mixed with formulas before being fed
breast-fed infants, complementary foods are an to infants
important source of nutrients. Complementary foods C. must completely replace the mother’s breast milk
(i.e., the additional foods, including formulas, given to in the diet
the breast-fed infant) or replacement foods (i.e., food D. should only be given when a mother’s milk lacks
other than formula given to formula-fed infants) the necessary iron
should be introduced step by step to both breast-fed E. are best added to an infant’s diet gradually
and formula-fed infants, beginning between four and
six months of age.

61. We see from the passage that breast feeding


an infant older than six months of age ----.

A. carries the possible risk of leaving the child short


of needed nutrients
B. must be replaced altogether by other nutrients
C. is much better than trying to replace it with
formula
D. meets all the prescribed nutritional requirements
of the child
E. has a long-term effect on inadequate or
excessive intake of food

62. It is pointed out in the passage that infant


formulas given in a child’s first year ----.

A. bring the digestive efficiency of the infant almost


up to the level of an adult
B. should never be combined with replacement
foods
C. must not be a cause of concern to mothers who
are still breast feeding
D. will interfere with the effectiveness of
complementary foods
E. are a reliable way to provide needed nutrition
after the first six months of life

268
65. – 68. soruları aşağıdaki parçaya göre 68. According to the passage, just appearing aged
cevaplayınız. without actually being so ----.

Following the growth of biological knowledge in the A. harms the community of potential partners
past few decades, a few researchers now believe B. means you will think fewer people are attractive
extension of the human lifespan might be within C. shows your real attitude towards babies
reach. Why do organisms – people included – age in D. affects your body’s ability to reproduce
the first place? Like machines, people wear out. E. makes it more difficult to have a baby
However, a machine can usually be repaired. A good
mechanic with a stock of spare parts can keep it
going indefinitely, to the point where no part of the
original remains. The question arises, of course, of
whether the machine is worth repairing. It is here that
people and nature disagree. From the individual’s
point of view, survival is a must. You cannot
reproduce unless you are alive. Since ageing is a
sure way of dying, it is no surprise that people want to
stop it from advancing. Moreover, even the
appearance of ageing can be harmful. It reduces the
range of potential partners who find you attractive –
since it is a sign that you are not going to be around
for a very long time to help bring up the baby – and
this, in turn, restricts your ability to reproduce. There
is a paradox, however: the individual’s evolved desire
not to age is opposed by another evolutionary force,
the disposable soma. The soma is all of a body’s cells
apart from the sex cells. Its role is to get the sex cells,
and thus the organism’s genes, into the next
generation. But evolutionary logic seems to require
the soma to age and die in order for a species to
continue. There is thus a premium on reproducing
early rather than conserving resources for a future
that may never come.

65. As we can infer from the passage, nature


tends to ----.

A. question whether human bodies can be repaired


B. favour shorter lifespan for living creatures
C. limit the number of partners a person can find
D. make people look old before they become old
E. oppose the evolutionary force of soma

66. It can be understood from the passage that


some researchers ----.

A. are trying to extend their own lives to the


maximum
B. are unsure if extending the human lifespan is
worth doing
C. now think it may be possible to help people to live
longer
D. are preparing a large supply of spare parts to
help people live indefinitely
E. have the goal of making people like machines

67. The passage indicates that the purpose of the


soma is to ----.

A. oppose a person’s desire not to become old


B. transmit genes to one’s child or children
C. separate itself from the organism’s sex cells
D. persuade the organism to reproduce earlier
E. prefer to conserve the organism’s resources

269
69. – 72. soruları aşağıdaki parçaya göre 71. According to the passage, the research team’s
cevaplayınız. experiments demonstrated that ----.

Individual “banks” of immune cells taken from pigs A. humans’ immune systems diminish in power after
might one day be used to boost our own immune childhood
systems or to fight HIV and cancer. Our immune B. immune cells that had been grown in pigs could
system’s T-cells, which play a key role in fighting off fight HIV and cancer
diseases, are sharpened during childhood to attack C. human T-cells need to be alongside pig immune
particular pathogens after encountering them. This cells to be fully functional
flexibility diminishes after a child reaches young D. piglets that were injected with human stem cells
adulthood, but researchers at a US university have soon produced a variety of human T-cells
come up with a way to revive it. According to them, if E. pig foetuses could be made to produce human T-
a human’s immune cells are transferred into a young cells
pig, they could be brought up to maximum
effectiveness (as in a child’s body), then implanted
back into the person they came from. The research 72. The passage suggests that the implantation of
team has already had success with experiments pig-boosted T-cells back into the original human
where human stem cells were injected into developing donor ----.
pig foetuses; when the piglets were born, the injected
cells had multiplied and matured into a diverse range A. has not been attempted yet by the researchers
of human T-cells, alongside the pig’s own immune B. was successful in restoring the person’s immune
cells, that were shown to be fully functional. The chief system to childhood levels
researcher envisions this approach eventually being C. resulted in a new immune system for him or her
used to make human cells that fight specific diseases. that was capable of fighting specific diseases
The necessary technology is available now to D. was stopped by the researchers’ discovery of a
introduce the technique widely, provided that dormant pig virus in the DNA
regulatory authorities can be convinced that it can be E. was refused by him or her out of fear of the
safely tested in humans. However, the fear is that possible dangers it might bring
dormant pig viruses buried in their DNA could be
spread to humans. Another potential danger is that
human derived cells might pick up surface molecules
from the pig. This could make the transferred cells
themselves targets for immune destruction. The pigs
might also produce too few human cells to fight
disease.

69. We can understand from the passage that


regulatory authorities ----.

A. are concerned that using pigs to grow human


immune cells might be harmful
B. should first convince the researchers to safely
test the technique against specific diseases
C. are responsible for protecting the health of baby
pigs implanted with human cells
D. are most afraid of the possibility of the pigs’
inability to produce enough human immune cells
E. have been working closely with the researchers
to ensure the experiments were done correctly

70. The passage points out that a human’s


immune system ----.

A. grows gradually stronger as he or she gets older


B. is exactly the same as a pig’s
C. functions at peak levels during childhood
D. eventually declines to zero in old age
E. consists of many copies of one kind of T-cell

270
73. – 76. soruları aşağıdaki parçaya göre 75. According to the passage, Washington
cevaplayınız. University researchers ----.

Asthma is a life-threatening, allergy-driven lung A. have been harshly criticized by their colleagues
disease common in wealthy countries. But exactly around the world
what causes it is unknown. Researchers at B. are strongly of the opinion that there is a link
Washington University believe the direct cause of between eczema and asthma
asthma is a chemical distress signal produced in skin C. feel that they still have to carry out new
that is damaged by another hazard of modern life: experiments as regards asthma
eczema. Unlike asthma, it is not dangerous, so D. have already come up with a set of solutions to
people rarely worry about it. Nevertheless, 17% of reduce the rate of eczema among American
children in America have it, and similarly high figures children
are found in Australia, Britain, and New Zealand. E. have collaborated with their British, Australian
What is particularly intriguing is that many people with and New Zealander colleagues to study eczema
eczema go on to develop asthma (in America, the and asthma
figure is 70%). That compares with an asthma
prevalence of 4-8% in the general population. The
Washington University group theorizes that the link 76. It is clearly pointed out in the passage that
between the two conditions is formed by thymic asthma ----.
stromal lympho-poietin (TSLP), a signalling molecule
secreted by damaged skin cells that elicits a strong A. is most widespread in countries with prosperous
immune response from the body to fight off invaders. populations
Thus, eczema-induced TSLP enters the bloodstream B. always causes serious damage in the lungs,
and, when it arrives at the lungs, sensitizes them so which try to fight it off
that they react to allergens that would not previously C. has only recently become a serious concern
have bothered them. In other words, they become among researchers in English-speaking countries
asthmatic. Several experiments carried out by the D. causes a great deal of TSLP secretion in the
researchers, only on mice, have confirmed that skin body, which leads to skin-allergy
damage creates susceptibility to asthma by releasing E. is generally ignored among the people who are
TSLP. not wealthy enough to seek medical aid

73. It is clear from the passage that a causative


relationship between skin damage and asthma ----.

A. has been proven only in English-speaking


countries
B. would mean that allergens would no longer play a
role in asthma attacks
C. has been established in mice but not yet in
human beings
D. means over-exposure to the sun may hurt the
lungs
E. does not provide immunity to sufferers of both
conditions

74. The passage points out that the prevalence of


eczema among children in America ----.

A. is related to their distress levels


B. is much higher than it is in other English speaking
countries
C. may be due to people’s failure to worry about it
D. results from the abundance of TSLP in that
country
E. is much higher than that of asthma in the general
population

271
77. – 80. soruları aşağıdaki parçaya göre 79. According to the passage, standard
cevaplayınız. procedures in a critical care unit ----.

Patients are admitted to critical care units from a A. must be carried out as soon as a patient is
variety of settings, including the emergency admitted
department, medical or surgical service, or operating B. are primarily restricted to patients with
room. Most critical care patients are acutely and cardiopulmonary arrest
severely ill, commonly with dysfunction or failure of C. can begin only after the immediate initial
more than one organ system. The initial assessment assessment has been completed
must be rapid and focus on real or potentially life D. should never be limited even though the patient
threatening processes that require immediate is receiving immediate intervention
intervention. An example is the resuscitation of a E. should be clearly defined before the patient’s
patient with cardiopulmonary arrest. The pace of case is fully specified
resuscitation is necessarily quick; physical
examination may be restricted initially to the central
nervous, cardiovascular, and respiratory systems and 80. The passage states that critical care units ----.
interventions may be limited to the essential ABCs of
airway, breathing, and circulation. Later, continuous A. only deal with patients having multi-organ
electrocardiographic monitoring, measurement of dysfunction or failure
blood pressure, and other standard procedures B. specialize in resuscitating those with
should start. In general, management of the critically cardiopulmonary arrest
ill patient should be based on an understanding of C. take into consideration the principles of
physiology and pathophysiology. Indeed, although the physiology when accepting patients
contributions of cell and molecular biology to critical D. have benefited financially from cell and molecular
care medicine are substantial, the critical care unit biology institutions
more resembles a physiology laboratory, since the E. accept patients from several different areas within
effects of its interventions can be directly observed. a hospital

77. The passage explains that initial assessment TEST BĐTTĐ.


in the critical care unit ----. CEVAPLARINIZI KONTROL EDĐNĐZ.

A. aims to resuscitate patients with cardiopulmonary


arrest
B. is only done for patients who are suffering from
various diseases
C. can itself become a life-threatening process if the
ABCs are not limited
D. must be immediate and centre on saving life
E. determines the ultimate survival of patients
suffering from heart disease

78. We learn from the passage that critical care


medicine ----.

A. rests primarily on the underlying disciplines of


physiology and pathophysiology
B. has received the greatest amount of support from
emergency departments
C. often must be based within an actual physiology
laboratory
D. revolves around the essential ABCs of airway,
breathing, and circulation
E. is necessarily concerned with the pace of
resuscitation in cardiopulmonary arrest

272
2006 ÜDS MART 2006 ÜDS EKĐM 2007 ÜDS MART 2007 ÜDS EKĐM
FEN SOSYAL SAĞLIK FEN SOSYAL SAĞLIK FEN SOSYAL SAĞLIK FEN SOSYAL SAĞLIK
1 E 41 A 1 A 41 B 1 C 41 E 1 D 41 A 1 C 41 E 1 A 41 B 1 A 41 B 1 C 41 C 1 B 41 D 1 E 41 D 1 C 41 A 1 A 41 C
2 D 42 D 2 D 42 C 2 E 42 B 2 C 42 D 2 A 42 A 2 B 42 E 2 D 42 C 2 B 42 D 2 D 42 A 2 B 42 C 2 B 42 C 2 E 42 E
3 A 43 A 3 B 43 D 3 A 43 E 3 B 43 B 3 B 43 D 3 C 43 A 3 C 43 D 3 E 43 B 3 A 43 C 3 C 43 A 3 D 43 A 3 D 43 D
4 B 44 D 4 C 44 E 4 B 44 D 4 E 44 E 4 D 44 B 4 A 44 B 4 E 44 E 4 C 44 C 4 C 44 B 4 D 44 B 4 A 44 B 4 C 44 A
5 C 45 E 5 B 45 A 5 D 45 A 5 D 45 A 5 E 45 A 5 E 45 C 5 B 45 C 5 E 45 D 5 E 45 E 5 A 45 E 5 E 45 A 5 B 45 B
6 A 46 B 6 E 46 D 6 B 46 C 6 A 46 C 6 A 46 E 6 D 46 D 6 D 46 A 6 D 46 E 6 D 46 A 6 C 46 C 6 C 46 D 6 D 46 C
7 B 47 C 7 D 47 C 7 A 47 A 7 E 47 B 7 D 47 D 7 E 47 B 7 E 47 D 7 B 47 A 7 C 47 E 7 D 47 D 7 A 47 E 7 A 47 D
8 E 48 D 8 A 48 D 8 E 48 E 8 B 48 D 8 E 48 B 8 D 48 E 8 C 48 B 8 D 48 B 8 B 48 B 8 E 48 B 8 B 48 B 8 B 48 E
9 D 49 A 9 C 49 A 9 B 49 D 9 D 49 E 9 C 49 A 9 C 49 C 9 A 49 C 9 C 49 D 9 E 49 D 9 A 49 E 9 D 49 D 9 E 49 B
10 B 50 B 10 E 50 B 10 C 50 B 10 E 50 A 10 B 50 C 10 B 50 B 10 B 50 A 10 A 50 B 10 A 50 C 10 C 50 A 10 B 50 E 10 C 50 A
11 C 51 A 11 B 51 B 11 E 51 A 11 C 51 E 11 D 51 A 11 D 51 D 11 E 51 C 11 B 51 E 11 E 51 E 11 B 51 B 11 E 51 A 11 D 51 B
12 B 52 D 12 A 52 D 12 D 52 E 12 A 52 D 12 E 52 C 12 E 52 C 12 B 52 D 12 E 52 C 12 A 52 C 12 A 52 E 12 D 52 C 12 E 52 E
13 A 53 A 13 C 53 B 13 A 53 C 13 B 53 B 13 D 53 D 13 C 53 B 13 C 53 E 13 B 53 D 13 C 53 B 13 D 53 C 13 A 53 B 13 B 53 A
14 D 54 C 14 E 54 C 14 D 54 D 14 C 54 C 14 B 54 B 14 B 54 D 14 D 54 C 14 D 54 B 14 B 54 D 14 E 54 B 14 C 54 C 14 A 54 D
15 E 55 B 15 D 55 C 15 B 55 C 15 A 55 E 15 C 55 D 15 C 55 E 15 E 55 E 15 A 55 C 15 D 55 B 15 C 55 D 15 B 55 B 15 C 55 C
16 B 56 C 16 A 56 D 16 A 56 B 16 E 56 C 16 E 56 C 16 E 56 B 16 A 56 D 16 B 56 B 16 B 56 E 16 D 56 B 16 A 56 E 16 B 56 C
17 A 57 D 17 E 57 A 17 D 57 A 17 D 57 A 17 D 57 E 17 D 57 A 17 B 57 A 17 E 57 A 17 A 57 C 17 B 57 A 17 E 57 B 17 D 57 A
18 C 58 B 18 C 58 E 18 C 58 D 18 E 58 D 18 B 58 A 18 A 58 E 18 A 58 B 18 C 58 D 18 D 58 A 18 C 58 D 18 C 58 E 18 A 58 E
19 D 59 E 19 D 59 C 19 E 59 B 19 B 59 B 19 C 59 B 19 B 59 A 19 E 59 C 19 D 59 C 19 E 59 D 19 A 59 E 19 A 59 C 19 E 59 D
20 E 60 C 20 E 60 B 20 C 60 C 20 D 60 E 20 D 60 D 20 A 60 E 20 C 60 A 20 B 60 E 20 C 60 A 20 D 60 C 20 B 60 E 20 A 60 B
21 D 61 B 21 C 61 C 21 E 61 D 21 C 61 A 21 E 61 E 21 E 61 A 21 D 61 B 21 C 61 A 21 C 61 C 21 E 61 D 21 E 61 A 21 C 61 B
22 E 62 A 22 B 62 E 22 D 62 A 22 A 62 D 22 A 62 D 22 B 62 C 22 B 62 C 22 D 62 D 22 A 62 A 22 B 62 A 22 D 62 D 22 A 62 D
23 D 63 D 23 D 63 A 23 A 63 B 23 B 63 B 23 C 63 E 23 C 63 B 23 D 63 D 23 B 63 A 23 E 63 B 23 A 63 C 23 E 63 A 23 D 63 E
24 A 64 E 24 A 64 B 24 D 64 C 24 C 64 D 24 A 64 C 24 D 64 D 24 E 64 E 24 A 64 C 24 B 64 E 24 C 64 B 24 B 64 D 24 B 64 B
25 C 65 C 25 B 65 D 25 B 65 E 25 E 65 E 25 B 65 A 25 E 65 C 25 A 65 D 25 E 65 A 25 D 65 D 25 E 65 E 25 E 65 C 25 E 65 C
26 B 66 B 26 E 66 E 26 C 66 A 26 D 66 A 26 E 66 B 26 A 66 A 26 D 66 B 26 B 66 B 26 C 66 B 26 A 66 C 26 D 66 E 26 D 66 D
27 C 67 E 27 C 67 B 27 B 67 C 27 C 67 B 27 D 67 D 27 B 67 B 27 B 67 D 27 C 67 D 27 D 67 D 27 D 67 B 27 B 67 A 27 A 67 C
28 D 68 A 28 E 68 A 28 A 68 E 28 B 68 C 28 A 68 E 28 D 68 A 28 E 68 C 28 A 68 A 28 B 68 C 28 C 68 D 28 C 68 C 28 B 68 A
29 C 69 D 29 A 69 D 29 D 69 B 29 E 69 D 29 B 69 C 29 B 69 D 29 D 69 D 29 D 69 D 29 A 69 A 29 B 69 E 29 B 69 D 29 C 69 E
30 E 70 E 30 D 70 B 30 A 70 C 30 A 70 E 30 C 70 A 30 A 70 A 30 C 70 A 30 B 70 E 30 E 70 E 30 A 70 A 30 D 70 A 30 D 70 A
31 D 71 D 31 C 71 C 31 B 71 D 31 D 71 B 31 B 71 E 31 E 71 C 31 D 71 C 31 E 71 D 31 B 71 C 31 E 71 E 31 E 71 B 31 E 71 C
32 B 72 E 32 B 72 A 32 E 72 B 32 C 72 D 32 E 72 C 32 C 72 D 32 A 72 B 32 C 72 C 32 E 72 B 32 B 72 C 32 A 72 E 32 B 72 B
33 A 73 C 33 A 73 D 33 D 73 C 33 B 73 C 33 A 73 E 33 A 73 A 33 E 73 D 33 A 73 E 33 A 73 E 33 D 73 B 33 C 73 D 33 A 73 D
34 E 74 B 34 D 74 C 34 A 74 E 34 D 74 B 34 D 74 A 34 E 74 C 34 B 74 C 34 D 74 D 34 C 74 D 34 A 74 D 34 D 74 C 34 D 74 E
35 C 75 A 35 C 75 E 35 B 75 D 35 B 75 E 35 B 75 D 35 B 75 B 35 E 75 A 35 B 75 E 35 D 75 A 35 C 75 A 35 C 75 E 35 C 75 A
36 B 76 D 36 A 76 A 36 D 76 A 36 A 76 C 36 C 76 B 36 D 76 A 36 B 76 E 36 A 76 B 36 A 76 B 36 B 76 D 36 B 76 B 36 A 76 B
37 E 77 B 37 D 77 E 37 E 77 C 37 E 77 D 37 B 77 C 37 A 77 E 37 D 77 B 37 D 77 E 37 B 77 C 37 A 77 B 37 E 77 D 37 E 77 E
38 B 78 E 38 B 78 D 38 C 78 B 38 D 78 B 38 E 78 B 38 C 78 A 38 E 78 C 38 E 78 D 38 E 78 A 38 E 78 A 38 D 78 E 38 C 78 C
39 E 79 D 39 C 79 A 39 B 79 E 39 B 79 A 39 D 79 D 39 B 79 B 39 A 79 D 39 A 79 A 39 D 79 D 39 C 79 E 39 E 79 C 39 B 79 A
40 C 80 A 40 E 80 B 40 A 80 D 40 C 80 D 40 B 80 E 40 A 80 A 40 D 80 E 40 E 80 E 40 C 80 E 40 B 80 C 40 B 80 B 40 D 80 C
2008 ÜDS MART 2008 ÜDS EKĐM 2009 ÜDS MART 2009 ÜDS EKĐM
FEN SOSYAL SAĞLIK FEN SOSYAL SAĞLIK FEN SOSYAL SAĞLIK FEN SOSYAL SAĞLIK
1 D 41 A 1 C 41 B 1 E 41 E 1 C 41 B 1 A 41 E 1 E 41 A 1 D 41 C 1 B 41 B 1 A 41 C 1 E 41 A 1 E 41 E 1 C 41 C
2 A 42 E 2 E 42 E 2 C 42 D 2 E 42 E 2 C 42 C 2 D 42 E 2 B 42 A 2 D 42 A 2 E 42 D 2 C 42 D 2 C 42 D 2 E 42 E
3 B 43 C 3 C 43 D 3 E 43 B 3 D 43 B 3 D 43 B 3 A 43 C 3 B 43 B 3 C 43 C 3 A 43 B 3 E 43 C 3 E 43 E 3 B 43 A
4 E 44 B 4 A 44 C 4 D 44 C 4 E 44 E 4 C 44 A 4 D 44 A 4 E 44 D 4 A 44 A 4 C 44 A 4 B 44 D 4 A 44 A 4 D 44 D
5 A 45 D 5 B 45 B 5 B 45 A 5 D 45 D 5 B 45 C 5 B 45 D 5 C 45 E 5 C 45 B 5 B 45 D 5 D 45 E 5 D 45 E 5 A 45 E
6 B 46 C 6 D 46 E 6 A 46 D 6 B 46 A 6 E 46 E 6 E 46 E 6 B 46 C 6 E 46 E 6 D 46 E 6 E 46 A 6 B 46 C 6 B 46 B
7 C 47 D 7 C 47 A 7 D 47 B 7 A 47 B 7 B 47 A 7 C 47 B 7 A 47 C 7 A 47 D 7 E 47 C 7 D 47 C 7 C 47 B 7 E 47 C
8 A 48 A 8 A 48 D 8 C 48 C 8 D 48 A 8 A 48 B 8 E 48 D 8 C 48 B 8 B 48 E 8 A 48 B 8 C 48 D 8 A 48 D 8 C 48 D
9 D 49 B 9 E 49 B 9 B 49 A 9 E 49 D 9 C 49 C 9 C 49 C 9 C 49 E 9 C 49 A 9 D 49 D 9 A 49 B 9 D 49 A 9 D 49 E
10 B 50 E 10 B 50 C 10 D 50 B 10 B 50 E 10 E 50 D 10 D 50 A 10 E 50 A 10 A 50 C 10 B 50 A 10 B 50 D 10 E 50 C 10 A 50 A
11 E 51 D 11 D 51 E 11 E 51 D 11 C 51 C 11 B 51 A 11 A 51 C 11 B 51 C 11 D 51 B 11 A 51 C 11 C 51 B 11 B 51 A 11 E 51 C
12 A 52 B 12 B 52 C 12 B 52 E 12 D 52 B 12 D 52 C 12 B 52 A 12 E 52 A 12 E 52 B 12 C 52 B 12 E 52 D 12 C 52 D 12 D 52 D
13 B 53 E 13 A 53 E 13 D 53 D 13 C 53 D 13 C 53 A 13 C 53 E 13 D 53 D 13 D 53 C 13 D 53 D 13 D 53 B 13 E 53 A 13 C 53 A
14 D 54 C 14 D 54 C 14 A 54 E 14 D 54 E 14 A 54 C 14 A 54 C 14 A 54 B 14 C 54 A 14 E 54 E 14 C 54 C 14 D 54 C 14 B 54 B
15 C 55 B 15 C 55 B 15 C 55 B 15 A 55 C 15 E 55 E 15 E 55 B 15 B 55 E 15 B 55 E 15 B 55 B 15 D 55 D 15 C 55 B 15 D 55 E
16 A 56 C 16 E 56 D 16 B 56 A 16 E 56 A 16 D 56 D 16 D 56 E 16 C 56 C 16 C 56 D 16 C 56 C 16 E 56 D 16 A 56 D 16 B 56 C
17 B 57 E 17 A 57 A 17 D 57 C 17 C 57 D 17 E 57 E 17 E 57 B 17 C 57 A 17 A 57 C 17 E 57 B 17 C 57 E 17 B 57 C 17 E 57 A
18 E 58 A 18 B 58 B 18 A 58 E 18 A 58 A 18 B 58 B 18 B 58 E 18 E 58 B 18 B 58 E 18 D 58 A 18 D 58 B 18 E 58 B 18 A 58 B
19 D 59 D 19 C 59 E 19 E 59 D 19 B 59 E 19 A 59 D 19 A 59 D 19 D 59 E 19 E 59 A 19 A 59 E 19 A 59 C 19 C 59 C 19 B 59 E
20 A 60 E 20 D 60 C 20 A 60 C 20 D 60 B 20 D 60 C 20 B 60 B 20 B 60 D 20 C 60 C 20 D 60 A 20 B 60 A 20 D 60 B 20 C 60 D
21 C 61 A 21 A 61 A 21 C 61 B 21 E 61 C 21 61 A 21 A 61 A 21 E 61 B 21 D 61 B 21 A 61 D 21 E 61 D 21 E 61 E 21 A 61 A
22 B 62 C 22 B 62 C 22 D 62 A 22 B 62 A 22 B 62 D 22 D 62 D 22 B 62 C 22 E 62 C 22 E 62 E 22 D 62 E 22 D 62 A 22 E 62 E
23 D 63 D 23 E 63 D 23 A 63 E 23 E 63 E 23 D 63 B 23 E 63 B 23 D 63 E 23 C 63 E 23 B 63 B 23 E 63 B 23 B 63 E 23 C 63 D
24 A 64 B 24 D 64 E 24 D 64 C 24 D 64 A 24 A 64 E 24 C 64 D 24 B 64 C 24 B 64 C 24 D 64 C 24 B 64 C 24 A 64 B 24 D 64 E
25 E 65 A 25 A 65 B 25 B 65 D 25 E 65 C 25 C 65 D 25 D 65 C 25 C 65 A 25 A 65 D 25 E 65 A 25 C 65 A 25 C 65 D 25 E 65 B
26 A 66 E 26 C 66 C 26 E 66 A 26 C 66 E 26 C 66 B 26 E 66 E 26 B 66 D 26 C 66 A 26 E 66 D 26 E 66 E 26 E 66 A 26 D 66 C
27 D 67 B 27 A 67 A 27 A 67 C 27 A 67 A 27 E 67 A 27 C 67 D 27 C 67 B 27 D 67 D 27 D 67 B 27 D 67 B 27 D 67 C 27 A 67 B
28 E 68 D 28 B 68 C 28 E 68 B 28 E 68 B 28 D 68 C 28 D 68 E 28 E 68 A 28 E 68 C 28 C 68 A 28 A 68 A 28 E 68 A 28 C 68 E
29 C 69 C 29 D 69 E 29 C 69 D 29 B 69 D 29 A 69 D 29 E 69 C 29 A 69 C 29 D 69 B 29 E 69 E 29 C 69 C 29 B 69 D 29 A 69 A
30 B 70 A 30 E 70 D 30 D 70 E 30 D 70 E 30 D 70 B 30 B 70 B 30 D 70 E 30 A 70 C 30 B 70 B 30 B 70 A 30 C 70 C 30 E 70 C
31 E 71 B 31 B 71 B 31 B 71 C 31 E 71 D 31 B 71 D 31 D 71 E 31 A 71 A 31 B 71 E 31 D 71 C 31 C 71 D 31 D 71 E 31 D 71 E
32 C 72 D 32 A 72 C 32 A 72 B 32 A 72 A 32 A 72 C 32 B 72 B 32 B 72 D 32 D 72 A 32 C 72 A 32 D 72 E 32 A 72 D 32 A 72 A
33 A 73 E 33 C 73 A 33 C 73 A 33 B 73 C 33 E 73 B 33 C 73 D 33 D 73 E 33 A 73 D 33 A 73 E 33 E 73 C 33 E 73 E 33 B 73 C
34 B 74 C 34 D 74 D 34 E 74 D 34 E 74 E 34 C 74 E 34 E 74 C 34 E 74 B 34 C 74 B 34 C 74 D 34 B 74 B 34 B 74 B 34 A 74 E
35 D 75 E 35 E 75 E 35 D 75 E 35 C 75 D 35 D 75 C 35 B 75 E 35 C 75 C 35 E 75 A 35 D 75 C 35 C 75 A 35 D 75 A 35 C 75 B
36 C 76 A 36 A 76 D 36 C 76 D 36 E 76 A 36 E 76 A 36 D 76 C 36 E 76 E 36 C 76 C 36 B 76 B 36 E 76 C 36 A 76 E 36 A 76 A
37 A 77 D 37 E 77 E 37 A 77 A 37 C 77 D 37 A 77 C 37 C 77 E 37 B 77 C 37 E 77 B 37 A 77 A 37 C 77 A 37 C 77 D 37 D 77 D
38 E 78 E 38 D 78 C 38 E 78 C 38 D 78 B 38 C 78 D 38 A 78 C 38 C 78 A 38 D 78 A 38 E 78 E 38 E 78 B 38 A 78 A 38 B 78 A
39 D 79 B 39 B 79 B 39 B 79 B 39 A 79 A 39 D 79 B 39 E 79 E 39 D 79 B 39 E 79 B 39 D 79 C 39 B 79 A 39 B 79 B 39 D 79 C
40 C 80 D 40 C 80 E 40 A 80 A 40 D 80 D 40 A 80 E 40 D 80 B 40 A 80 D 40 A 80 D 40 E 80 E 40 C 80 D 40 D 80 C 40 E 80 E

You might also like